Notes On Labor Relations

Download as pdf or txt
Download as pdf or txt
You are on page 1of 262

• Sec.

3, Article XIII
Labor Relations
Atty. Nolasco SECTION 3. The State shall afford
full protection to labor, local and
overseas, organized and unorganized,
1.1 Constitutional Basis; State Policies
and promote full employment and
A. Constitution equality of employment opportunities
for all.
• Sec. 18, Article II, Constitution
It shall guarantee the rights of all
SECTION 18. The State affirms labor workers to self-organization,
as a primary social economic force. It collective bargaining and
shall protect the rights of workers and negotiations, and peaceful concerted
promote their welfare. activities, including the right to strike
in accordance with law. They shall be
• Sec. 4, 8 ,11, and 16, Article III entitled to security of tenure, humane
conditions of work, and a living wage.
SECTION 4. No law shall be passed
They shall also participate in policy
abridging the freedom of speech, of
and decision-making processes
expression, or of the press, or the right
affecting their rights and benefits as
of the people peaceably to assemble
may be provided by law.
and petition the government for
redress of grievances. The State shall promote the principle
of shared responsibility between
SECTION 8. The right of the people,
workers and employers and the
including those employed in the
preferential use of voluntary modes in
public and private sectors, to form
settling disputes, including
unions, associations, or societies for
conciliation, and shall enforce their
purposes not contrary to law shall not
mutual compliance therewith to foster
be abridged.
industrial peace.
SECTION 11. Free access to the
The State shall regulate the relations
courts and quasi-judicial bodies and
between workers and employers,
adequate legal assistance shall not be
recognizing the right of labor to its
denied to any person by reason of
just share in the fruits of production
poverty.
and the right of enterprises to
SECTION 16. All persons shall have reasonable returns on investments,
the right to a speedy disposition of and to expansion and growth.
their cases before all judicial, quasi-
judicial, or administrative bodies.

MNI NOTES |1
B. Labor Code court or administrative agency or official shall
have the power to set or fix wages, rates of pay,
• Art. 218 – Declaration of Policy hours of work or other terms and conditions of
employment, except as otherwise provided under
A. It is the policy of the State:
this Code. (As amended by Section 3, Republic
a. To promote and emphasize the primacy Act No. 6715, March 21, 1989)
of free collective bargaining and
1.2 Definitions
negotiations, including voluntary
arbitration, mediation and conciliation, as • Art. 219, LC
modes of settling labor or industrial
disputes; a. "Commission" means the National Labor
Relations Commission or any of its divisions,
b. To promote free trade unionism as an as the case may be, as provided under this
instrument for the enhancement of Code.
democracy and the promotion of social
justice and development; b. "Bureau" means the Bureau of Labor
Relations and/or the Labor Relations
c. To foster the free and voluntary Divisions in the regional offices established
organization of a strong and united labor under Presidential Decree No. 1, in the
movement; Department of Labor.

d. To promote the enlightenment of c. "Board" means the National Conciliation


workers concerning their rights and and Mediation Board established under
obligations as union members and as Executive Order No. 126.
employees;
d. "Council" means the Tripartite Voluntary
e. To provide an adequate administrative Arbitration Advisory Council established
machinery for the expeditious settlement under Executive Order No. 126, as amended.
of labor or industrial disputes;
e. "Employer" includes any person acting in
f. To ensure a stable but dynamic and just the interest of an employer, directly or
industrial peace; and indirectly. The term shall not include any
labor organization or any of its officers or
g. To ensure the participation of workers
agents except when acting as employer.
in decision and policy-making processes
affecting their rights, duties and welfare. f. "Employee" includes any person in the
employ of an employer. The term shall not be
B. To encourage a truly democratic method of
limited to the employees of a particular
regulating the relations between the employers
employer, unless the Code so explicitly
and employees by means of agreements freely
states. It shall include any individual whose
entered into through collective bargaining, no

MNI NOTES |2
work has ceased as a result of or in m. "Managerial employee" is one who is
connection with any current labor dispute or vested with the powers or prerogatives to lay
because of any unfair labor practice if he has down and execute management policies
not obtained any other substantially and/or to hire, transfer, suspend, lay-off,
equivalent and regular employment. recall, discharge, assign or discipline
employees. Supervisory employees are those
g. "Labor organization" means any union who, in the interest of the employer,
or association of employees which exists in effectively recommend such managerial
whole or in part for the purpose of collective actions if the exercise of such authority is not
bargaining or of dealing with employers merely routinary or clerical in nature but
concerning terms and conditions of requires the use of independent judgment. All
employment. employees not falling within any of the above
definitions are considered rank-and-file
h. "Legitimate labor organization" means
employees for purposes of this Book.
any labor organization duly registered with
the Department of Labor and Employment, n. "Voluntary Arbitrator" means any
and includes any branch or local thereof. person accredited by the Board as such or any
person named or designated in the Collective
i. "Company union" means any labor
Bargaining Agreement by the parties to act as
organization whose formation, function or
their Voluntary Arbitrator, or one chosen
administration has been assisted by any act
with or without the assistance of the National
defined as unfair labor practice by this Code.
Conciliation and Mediation Board, pursuant
j. "Bargaining representative" means a to a selection procedure agreed upon in the
legitimate labor organization whether or not Collective Bargaining Agreement, or any
employed by the employer. official that may be authorized by the
Secretary of Labor and Employment to act as
k. "Unfair labor practice" means any unfair Voluntary Arbitrator upon the written request
labor practice as expressly defined by the and agreement of the parties to a labor
Code. dispute.

l. "Labor dispute" includes any controversy o. "Strike" means any temporary stoppage
or matter concerning terms and conditions of of work by the concerted action of employees
employment or the association or as a result of an industrial or labor dispute.
representation of persons in negotiating,
fixing, maintaining, changing or arranging p. "Lockout" means any temporary refusal
the terms and conditions of employment, of an employer to furnish work as a result of
regardless of whether the disputants stand in an industrial or labor dispute.
the proximate relation of employer and
q. "Internal union dispute" includes all
employee.
disputes or grievances arising from any

MNI NOTES |3
violation of or disagreement over any (d) "Code" means the Labor Code of the
provision of the constitution and by-laws of a Philippines, as amended.
union, including any violation of the rights
and conditions of union membership (e) "Employer" includes any person
provided for in this Code. acting in the interest of an employer,
directly or indirectly. The term shall not
r. "Strike-breaker" means any person who include any labor organization or any of
obstructs, impedes, or interferes with by its officers or agents except when acting
force, violence, coercion, threats, or as employer.
intimidation any peaceful picketing affecting
wages, hours or conditions of work or in the (f) "Employee" includes any person in
exercise of the right of self-organization or the employ of a particular employer. The
collective bargaining. term shall not be limited to the employees
of a particular employer, unless the Code
s. "Strike area" means the establishment, so explicitly states. It shall include any
warehouses, depots, plants or offices, individual whose work has ceased as a
including the sites or premises used as result of or in connection with any current
runaway shops, of the employer struck labor dispute or because of any unfair
against, as well as the immediate vicinity labor practice if he has not obtained any
actually used by picketing strikers in moving other substantially equivalent and regular
to and fro before all points of entrance to and employment.
exit from said establishment. (As amended by
Section 4, Republic Act No. 6715, March 21, (g) "Labor Organization" means any
1989) union or association of employees which
exists in whole or in part for the purpose
• Section 1, Rule 1, Book V, Omnibus Rules of collective bargaining or of dealing with
Implementing the Labor Code employers concerning terms and
conditions of employment.
SECTION 1. Definition of terms. —
(h) "Local Union" means any labor
(a) "Commission" means the National organization operating at the enterprise
Labor Relations Commission. level.

(b) "Bureau" means the Bureau of Labor (i) "National Union/Federation" means
Relations and/or the Industrial Relations any labor organization with at least ten
Division in the Regional Offices of the (10) locals or chapters each of which
Department of Labor and Employment. must be a duly recognized collective
bargaining agent.
(c) "Board" means the National
Conciliation and Mediation Board. (j) "Legitimate Labor Organization"
means any labor organization duly

MNI NOTES |4
registered with the Department of Labor clerical in nature but require the use of
and Employment and includes any independent judgment. All employees not
branch, local or affiliate thereof. falling within any of the above definitions
are considered rank-and-file employees
(k) "Company Union" means any labor for purposes of this Book.
organization whose formation, function
or administration has been assisted by any (p) "Voluntary Arbitrator" means any
act defined as unfair labor practice by the person accredited by the Board as such,
Code. or any person named or designated in the
collective bargaining agreement, by the
(l) "Bargaining Representative" means parties to act as their voluntary arbitrator,
a legitimate labor organization or any or one chosen, with or without the
duly authorized officer or agent of such assistance of the National Conciliation
organization whether or not employed by and Mediation Board, pursuant to a
the employer. selection procedure agreed upon in the
collective bargaining agreement, or any
(m) "Unfair Labor Practice" means
official that may be authorized by the
any unfair labor practice as expressly
Secretary of Labor and Employment to
defined in the Code.
act as voluntary arbitrator upon the
(n) "Labor or Industrial Dispute" written request and agreement of the
includes any controversy or matter parties to a labor dispute.
concerning terms or conditions of
(q) "Strike" means any temporary
employment or the association or
stoppage of work by the concerted action
representation of persons in negotiating
of employees as a result of a labor or
the fixing, maintaining, changing or
industrial dispute.
arranging of terms and conditions of
employment regardless of whether or not (r) "Strike-Breaker" means any person
the disputants stand in the proximate who obstructs, impedes, or interferes with
relationship of employers and employees. by force, violence, coercion, threats or
intimidation any peaceful picketing by
(o) "Managerial Employee" is one who
employees during any labor controversy
is vested with powers or prerogatives to
affecting wages, hours or conditions of
lay down and execute management
work or in the exercise of the right of self-
policies and/or to hire, transfer, suspend,
organization or collective bargaining.
layoff, recall, discharge, assign or
discipline employees. Supervisory (s) "Strike Area" means the
employees are those who, in the interest establishment, warehouse, depots, plants
of the employer, effectively recommend or offices, including the sites or premises
such managerial actions if the exercise of used as run-away shops, of the employer
such authority is not merely routinary or

MNI NOTES |5
struck against, as well as the immediate agent of the employees in an appropriate
vicinity actually used by picketing bargaining unit, for purposes of collective
strikers in moving to and fro before all bargaining.
points of entrance to and exits from said
establishment. (y) "Consent Election" means the
election voluntarily agreed upon by the
(t) "Lockout" means the temporary parties to determine the issue of majority
refusal of an employer to furnish work as representation of all the workers in the
a result of a labor or industry dispute. appropriate collective bargaining unit.

(u) "Internal Union Dispute" includes (z) "Run-Off" refers to an election


all disputes or grievances arising from between the labor unions receiving the
any violation of or disagreement over any two (2) higher number of voters when a
provision of the constitution and by-laws certification election which provides for
of a union, including any violation of the three (3) or more choices results in no
rights and conditions of union choice receiving a majority of the valid
membership provided for in this Code. votes cast, where the total number of
votes for all contending unions is at least
(v) "Appeal" means the elevation by an fifty percent (50%) of the number of
aggrieved party of any decision, order or votes cast.
award of a lower body to a higher body,
by means of a pleading which includes (aa) "Registration of Agreement" refers
the assignment of errors, memorandum of to the filing of the collective bargaining
arguments in support thereof, and the agreement with the Regional Office or the
reliefs prayed for. A mere notice of Bureau accompanied by verified proof of
appeal, therefore, does not constitute the posting and ratification and payment of
appeal as herein defined and understood, fee.
and shall not stop the running of the
period for perfecting an appeal. (bb) "Organized Establishment" refers
to a firm or company where there is a
(w) "Perfection of an Appeal" includes recognized or certified exclusive
the filing within the prescribed period, of bargaining agent.
the memorandum of appeal containing,
among others, the assignment of error/s, (cc) "Registration Proceedings" refer to
the argument in support thereof, the proceedings involving the application for
reliefs sought and posting of the appeal registration of labor organizations.
bond.
(dd) "Cancellation Proceeding" is the
(x) "Certification Election" means the process leading to the revocation of the
process of determining, through secret registration certificate of a labor
ballot, the sole and exclusive bargaining organization after due process.

MNI NOTES |6
including mandatory provisions for
grievances and arbitration machineries.
(ee) "Hearing Officers" are officers
appointed/designated in the Regional (kk) "Med-Arbiter" is an official in the
Office and authorized to hear and decide Regional Office authorized to hear,
cases under Section 2 of Republic Act conciliate, mediate and decide
No. 6715 and whose decision is representation cases, internal union and
appealable to the Commission. inter-union disputes.

(ff) "Union Accounts Examiners" are (ll) "Administrator" refers to the


officials in the Bureau or the Industrial Administrator of the Philippine Overseas
Relations Division in the Regional Office Employment Administration or the
empowered to audit books of accounts of National Conciliation and Mediation
the union. Board as the context so indicates.

(gg) "Representation Officer" refer to a • What is labor relations


person duly authorized to conduct and
supervise certification elections in “Labor Relations” refers to the interactions
accordance with Rule VI of this Book. between employer and employees or their
representatives and the mechanism by which
(hh) "Term of Office" means the tenure the standards and other terms and conditions
of office of elected officials of a labor of employment are negotiated, adjusted and
organization which is for a fixed period of enforced.
five (5) years.
1.3 Parties
(ii) "Cabo" refers to a person or group or
persons or to a labor group which, in the • Parties in labor relations and their
guise of a labor organization, supplies relationship'
workers to an employer, with or without
Employer
any monetary or other consideration
whether in the capacity of an agent of the Employee
employer or as an ostensible independent
contractor. Representative

(jj) "Collective Bargaining State


Agreement" refers to the negotiated
contract between a legitimate labor
organization and the employer
concerning wages, hours of work and all CASES
other terms and conditions of
employment in a bargaining unit,

MNI NOTES |7
DEVELOPMENT BANK OF THE To finally settle both the AA and GFPA issues,
PHILIPPINES, PETITIONER, V. it will be better to pay the AA, to be offset from
COMMISSION ON AUDIT, RESPONDENT. the amount already paid as GFPA with the
GR 210838, July 03, 2018 following suggested conditions:

FACTS: a. If the amount of the AA is more than


the GFPA, the differential amount will be
DBP, a government financial institution created paid to the employees.
and operating under its own charter was faced
with labor unrest in 2003 due to its employees' b. If the AA is less than the GFPA,
insistence that they be paid their benefits concerned employees shall no longer be
required to return the amount.
• Amelioration Allowance (AA),
• Cost of Living Allowance (COLA) and c. Those who did not receive the GFPA
• the Bank Equity Benefit Differential Pay will get their AA in full.
(BEBDP),
d. Retirees/resignees without the usual
for the year that the Department of Budget and waiver will likewise receive their AA in
Management Corporate Compensation Circular full. Those with waivers, do not get
No. 10 (DBM CCC No. 10) was declared anything more.
ineffective by this Court for non-publication.
DBP received Notice of Disallowance,
the employees' group and DBP arrived at an disallowing the grant of the GFPA. According to
agreement to put an end to the division causing COA's Legal and Adjudication Team,
disruptions in bank operations. industrial peace may not be used as a legal and
sufficient basis in granting monetary awards.
They adopted a board resolution approving a
one-time grant called the Governance Forum Furthermore, the GFPA partakes the nature of a
Productivity Award (GFPA) to DBP's officers compromise agreement and circumvents the rule
and employees. The total amount distributed was that only a settled claim may be a subject of
PhP170,893,689.00. compromise.

Audit Observation Memorandum (AOM) No. DBP assailed the ND by arguing that payment of
001 found the grant of the GFPA without legal the GFPA was made pursuant to the power of its
basis and recommended its refund. Board of Directors (BOD) to enter into a
compromise agreement for settlement of
the Executive Committee (Execom) of the DBP employees' claims; that industrial peace is a valid
adopted Resolution No. 0151 which granted the consideration for a compromise agreement; and
payment of Amelioration Allowance (AA) to bank that the GFPA was superseded and rendered
employees. The amount due as AA for individual inexistent by the grant of the AA to DBP's
employees was offset against the GFPA already employees.
received by them, in the following manner:

MNI NOTES |8
COA upheld the disallowance. agreement to settle a labor dispute, allegedly
an ultra vires act of DBP's BOD.
The FAIO ruled that the power of DBP's Board
to fix the remuneration and emoluments of its RULING:
officials and employees is not absolute and is
subject to Sections 5 and 6 of Presidential There is no quibbling over the fact that labor
Decree (PD) No. 1597 and Section 3 of unrest impelled the DBP, in the interest of
Memorandum Order (MO) No. 20 of the industrial peace, to grant the GFPA to its
Office of the President requiring prior employees.
presidential approval. It held that the power of
while Sec. 13 of DBP's charter as amended on
DBP's BOD to enter into a compromise
February 14, 1998, exempts it from existing
agreement has no basis in law. Furthermore, the
laws on compensation and position
subsequent payment of the AA was a separate
classification, it concludes by expressly stating
matter that does not render the disallowance of
that DBP's system of compensation shall
the GFPA moot and academic.
nonetheless conform to the principles under
On petition for certiorari, DBP insists: the SSL.

o that under its charter, the BOD was From this, there is no basis to conclude that the
authorized to settle its employees' claims, DBP's BOD was conferred unbridled authority to
which it did, by way of the grant of fix the salaries and allowances of its officers and
GFPA. employees. The authority granted DBP to freely
fix its compensation structure under which it may
o It reiterated its exemption the Salary grant allowances and monetary awards remains
Standardization Law (SSL). circumscribed by the SSL; it may not entirely
depart from the spirit of the guidelines therein.
o DBP also maintains that the GFPA
recipients and DBP Directors who What made the GFPA granted by the DBP to its
approved the disbursement all acted in officers and employees in 2003 unique was that
good faith; consequently, should the it was the product of a compromise arrived at
disallowance be upheld, they may not be after negotiations between DBP employees and
held liable for the return of the disallowed management referred to as a governance forum.
amount. The COA considered the process undertaken as
labor negotiations.’
ISSUE:
It appears that DBP misconstrued its authority to
whether or not the COA acted without or in compromise.
excess of its jurisdiction, or with grave abuse of
discretion amounting to lack or excess of The COA's insistence that industrial peace is not
jurisdiction, when it disallowed the GFPA on the a determining factor under the principles of the
basis that it was in the nature of a compromise

MNI NOTES |9
SSL in fixing the compensation of DBP's DOCTRINE: The right to form a union or
employees, is correct. association or to selforganization comprehends
two (2) broad notions, to wit: (a) the liberty or
Subject to the minimum requirements of wage freedom, i.e., the absence of restraint which
laws and other labor and welfare legislation, the guarantees that the employee may act for himself
terms and conditions of employment in the without being prevented by law, and (b) the
unionized private sector are settled through the power, by virtue of which an employee may, as
process of collective bargaining. In government he pleases, join or refrain from joining an
employment, however, it is the legislature and, association.
where properly given delegated power, the
administrative heads of government which fix the FACTS:
terms and conditions of employment. And this is
effected through statutes or administrative Petitioner KNITJOY had a collective
circulars, rules, and regulations, not through bargaining agreement (CBA) with the
collective bargaining agreements Federation of Filipino Workers (FFW). The
bargaining unit covered only the regular rank-
the grant of GFPA was indeed an ultra vires act and-file employees of KNITJOY paid on a daily
or beyond the authority of DBP's BOD. or piece-rate basis. It did not include regular
rank-and-file office and production employees
There was no grave abuse of discretion on the paid on a monthly basis
part of COA when it disallowed the GFPA on
the basis of a compromise agreement to settle a Prior to the expiration of the CBA the Trade
labor dispute. We thus, sustain the disallowance. Union of the Philippines and Allied Services
(TUPAS) filed a petition for the holding of a
It is settled that Government officials and certification election among KNITJOY's regular
employees who received benefits or rank-and-file employees paid on a daily and
allowances, which were disallowed, may keep piece-rate basis.
the amounts received if there is no finding of bad
faith and the disbursement was made in good Excluded were the regular rank-and-file
faith. On the other hand, officers who employees paid on a monthly basis.
participated in the approval of the disallowed
allowances or benefits are required to refund only In the certification election conducted and CFW
the amounts received when they are found to be (Confederation of Filipino Workers) emerged as
in bad faith or grossly negligent amounting to bad the winner; thereafter, negotiations for a new
faith. CBA between CFW and KNITJOY
commenced.
KNITJOY MANUFACTURING, INC. V.
FERRER-CALLEJA G.R. No. 81883, During the pendency of the said negotiations,
September 23, 1992 Davide, Jr. private respondent KMEU (Knitjoy Monthly
Employees Union), a separate union representing
monthly paid empolyees, filed a petition for

MNI NOTES |10


certification election among KNITJOY's regular WON the inclusion of the monthly paid
rankand-file monthly paid employees employees in the coverage of the new CBA bars
the certification election – NO
The petition was dismissed by the Med-Arbiter.
RULING:
KMEU filed a motion to reconsider this order,
which was treated as an appeal by the Bureau of 1) ANSWER: Yes, because the KMEU
Labor Relations (BLR). represents a separate and distinct bargaining unit.

BLR Director Pura Ferrer-Calleja, reversed the The suggested bias of the Labor Code in
order of MedArbiter de la Cruz, allowing the favor of the one company-one union
certification election. policy, anchored on the greater mutual
benefits which the parties could derive, is
Arguments of Knitjoy the employer: not without exceptions.

the monthly-paid employees have the The one company-one union policy must
same working incentives as their yield to the right of the employees to form
counterparts, the daily-paid workers; unions or associations for purposes not
contrary to law, to self-organization and
that the existing collective bargaining
to enter into collective bargaining
agent (CFW) is willing to include the
negotiations, among others, which the
monthly-paid employees; and
Constitution guarantees.
that out of the 212 monthly-paid
LAW: Article 245 of the Labor Code expressly
employees, 116 qualify as managerial
allows supervisory employees who are not
employees while the rest who are holding
performing managerial functions to join, assist or
confidential or technical positions should
form their separate union but bars them from
likewise be excluded.
membership in a labor organization of the rank-
the CBA also rendered the case moot and and-file employees. This allows more than one
academic since the monthly paid union in a company
employees are now included in the
This provision obviously allows more than one
provisions of the said CBA.
union to exist in a company.
ISSUES:
The right to form a union or association or to self-
WON KMEU can constitute as an appropriate organization comprehends two (2) broad notions,
bargaining unit separate and distinct from the to wit: (a) the liberty or freedom, i.e., the absence
existing unit composed of daily or piece-rate paid of restraint which guarantees that the employee
employees- YES may act for himself without being prevented by
law, and (b) the power, by virtue of which an

MNI NOTES |11


employee may, as he pleases, join or refrain from agreement registered before or during the last 60
joining an association. days of a subsisting agreement or during the
pendency of the representation case. (but
In Victoriano vs. Elizalde Rope Worker’s Union, basically KMEU is not a party to the current CBA
the SC stated: It is, therefore, the employee who so it does not matter) •
should decide for himself whether he should
join or not an association; and should he choose In the bargaining history of KNITJOY, the CBA
to join, he himself makes up his mind as to which has been consistently limited to the regular rank-
association he would join; and even after he has and-file employees paid on a daily or piece-rate
joined, he still retains the liberty and the power to basis. On the other hand, the rankand-file
leave and cancel his membership with said employees paid on a monthly basis were never
organization at any time. included within its scope. •

Inasmuch as what both the Constitution and the KMEU is limited only to rank and file EEs paid
Industrial Peace Act have recognized, and on a monthly basis. It does not seek to dislodge
guaranteed to the employee, is the ‘right’ to join CFW as the exclusive bargaining rep of
associations of his choice, it would be absurd to KNITJOY. •
say that the law also imposes, in the same breath,
upon the employee the duty to join associations. In the elections that happened, KMEU was
The law does not enjoin an employee to sign expressly excluded. Hence, the negotiations
up with any association. between CFW and KNITJOY refer ONLY to
rank and file EEs paid on a daily or piece-rate
CONCLUSION: KMEU may legitimately basis. •
represent the monthly paid employees as a
separate Union since the employee’s right to self- Clearly, insofar as the monthly paid EES are
organization shall be protected and that the labor concerned (KMEU), KNITJOY AND CFW
law provisions allows multiple unions in a recognize that KMEU’s constitution of a separate
company. union as sole bargaining rep CAN NEITHER BE
PREVENTED NOR AVOIDED without
(2) ANSWER: No, the new CBA cannot infringing on these employees’ rights to form a
validly include KMEU and does not bar the union and to enter into collective bargaining
holding of a certification election to determine negotiations. •
their sole bargaining agent, and the negotiation
for and the execution of a subsequent CBA KMEU had the unquestioned and undisputed
right to seek certification as the exclusive
LAW: Section 4, Rule V, Book V of the Rules bargaining representative for the monthly paid
Implementing the Labor Code. Effects of early rank-and-file employees; both KNITJOY and
agreements.— CFW cannot block the same. •

The representation case shall not, however, be It has not been shown that KMEU was privy to
adversely affected by a collective bargaining this agreement. And even if it were so, it can

MNI NOTES |12


never bind subsequent federations and unions however, be adversely affected by a
particularly private respondent-union because it collective bargaining agreement
is a curtailment of the right to self-organization registered before or during the last 60
guaranteed by the labor laws. • days of a subsisting agreement or during
the pendency of the representation case.
The monthly-paid rank-and-file employees
should be allowed to join the union of the daily- The records disclose that the intervenors in the
paid-rank-and-file employees of petitioner so petition for certification are the KMEA-CCLU
that they can also avail of the CBA benefits or to and CFW. They should be included as among the
form their own rank-and-file union • certification election besides KMEU and no
union.
Considering that (a) the TUPAS solicited
certification election was strictly confined to the CONCLUSION: The existing CBA agreement
rank-and-file employees who are paid on a daily does not bar the certification election filed by
or piece-rate basis, (b) the results of the election KMEU and including the monthly paid
must also necessarily confine the certified employees in the coverage of the current CBA
union’s representation to the group it represents would be violative of their right to self-
and (c) the issue of the plight of the monthly-paid organization since they were never part of the
employees was still pending, KNITJOY and negotiation neither they were represented.
CFW clearly acted with palpable bad faith and
malice in including within the scope of the new DISPOSITIVE PORTION / RULING:
CBA these monthly paid employees. • WHEREFORE, the instant petitions are
DISMISSED. However, the challenged decision
Thus, there was a conspiracy to suppress the right of public respondent of 1 December 1987 is
of KMEU to bargain collectively and negotiate modified to include in the choices for the
for themselves, by imposing to them a contract certification election petitioner Confederation of
for the negotiation which THEY WERE NOT Filipino Workers (CFW) and the Knitjoy
GIVEN NOTICE OF and neither was they Monthly Employees Association and
allowed to participate. • Confederation of Citizens Labor Unions
(KMEU-CCLU)
The new CBA cannot validly include KMEU and
does not bar the holding of a certification election SAMAHAN NG MANGGAGAWA SA
to determine their sole bargaining agent, and the HANJIN SHIPYARD V. BUREAU OF
negotiation for and the execution of a subsequent LABOR RELATIONS
CBA between KNITJOY and the eventual
winner in said election. Section 4, Rule V, Book DOCTRINE: The right to form or join a labor
V of the Rules Implementing the Labor Code organization necessarily includes the right to
expressly provides: • refuse or refrain from exercising the said right. It
is self-evident that just as no one should be
SEC. 4. Effects of early agreements.— denied the exercise of a right granted by law, so
The representation case shall not, also, no one should be compelled to exercise such

MNI NOTES |13


a conferred right. Also inherent in the right to On March 15, 2010: Respondent Hanjin Heavy
self-organization is the right to choose whether to Industries and Construction Co (Hanjin) filed
form a union for purposes of collective a petition with DOLE-Pampanga praying for the
bargaining or a workers' association for purposes cancellation of registration of Samahan’s
of providing mutual aid and protection. There is association on the ground that its members did
no provision in the Labor Code that states that not fall under any of the types of workers
employees with definite employers may form, enumerated in the 2nd sentence of Art 249
join or assist unions only. A union refers to any
labor organization in the private sector organized Opined that only ambulant, intermittent,
for collective bargaining and for other legitimate itinerant, rural workers, self- employed, and
purpose, while a workers' association is an those without definite employers may form a
organization of workers formed for the mutual workers' association.
aid and protection of its members or for any
• It further posited that one third (1/3) of
legitimate purpose other than collective
the members of the association had
bargaining. Any labor organization which may or
definite employers and the continued
may not be a union may deal with the employer.
existence and registration of the
This explains why a workers' association or
association would prejudice the
organization does not always have to be a labor
company's goodwill.
union and why employer-employee collective
• Hanjin also claimed that Samahan made a
interactions are not always collective bargaining
misrepresentation in connection with the
FACTS: list of members and/or voters who took
part in the ratification of their constitution
On Feb 16, 2010: Samahan filed an application and by laws in its application for
for registration of itsname “Samahan ng mga registration.
Manggagawa sa Hanjin Shipyard” with the • Samahan made it appear that its members
DOLE. were all qualified to become members of
the worker’s association.
• Attached to the application were the list
of names of the association’s officers and RULING OF THE DOLE REGIONAL
members, signatures of the attendees of DIRECTOR:
the Feb 7, 2010 meetings, copies of their
Constitution and By-Laws. In favor of Hanjin
• The application also stated that the
RULING OF THE BLR:
association had a total of 120 members.
• DOLE then issued the corresponding In favor of the Samahan but ordered it to drop
certificate of registration in favor of “Hanjin Shipyard” from its name.
Samahan.
• It stated that the law clearly afforded the
right to self organization to all workers

MNI NOTES |14


including those without definite argues that the right to form a workers'
employers. association is not exclusive to intermittent,
• As an expression of the right to self- ambulant and itinerant workers.
organization, industrial, commercial and
self-employed workers could form a • While the Labor Code allows the
workers' association if they so desired but workers "to form, join or assist labor
subject to the limitation that it was only organizations of their own choosing" for
for mutual aid and protection. the purpose of collective bargaining, it
• Nowhere could it be found that to form a does not prohibit them from forming a
workers' association was prohibited or labor organization simply for purposes of
that the exercise of a workers' right to mutual aid and protection.
selforganization was limited to collective • All members of Samahan have one
bargaining. common place of work, Hanjin Shipyard.
• Thus, there is no reason why they cannot
RULING OF THE CA: use "Hanjin Shipyard" in their name.
• Reiterates its stand that workers with a
In favor of Hanjin. definite employer can organize any
association for purposes of mutual aid
• The registration of Samahan as a
and protection. Inherent in the workers'
Legitimate Worker’s Association was
right to self-organization is its right to
contrary to the provisions of Art 243 of
name its own organization.
the LC.
• Samahan referred "Hanjin Shipyard" as
• It stressed that only 57 out of the 120
their common place of work. Therefore,
members were actually working in
they may adopt the same in their
Hanjin while the phrase in the preamble
association's name.
of Samahan's Constitution and By- laws,
"KAMI, ang mga Manggagawa sa Hanjin HANJIN: posits that the members of Samahan
Shipyard," created an impression that all have definite employers, hence, they should have
its members were employees of HHIC. formed a union instead of a workers' association.
• Such unqualified manifestation which
was used in its application for ISSUE:
registration, was a clear proof of
misrepresentation which warranted the 1. Whether SAMAHAN’s registration was
cancellation of Samahan's registration. correctly cancelled on the ground that it cannot
form a Workers’ Association of Employees in
SUMMARY OF THE ARGUMENTS OF HANJIN, and should have instead of formed a
THE PARTIES SAMAHAN: union. NO

2. Whether the decision ordering the removal of


the word “HANJIN” in the name of SAMAHAN

MNI NOTES |15


was proper, by reason of the company’s property • A labor organization has two broad
right over the company name “Hanjin.” YES rights: (1) to bargain collectively (2) to
deal with the employer concerning terms
RULING: and conditions of employment.
• To bargain collectively is a right given to
NO. Samahan’s registration should not have
a union once it registers itself with the
been cancelled. As provided for by both the
DOLE. Dealing with the employer, on the
Constitution and the Labor Code, the right to
other hand, is a generic description of
self-organization includes the right to form, join
interaction between employer and
or assist labor organizations for the purpose of
employees concerning grievances,
collective bargaining through representatives of
wages, work hours and other terms and
their own choosing and to engage in lawful
conditions of employment, even if the
concerted activities for the same purpose for their
employees' group is not registered with
mutual aid and protection.
the DOLE.
• This is in line with the policy of the State • Collective bargaining is just one of the
to foster the free and voluntary forms of employee participation. Despite
organization of a strong and united so much interest in and the promotion of
labor movement as well as to make sure collective bargaining, it is incorrect to say
that workers participate in policy and that it is the device and no other, which
decision-making processes affecting their secures industrial democracy.
rights, duties and welfare. • It is equally misleading to say that
collective bargaining is the end-goal of
The right to form a union or association or to employee representation. Rather, the real
self-organization comprehends two notions, to aim is employee participation in whatever
wit: form it may appear, bargaining or no
bargaining, union or no union. Any labor
• the liberty or freedom, that is, the absence organization which may or may not be a
of restraint which guarantees that the union may deal with the employer.
employee may act for himself without
• This explains why a workers' association
being prevented by law
or organization does not always have to
• the power, by virtue of which an be a labor union and why employer-
employee may, as he pleases, join or employee collective interactions are not
refrain from joining an association. always collective bargaining.
A labor organization is defined as "any union or A cursory reading of the law demonstrates that
association of employees which exists in whole a common element between unionism and the
or in part for the purpose of collective bargaining formation of Labor Management Councils is the
or of dealing with employers concerning terms existence of an employer-employee relationship.
and conditions of employment."

MNI NOTES |16


• Where neither party is an employer nor an The policy underlying the prohibition in Sec 18
employee of the other, no duty to bargain against the registration of a corporate name
collectively would exist. In the same which is "identical or deceptively or confusingly
manner, expressed in Article 255 (now similar" to that of any existing corporation or
261) is the requirement that such workers which is "patently deceptive" or "patently
be employed in the establishment before confusing" or "contrary to existing laws," is the
they can participate in policy and decision avoidance of fraud upon the public which
– making processes. would have occasion to deal with the entity
concerned, the evasion of legal obligations and
In contrast, the existence of employer-employee duties, and the reduction of difficulties of
relationship is not mandatory in the formation administration and supervision over
of workers' association. corporations.

What the law simply requires is that the For the same reason, it would be misleading for
members of the workers' association, at the very the members of Samahan to use "Hanjin
least, share the same interest. Shipyard" in its name as it could give the wrong
impression that all of its members are employed
The very definition of a workers' association
by Hanjin.
speaks of "mutual aid and protection."
Further, Section 9, Rule IV of D.O. No. 40-03,
2. YES. As there is no provision under our labor
Series of 2003 explicitly states:
laws which speak of the use of name by a
workers' association, the Court refers to the The change of name of a labor
Corporation Code, which governs the names organization shall not affect its legal
of juridical persons. Sec 18 of the Corporation personality. All the rights and obligations
Code provides that of a labor organization under its old name
shall continue to be exercised by the labor
“No corporate name may be allowed by
organization under its new name.
the Securities and Exchange Commission
if the proposed name is identical or Thus, in the directive of the BLR removing the
deceptively or confusingly similar to that words "Hanjin Shipyard," no abridgement of
of any existing corporation or to any other Samahan's right to self-organization was
name already protected by law or is committed. WHEREFORE, the petition is
patently deceptive, confusing or contrary PARTIALLY GRANTED
to existing laws. When a change in the
corporate name is approved, the ASIAN INSTITUTE OF MANAGEMENT
Commission shall issue an amended (AIM) vs. AIM FACULTY ASSOCIATION
certificate of incorporation under the
amended name.” DOCTRINE: Managerial employees are not
allowed to join labor organizations.

MNI NOTES |17


FACTS: The CA reversed the decision of the SOLE,
agreeing the AFA is composed of managerial
Petitioner AIM is a registered non-stock, non- employees. AFA brought this case before the SC
profit educational institution. via Pet. for Review on Certiorari, docketed as
GR. 197089. This case remains pending before
Respondent AIM Faculty Association (AFA)
the SC up to this date.
is a duly registered labor organization composed
of members of the AIM faculty. AIM also filed a Petition for Certiorari before the
CA, questioning the decision of the BLR that had
AFA filed a Petition for Certification Election,
denied its Petition for Cancellation.
seeking to represent a bargaining unit in AIM
consisting of 40 faculty members. The CA denied the petition. AIM’s MR was
likewise denied. Hence, the instant petition
AIM opposed the petition, claiming that AFA’s
before the SC.
members are managerial employees.
ISSUE:
AIM thereafter filed a Petition for Cancellation
of AFA’s certificate of registration on the ground WON the CA erred in not cancelling the
of misrepresentation in registration and that AFA certificate of registration of AFA -NO
is composed of managerial employees.
RULING:
AFA’s Petition for Certification Election was
denied by the Med-Arbiter, holding that AFA ANSWER:
was composed of managerial employees.
No, because the issue of whether AFA’s
AFA appealed the denial to the Sec. of Labor members are managerial employees is still
(SOLE), who in turn reversed the Med-Arbiter pending resolution in GR. 197089. The issue was
and ordered the conduct of a certification first raised by AIM in said case. The resolution
election. • Meanwhile however, AIM’s Petition of the same cannot be pre-empted.
for Cancellation of AFA’s certificate of
registration was granted. LAW:

AFA’s appealed the same to the BLR, which The Labor code, specifically Art. 255, provides
reversed the cancellation. The BLR ruled that the that “managerial employees are not eligible to
grounds relied upon by AIM are not among the join, assist, or form any labor organization.”
grounds authorized in the LC and that AFA is not
CONCLUSION: As it has not yet been
composed of managerial employees.
determined that AFA’s members are managerial,
AIM appealed the decision of the SOLE which the instant petition cannot be resolved. Until said
had granted the Petition for Certification issue is resolved with finality, AIM’s Petition for
Election. Cancellation of AFA’s certificate of registration
cannot be resolved. This is because said petition

MNI NOTES |18


relies of the allegation that AFA’s members are IPTEU filed a verified petition for certification
managerial employees. Thus, as a matter of election seeking to represent a bargaining unit
courtesy and in order to avoid conflicting consisting of approximately 22 rank-and-file
decisions, GR. 197089 must first be determined professional and technical employees of CCBPI
whether AFA is composed of managerial Ilocos Norte Plant.
employees.
CCBPI prayed, among others, for the denial and
COCA-COLA BOTTLERS PHILIPPIES, dismissal of the petition, arguing that members of
INC v. ILOCOS PROFESSIONAL AND the bargaining unit sought to be represented by
TECHNICAL EMPLOYEES UNION IPTEU are either supervisory employees or
confidential employees, hence, ineligible for
DOCTRINE: To be classified as a confidential inclusion as members of IPTEU.
employee, access to vital labor information is an
imperative consideration. Employees who A preliminary hearing of the petition was
encounter or handle trade secrets and financial scheduled and held, wherein the possibility of
information are not automatically classified as voluntary recognition or consent election was not
confidential employees. A confidential employee acceded to by CCBPI.
must assist or act in a confidential capacity and
obtain confidential information relating to labor Mediator-Arbiter Florence Marie Gacad-
relations policies. Exposure to internal business Ulep: granted IPTEU’s petition being convinced
operations of the company is not per se a ground that the union members are rank-and-file
for the exclusion in the bargaining unit employees and not occupying positions that are
supervisory or confidential in nature. The
PARTIES: conduct of the Preelection Conference was
ordered.
Coca-Cola Bottlers Philippines, Inc. or CCBPI
(petitioner) is a domestic corporation duly • During the Preelection Conference,
organized and operating under the Philippines CCBPI and IPTEU mutually agrees to
laws. It is primarily engaged in the beverage conduct the certification election on 21
business, which includes the manufacture of September 2007.
carbonated soft drinks. • On election day, only 16 of the 22
employees in the IPTEU list votes.
2. Ilocos Professional and Technical Employees However, no votes were canvassed.
Union or IPTEU (respondent) is a registered
independent labor organization with address at CCBPI filed and registered a Protest questioning
CCBPI Ilocos Plant in Barangay Catuguing, San the conduct and mechanics of the election and a
Nicalos, Ilocos Norte. Challenge to Votes on the ground that the voters
are supervisory and confidential employees. • By
FACTS: agreement, the parties met for the opening and
counting of the challenged votes.

MNI NOTES |19


Med-Arbiter: denied CCBPI’s challenged to the regard (2) to persons who formulate, determine,
16 votes. She found that the voters are rank-and- and effectuate management policies in the field
file employees holding positions that are not of labor relations.
confidential in nature, and who are not, or used
to be, members of Ilocos Monthlies Union (IMU) The two criteria are cumulative, and both
due to the reclassification of their positions by must be met if an employee is to be
CCBPI and have been excluded from the CBA considered a confidential employee —
entered into by IMU and CCBPI from 1997 to thexist between the employee and his
2005. supervisor, and the supervisor must
handle the prescribed responsibilities
• Consequently, the challenged votes were relating to labor relations.
opened and canvassed. After garnering 14
out of the 16 votes cast, IPTEU was The exclusion from bargaining units of
proclaimed as the SEBA of the rank-and- employees who, in the normal course of
file exempt workers in CCBPI Ilocos their duties, become aware of
Norte Plant. management policies relating to labor
• CCBPI elevated the case to the SOLE. relations is a principal objective sought to
SOLE: denied the appeal, holding that, as be accomplished by the “confidential
shown by the certification of the IMU employee rule.” at is, the confidential
President and the CBAs forged between relationship must
CCBPI and IMU from 1997 to 2007, the
Corollarily, although Article 245 of the Labor
22 employees sought to be represented by
Code limits the ineligibility to join, form and
IPTEU are not part of IMU and are
assist any labor organization to managerial
excluded from its CBA coverage.
employees, jurisprudence has extended this
CA: denied the petition. prohibition to confidential employees or those
who by reason of their positions or nature of work
ISSUE: are required to assist or act in a fiduciary manner
to managerial employees and, hence, are likewise
WON the 22 employees of the bargaining unit privy to sensitive and highly confidential records.
sought to be represented by IPTEU are
confidential employees, hence, ineligible for Confidential employees are thus excluded from
inclusion as members of IPTEU – NO the rank-andfile bargaining unit.

RULING: The rationale for their separate category


and disqualification to join any labor
NO. CCBPI failed to prove that the 22 organization is similar to the inhibition
employees are confidential employees. for managerial employees, because if
allowed to be affiliated with a union, the
Confidential employees are defined as those who latter might not be assured of their loyalty
(1) assist or act in a confidential capacity, in in view of evident conflict of interests and

MNI NOTES |20


the union can also become company In the present case, the subject employees
denominated with the presence of encounter and handle financial as well as
managerial employees in the union physical production data and other information
membership. which are considered vital and important from
business operations’ standpoint.
Having access to confidential
information, confidential employees may Such information is not the kind of
also become the source of undue information that is relevant to collective
advantage. Said employees may act as a bargaining negotiations and settlement of
spy or spies of either party to a collective grievances as would classify them as
bargaining agreement. (San Miguel confidential employees.
Foods, Inc. v. San Miguel Corp.
Supervisors and Exempt Union, 670 Phil. As to whether the 16 votes sought to be excluded
421, 432-434; 655 SCRA 1, 14 [2011]. from the appropriate bargaining unit are
confidential employees, such query is a question
To be considered as a confidential employee, an of fact, which is not a proper issue in a petition
employee must assist or act in a confidential for review under Rule 43 of the Rules. This holds
capacity and obtain confidential information more true in the present case in view of the
relating to labor relations policies. Exposure to consistent findings of the Med-Arbiter, the SOLE
internal business operations of the company is and the CA.
not per se a ground for the exclusion in the
bargaining unit. Employees who encounter or In this case, organization charts, detailed
handle trade secrets and financial information are job descriptions, and training programs
not automatically classified as confidential were presented by CCBPI before the
employees. Med-Arbiter, the SOLE, and the CA.

The determination of factual issues is vested in Despite these, the Med-Arbiter ruled that
the Mediator-Arbiter and the Department of the subject employees should not be
Labor and Employment. Pursuant to the doctrine automatically classified as confidential
of primary jurisdiction, the Court should refrain employees although they encounter or
from resolving such controversies unless the case handle trade secrets and financial
falls under recognized and well-established information. The SOLE, which the CA
exceptions. The doctrine of primary jurisdiction affirmed, likewise held that the
does not warrant a court to arrogate unto itself the questioned voters do not have access to
authority to resolve a controversy the jurisdiction confidential labor relations information.
over which is initially lodged with an
Lastly, As proven by the certification of the IMU
administrative body of special competence.
President as well as the CBAs executed between
APPLICATION: IMU and CCBPI, the 22 employees sought to be
represented by IPTEU are not IMU members and
are not included in the CBAs due to

MNI NOTES |21


reclassification of their positions. If these * never-ending quest of the petitioner employer
documents were false, the IMU should have to stop the conduct of the certification election on
manifested its vigorous opposition. the ground of the pendency of proceedings to
cancel the labor organization's registration - on
CONCLUSION: the ground that the membership of the labor
organization was a mixture of managerial and
Therefore, the subject employees are not
supervisory employees with the rank-and-file
excluded to be members of IPTEU because they
do not handle the kind of information that is employees. (the pendency of the cancellation of
relevant to collective bargaining negotiations union registration brought against the labor
settlement of grievances as would classify them organization applying for the certification
as confidential employees.
election should not prevent the conduct of the
DISPOSITIVE PORTION / RULING: certification election)
WHEREFORE the petition is DENIED.
December 13, 2005 – CA dismissed its petition
The Decision of CA, which affirmed Resolution for certiorari to assail the resolutions of
of the Secretary of Labor and Employment, respondent Secretary of Labor and Employment
dismissing petitioner’s appeal that assailed the sanctioning the conduct of the certification
Decision (On the Challenged Voters) and election initiated by respondent labor
Proclamation of the Winner, of the Mediator- organization.
Arbiter, are hereby AFFIRMED.
October 11, 1995, respondent National Union of
Workers in Hotel Restaurant and Allied
Industries-Heritage Hotel Manila Supervisors
G.R. No. 172132, July 23, 2014
Chapter (NUWHRAIN-HHMSC) filed a petition
THE HERITAGE HOTEL MANILA, for certification election – their OPPOSITiON –
ACTING THROUGH ITS OWNER, GRAND but it was denied by Med-Arbiter Fernando on
PLAZA HOTEL CORPORATION, Feb. 14, 1996 and issued ORDER FOR THE
Petitioner, v. SECRETARY OF LABOR AND CONDUCT OF THE CERTIFICATION
EMPLOYMENT; MED-ARBITER TOMAS ELECTION; Petitioner appealed the order but
F. FALCONITIN; AND NATIONAL UNION was denied again
OF WORKERS IN THE HOTEL,
Pre-Election Conference – Feb. 20, 1998,
RESTAURANT AND ALLIED
suspended until further notice due to non-
INDUSTRIES–HERITAGE HOTEL
repeated non-appearance of UWHRAIN-
MANILA SUPERVISORS CHAPTER
HHMSC.
(NUWHRAIN-HHMSC), Respondents
January 29, 2000, NUWHRAIN-HHMSC
FACTS:
moved for the conduct of the pre-election
conference.

MNI NOTES |22


April 17, 2000 - petitioner filed a motion to Petitioner timely appealed to the DOLE
dismiss Secretary

raising the prolonged lack of interest of (a) the membership of NUWHRAIN-HHMSC


NUWHRAIN-HHMSC to pursue its petition for
certification election. consisted of managerial, confidential, and rank-
and-file employees;
My 12, 2000 - petitioner filed a petition for the
cancellation of NUWHRAIN-HHMSC’s (b) NUWHRAIN-HHMSC failed to comply with
registration as a labor union for failing to submit
the reportorial requirements; and
its annual financial reports and an updated list of
members as required by Article 238 and Article (c) Med-Arbiter Falconitin simply brushed aside
239 of the Labor Code. serious questions on the illegitimacy of
NUWHRAINHHMSC.
Filed another motion on June 1, 2000 to seek
either the dismissal or the suspension of the Ruling of the DOLE Secretary
proceedings on the basis of its pending petition
for the cancellation of union registration. Augst 21, 2002, DOLE Secretary Patricia A. Sto.
Tomas issued a resolution denying the appeal,
June 23, 2000 - (DOLE) issued a notice and affirming the order of MedArbiter
scheduling the certification elections. Falconitin.
Dissatisfied, the petitioner commenced in the CA The petitioner moved for reconsideration.
on June 14, 2000 a special civil action for
certiorari - DOLE gravely abused its discretion in October 21, 2002 – Motion Denied:
not suspending the certification election
proceedings – CA dismissed for non- DOLE Secretary declared that:
exhaustion of administrative remedies
• the mixture or co-mingling of employees
Ruling of the Med-Arbiter in a union was not a ground for
dismissing a petition for the certification
January 26, 2001 - Falconitin issued an order, election and;
ruling that the petition for the cancellation • appropriate remedy was to exclude the
ineligible employees from the bargaining
of union registration was not a bar to the unit during the inclusion-exclusion
holding of the certification election. proceedings;
* premises considered, respondent • the dismissal of the petition for the
employer/protestant’s protest with motion to certification
defer certification of results and winner is hereby • election based on the legitimacy of the
dismissed for lack of merit. petitioning union would be inappropriate
because it would effectively allow a

MNI NOTES |23


collateral attack against the union’s legal provide what the effect is if a rank-and-fileunion
personality; and takes in supervisory employees as members, or
• that a collateral attack against the vice versa.
personality of the labor organization was
prohibited The fact that the cancellation proceeding has not
yet been resolved makes it obvious that the legal
Petitioner elevated the matter to the CA by personality of the respondent union is still very
petition for certiorari. much in force – DOLE CAN STILL PROCEED
WITH THE CERTIFICATION ELECTION
Ruling of the CA AND NO GRAVE ABUSE OF DISCRETION
HAS BEEN COMMITTED - the right to be
Dec. 13, 2005 – CA dismissed.
certified as
The petition for certiorari filed by the petitioner
collective bargaining agent is one of the
is, a continuation of the debate on the relevance
legitimate privileges of a registered union.
of the Toyota Motor, Dunlop Slazenger and
Progressive Development cases to the issues ISSUES:
raised - anchored on the provisions of Article 245
of the Labor Code which prohibit managerial • should the petition for the cancellation of
employees from joining any labor union and union registration based on mixed
permit supervisory employees to form a separate membership of supervisors and managers
union of their own. in a labor union, and the non-submission
of reportorial requirements to the DOLE
Tagaytay Highlands proclaims Department justify the suspension of the proceedings
Order 9, that after a certificate of registration is for the certification elections or even the
issued to a union, its legal personality cannot be denial of the petition for the certification
subject to a collateral attack election? - NO

** courts have held that a union cannot become a RULING:


legitimate labor union if it shelters under its wing
both types of employees. But there are elements Petition for Certiorari is denied
of an elliptical reasoning in the holding of these
two cases that a petition for certification election the petitioner lacked the legal personality to
may not prosper until the composition of the assail the proceedings for the certification
union is settled therein. election, and should stand aside and not oppose
the petition, or even appeal the Med-Arbiter’s
** Article 245 prohibits supervisory employees orders relative to the conduct of the certification
from joining a rank-and-file union, it does not election because the sole concern deemed an
intruder as far as the certification election is
concerned. (Except when it is requested to
bargain collectively)

MNI NOTES |24


In that respect, only a final order for the (a) Misrepresentation, false statement or fraud in
cancellation of the registration would have connection with the adoption or ratification of
prevented NUWHRAINHHMSC from the
continuing to enjoy all the rights conferred on it
as a legitimate labor union, including the right to constitution and by-laws or amendments thereto,
the petition for the certification election. This the minutes of ratification, and the list of
rule is now enshrined in Article 238-A of the members who took part in the ratification;
Labor Code, as amended by Republic Act No.
(b) Misrepresentation, false statements or fraud
9481.
in connection with the election of officers,
Article 238-A. Effect of a Petition for minutes of the election of officers, and the list of
Cancellation of Registration. – A petition for voters;
cancellation of union registration
(c) Voluntary dissolution by the members.
shall not suspend the proceedings for
R.A. No. 9481 also inserted in the Labor Code
certification election nor shall it prevent the
filing of a petition for certification election. Article 242-A, which provides:
the Court declared that the dismissal of the ART. 242-A. Reportorial Requirements.--The
petition for the cancellation of the registration of following are documents required to be
NUWHRAIN-HHMSC was proper when viewed submitted to the Bureau by the legitimate labor
against the primordial right of the workers to self organization concerned:
organization, collective bargaining negotiations
and peaceful concerted actions. (a) Its constitution and by-laws, or amendments
thereto, the minutes of ratification, and the list of
Labor authorities should act with circumspection members who took part in the ratification of the
in treating petitions for cancellation of union constitution and by-laws within thirty (30) days
registration or be accused of interfering with from adoption or ratification of the constitution
union activities. and by-laws or amendments thereto;
In resolving the petition, consideration must be (b) Its list of officers, minutesof the election of
taken of the fundamental rights guaranteed by officers, and list of voters within thirty (30) days
Article XIII, Section 3 of the Constitution, i.e., from election;
the rights of all workers to self-organization,
(c) Its annual financial report within thirty (30)
collective bargaining and negotiations, and days after the close of every fiscal year; and
peaceful concerted activities.
(d) Its list of members at least once a year or
ART. 239. Grounds for Cancellation of Union whenever required by the Bureau.
Registration.--The following may constitute
grounds for cancellation of union registration:

MNI NOTES |25


Failure to comply with the above requirements (STEAM-PCWF), a labor organization of staff
shall not be a ground for cancellation of union and technical employees of MERALCO, filed a
registration but shall subject the erring officers petition for certification election, seeking to
or members to suspension, expulsion from represent regular employees of MERALCO who
membership, or any appropriate penalty. are: (1) non-managerial employees with Pay
Grades VII and above; (2) non-managerial
Presently, then, the mixed membership does not employees in the Patrol Division, Treasury
result in the illegitimacy of the registered labor Security Services Section, Secretaries who are
union unless the same was done through automatically removed from the bargaining unit;
misrepresentation, false statement or fraud and (3) employees within the rank and file unit
according to Article 239 of the Labor Code. who are automatically disqualifies from
becoming union members of any organization
"What is important is that there is an
within the same bargaining unit.
unmistakable intent of the members of [the]
union to exercise their right to organize. We Allegation of the petition: While there exists a
cannot impose rigorous restraints on such right if duly-organized union for rank and file
we are to give meaning to the protection to labor employees in Pay Grade I-VI, which is the
and social justice clauses of the Constitution." MERALCO Employees and Worker’s
Association (MEWA) which holds a valid CBA
the Court DENIES the petition for review on
for the rank and file employees, there is no other
certiorari; AFFIRMS the decision promulgated
labor organization except STEAM-PCWF
on
claiming to represent the MERALCO
December 13, 2005 by the Court of Appeals; and employees.
ORDERS the petitioner to pay the costs of suit
The petition was premised on the
exclusion/disqualification of certain MERALCO
employees pursuant to Article 1, Section 2 and 3
G.R. No. 91902 May 20, 1991 of the existing MEWA CBA.

MANILA ELECTRIC COMPANY, petitioner MERALCO moved for the dismissal of the
vs. THE HON. SECRETARY OF LABOR petition on the grounds that the employees
AND EMPLOYMENT, STAFF AND sought to be represented by petitioner are either
TECHNICAL EMPLOYEES managerial who are prohibited by law from
ASSOCIATION OF MERALCO, and FIRST forming or joining supervisory union, security
LINE ASSOCIATION OF MERALCO services personnel who are prohibited from
SUPERVISORY EMPLOYEES, respondents. joining or assisting the rank-and-file union,
secretaries who do not consent to the petitioner’s
FACTS: representation and whom petitioner cannot
represent, and rank-and-file employees
On November 22, 1988, the Staff and Technical
represented by the certified or duly recognized
Employees Association of MERALCO

MNI NOTES |26


bargaining representative of the only rank-and- certification election.
file bargaining unit in the company.
Secretary of Labor affirmed the assailed order
MERALCO’s Contention: The employees of the Med-Arbiter.
from Pay Grades VII and above are classified as
managerial employees who, under the law, are ISSUES:
prohibited from forming, joining or assisting a
1. Whether or not another rank-and-file
labor organization of the rank-and-file.
bargaining unit should be established
As regards those in the Patrol Division and independent, distinct and separate from the
Treasury Security Service Section, MERALCO existing rank-and-file bargaining unit. - NO
maintains that since these employees are tasked
2. Whether or not employees from pay grades
with providing security to the company, they are
VII and above are rank-and-file employees. –
not eligible to join the rank and file bargaining
NO
unit, pursuant to Sec. 2(c), Rule V, Book V of
the then Implementing Rules and Regulation of 3. Whether or not security personnel are
the Labor Code (1988). prohibited from membership in a labor
organization of the rank and file. - NO
As regards those rank and file employees
enumerated in Sec. 3, Art. I, MERALCO RULING:
contends that they are already beneficiaries of
the MEWA-CBA, they may not be treated as a 1. No. In this petition, MERALCO has admitted
separate and distinct appropriate bargaining unit. that the employees belonging to Pay Grades VII
MERALCO raised the same argument with and up are supervisory. The records also show
respect to employees sought to be represented by that STEAM-PCWF had "renounced its
STEAM- PCWF, claiming that these were representation of the employees in Patrol
already covered by the MEWA- CBA. Division, Treasury Security Service Section and
rank and file employees in Pay Grades I-VI";
Med-Arbiter’s Ruling: Having been excluded while FLAMES (First Line Association of
from the existing Collective Bargaining Meralco Supervisory Employees), on the other
Agreement for rand and file employees, these hand, had limited its representation to employees
employees have the right to form a union of their belonging to Pay Grades VII-XIV, generally
own, except those employees performing accepted as supervisory employees, as follows:
managerial functions.
Supervisory Employees with Pay Grades VII to
With respect to those employees who has XIV are not managerial employees. In fact the
resented their alleged involuntary membership in petition itself of petitioner Manila Electric
the existing CBA, the Med-Arbiter stated that the Company on page 9, paragraph 3 of the petition
holding of a certification election would allow stated as follows, to wit:
them to fully translate their sentiment on the
matter, and thus directed the holding of a There was no need for petitioner to prove that

MNI NOTES |27


these employees are not rank-and-file. As law itself, Art. 212-m, (as amended by Sec. 4 of
adverted to above, the private respondents RA 6715) has already laid down the
admit that these are not the rank-and-file but the corresponding guidelines:
supervisory employees, whom they seek to
represent. What needs to be established is the Art. 212. Definitions. . . .
rank where supervisory ends and managerial
(m) "Managerial employee" is one who is
begins.
vested with powers or prerogatives to lay
and First Line Association of Meralco down and execute management policies and/or
Supervisory Employees herein states that Pay to hire, transfer, suspend, lay-off, recall,
Grades VII to XIV are not managerial discharge, assign or discipline employees.
employees. In fact, although employees with Supervisory employees are those who, in the
Pay Grade XV carry the Rank of interest of the employer, effectively
Department Managers, these employees only recommend such managerial actions if the
enjoys the Rank Manager but their exercise of such authority is not merely
recommendatory powers are subject to routinary or clerical in nature but requires the
evaluation, review and final action by the use of independent judgment. All employees
department heads and other higher executives of not falling within any of the above definitions
the company. are considered rank-and-file employees for
purposes of to Book.
Based on the foregoing, it is clear that the
employees from Pay Grades VII and up have In his resolution, the Secretary of Labor further
been recognized and accepted as supervisory. elaborated:

Anent the instant petition therefore, STEAM- . . . Thus, the determinative factor in
PCWF, and FLAMES would therefore classifying an employee as managerial,
represent supervisory employees only. In this supervisory or rank-and-file is the nature of the
regard, the authority given by the Secretary work of the employee concerned.
of Labor for the establishment of two labor
In National Waterworks and Sewerage
organizations for the rank and file will have
Authority vs. National Waterworks and
to be disregarded since We hereby uphold
Sewerage Authority Consolidated Unions, the
certification elections only for supervisory
Supreme Court had the occasion to come out
employees from Pay Grade VII and up, with
with an enlightening dissertation of the nature
STEAM-PCWF and FLAMES as choices.
of the work of a managerial employees as
2. No. As to the alleged failure of the Secretary follows:
of Labor to establish a demarcation line for
. . . that the employee's primary duty consists of
purposes of segregating the supervisory from
the management of the establishment or of a
the managerial employees, the required
customarily recognized department or
parameter is really not necessary since the
subdivision thereof, that he customarily and

MNI NOTES |28


regularly directs the work of other employees With the elimination, security personnel were
therein, that he has the authority to hire or thus free to join a rank and file organization.
discharge other employees or that his
suggestions and recommendations as to the On March 2, 1989, the present Congress passed
hiring and discharging and or to the RA 6715. Section 18 thereof amended Art. 245,
advancement and promotion or any other to read as follows:
change of status of other employees are given
Art. 245. Ineligibility of managerial
particular weight, that he customarily and
employees to join any labor
regularly exercises discretionary powers . . .
organization; right of supervisory
3. NO. With regards to the rights of the security employees.— Managerial employees
guards to self- organize, MERALCO has are not eligible to join, assist or form
questioned the legality of allowing them to join any labor organization. Supervisory
either the rank and file or the supervisory union, employees shall not be eligible for
claiming that this is a violation of par. 2, Sec. 1, membership in a labor organization of
Rule II, Book V of the Implementing Rules of the rank-and-file employees but may
RA 6715, which states as follows: join, assist, or form separate labor
organizations of their own. (emphasis
Sec 1. Who may join unions. . . . ours)

Supervisory employees and security As will be noted, the second sentence of Art.
guards shall not be eligible for 245 embodies an amendment disqualifying
membership in a labor organization of supervisory employees from membership in a
the rank- and-file employees but may labor organization of the rank-and-file
join, assist or form separate labor employees. It does not include security guards
organizations of their own; . . . in the disqualification.

On December 24, 1986, Pres. Corazon C. While therefore under the old rules, security
Aquino issued E.O. No. guards were barred from joining a labor
organization of the rank and file, under RA
111 which eliminated the above-cited provision 6715, they may now freely join a labor
on the disqualification of security guards. What organization of the rank and file or that of
was retained was the disqualification of the supervisory union, depending on their
managerial employees, renumbered as Art. 245 rank. By accommodating supervisory
(previously Art. 246), as follows: employees, the Secretary of Labor must
likewise apply the provisions of RA 6715 to
Art. 245. Ineligibility of managerial
security guards by favorably allowing them free
employees to joint any labor
access to a labor organization, whether rank and
organization.—Managerial employees
file or supervisory, in recognition of their
are not eligible to join, assist or form
constitutional right to self-organization.
any labor organization.

MNI NOTES |29


The Court is aware however of possible employees for a term of three years retroactive to
consequences in the implementation of the law April 1, 1987 and extending up to March 31,
in allowing security personnel to join labor 1990.
unions within the company they serve. The law
is apt to produce divided loyalties in the faithful -On December 28, 1989, CURE wrote CENECO
performance of their duties. Economic reasons proposing that negotiations be conducted for a
would present the employees concerned with new collective bargaining agreement.
the temptation to subordinate their duties to the
-On January 18, 1990, CENECO denied CURE's
allegiance they owe the union of which they are
request on the ground that employees who at the
members, aware as they are that it is usually
same time are members of an electric cooperative
union action that obtains for them increased
are not entitled to form or join a union.
pecuniary benefits.
- Prior to the submission of the proposal for CBA
Thus, in the event of a strike declared by their
renegotiation, CURE members, in a general
union, security personnel may neglect or
assembly held on December 9, 1989, approved
outrightly abandon their duties, such as
Resolution No. 35 whereby it was agreed that "all
protection of property of their employer and the
union members shall withdraw, retract, or recall
persons of its officials and employees, the
the union members' membership from Central
control of access to the employer’s premises,
Negros Electric Cooperative, Inc. in order to
and the maintenance of order in the event of
avail of the full benefits under the existing
emergencies and untoward incidents.
Collective Bargaining Agreement entered into by
It is hoped that the corresponding amendatory and between CENECO and CURE, and the
and/or suppletory laws be passed by Congress supposed benefits that our union may avail (of)
to avoid possible conflict of interest in security under the renewed CBA ratified by 259 of the
personnel. 362 union members.

G.R. No. 94045 September 13, 1991 - The withdrawal from membership was denied
by CENECO on February 27, 1990 under
CENTRAL NEGROS ELECTRIC Resolution No. 90 "for the basis of withdrawal is
COOPERATIVE, INC. (CENECO), not among the grounds covered by Board
petitioner, vs. HONORABLE SECRETARY, Resolution No. 5023, dated November 22, 1989
DEPARTMENT OF LABOR AND and is contrary to Board Resolution No. 5033
EMPLOYMENT, and CENECO UNION OF dated December 13, 1989."
RATIONAL EMPLOYEES (CURE),
respondents. - CURE filed a petition for direct recognition or
for certification election, supported by 282 or
FACTS: 72% of the 388 rank-and-file employees in the
- On August 15, 1987, CENECO entered into a bargaining unit of CENECO.
collective bargaining agreement with CURE, a
labor union representing its rank-and-file

MNI NOTES |30


- CENECO filed a motion to dismiss to the effect 1. Under Article 256 of the Labor Code, to have
that "employees who at the same time are a valid certification election at least a majority of
members of an electric cooperative are not all eligible voters in the unit must have cast their
entitled to form or join unions for purposes of votes. It is apparent that incidental to the power
collective bargaining agreement, for certainly an of the med-arbiter to hear and decide
owner cannot bargain with himself or his co- representation cases is the power to determine
owners." who the eligible voters are. In so doing, it is
axiomatic that the med-arbiter should determine
- Med-Arbiter Felizardo T. Serapio issued an the legality of the employees' membership in the
order granting the petition for certification union. In the case at bar, it obviously becomes
election in effect, was a denial of CENECO's necessary to consider first the propriety of the
motion to dismiss, and directing the holding of a employees' membership withdrawal from the
certification election between CURE and No cooperative before a certification election can be
Union. had.

- CENECO appealed to the Department of Labor 2. The argument of CENECO that the withdrawal
and Employment which issued the questioned was merely to subvert the ruling of this Court in
order modifying the order of the med-arbiter by the BATANGAS case is without merit. The case
directly certifying CURE as the exclusive referred to merely declared that employees who
bargaining representative of the rank-and-file are at the same time members of the cooperative
employees of CURE. cannot join labor unions for purposes of
collective bargaining. However, nowhere in said
ISSUE:
case is it stated that member-employees are
1. W/N med-arbiter has no jurisdiction to rule on
prohibited from withdrawing their membership
the issue of withdrawal from membership of its
in the cooperative in order to join a labor union.
employees in the cooperative which is properly
vested in the NEA which has control and As discussed by the Solicitor General, Article I,
supervision over all electric cooperatives. Section 9 of the Articles of Incorporation and By-
Laws of CENECO provides that "any member
2. Whether or not the employees of CENECO
may withdraw from membership upon
who withdrew their membership from the
compliance with such uniform terms and
cooperative are entitled to form or join CURE for
conditions as the Board may prescribe." The
purposes of the negotiations for a collective
same section provides that upon withdrawal, the
bargaining agreement proposed by the latter.
member is merely required to surrender his
3. W/N the direct certification ordered by membership certificate and he is to be refunded
respondent Secretary is proper? his membership fee less any obligation that he
has with the cooperative. There appears to be no
RULING: other condition or requirement imposed upon a
withdrawing member. Hence, there is no just
cause for petitioner's denial of the withdrawal

MNI NOTES |31


from membership of its employees who are also join a union of his choice. Besides, the 390
members of the union. employees of CENECO, some of whom have
never been members of the cooperative,
The alleged board resolutions relied upon by represent a very small percentage of the
petitioner in denying the withdrawal of the cooperative's total membership of 44,000. It is
members concerned were never presented nor inconceivable how the withdrawal of a negligible
their contents disclosed either before the med- number of members could adversely affect the
arbiter or the Secretary of Labor if only to prove business concerns and operations of CENECO.
the ratiocination for said denial. Furthermore,
CENECO never averred noncompliance with the 3. However, the direct certification ordered by
terms and conditions for withdrawal, if any. It respondent Secretary is not proper. By virtue of
appears that the Articles of Incorporation of Executive Order No. 111, which became
CENECO do not provide any ground for effective on March 4, 1987, the direct
withdrawal from membership which accordingly certification originally allowed under Article 257
gives rise to the presumption that the same may of the Labor Code has apparently been
be done at any time and for whatever reason. In discontinued as a method of selecting the
addition, membership in the cooperative is on a exclusive bargaining agent of the workers. This
voluntary basis. Hence, withdrawal therefrom amendment affirms the superiority of the
cannot be restricted unnecessarily. The right to certification election over the direct certification
join an organization necessarily includes the which is no longer available now under the
equivalent right not to join the same. change in said provision.

The right of the employees to self-organization is It is not alone sufficient that a union has the
a compelling reason why their withdrawal from support of the majority. What is equally
the cooperative must be allowed. As pointed out important is that everyone be given a democratic
by CURE, the resignation of the member- space in the bargaining unit concerned. The most
employees is an expression of their preference effective way of determining which labor
for union membership over that of membership organization can truly represent the working
in the cooperative. The avowed policy of the force is by certification election.
State to afford full protection to labor and to
promote the primacy of free collective bargaining WHEREFORE, the questioned order for the
mandates that the employees' right to form and direct certification of respondent CURE as the
join unions for purposes of collective bargaining bargaining representative of the employees of
be accorded the highest consideration. petitioner CENECO is hereby ANNULLED and
SET ASIDE. The med-arbiter is hereby ordered
Membership in an electric cooperative which to conduct a certification election among the
merely vests in the member a right to vote during rank-and-file employees of CENECO with
the annual meeting becomes too trinal and CURE and No Union as the choices therein.
insubstantial vis-a-vis the primordial and more
important constitutional right of an employee to SO ORDERED.

MNI NOTES |32


G.R. No. 85750 September 28, 1990 ICMC case:

INTERNATIONAL CATHOLIC • After the Vietnam War, Vietnamese


IMMIGRATION COMMISSION, petitioner refugees started fleeing from South
vs HON. PURA CALLEJA IN HER Vietnam to other countries. The
CAPACITY AS DIRECTOR OF THE Philippine Government and the United
BUREAU OF LABOR RELATIONS AND Nations High Commissioner for
TRADE UNIONS OF THE PHILIPPINES Refugees forged an agreement whereby
AND ALLIED SERVICES (TUPAS) WFTU an operating center for processing Indo-
respondents. Chinese refugees for eventual
resettlement to other countries was to be
G.R. No. 89331 September 28, 1990 established in Bataan. ICMC was one of
those accredited by the Philippine
KAPISANAN NG MANGGAGAWA AT
Government to operate the processing
TAC SA IRRI-ORGANIZED LABOR
center in Bataan. It is a non-profit agency
ASSOCIATION IN LINE INDUSTRIES
involved in international humanitarian
AND AGRICULTURE, petitioner, vs
and voluntary work, enjoys Consultative
SECRETARY OF LABOR AND
Status Category II, it is an international
EMPLOYMENT AND INTERNATIONAL
organization rendering voluntary and
RICE RESEARCH INSTITUTE, INC.,
humanitarian services in the Philippines.
respondents.
DOCTRINE: • The Trade Unions of the Philippines and
The immunity granted being "from every form of Allied Services (TUPAS) filed with the
legal process except in so far as in any particular Ministry of Labor and Employment a
case they have expressly waived their immunity," Petition for Certification Election among
it is inaccurate to state that a certification election the rank and file employed members of
is beyond the scope of that immunity for the ICMC. ICMC opposed because it was an
reason that it is not a suit against ICMC. international organization registered with
the UN and enjoys diplomatic immunity.
A certification election cannot be viewed as an
independent or isolated process. It could tugger • Med-Arbiter: dismissed petition.
off a series of events in the collective bargaining Director Calleja of Bureau of Labor
process together with related incidents and/or Relations reversed Med-Arbiter's
concerted activities, which could inevitably decision and ordered the immediate
involve ICMC in the "legal process," which conduct of a certification election. At that
includes "any penal, civil and administrative time, ICMC's request for recognition as a
proceedings." specialized agency was still pending with
DFA, but was later granted, with
FACTS: corresponding diplomatic privileges and
immunities. ICMC sought the immediate

MNI NOTES |33


dismissal of the petition for certification • Kapisanan filed a petition for direct
election, but was denied. certification election. IRRI opposed due
to it having the status of an international
• ICMC filed petition for certiorari with organization which grants it immunity
preliminary injunction. SC: issued TRO from all civil, criminal, and
on the certification election administrative proceedings under
Philippine laws.
IRRI case:
• Med-Arbiter: dismissed petition for
• The Philippine Government and the Ford
direct election. BLR Director Calleja
and Rockefeller Foundations signed a
reversed Med-Arbiter's decision and
memorandum of understanding
called for a certification election among
establishing the International Rice
the rank and file employees of IRRI. The
Research Institute (IRRA), to be an
Secretary of Labor set aside Calleja's
autonomous, philanthropic, tax-free, non-
decision.
profit, non-stock organization designed to
carry out the principal objective of ISSUE::
conducting basic research on the rice
plant, on all phases of rice production, Whether or not the grant of diplomatic privileges
management, distribution and utilization and immunities extends to immunity from the
with a view to attaining nutritive and application of Philippine labor laws - YES
economic advantage or benefit for the
people of Asia and other major rice- HELD:
growing areas through improvement in
- Yes, Diplomatic Immunity has been granted to
quality and quantity of rice.
both ICMC and IRRI.
• Initially, it was organized and registered
For ICMC, Article 3 Section 4 of the
with the SEC as a private corporation
Convention on the Privileges and Immunities
subject to all laws and regulations.
of Specialized Agencies states that specialized
However, it was granted the status,
agencies like ICMC shall enjoy immunity from
prerogatives, privileges and immunities
every form of legal process except insofar as in
of an international organization by PD
any particular case they have expressly waived
1620.
their immunity. Their property and assets shall be
• The Organized Labor Association in Line immune from search, requisition, confiscation,
Industries and Agriculture (OLALIA) is a expropriation and any other form of interference.
legitimate labor organization with an
For IRRI, Article 3 of PD 1620 likewise states
existing local union, the Kapisanan ng
that it shall enjoy immunity from any penal, civil
Manggagawa at TAC sa IRRI
and administrative proceedings, except where it
(Kapisanan).
has expressly waived its immunity.

MNI NOTES |34


The objective in granting these immunities is Wherefore, petition granted in ICMC case and in
to avoid the danger of partiality and interference IRRI case, the petition was dismissed.
by the host country in their internal workings, and
to shield the affairs of international organizations G.R. No. 181531 July 31, 2009
from political pressure or control by the host
NATIONAL UNION OF WORKERS IN
country to the prejudice of the member States of
HOTELS, RESTAURANTS AND ALLIED
the organization and to ensure the unhampered
INDUSTRIES- MANILA PAVILION
performance of their functions.
HOTEL CHAPTER, Petitioner, vs.
The exercise of jurisdiction by the Department of SECRETARY OF LABOR AND
Labor defeats this purpose. EMPLOYMENT, BUREAU OF LABOR
RELATIONS, HOLIDAY INN MANILA
This immunity does not deprive labor of its PAVILION HOTEL LABOR UNION AND
basic rights as both ICMC and IRRI have ACESITE PHILIPPINES HOTEL
their own modes of protecting the rights of CORPORATION, Respondents.
those employed by them.
FACTS:
For ICMC, the Government is given the
discretion to withdraw their immunities and A certification election was conducted on June
privileges if there has been abuse of those 16, 2006 among the rank-and-file employees of
privileges granted. respondent Holiday Inn Manila Pavilion Hotel
(the Hotel) with the following results:
For IRRI, there is the formation of the Council
of IRRI Employees and Management (CIEM), EMPLOYEES IN VOTERS’ LIST = 353
wherein both management and employees are
represented, for the purpose of maintaining
TOTAL VOTES CAST = 346
mutual and beneficial cooperation between IRRI
and its employees.
NUWHRAIN-MPHC = 151
It is inaccurate to state that a certification election
is beyond the scope of that immunity as it is not
HIMPHLU = 169
a suit against ICMC because a certification
election should not be viewed as an isolated event
but one that could trigger a series of events in the NO UNION = 1
process of collective bargaining which could
inevitably involve both organizations in the legal SPOILED = 3
process/proceedings.

ICMC case: TRO on the certification election is SEGREGATED = 22


made permanent. IRRI case: petition dismissed,
decision of the Secretary of Labor affirmed.

MNI NOTES |35


In view of the significant number of segregated garnered would be one vote short of the majority
votes, contending unions, petitioner, which would then become 169.
NUHWHRAIN-MPHC, and respondent Holiday
Inn Manila Pavillion Hotel Labor Union Secretary affirmed the decision of the med-
(HIMPHLU), referred the case back to Med- arbiter. In fine, the SOLE concluded that the
Arbiter to decide which among those votes would certification of HIMPHLU as the exclusive
be opened and tallied. 11 votes were initially bargaining agent was proper.
segregated because they were cast
ISSUES:
by dismissed employees, albeit the legality of
their dismissal was still pending before the Court (1) whether employees on probationary status at
of Appeals. Six other votes were segregated the time of the certification elections should be
because the employees who cast them allowed to vote - YES
were already occupying supervisory positions at
the time of the election. Still five other votes were (2) whether HIMPHLU was able to obtain the
segregated on the ground that they were cast required majority for it to be certified as the
by probationary employees and, pursuant to the exclusive bargaining agent.- NO
existing Collective Bargaining Agreement
(CBA), such employees cannot vote. It bears HELD:
noting early on, however, that the vote of one
I. On the first issue, the Court rules in the
Jose Gatbonton (Gatbonton), a probationary
affirmative.
employee, was counted.
The inclusion of Gatbonton’s vote was proper not
Med-Arbiter Calabocal ruled for the opening of
because it was not questioned but because
17 out of the 22 segregated votes, specially those
probationary employees have the right to vote in
cast by the 11 dismissed employees and those
a certification election. The votes of the six other
cast by the six supposedly supervisory employees
probationary employees should thus also have
of the Hotel.
been counted. As Airtime Specialists, Inc. v.
Petitioner, which garnered 151 votes, appealed to Ferrer-Calleja holds:
the Secretary of Labor and Employment (SOLE),
In a certification election, all rank and file
arguing that the votes of the probationary
employees in the appropriate bargaining unit,
employees should have been opened considering
whether probationary or permanent are entitled to
that probationary employee Gatbonton’s vote
vote. This principle is clearly stated in Art. 255
was tallied. And petitioner averred that
of the Labor Code which states that the “labor
respondent HIMPHLU, which garnered 169
organization designated or selected by the
votes, should not be immediately certified as the
majority of the employees in an appropriate
bargaining agent, as the opening of the 17
bargaining unit shall be the exclusive
segregated ballots would push the number of
representative of the employees in such unit for
valid votes cast to 338 (151 + 169 + 1 +
purposes of collective bargaining.”
17), hence, the 169 votes which HIMPHLU

MNI NOTES |36


Collective bargaining covers all aspects of the not contrary to law, morals, good customs, public
employment relation and the resultant CBA order or public policy.
negotiated by the certified union binds all
employees in the bargaining unit. Hence, all II. As to whether HIMPHLU should be
rank and file employees, probationary or certified as the exclusive bargaining agent, the
permanent, have a substantial interest in the Court rules in the negative.
selection of the bargaining representative. The
It is well-settled that under the so-called
Code makes no distinction as to their
“double majority rule,” for there to be a valid
employment status as basis for eligibility in
certification election, majority of the
supporting the petition for certification
bargaining unit must have voted AND the
election.
winning union must have garnered majority
The law refers to “all” the employees in the of the valid votes cast.
bargaining unit. All they need to be eligible to
Prescinding from the Court’s ruling that all the
support the petition is to belong to the
probationary employees’ votes should be deemed
“bargaining unit.”
valid votes while that of the supervisory
For purposes of this section (Rule II, Sec. 2 of employees should be excluded, it follows that the
Department Order No. 40-03, series of 2003), number of valid votes cast would increase – from
any employee, whether employed for a 321 to 337. Under Art. 256 of the Labor Code,
definite period or not, shall beginning on the the union obtaining the majority of the valid
first day of his/her service, be eligible for votes cast by the eligible voters shall be certified
membership in any labor organization. as the sole and exclusive bargaining agent of all
the workers in the appropriate bargaining unit.
All other workers, including ambulant, This majority is 50% + 1. Hence, 50% of 337 is
intermittent and other workers, the self- 168.5 + 1 or at least 170.
employed, rural workers and those without any
definite employers may form labor organizations HIMPHLU obtained 169 while petitioner
for their mutual aid and protection and other received 151 votes. Clearly, HIMPHLU was not
legitimate purposes except collective bargaining. able to obtain a majority vote. The position of
both the SOLE and the appellate court that the
The provision in the CBA disqualifying opening of the 17 segregated ballots will not
probationary employees from voting cannot materially affect the outcome of the certification
override the Constitutionally-protected right election as for, so they contend, even if such
of workers to self-organization, as well as the member were all in favor of petitioner, still,
provisions of the Labor Code and its HIMPHLU would win, is thus untenable.
Implementing Rules on certification elections
and jurisprudence thereon. It bears reiteration that the true importance of
ascertaining the number of valid votes cast is for
A law is read into, and forms part of, a contract. it to serve as basis for computing the required
Provisions in a contract are valid only if they are majority, and not just to determine which union

MNI NOTES |37


won the elections. The opening of the segregated
but valid votes has thus become material.

To be sure, the conduct of a certification election


has a two-fold objective: to determine the
appropriate bargaining unit and to ascertain the
majority representation of the bargaining
representative, if the employees desire to be
represented at all by anyone. It is not simply the
determination of who between two or more
contending unions won, but whether it
effectively ascertains the will of the members of
the bargaining unit as to whether they want to be
represented and which union they want to
represent them.

Having declared that no choice in the


certification election conducted obtained the
required majority, it follows that a run-off
election must be held to determine which
between HIMPHLU and petitioner should
represent the rank-and-file employees.

PETITION GRANTED.

____________

NOTES:

A run-off election refers to an election between


the labor unions receiving the 2 highest number
of votes in a certification or consent election with
3 or more choices, where such a certified or
consent election results in none of the 3 or more
choices receiving the majority of the valid votes
cast; provided that the total number of votes for
all contending unions is at least 50% of the
number of votes cast.

MNI NOTES |38


MODULE 2 requiring the attendance of the parties to
such meetings. It shall be the duty of the
2.1 What is a Bargaining Unit? parties to participate fully and promptly
in the conciliation meetings the Board
-it refers to a group of employees sharing may call;
mutual interests within a given employer
unit, comprised of all or less than all of d. During the conciliation proceedings in
the entire body of employees in the the Board, the parties are prohibited from
employer unit or any specific doing any act which may disrupt or
occupational or geographical grouping impede the early settlement of the
within such employer unit. disputes; and
-a union represents a collective
e. The Board shall exert all efforts to settle
bargaining unit.
disputes amicably and encourage the
2.2 Principles in Determining a parties to submit their case to a voluntary
arbitrator. (As amended by Section 20,
Bargaining Unit
Republic Act No. 6715, March 21, 1989)
Procedure in collective bargaining. The
2.3 Reference Materials
following procedures shall be observed in
collective bargaining: Article 250, LC – Rights and Conditions of
a. When a party desires to negotiate an Membership
agreement, it shall serve a written notice
upon the other party with a statement of Art. 250. Rights and conditions of
its proposals. The other party shall make membership in a labor organization. The
a reply thereto not later than ten (10) following are the rights and conditions of
calendar days from receipt of such notice; membership in a labor organization:

b. Should differences arise on the basis of (a) No arbitrary or excessive initiation


such notice and reply, either party may fees shall be required of the members of a
request for a conference which shall legitimate labor organization nor shall
begin not later than ten (10) calendar days arbitrary, excessive or oppressive fine
from the date of request. and forfeiture be imposed;

(b) The members shall be entitled to full


c. If the dispute is not settled, the Board
and detailed reports from their officers
shall intervene upon request of either or
and representatives of all financial
both parties or at its own initiative and
transactions as provided for in the
immediately call the parties to
constitution and by-laws of the
conciliation meetings. The Board shall
organization;
have the power to issue subpoenas

MNI NOTES |39


(c) The members shall directly elect their (f) No person who has been convicted of
officers in the local union, as well as their a crime involving moral turpitude shall be
national officers in the national union or eligible for election as a union officer or
federation to which they or their local for appointment to any position in the
union is affiliated, by secret ballot at union;
intervals of five (5) years. No
qualification requirement for candidacy (g) No officer, agent or member of a labor
to any position shall be imposed other organization shall collect any fees, dues,
than membership in good standing in or other contributions in its behalf or
subject labor organization. The secretary make any disbursement of its moneys or
or any other responsible union officer funds unless he is duly authorized
shall furnish the Secretary of Labor and pursuant to its constitution and by-laws;

Employment with a list of the newly- (h) Every payment of fees, dues or other
elected officers, together with the contributions by a member shall be
appointive officers or agents who are evidenced by a receipt signed by the
entrusted with the handling of funds officer or agent making the collection and
within thirty (30) calendar days after the entered into the record of the organization
election of officers or from the to be kept and maintained for the purpose;
occurrence of any change in the list of
(i) The funds of the organization shall not
officers of the labor organization. (As
be applied for any purpose or object other
amended by RA 6715);
than those expressly provided by its
(d) The members shall determine by constitution and by-laws or those
secret ballot, after due deliberation, any expressly authorized by written
question of major policy affecting the resolution adopted by the majority of the
entire membership of the organization, members at a general meeting duly called
unless the nature of the organization or for the purpose;
force majeure renders such secret
(j) Every income or revenue of the
balloting impractical, in which case the
organization shall be evidenced by a
board of directors of the organization may
record showing its source, and every
make the decision in behalf of the general
expenditure of its funds shall be
membership.
evidenced by a receipt from the person to
(e) No labor organization shall knowingly whom the payment is made, which shall
admit as member or continue in state the date, place and purpose of such
membership any individual who belongs payment. Such record or receipt shall
to a subversive organization or who is form part of the financial records of the
engaged directly or indirectly in any organization.
subversive activity;

MNI NOTES |40


Any action involving the funds of impeachment or expulsion from the
the organization shall prescribe after organization; lawphi1.net
three (3) years from the date of
submission of the annual financial report (l) The treasurer of any labor organization
to the Department of Labor and and every officer thereof who is
Employment or from the date the same responsible for the accounts of such
should have been submitted as required organization or for the collection,
by law, whichever comes earlier: management, disbursement, custody or
control of the funds, moneys and other
Provided, That this provision properties of the organization, shall
shall apply only to a legitimate labor render to the organization and to its
organization which has submitted the members a true and correct account of all
financial report requirements under this moneys received and paid by him since
Code: Provided, further, That failure of he assumed office or since the last date on
any labor organization to comply with the which he rendered such account and of
periodic financial reports required by law the balance remaining in his hands at the
and such rules and regulations time of rendering such account, and of all
promulgated thereunder six (6) months bonds, securities and other properties of
after the effectivity of this Act shall the organization entrusted to his custody
automatically result in the cancellation of or under his control.
union registration of such labor
organization. (As amended by RA 6715); The rendering of such account shall be
made.
(k) The officers of any labor organization
shall not be paid any compensation other (1) At least once a year within
than the salaries and expenses due to their thirty (30) days after the close of
positions as specifically provided for in its fiscal year;
its constitution and by-laws or in a written
(2) At such other times as may be
resolution duly authorized by a majority
required by a resolution of the
of all the members at a general
majority of the members of the
membership meeting duly called for the
organization; and
purpose.
(3) Upon vacating his office.
The minutes of the meeting and
the list of participants and ballots cast The account shall be duly audited and
shall be subject to inspection by the verified by affidavit and a copy thereof
Secretary of Labor and Employment or shall be furnished the Secretary of Labor
his duly authorized representatives. Any and Employment;
irregularities in the approval of the
resolutions shall be a ground for

MNI NOTES |41


(m) The books of accounts and other For this purpose, registered labor
records of the financial activities of any organizations may assess reasonable dues
labor organization shall be opened to to finance labor relations seminars and
inspection by any officer or member other labor education activities.
thereof during office hours;
Any violation of the above rights
(n) No special assessment or other and conditions of membership shall be a
extraordinary fees may be levied upon the ground for cancellation of union
members of a labor organization unless registration or expulsion of an officer
authorized by a written resolution of a from office, whichever is appropriate. At
majority of all the members at a general least thirty (30%) per cent of all the
membership meeting duly called for the members of a union or any member or
purpose. The secretary of the members specifically concerned may
organization shall record the minutes of report such violation to the Bureau. The
the meeting including the list of all Bureau shall have the power to hear and
members present, the votes cast, the decide any reported violation and to mete
purpose of the special assessment or fees appropriate penalty.
and the recipient of such assessment or
fees. The record shall be attested to by the Criminal and civil liabilities
president; arising from violations of above rights
and conditions of membership shall
(o) Other than for mandatory activities continue to be under the jurisdiction of
under the Code, no special assessment, ordinary courts.
attorney's fees, negotiation fees or any
other extraordinary fees may be checked Section 1(d) and ® Rule I, Book V, Omnibus
off from any amount due an employee Rules Implementing the Labor Code
without an individual written
(d) Bargaining Unit
authorization duly signed by the
employee. The authorization should → refers to a group of employees sharing mutual
specifically state the amount, purpose and interests within a given employer unit, comprised
beneficiary of the deduction; and of all or less than all of the entire body of
employees in the employer unit or any specific
(p) It shall be the duty of any labor
occupational or geographical grouping within
organization and its officers to inform its
such employer unit.
members on the provisions of its
constitution and by-laws, collective (r) Employee
bargaining agreement, the prevailing
labor relations system and all their rights → refers to any person working for an employer.
and obligations under existing labor laws. It includes one whose work has ceased in
connection with any current labor dispute or

MNI NOTES |42


because of any unfair labor practice and one who but may form, join, or assist separate labor unions
has been dismissed from work but the legality of of their own. Managerial employees shall not be
the dismissal is being contested in a forum of eligible to form, join, or assist any labor unions
appropriate jurisdiction. for purposes of collective bargaining.

Rule II, Book V, Omnibus Rules Alien employees with valid working
Implementing the Labor Code permits issued by the Department may exercise
the right to self-organization and join or assist
Coverage of the Right to Self-Organization labor unions for purposes of collective
bargaining if they are nationals of a country
Section 1. Policy- It is the policy of the State to
which grants the same or similar rights to Filipino
promote the free and responsible exercise of the
workers, as certified by the Department of
right to self-organization through the
foreign Affairs, or which has ratified either ILO
establishment of a simplified mechanism for the
Convention No. 87 and ILO Convention No. 98.
speedy registration of labor unions and workers’
association, determination or representation For purposes of this Section, any
status and resolution of inter/intra-union and employee whether employed for a definite period
other related labor relations dispute. Only or not, shall beginning on the first day of his/her
legitimate or registered labor unions shall have service, be eligible for membership in any labor
the right to represent their members for collective organization.
bargaining and for other purposes. Worker’s
associations shall have the right to represent their All other workers, including ambulant,
members for purposes other than collective intermittent and other workers, the self-
bargaining. employed, rural workers and those without any
definite employers may form labor organizations
Section 2. Who May Join Labor Unions and for their mutual aid and protection and other
Worker’s Associations- All persons employed legitimate purposes except collective bargaining.
in commercial, industrial, and agricultural
enterprises, including employees of government- 2.4 Readings / Cases
owned or controlled corporations without
original charters established under the Erson Ang Lee v. Samahang
Corporation Code, as well as employees of Manggagawa ng Super Lamination GR
religious charitable, medical or educational No 193816
institutions whether operating for profit or not,
shall have the right to self-organization and to Doctrine: A settled formulation of piercing the
form, join, or assist labor unions for purposes of corporate veil is that when two business are
collective bargaining: owned, conducted, and controlled by the same
parties, both law and equity will, when necessary
Provided, however, that supervisory
to protect the rights of third parties, disregard the
employees shall not be eligible for membership
in a labor union of the rank-and-file employees

MNI NOTES |43


legal fiction that these two entities are distinct Super Lamination, Express Lamination and
and treat them as identical or as one and the same. Express Coat (Companies) all represented by one
counsel filed a Motion to Dismiss on the ground
FACTS: that there is no employer-employee relationship
(EER) between these establishments and the
Petitioner Erson Ang Lee, through Super
bargaining units that Unions A, B and C seek to
Lamination, is a duly registered entity principally
represent as well as these unions’ respective
engaged in the business of providing lamination
members
services to the general public
All the filed PCEs were denied.
• Respondent Samahan ng mga
Manggagawa ng Super Lamination On appeal before the Office of the DOLE
Services (Union A) is a legitimate labor Secretary: ruled in favor of the unions and
organization, which is also a local chapter granted the PCEs
af liate of the National Federation of
Labor Unions — Kilusang Mayo Uno. DOLE found that Super Lamination, Express
• It appears that Super Lamination is a Lamination, and Express Coat were sister
sole proprietorship under petitioner's companies that had a common human resource
name, while Express Lamination and department responsible for hiring and
Express Coat are duly incorporated disciplining the employees of the three
entities separately registered with the companies.
Securities and Exchange Commission
(SEC). The same department was found to have also
given them daily instructions on how to go about
On Mar 7, 2008: Unions A, B, and C all filed their work and where to report for work.
petitions on the same day.
It also found that the three companies involved
• Union A filed a Petition for Certification constantly rotated their workers, and that the
Election (PCE) to represent all the rank- latter's identification cards had only one
and-file employees of Super Lamination signatory.
• Notably on the same date, Express
These circumstances showed that the companies
Lamination Workers’ Union (Union B)
were engaged in a work-pooling scheme, in
also filed a PCE to represent all the rank-
light of which they might be considered as one
and-file employees of Express
and the same entity for the purpose of
Lamination.
determining the appropriate bargaining unit in a
• the Samahan ng mga Manggagawa ng
certification election
Express Coat Enterprise, Inc. (Union C)
also filed a PCE to represent the rank- ISSUE:
and-file employees of Express Coat.

MNI NOTES |44


1. Whether the rank-and-file employees of Super juridical personalities there are instances when
Lamination, Express Lamination and Express their separate personality will be disregarded
Coat constitute an appropriate bargaining unit. under the doctrine of piercing the corporate
veil.
2. Whether the application of the doctrine of
piercing the corporate veil is warranted the pieces of evidence (such as the companies
were engaged in the same business, employees
RULING: were hired through a common human resource
department, the workers were constantly rotated
The basic test for determining the appropriate
and that the common human resource department
bargaining unit is the application of a standard
imposed disciplinary sanctions and directed the
whereby a unit is deemed appropriate if it affects
daily performance of all the members of Unions
a grouping of employees who have substantial,
A, B & C) adduced by the respondents show that
mutual interests in wages, hours, working
the three companies are under the control and
conditions, and other subjects of collective
management of the same party – petitioner Ang
bargaining
Lee.
We have ruled that geographical location can be
We hold that if we allow petitioner and the two
completely disregarded if the communal or
other companies to continue obstructing the
mutual interests of the employees are not
holding of the election in this manner, their
sacrificed.
employees and their respective unions will never
In the present case, there was communal have a chance to choose their bargaining
interest among the rank-and-file employees of representative.
the three companies based on the finding that
Therefore, it is only proper that, in order to
they were constantly rotated to all three
safeguard the right of the workers and Unions A,
companies, and that they performed the same
B, and C to engage in collective bargaining, the
or similar duties whenever rotated
corporate veil of Express Lamination and
As correctly observed by the lower courts, while Express Coat must be pierced.
there is no prohibition on the mere act of
engaging in a work-pooling scheme as sister
Leoncia Yumang v. RPN9GR No
companies, that act will not be tolerated, and the 201016
sister companies' separate juridical personalities
Doctrine: Pursuant to Article 250 of the Labor
will be disregarded, if they use that scheme to
Code, one of the rights of a union member is the
defeat the workers' right to collective bargaining.
right to call for the investigation of any
While it is true that separate corporations cannot irregularity within the union. Thus, a complaint
be treated as a single bargaining unit even if their filed regarding such an irregularity cannot be
businesses are related as these companies are considered a misconduct or disloyalty under the
indubitably distinct entities with separate union’s constitution and by-laws.

MNI NOTES |45


FACTS: violated the prohibition against forming another
union outside of the freedom period, they can be
Leoncia Yumang was employed in Radio held guilty of malicious attack against the union
Philippines Network, Inc. She was a member of or union officers under the CBL
the Radio Philippines Network Employees Union -They were found guilty for urging or
(RPNEU) which had a CBA with RPN 9 (2004- advocating to the members of filing cases
2009) with the DOLE without availment of
exhaustion of all remedies
Yumang and 14 other union members filed
complaints with the DOLE-NCR against the
RPNEU officers and members of the Board of RPNEU officers and directors asked RPN 9 to
Directors for: impeachment, an audit of union terminate the employment of the expelled union
funds, and the conduct of a snap election. members, pursuant to the CBA’S union security
clause
o Because allegedly the union president
was driving a new Toyota revo which was Yumang wrote to RPN 9 claiming that their
registered under the name of RPN’s expulsion had been reversed by 118 union
general manager members or more than 30% of RPNEU’s general
membership
In the meantime, two complaints were filed with
the RPNEU Executive Board against Yumang
and her group with the Grievance Committee.
The complaints involved alleged violations of the But RPNEU that those 30% could not have
RPNEU Constitution and Bylaws (CBL), validly convened since Yumang and her group
principally: failed to appeal the BOD resolution expelling
them from the union as required by the CBL
1) the commission of acts inimical to the
interests of the union and the general Inquiry by RPN on the said matter ensued but
membership; Yumang sensed that the RPN panel handling the
case was conducting an inquiry only to effect a
2) the attempt to form another union; and reconciliation between them and the officers and
not to determine the validity of their expulsion
3) an appeal to the general membership
urging them to commence legal action Nonetheless, they expressed no objection to a
without exhausting remedies under the reconciliation on condition that a referendum
RPNEU CBL. should be held, the union should shoulder their
attorney’s fees and that they be paid damages

The grievance committee submitted its report to The Union president (Siozon) wanted all the
the union board of directors stating that while cases dropped. The next day, the expelled union
Yumang and others cannot be said to have members informed the panel that they would no

MNI NOTES |46


longer answer any questions and allegedly for o That inquiry was conducted for the sole
this reason, the panel concluded the inquiry purpose for reconciliation and not determining
the validity of her expulsion
The panel recommended that RPN management
comply with the CBA’S union security clause (so o That it merely questioned the resolution of at
terminated na nga ang Yumang and friends) least 30% of union membership reversing their
expulsion
Yumang filed a complaint for unpaid CBA
benefits and applicable wage orders • Yumang denied the RPNEU’s charges against
her and invoked her right under Art. 241 of the
After which, she filed a complaint for illegal Labor Code that she has a right to call for the
dismissal against RPN investigation of any irregularity within the union

The Labor Arbiter declared that Yumang had • The CA denied the petition and affirmed the
been illegally dismissed and ordered her NLRC ruling stating that it was well within the
reinstatement with back wages, monetary NLRC’S jurisdiction to uphold petitioners
benefits dismissal as per the CBA

o Also declared that although Yumang’s o It also concluded that Yumang and other
dismissal was in compliance with the CBA’s complainants were given several opportunities to
union security clause, her expulsion from the defend themselves but they responded with
union was without due process suspicion and animosity

• RPN appealed and NLRC reversed and declared • Yumang filed an MR and again raised the
that Yumang’s dismissal valid matter of non-perfection of the respondents
appeal and for failure of CA to explain why it
• Yumang sought appeal to the CA via petition
departed from the established facts as ruled by
for certiorari charging NLRC with grave abuse of
other divisions in at least 2 identical cases –
discretion when it entertained RPN’s appeal
DENIED
despite non-perfection and for declaring her
dismissal as valid ISSUE:

• Yumang futher argued that while her WON Yumang can be held guilty of malicious
employment was terminated in compliance with attack against the union officers and thus warrant
the CBA’s union security clause, she was not her expulsion from the union and from the
accorded due process before she was dismissed establishment itself by virtue of the CBA’s union
security clause
o She assails that RPN’s inquiry into her
expulsion without the company investigating HELD:
whether it was justified

MNI NOTES |47


No, Yumang cannot be held guilty of malicious the first, is the comprehensive power vested in
attack against the union officers that will warrant the union with respect to the individual, The
the termination of her employment. union may be considered but the agent of its
members for the purpose of securing for them fair
Pursuant to Article 250 of the Labor Code and just wages and good working conditions and
which lists down the rights and conditions of is subject to the obligation of giving the members
membership in a labor organization, it is her right as its principals all information relevant to union
to be informed of what is going on with the union, and labor matters entrusted to it.
especially in handling of union funds, the
Where, however, collective bargaining process is
negotiation and conclusion of the CBA, in labor
not involved, and what is at stake are back wages
education and in all the rights and obligations of
already earned by the individual workers by way
union members under existing laws.
of overtime, premium and differential pay, and
In the case at hand, apparently, Yumang and the
final judgment has been rendered in their favor,
other expelled union members were not informed
as in the present case, the real parties in interest
about these matters, prompting them to seek an
with direct material interest, as against the union
investigation on how the union affairs were being
which has only served as a vehicle for collective
administered. The petitioner cannot be made
action to enforce their just claims, are the
answerable for malicious attack against the
individual workers themselves.
RPNEU and its officers as she was merely
exercising her right, as a union member, to Authority of the union to waive or quit claim all
ventilate before the public authorities her or part of the judgment award in favor of the
perceived grievance against the union leadership. individual workers cannot be lightly presumed
In sum, we find merit in the petition. The but must be expressly granted, and the employer,
petitioner was illegally dismissed as her as judgment debtor, must deal in all good faith
expulsion from the union had no basis. with the union as the agent of the individual
workers.
The court in turn should certainly verify and
Heirs of Teodolo Cruz v. CIR assure itself of the fact and extent of the authority
GR No L-23361-62 of the union leadership to execute any
compromise or settlement of the judgment on
DOCTRINE: The union has been evolved as an behalf of the individual workers who are the real
organization of collective strength for the judgment creditors.
protection of labor against the unjust exactions of
capital, but equally important is the requirement FACTS:
of fair dealing between the union and its • On June 21, 1952, the Santiago Labor Union,
members, which is fiduciary in nature, and arises composed of workers of the Santiago Rice Mill,
out of two factors: one is the degree of a business enterprise engaged in the buying and
dependence of the individual employee on the milling of palay at Isabela, and owned operated
union organization; and the other, a corollary of by King Hong Co., filed before the respondent

MNI NOTES |48


Court of Industrial Relations a petition for • Before and after the submittal of the Chief
overtime pay, premium pay for night, Sunday Examiner's Report, the union pressed for
and holiday work, and for reinstatement of execution of the final judgment in favor of its
workers illegally laid off. claimants-members.
• The court dismissed the petition of the union It filed an Urgent Motion for Preliminary
for lack of merit and want of jurisdiction; but Attachment, in view of the disposition by
upon a motion for reconsideration, the Court of respondent firm of its trucks and automotive
Industrial Relations en banc, by a split decision equipment, and by virtue of the fact admitted by
of 3-2 vote, issued a resolution reversing the respondent firm that it had stopped operations
decision of the trial judge. preparatory to liquidation, by reason of the alien
nationality of most of its stockholders, under the
• After the remand of the records for enforcement
provisions of Republic Act No. 3018
by CIR, the Court's Chief Examiner filed his
nationalizing the rice and corn industry. In
Partial Report wherein the judgment award in
another motion, the union had asked that the
favor of the workers totaled to P423,756.74.
Court at least order respondent firm to put up a
However, petitioners claim that in this
bond of P500,000.00.
computation, the filed examiners did not include
the claims of seventy (70) other laborers whose • The trial Judge released an Order for the
total claims for back wages would be respondent firm to deposit in Court the sum of
P441,000.00. Therefore, the correct grand total one hundred thousand (P100,000.00) pesos and
amount due the laborers would be P864,756.74. to file a surety bond of equal amount. This Order
was affirmed by respondent CIR en banc, in its
• The Chief Examiner's Report showed
Resolution denying respondent firm's motion for
respondent firm's total assets to be P191.151.08,
reconsideration.
and its net worth to be in the same amount of
P191,151.08. The Report further stated • The trial judge took no action on the latest
respondent firm sold its trucks, jeep and one car, Urgent Motion of the union, wherein it
with a net book value of P2,628.71 for emphasized that respondent, with Court’s action
P27,000.00 or a net gain of P24,371.29. rejecting its appeal, no longer had any excuse for
refusing to comply with the deposit Order.
Petitioners claim that the book value of
Instead, an unscheduled conference was called
respondent firm's fixed assets is only one-sixth of
and held on October 31, 1963 in the chambers of
their actual market value of P240.442.50, and
the trial judge, and attended by representatives of
that its total leviable assets therefore amounted to
respondent firm, including their counsels of
close to P390,000.00, without taking into account
record and the President of the union and 8
the huge income potential of its rice mill
directors of the union. Four of these nine union
operations. Respondent firm disputes such a
representatives, including the union president
figure as "completely gratuitous and without
himself, had no claims or awards whatever under
basis in fact."
the judgment. Said union officials were not
assisted by counsel, as petitioner Mary

MNI NOTES |49


Concepcion, counsel of record of the union, was The law provides that due process must be
not present, not having been notified of the followed.
conference.
In the present case, the union was deprived of
• In this conference respondent firm made again the assistance of its counsel.
the same offer to settle and quitclaim the
The lack or due deliberation and caution in the
judgment in favor of the union members for the
trial judge’s instant approval of the settlement is
same amount of P110,000.00, which offer had
seen from the stipulations therein that the union
already been ‘rejected by the union at the earlier
thereby waived and quitclaimed any and all
conference held on June 25, 1963. But this time,
claims which it may have against the respondent,
respondent and the directors of the union decided
as well as the claim of each and every one of the
to settle the case amicably with the payment by
members of the union against respondent, when
the firm of the same amount of P110,000.00
precisely the authority of the union board
which was deposited with the Court’s disbursing
members to enter into any such compromise or
officer “immediately upon the signing of the
settlement was under express challenge by
settlement which will be prepared by the
petitioner Magalpo, a board member herself
respondent firm through its counsel.”
which the trial judge completely disregarded.
• One of the union directors together with 49 of
Petitioners were deprived of the formal
its members questioned the amicable settlement
conference and of their right to be assisted by the
that took place. They claim that the Board of
union counsel as expressly requested, so that a
Directors did not have any express authority of
fair hearing could be accorded petitioners and an
the members of the Santiago Labor Union to
opportunity afforded them to air their serious
enter into any compromise for the sum of
charges of bad faith and lack authority against the
P110,000.00, that it was tainted by apparent bad
Union leadership.
faith on the part of the President of the Union,
that the amount of P110,000.00 is Certainly, all these serious questions and charges
unconscionable, considering, that the total claims made by petitioners could have been threshed out
of the members of the union is more than and verified, if the formal conference had been
P400,000.00. held with the presence of union counsel.
ISSUE: The transcript of the conference is deficient and
does not reflect the actual discussions and
WON this Court can give its sanction to
proceedings. This is to be deplored, for in a
respondent Court's majority resolution upholding
matter of such great importance, especially
the trial judge's approval of the union board's
where the union officials were unassisted by
settlement for P110,000.00 of the estimated
counsel in an unscheduled conference, care
P423,766.74? - NO.
should be taken by the trial judge that the
RULING: proceedings are faithfully recorded.

MNI NOTES |50


Moreover, the law provides that the lack of any comprehensive power vested in the union with
express or specific authority of the president and respect to the individual.”
majority of the union Board of Directors shall
The union may, be considered but the agent of its
render the settlement invalid.
members for the purpose of securing for them fair
In the present case, the petitioner board member and just wages and good working conditions and
Magalpo timely challenged the authority or the is subject to the obligation of giving the members
union board to execute any such settlement, as its principals all information relevant to union
expressly informing the trial judge that the union and labor matters entrusted to it.
had specifically appointed an entity in Manila,
The union leadership in the case at bar was
the “CREAM, Inc.,” as its attorney in-fact and
recreant in its duty towards the union members in
“exclusive authorized representative for the
apparently having failed to disclose to the union
evaluation, adjustment and -liquidation or its
members the full situation of their judgment
claim against respondent. These union members
credit against respondent, to wit, that they were
have repudiated the former union president,
in the advantageous position of being able to
Maylem and his board of directors, for having
require enforcement of the respondent court’s
betrayed the union members, and the new union
P200,000.00-deposit order, and in presuming that
leadership. The authority of the union, to execute
it had authority to waive and quitclaim the
a settlement of the judgment award in favor of the
estimated P423,756.74-judgment credit of the
individual union members, cannot be presumed
union members for the unconscionable amount of
but must be expressly granted.
P110,000.00, which had already been previously
Just as this Court has stricken down unjust rejected by the workers.
exploitation of laborers by oppressive employers,
Respondent firm could not claim that it dealt in
so will it strike down their unfair treatment by
good faith with the union officials, for it hastily
their own unworthy leaders.
executed the purported settlement
The Constitution enjoins the State to afford notwithstanding the serious charges of bad faith
protection to labor. Fair dealing is equally against the union leadership, and the nonholding
demanded of unions as well as of employers in of the scheduled conference where the union
their dealings with employees. leaders, at their express request, could be duly
assisted by union counsel.
The union has been evolved as an organization of
collective strength for the protection of labor The interests of the individual worker can be
against the unjust exactions of capital, but better protected on the whole by a strong union
equally important is the requirement of fair aware of its moral and legal obligations to
dealing between the union and its members, represent the rank and file faithfully and secure
which is fiduciary in nature, and arises out of two for them the best wages and working terms and
factors: “one is the degree of dependence of the conditions in the process of collective
individual employee on the union organization; bargaining. As has been aptly pointed out, the
the other, a corollary of the first, is the will of the majority must prevail over that of the

MNI NOTES |51


minority in the process, for “under the Benjamin Victoriano v. Elizalde Rope
philosophy of collective responsibility, an Workers' Union et. al.
employer who bargains in good faith should be
GR No L-25246
entitled to rely upon the promises and agreements
of the union representatives with whom he must
DOCTRINE: It must be pointed out that the free
deal under the compulsion of, law and contract.
exercise of religious profession or belief is
The “collective bargaining process should be superior to contract rights. In case of conflict, the
carried on between parties who can mutually latter must, therefore, yield to the former.
respect and rely upon the authority of each
other.” Where, however, collective bargaining FACTS:
process is not involved, and what is at stake are
back wages already earned by the individual • Benjamin Victoriano, a member of the
workers by way of overtime, premium and religious sect known as the "Iglesia ni Cristo",
differential pay, and final judgment has been had been in the employ of the Elizalde Rope
rendered in their favor, as in the present case, the Factory, Inc. since 1958.
real parties in interest with direct material
interest, as against the union which has only As such employee, he was a member of the
served as a vehicle for collective action to Elizalde Rope Workers' Union which had with
enforce their just claims, are the individual the Company a collective bargaining agreement
workers themselves. Authority of the union to containing a closed shop provision which
waive or quitclaim all or part of the judgment requires membership in the union as a condition
award in favor of the individual workers cannot of employment for all permanent employees’
be lightly presumed but must be expressly workers covered by this Agreement.
granted, and the employer, as judgment debtor,
must deal in all good faith with the union as the The collective bargaining agreement expired on
agent of the individual workers. March 3, 1964 but was renewed the following
day, March 4, 1964.
The Court in turn should certainly verify and
assure itself of the fact and extent of the authority
Under Section 4(a), paragraph 4, of Republic
of the union leadership to execute any
Act No. 875, prior to its amendment by Republic
compromise or settlement of the judgment on
Act No. 3350, the employer was not precluded
behalf of the individual workers who are the real
"from making an agreement with a labor
judgment creditors. The settlement in the case at
organization to require as a condition of
bar was precipitately approved without
employment membership therein, if such labor
verification of the union boards authority to
organization is the representative of the
execute the compromise settlement and that there
employees."
was no such authority.
CONCLUSION: Therefore, the Union Board’s On June 18, 1961, however, Republic Act No.
settlement is invalid. 3350 was enacted, introducing an amendment to

MNI NOTES |52


- paragraph (4) subsection (a) of section 4 of consequently, deprives said members of
Republic Act No. 875, as follows: their constitutional right to form or join
... "but such agreement shall not cover members lawful associations or organizations
of any religious sects which prohibit affiliation of guaranteed by the Bill of Rights, and thus
their members in any such labor organization" becomes obnoxious [to the] Constitution;
• Being a member of a religious sect that 2) Impairs the obligation of contracts;
prohibits the affiliation of its members with any 3) discriminates in favor of certain religious sects
labor organization, Appellee presented his and affords no protection to labor unions;
resignation to appellant Union in 1962, and when 4) violates the constitutional provision that no
no action was taken thereon, he reiterated his religious test shall be required for the exercise of
resignation on September 3, 1974. a civil right;
5) violates the equal protection clause; and
• Thereupon, the Union wrote a formal letter to 6) the act violates the constitutional provision
the Company asking the latter to separate regarding the promotion of social justice
Appellee from the service in view of the fact that
he was resigning from the Union as a member. ISSUE:

• The management of the Company in turn WON RA 3350 infringes on the fundamental
notified Appellee and his counsel that unless the right to form lawful associations when it
Appellee could achieve a satisfactory "prohibits all the members of a given religious
arrangement with the Union, the Company would sect from joining any labor union if such sect
be constrained to dismiss him from the service. prohibits affiliations of their members thereto";
and, consequently, deprives said members of
• This prompted Appellee to file an action for their constitutional right to form or join lawful
injunction, in the Court of First Instance of associations or organizations guaranteed by the
Manila to enjoin the company and the Union Bill of Rights
from dismissing Appellee.
RULING: NO.
• CFI: judgment is rendered enjoining the
defendant Elizalde Rope Factory, Inc. from The purpose of Republic Act No. 3350 is
dismissing the plaintiff from his present secular, worldly, and temporal, not spiritual or
employment. religious or holy and eternal.

• In its appeal, the Union claimed that R.A. no. It was intended to serve the secular purpose of
3350 was unconstitutional on the ground that: advancing the constitutional right to the free
1) prohibits all the members of a given exercise of religion, by averting those certain
religious sect from joining any labor persons be refused work, or be dismissed from
union if such sect prohibits affiliations of work, or be dispossessed of their right to work
their members.thereto; and, and of being impeded to pursue a modest means

MNI NOTES |53


of livelihood, by reason of union security Tancinco v. Pura Ferrer-Calleja
agreements. Congress acted merely to relieve the GR No 78131
exercise of religion, by certain persons, of a
burden that is imposed by union security DOCTRINE: Eligibility to vote may be
agreements. It was Congress itself that imposed determined through the use of the applicable
that burden when it enacted the Industrial Peace payroll period and employee's status during the
Act (Republic Act 875), and, certainly, Congress, applicable payroll period. The payroll of the
if it so deems advisable, could take away the month next preceding the labor dispute in case of
same burden regular employees and the payroll period at or
near the peak of operations in case of employees
In this case, the means adopted by the Act to in seasonal industries
achieve that purpose (exempting the members of FACTS:
said religious sects from coverage of union
security agreements) is reasonable. What the • The respondents are the organizers of Imperial
exception provides, is that members of said Textile Mills Inc. Monthly Employees
religious sects cannot be compelled or coerced to Association (ITM-MEA)
join labor unions even when said unions have
closed shop agreements with the employers and • While respondents were preparing to file a
that in spite of any closed shop agreement, petition for direct certification of the union as
members of said religious sects cannot be refused sole and exclusive bargaining agent of ITM’s
employment or dismissed from their jobs on the bargaining unit, the union’s VP (Dalamaco) was
sole ground that they are not members of the promoted to Department Head, thereby
collective bargaining union. disqualifying him for union membership

Infringement of religious freedom may be • This incident, among others, lead to strike
justified only if there were an immediate and headed by the Lacanilao group, while the
grave danger to the security and welfare of the Tancinco group staged a strike inside the
community where it is unavoidably necessary to company premises.
prevent or avoid such danger.
• After 4 days, strike was settled, agreement was
It is clear, therefore, that the assailed act entered between Lacanilao and Tancinco group
(Republic Act No. 3350), far from infringing the
constitutional provision on freedom of • Pre-election conference was held but parties
association, upholds and reinforces it. It does not failed to agree on the list of voters
prohibit the members of said religious sects from
affiliating with labor unions. It still leaves to said • In another pre-election conference attended by
members the liberty and the power to affiliate, or MOLE
not to affiliate, with labor unions. officers, ANGLO through its National Secretary,
a certain Mr. Cornelio A. Sy made a unilateral

MNI NOTES |54


ruling excluding some 56 employees consisting RULING:
of the Manila office employees, members of
YES. Submission of the employees names with
Iglesia ni Kristo, non-time card employees,
the BLR as qualified members of the union is not
drivers of Mrs. Salazar and the cooperative
a condition sine qua non to enable said members
employees of Mrs. Salazar.Cooperative
to vote in the election of union's officers
employees of Mrs. Salazar
Per public respondent's findings the 1986 list
• MOLE Pampanga protested the ruling but no
consists of 158 union members only wherein 51
action was taken
of the 56 challenged voters' names do not appear.
Adopting however a rough estimate of a total
• Election of officers were conducted, votes of
number of union members who cast their votes of
said 56 employees were not counted
some 333 and excluding therefrom the 56
challenged votes, if the list is to be the basis as to
• Lacnilao’s group won, 119 votes (3 votes over
who the union members are then public
Tancinco)
respondent should have also disqualified some
175 of the 333 voters
• Tancinco filed a protest for inclusion of the 56
votes
Under article 242(c) of the LC, as amended,
only members of the union can participate in the
• MOLE directed the inclusion of the votes
election of union officers. The question however
of eligibility to vote may be determined through
• Lacanilao appealed with BLR
the use of the applicable payroll period and
o Iglesia ni Kristo - allowing them to vote
employee's status during the applicable payroll
will be
period. The payroll of the month next
anomalous since it is their policy not to
preceding the labor dispute in case of regular
participate in any form of union activities
employees and the payroll period at or near the
o non-time card employees, that they are
peak of operations in case of employees in
managerial employees
seasonal industries
o employees of the cooperative as non-
ITM employees
In this case, considering that none of the parties
insisted on the use of the payroll period-list as
• BLR said that exclusion was arbitrary BUT set
voting list and considering further that the 51
aside the MOLE order since 51/56 are not yet
remaining employees were correctly ruled to be
union members at the time of election
qualified for membership, their act of joining the
election by casting their votes on after the 1986
ISSUE:
agreement is a clear manifestation of their
intention to join the union.
Whether or not the 56 votes should be included.
YES.

MNI NOTES |55


Manalad v. Trajano the positions and ordered the winners of the
GR No 72772-73 November 26, 1984 elections to stop acting as
officers, and turn over immediately the
TOPIC: Rights and Conditions of Membershi management of the union affairs to respondent
FACTS: Director of the Bureau of Labor Relations as well
as a call for a special election.
• The parties herein are employees of United
Dockhandlers, Inc.; they are members of rival • Pursuant thereto, the Director of the Bureau of
groups in the Associated Port Checkers and Labor Relations issued an order on July 10, 1985
Workers' Union (APCWU for short) in said to the effect that he was taking over the
company, management of the affairs of said union, ordering
private respondents Babula and all other persons
• Sometime in 1982, the petitioners were to cease acting as officers of the union, and
disqualified from running as candidates in the requiring them to turn over the union funds to
election of APCWU officers by the Med-Arbiter. said director.
However, on appeal, said order was set aside by
the Director of the Bureau of Subsequently, the Court’s aforesaid resolution of
Labor Relations on October 31, 1984. July 3, 1985 was modified on July 17, 1985 by
providing that the special election scheduled on
• Thereafter, the election of officers and board July 20, 1985 shall be held under the personal
members of the union was held on November 26, supervision of respondent
1984, with the candidates of the petitioners, that Director Trajano. Private respondents won the
is, Manalad, Leano and Puerto, winning over elections
those of the private respondents, who were although there was an attempt from the
Babula, Mijares and Navarro, for the positions of petitioners to have them disqualified for their
president, treasurer and auditor, respectively. alleged refusal to comply with the resolution of
the SC.
• As a consequence, the respondents filed a
petition for review with this Court assailing the • On July 26, 1985,Director Trajano still issued a
aforesaid order of October 31, 1984 of the Bureau resolution proclaiming private respondents as the
of Labor Relations which had declared the winners in the special election and duly elected
aforesaid petitioners eligible to run for said union officers of APCWU, with the following
offices. observation: “The submission that Mr. Babula
failed to completely turn over management of the
• While the petition for review filed by the union to the undersigned is within the
respondents was pending, the SC, in another case competence and authority of the Supreme Court
entitled Associated Port Checkers and Workers to pass upon considering that the mandate for
Union, et al. vs. Ricardo R. Manalad, et al.” such a turn-over came from the Court.
however issued a resolution declaring vacant all

MNI NOTES |56


• Meanwhile, the three-year term of the private overriding considerations to the contrary, to
respondents under the July 20, 1985 elections respect the will of the majority of the workers
expired on July 20, 1988. who voted in the November 28, 1988 elections.
Although
• However petitioners insist that they be declared decreed under a different setting, it is apropos to
the winners in said election (held on Nov. 26, recall in this case.
1984) with their terms of three (3) years to
commence from the time they assume office in Our ruling that where the people have elected a
execution of a final and executory resolution of man to office, it must be assumed that they did
the SC. this with knowledge of his life and character, and
that they disregarded or forgave his faults or
• Consequently, an election was held on misconduct, if he had been guilty of any.
December 1, 1988 to elect new set of officers due
to the expiration of Bulba et al’s term of office. At this juncture, it would further be appropriate
to remind petitioners that even if the
ISSUE: disqualification of private respondents could be
Whether the case became moot and academic justified, the candidates of petitioners certainly
because of the expiration of the terms of office of cannot be declared as the winners in the disputed
the union officers and the election of officers on election. The mere fact that they obtained the
November 28, 1988. second highest number of votes does not mean
that they will thereby be considered as the elected
RULING: officers if the true winners are disqualified.

Yes, because the consistent ruling is that ACCORDINGLY, this case is DISMISSED for
whenever certain events or circumstances have being moot and academic.
taken place during the pendency of the case
which would render the case moot and academic,
the petition should be dismissed.
Del Pilar Academy, et. al. v. Del Pilar
Academy Employees Union
Under Philippine Jurisprudence, It is pointless
and unrealistic to insist on annulling an election GR No 170112
of officers whose terms had already expired. We
would have thereby a judgment on a matter DOCTRINE: Employees of an appropriate
which cannot have any practical legal effect upon collective bargaining unit who are not members
a controversy, even if existing, and which, in the of the recognized collective bargaining agent
nature of things, cannot be enforced. may be assessed reasonable fees equivalent to the
dues and other fees paid by the recognized
In this case, it is the better part of conventional or collective bargaining agent, if such non-union
pragmatic solutions in cases of this nature, absent members accept the benefits under the collective
bargaining agreement;

MNI NOTES |57


When so stipulated in a collective bargaining • UNION objected to the proposal claiming
agreement or authorized in writing by the diminution of benefits. DP refused to sign the
employees concerned, the Labor Code and its CBA, resulting in a deadlock.
Implementing Rules recognize it to be the duty of
• UNION requested DP to submit the case for
the employer to deduct the sum equivalent to the
voluntary arbitration, but the latter allegedly
amount of union dues, as agency fees, from the
refused, prompting the UNION to file a case for
employees’ wages for direct remittance to the
unfair labor practice with the Labor Arbiter
union.
against DP; Eduardo Espejo, its president; and
The system is referred to as check off. No Eliseo Ocampo, Jr., chairman of the Board of
requirement of written authorization from the Trustees.
non-union employees is necessary if the
• DP denied committing unfair labor practices
nonunion employees accept the benefits resulting
against the UNION. It justified the non-
from the CBA.
deduction of the agency fees by the absence of
FACTS: individual check off authorization from the
nonunion employees.
• Respondent Union (UNION) is the certified
collective bargaining representative of teaching ISSUE:
and non-teaching personnel of Petitioner Del
1. whether or not the UNION is entitled to collect
Pilar Academy (DP), an educational institution
agency fees from non-union members; YES
operating in Imus, Cavite.
2. and if so, whether an individual written
• Sep 15, 1994 – UNION and DEL PILAR authorization is necessary for a valid check off.
entered into a CBA granting salary increase and
RULING:
other benefits to the teaching and non-teaching
Pursuant to Article 248 of the Labor Code.
staff.
• The collection of agency fees in an amount
• UNION assessed agency fees from non-union
equivalent to union dues and fees, from
employees, and requested DP to deduct said
employees who are not union members, is
assessment from the employees’ salaries and
recognized by Article 248(e) of the Labor Code.2
wages. DP refused to effect deductions claiming
that the non-union employees were not amenable o When so stipulated in a collective bargaining
to it. agreement or authorized in writing by the
employees concerned, the Labor Code and its
• Sept 1997 – UNION negotiated for the renewal
Implementing Rules recognize it to be the duty of
of the CBA. DP refused to renew unless the
the employer to deduct the sum equivalent to the
provision regarding entitlement to two (2)
amount of union dues, as agency fees, from the
months summer vacation leave with pay will be
employees’ wages for direct remittance to the
amended by limiting the same to teachers, who
union.
have rendered at least three (3) consecutive
academic years of satisfactory service. o The system is referred to as check off.

MNI NOTES |58


o No requirement of written authorization DOCTRINE: In a check-off, the employer, on
from the nonunion employees is necessary if agreement with the Union, or on prior
the non-union employees accept the benefits authorization from employees, deducts union
resulting from the CBA. dues or agency fees from the latters’ wages and
remits them directly to the union. Employee’s
• The grant of annual salary increase is not the
written consent is required for a valid reduction
only provision in the CBA that benefited the non-
or check off of attorneys fees from any amount
union employees. These provisions in the CBA
due to him.
surely benefited the non-union employees,
justifying the collection of, and the UNION’s FACTS:
entitlement to, agency fees.
• Petitioners comprise the Executive Board (EB)
o Accordingly, no requirement of written of the SolidBank Union, the duly recognized
authorization from the non-union employees is collective bargaining agent for the rank and file
needed to effect a valid check off. Article 248(e) employees of Solid Bank Corporation. Private
makes it explicit that Article 241, paragraph respondents are members of said union.
(o), requiring written authorization is
• October 1991 - the union’s EB decided to retain
inapplicable to non-union members, especially in
anew the service of Atty. Ignacio Lacsina (now
this case where the non-union employees receive
deceased) as union counsel for the negotiations
several benefits under the CBA.
for a new CBA, to which the majority of all union
o The employee’s acceptance of benefits members approved and signed a resolution
resulting from a collective bargaining agreement confirming EB’s decision
justifies the deduction of agency fees from his
• The resolution
pay and the union’s entitlement thereto.
o Provided that: 10% of the total
§ In this aspect, the legal basis of the
economic benefits that may be secured
union’s right to agency fees is neither
through the negotiations be given to Atty.
contractual nor statutory, but quasi-
Lacsina as attorneys fees
contractual, on the proscription against
unjust o Contained an authorization for
SolidBank Corporation to check-off said
By this jurisprudential yardstick, this Court finds
attorneys fees from the first lump sum
that the CA did not err in upholding the UNION’s
payment of benefits to the employees
right to collect
under the new CBA and to turn over said
agency fees.
amount to Atty. Lacsina and/or his duly
authorized representative • 21 Feb 1992 -
Evangeline Gabriel v. Sec. of Labor and the new CBA was signed.
Employment
• The bank, on request of the union, made payroll
GR No 115949
deductions for attorneys fees from the CBA
benefits paid to the union members.

MNI NOTES |59


• Respondents instituted a complaint against the ISSUE/S:
petitioners and the union counsel before the
DOLE for illegal deduction of attorneys fees as Whether or not the deduction of attorney’s fees
well as for quantification of the benefits in the by petitioners was valid
1992 CBA.
RULING:
• Petitioners moved for the dismissal of the
:
complaint citing litis pendentia, forum shopping
No, attorneys fees may not be deducted or
and failure to state a cause of action as their
checked off from any amount due to an employee
grounds.
without his written consent.
• RULING OF MED-ARBITER: Granted the
In check-off, the employer, on agreement with
complaint
the Union, or on prior authorization from
o Petitioners are directed to refund to the
employees, deducts union dues or agency fees
respondents the illegally deducted
from the latters wages and remits them directly to
amount of attorneys fees from the
the union.
package of benefits
Article 222 (b) :
• RULING OF SECRETARY OF LABOR:
"No attorneys fees, negotiation fees or similar
Partially granted
charges of any kind arising from any collective
o that the ordered refund shall be limited
bargaining negotiations or conclusions of the
to those union members who have not
collective agreement shall be imposed on any
signified their conformity to the check-
individual member of the contracting union:
off of attorney’s fees;
Provided, however, that attorneys fees may be
o the directive on the payment of 5%
charged against union funds in an amount to be
attorney’s fees should be deleted for lack
agreed upon by the parties. Any contract,
of basis
agreement or arrangement of any sort to the
contrary shall be null and void."
• Petitioners argue that the General Membership
Resolution authorizing the bank to check-off
Article 241(o):
attorneys fee from the first lump sum payment of
"Other than for mandatory activities under the
the benefits to the employees under the new CBA
Code, no
satisfies the legal requirements for such
special assessment, attorneys fees, negotiation
assessment
fees or any other extraordinary fees may be
checked off from any amount due to an employee
• Respondents claim that the check-off provision
without an individual written authorization
in question is illegal because it was never
duly signed by the employee. The
submitted for approval at a general membership
authorization should specifically state the
meeting called for the purpose and that it failed
to meet the legal formalities

MNI NOTES |60


amount, purpose and beneficiary of the No deduction can be made from the salaries of
deduction." the concerned employees other than those
mandated by law.
Requisites for the validity of the special
assessment for unions incidental expenses, Holy Cross of Davao College Inc. v.
attorneys fees and representation expenses. Jerome Joaquin GR No 110007
(1) Authorization by a written resolution
of the majority of all the members at the DOCTRINE: No provision of law makes the
general membership meeting called for employer directly liable for the payment to the
the purpose; labor organization of union dues and assessments
(2) Secretary’s record of the minutes of that the former fails to deduct from its
the meeting; employees’ salaries and wages pursuant to a
(3) Individual written authorization for check-off stipulation. The employer’s failure to
check off duly signed by the employees make the requisite deductions may constitute a
concerned violation of a contractual commitment for which
it may incur liability for unfair labor practice. But
• General Membership Resolution of the it does not by that omission, incur liability to the
SolidBank Union did not satisfy the requirements union for the aggregate of dues or assessments
laid down by law for the validity of the ten uncollected from the union members, or agency
percent (10%) special assessment for unions fees for non-union employees.
incidental expenses, attorneys fees and
The obligation to pay union dues and agency fees
representation expenses.
obviously devolves not upon the employer, but
the individual employee. It is a personal
• There were no individual written check off
obligation not demandable from the employer
authorizations by the employees concerned and
upon default or refusal of the employer to consent
so the assessment cannot be legally deducted by
to a check-off. The only bligation of the
their employer.
employer under a check-off is to effect the
deductions and remit the collections to the union.
From all the foregoing, we are of the considered
view that public respondent did not act with FACTS:
grave abuse of discretion in ruling that the
workers through their union should be made to • A CBA was entered into between petitioner
shoulder the expenses incurred for the services of Holy Cross, an educational institution, and the
a lawyer. affiliate labor organization representing its
employees, respondent KAMAPI.
And accordingly the reimbursement should be
charged to the union's general fund or account. • Holy Cross stopped deducting from the salaries
and wages of its teachers and employees the
corresponding union dues and special assessment
(payable by union members), and agency fees

MNI NOTES |61


(payable by non-members), in accordance with recognized as the proper bargaining
the checkoff clause of the CBA, prompting representatives, or on prior authorization from its
KAMAPI to demand an explanation. employees, deducts union dues or agency fees
from the latter's wages and remits them directly
• Several conciliation meetings were held to the union.
between them, and when these failed to bring
about any amicable settlement, the parties agreed When stipulated in a collective bargaining
to submit the case to voluntary arbitration. agreement, or authorized in writing by the
employees concerned, the Labor Code and its
• Holy Cross asserts that it could not comply with Implementing Rules recognize it to be the duty of
the check-off provisions because contrary to the employer to deduct sums equivalent to the
established practice, KAMAPI failed to submit to amount of union dues from the employees' wages
the college comptroller every 8th day of the for direct remittance to the union, in order to
month, a list of employees from whom union facilitate the collection of funds vital to the role
dues and the corresponding agency fees were to of the union as
be deducted.
representative of employees in a bargaining unit
• Voluntary Arbitrator required Holy Cross to if not, indeed, to its very existence.
assume liability for the union dues and
assessments, and agency fees that it had failed to No provision of law makes the employer directly
deduct from its employees salaries. liable for the payment to the labor organization
of union dues and assessments that the former
ISSUE: fails to deduct from its employees’ salaries and
wages pursuant to a check-off stipulation. The
Whether or not an employer is liable to pay to the
employer’s failure to make the requisite
union of its employees, the amounts it failed to
deductions may constitute a violation of a
deduct from their salaries — as union dues (with
contractual commitment for which it may incur
respect to union members) or agency fees (as
liability for unfair labor practice. But it does not
regards those not union members) — in
by that omission, incur liability to the union for
accordance with the check-off provisions of the
the aggregate of dues or assessments uncollected
collective bargaining contract (CBA) which it
from the union members, or agency fees for non-
claims to have been automatically extended.
union employees.
RULING:
The obligation to pay union dues and agency fees
NO. Holy Cross is not liable for allegedly failing obviously devolves not upon the employer, but
to deduct union dues, assessments and agency the individual employee. It is a personal
fees. obligation not demandable from the employer
upon default or refusal of the employer to consent
A check-off is a process or device whereby the to a check-off. The only obligation of the
employer, on agreement with the union

MNI NOTES |62


employer under a check-off is to effect the respects, the decision of the Voluntary Arbitrator
deductions and remit the collections to the union. is hereby AFFIRMED.

The principle of unjust enrichment necessarily


precludes recovery of union dues—or agency
fees—from the employer, these being obligations Verceles v. DOLE GR No 193816
pertaining to the individual worker in favor of the FACTS:
bargaining union. Where the employer fails or ● Private respondents Rodel E. Dalupan, Efren J.
refuses to implement a check-off agreement, De Ocampo, Proceso Totto, Jr., Elizabeth Alarca,
logic and prudence dictate that the union itself and Elvira S. Manalo are members of the
undertake the collection of union dues and University of the East Employees’ Association
assessments from its members (and agency fees (UEEA).
from non-union employees); this, of course,
without prejudice to suing the employer for ● On 15 September 1997, they each received a
unfair labor practice. Memorandum from the UEEA charging them
with spreading false rumors and creating
In this case, there was no basis for the Voluntary disinformation among the members of the said
Arbitrator to require Holy Cross to assume association.
liability for the union dues and assessments, and
● They were given 72 hours to reply to the
agency fees that it had failed to deduct from its
memorandum
employees’ salaries on the proffered plea that
contrary to established practice, KAMAPI had ● In their reply, the private respondents denied
failed to submit to the college comptroller every the allegations - the said allegations were vague
8th day of the month, a list of employees from and are without legal basis that no intelligent
whose pay union dues and the corresponding reply could be made of it
agency fees were to be deducted.
● Petitioners issued another memorandum
Therefore, Holy Cross is not liable for allegedly directing private respondents to reply within 72
failing to deduct union dues, assessments and hours, again
agency fees. ● Eventually, the membership of the private
respondents were suspended by Verceles, in his
WHEREFORE, the requirement imposed on
capacity as president of the association
petitioner Holy Cross by the challenged decision
of the Voluntary Arbitrator, to pay respondent ● Private respondents Dalupan et al, filed a
KAMAPI the amount equivalent to the complaint for illegal suspension willful and
uncollected union dues and agency fees from unlawful violation of UEEA constitution and by-
August 1989 up to the time a new collective laws, refusal to render financial and other reports,
bargaining agreement is concluded, is deliberate refusal to call general and special
NULLIFIED and SET ASIDE; but in all other meetings, illegal holdover of terms and damages
was filed by the respondents against herein

MNI NOTES |63


petitioners Ernesto C. Verceles, Diosdado F. WON non-holding of meetings and alleged non-
Trinidad, Salvador G. Blancia, Rosemarie De submission of reports are moot and academic -
Lumban, Felicitas Ramos, Miguel Teaño, Jaime NO
Bautista and Fidel Acero before the Department
RULING:
of Labor and Employment, National Capital
Region (DOLE-NCR).
No, the Supreme Court held that the non-
● RDO’s decision:
submission of the reports are NOT moot and
o Lift suspension academic by their ventual compliance.
o Make open and available the As found by the Court of Appeals, the financial
union’s/association/s books of accounts and statements for the years 1995 up to 1997 were
other documents pertaining to the union funds submitted to DOLE-NCR only on 06 February
and thereby explain the financial status of the 1998 while that for the year 1998 was submitted
union only on 16 March 1999. The last association’s
meeting was conducted on 21 April 1995, and the
o To regularly conduct special and general
copy of the minutes thereon was submitted to
membership meetings in accordance with the
BLR-DOLE only on 24 February 1998.
union’s constitution and by-laws;
The passage of General Assembly Resolution
o To immediately hold/conduct an election of
No. 10 dated 10 December 1997 and Resolution
officers in accordance with the union’s
No. 8, Series of 2000, which supposedly cured
constitution and by-laws.
the lapses committed by the association’s officers
● Petitioner’s appealed but the BLR-DOLE and reiterated the approval of the general
dismissed their appeal membership of the acts and collateral actions of
the association’s officers cannot redeem the
● Petitioner’s argument before the SC:
petitioners from their predicament. The
o That the non-holding of meetings and alleged obligation to hold meetings and render financial
on submission of reports are moot and academic reports is mandated by UEEA’s constitution and
by-laws. This fact was never denied by the
o The petitioners likewise maintain that the
petitioners. Their eventual compliance, as what
passage of General Assembly Resolution No. 10
happened in this case, shall not release them from
dated 10 December 1997 and Resolution No. 8,
the obligation to accomplish these things in the
Series of 2000, following the application of the
future.
principle that the sovereign majority rules, cured
any liability that may have been brought about by Prompt compliance in rendering financial reports
their belated actions. together with the holding of regular meetings
with the submission of the minutes thereon with
ISSUE:
the BLR-DOLE and DOLE-NCR shall negate
any suspicion of dishonesty on the part of
UEEA’s officers. This is not only true with

MNI NOTES |64


UEEA, but likewise with other Although both of them work in the same school,
unions/associations, as this matter is imbued with they do not belong in the same bargaining unit.
public interest. Undeniably, transparency in the
official undertakings of union officers will B as a math teacher has a different set of
bolster genuine trade unionism in the country. employee related interest as compared to A who
is a supervisor.
Third Issue: was the indefinite suspension of the
private respondents illegal? - We rule in the The same holds true if for instance, both A and B
affirmative. are both rank-and-file employees. But A works
as a site technician, while B is a security
Fourth Issue: is the non-holding of meetings and
personnel. Due to the skills required to perform
non-submission of reports by the petitioners
their respective jobs they may be treated as
moot and academic, and whether the decision to
members of different bargaining units.
hold meetings and submit reports contradict and
override the sovereign will of the majority? - We The importance of knowing whether an
do not believe so. employee belongs to a bargaining unit comes in
Prompt compliance in rendering financial reports to play when labor unions is elected and certified
together with the holding of regular meetings as their representative or bargaining agent,
with the submission of the minutes thereon with because under the Labor Code a labor union
the BLR-DOLE and DOLE-NCR shall negate may only be certified as a representative of all
any suspicion of dishonesty on the part of employees in an appropriate bargaining unit.
UEEA’s officers. This is not only true with
Thus, to determine the propriety of the Labor
UEEA, but likewise with other
Union to represent a bargaining unit, we apply
unions/associations, as this matter is imbued with
the doctrines of:
public interest. Undeniably, transparency in the
official undertakings of union officers will • Mutuality of interest;
bolster genuine trade unionism in the country. • Collective bargaining history;
• Employment status, or
2.5 3-minute Vid Bargaining Unit
• Globe doctrine
Bargaining Unit refers to a group of employees
As a member of a labor organization an employee
sharing mutual interests within a given employer
enjoys certain rights and also burdened with
unit, comprised of all or less than all of the entire
concomitant obligations, and one of the most
body of employees in the employer unit or any
important obligation of any member is that of
specific occupational or geographical grouping
paying union dues which may be collected
within such employer unit.
through a valid check off.
Ex: A and B working in Y school. A is a
But do not forget that an individual written
supervisor at the admin dept. B is a math teacher.
authorization is required before there may be a
valid wage deduction through a valid check off.

MNI NOTES |65


However, union dues should not be confused There is no correct manner in determining the
with agency fees. Agency fees, are collected propriety of a bargaining unit , no hard and fast
against non-union members, but belong to the rule as it depends on the circumstances.
bargaining unit being represented by the labor
union. These non-union members shall be subject Ideally, a bargaining unit should be comprised of
to agency fees, once they receive benefits from a employees belonging to an establishment in the
collective bargaining agreement forged by the same geographical location.
labor union.
However, in the case of ERSON ANG LEE vs.
Collection of agency fees is not subject to the SAMAHANG MANGGAGAWA NG SUPER
same requirements as a check off. LAMINATION, the SC pierced the veil of
corporate fiction and said that there is communal
2.6 Podcast interest among the rank-and-file employees of
three companies that are dispersed
Bargaining Unit - refers to a group of employees geographically. SC said that because rank and
who share the mutual interests within a given file employees had been constantly rotated in all
employer unit and they may be comprised by all three companies and the fact that they are
or less than all of the entire body of the performing the same duties whenever they are
employees in the employer unit or any specific placed in rotation, then these employees
occupational or geographical grouping within represent only one bargaining unit. They share
that employer unit. the same interests. Hence, it would be proper that
these rank-and-file employees form one
Example: Rank and file employees and bargaining unit.
Supervisory employees do not share the same
interests. DE LA SALLE UNIVERSITY and ST.
BENILDE case - SC said that the employees of
A labor union is expected to represent only one ST. BENILDE cannot form one bargaining unit
bargaining unit with the employees of De La Salle University,
regardless of whether they are forming the
MODES to determine the propriety of a
bargaining unit or representing the rank-and-file
bargaining unit:
employees. SC said that DLSU and Benilde are
o Employment Status Principle different and separate establishments, they
cannot be treated as one. Therefore, their
employees cannot form one bargaining unit.
o Bargaining History Principle
How to differentiate the ruling in the DLSU
o "Globe Doctrine" case and the Erson Ang Lee case? - In Erson
Ang case, the SC has pierced the veil of corporate
o Mutual Interests Principle fiction because there appears an unlawful and
oppressive practices being employed by the

MNI NOTES |66


establishments with respect to their employees organization fails to file the report within
which are not actually existing in the DLSU case. the period provided under the Labor
Code.
When a member of a bargaining unit becomes a
member of a labor union, the members are "Failure of the labor unions to submit the
considered the principal of the labor financial statements shall be a ground to
organization. The rights should be protected, cancel the registration of the labor union"
respected and promoted by the labor union. is not anymore allowed because the
Labor Union should not act on its own but always limitations on the grounds to cancel the
take into consideration the interests, the rights, registration of a legitimate labor union.
and the prerogatives of the members of the labor
union. The political rights of the members:

Rights of a member of a labor union provided The Right to vote: The members shall be
under the Labor Code: entitled to directly elect their officers by secret
ballot at an interval of five years. (The tenure of
The rights of the members against arbitrary, the labor union officers under the Labor Code
excessive initiation fees and fines and forfeitures shall be five years)
that may be imposed against them by the labor
union for violations of whatever policies there Mandate of officers – protect, promote and
may be provided under the by-laws and articles preserve the rights and interests of the members
of incorporation of the labor organization. of LU.

The rights of the members to be entitled to a full If a member of a labor union would want to run
and detailed reports of the financial transactions for office, the only requirement under the Labor
of the labor organization. Code would be that the person should not be
convicted of a crime involving moral turpitude
YUMANG vs. RPN9 - SC upheld the and should not be a member of any subversive
rights of the member of a labor union who organization.
was expelled from the union just because
that member demanded for the full Assessment Fees/Union Fees (subject to check
disclosure of the financial transactions of off) - a member of a labor union should not be
the labor organization. subject to arbitrary, excessive and oppressive
fees. A member of the labor union should also not
Any complaints involving the funds and be subject to special assessments or other
financial transactions of the labor union extraordinary fees, unless authorized by a written
should be filed within three years from resolution of majority of all the members during
the date that the financial statements the general membership meeting duly called for
should have been filed or from the date the purpose.
that the reports should have been
submitted , if in case the labor

MNI NOTES |67


ASSESSMENT of UNION DUES vs. written consent. Requirement for
COLLECTION of AGENCY FEES individual written consent only applies
for union dues that is subject to a check
Agency Fees - fees collected from the off under the CBA.
employees who are members of a
bargaining unit but are not members of UNION DUES - will only affect the
the labor organization that is designated members of the union and will not affect
or recognized as the bargaining the nonmembers.
representative.
The provision on collection of Agency
(No agency fees, NO CBA. No CBA, NO Fees will not apply if A did not receive
designated CEBA) any benefit from the CBA.

May be assessed against employees who The provision will also not apply if A is
are being represented by a CEBA but not a member of the bargaining unit being
unfortunately are not members of that represented by the Union C.
CEBA
The right wherein the employee, a member of
Example: the labor union, may file a complaint against
any errant members or officers of the labor
In an establishment, A is an employee union. The Labor Code provides that "any
who is not a member of any labor union. violation of the rights and conditions of
membership shall become a ground to cancel the
However, A is a member of the
union registration or the expulsion of officers
bargaining unit that is being represented
from office" is no longer applicable.
by Union C. Union C was able to
successfully negotiate a CBA which The violation should be reported by at
provides for a yearly salary increase least 30% of the members of the union, or
amongst others that benefited all the any member concerned - not a
members of the bargaining unit including mandatory requirement but a single
A. complaint should be investigated by the
DOLE, specifically by the Bureau of
Despite the fact that A is not a member of
Labor Relations.
Union C, A may be assessed agency fees
because A had benefited from the CBA
that was successfully negotiated by the
Union C.

It will be incorrect for A to refuse the


payment of agency fees by saying that
he/she did not provide for any individual

MNI NOTES |68


MODULE 3 - composed of employees in the same company.
i. COMPANY UNION
3.1 Kinds of Labor Organizations
- the formation, function or administration of
Kinds of Labor Union
which has been assisted by any act defined as
a. NATIONAL UNION or FEDERATION ULP.

- is a mother labor organization with at least 10 3.2 Registration of Labor


locals/chapters or affiliates.
Organizations; Cancellation of
b. LOCAL UNION Registration
- operating at the enterprise level.
3.2.1 Reference Materials
c. CHARTERED LOCAL
-labor organization without an independent Articles 240-249, LC
registration whose legal personality is derived
ART. 240. [234] Requirements of
from its mother union or federation upon
Registration.
issuance of a certificate of creation of chartered
local. A federation, national union or industry or trade
d. AFFLIATE union center or an independent union shall
acquire legal personality and shall be entitled to
- independent registered union attached to a the rights and privileges granted by law to
national union or federation. legitimate labor organizations upon issuance of
e. INDEPENDENT UNION the certificate of registration based on the
following requirements:
- operating at the enterprise level that acquired
legal personality through independent (a) Fifty pesos (P50.00) registration fee;
registration and is not affiliated with a national
union or federation. (b) The names of its officers, their
addresses, the principal address of the
f. INDUSTRIAL UNION labor organization, the minutes of the
- composed of workers in a particular industry. organizational meetings and the list of the
workers who participated in such
meetings;
g. CRAFT UNION
(c) In case the applicant is an independent
- composed of workers engaged in a particular union, the names of all its members
trade or occupation of a kind that requires skill comprising at least twenty percent (20%)
and training. of all the employees in the bargaining unit
where it seeks to operate;
h. COMPANY- TYPE UNION

MNI NOTES |69


(d) If the applicant union has been in All requisite documents and papers shall be
existence for one or more years, copies of certified under oath by the secretary or the
its annual financial reports; and treasurer of the organization, as the case may be,
and attested to by its president.
(e) Four copies of the constitution and by-
laws of the applicant union, minutes of its ART. 243. [236] Denial of Registration;
adoption or ratification, and the list of the Appeal. - The decision of the Labor Relations
members who participated in it. Division in the regional office denying
registration may be appealed by the applicant
ART. 241. [234-A] Chartering and Creation union to the Bureau within ten (10) days from
of a Local Chapter. A duly registered receipt of notice thereof.
federation or national union may directly create ART. 244. [237] Additional Requirements for
a local chapter by issuing a charter certificate Federations or National Unions. - Subject to
indicating the establishment of the local chapter. Article 238, if the applicant for registration is a
The chapter shall acquire legal personality only federation or a national union, it shall, in addition
for purposes of filing a petition for certification to the requirements of the preceding Articles,
election from the date it was issued a charter submit the following:
certificate.
Proof of the affiliation of at least ten (10) locals
The chapter shall be entitled to all other rights or chapters, each of which must be a duly
and privileges of a legitimate labor organization recognized collective bargaining agent in the
only upon the submission of the following establishment or industry in which it operates,
documents in addition to its charter certificate: supporting the registration of such applicant
federation or national union; and
(a) The names of the chapter's
The names and addresses of the companies
officers, their addresses, and the principal
where the locals or chapters operate and the list
office of the chapter; and
of all the members in each company involved.

(b) The chapter's constitution and by-


laws: Provided, That where the chapter's ART. 245. [238] Cancellation of Registration
constitution and by-laws are the same as that - The certificate of registration of any legitimate
of the federation or the national union, this labor organization, whether national or local,
fact shall be indicated accordingly. may be cancelled by the Bureau, after due
hearing, only on the grounds specified in Article
239 hereof.
The additional supporting requirements shall be
certified under oath by the secretary or treasurer ART. 246. [238-A] Effect of a Petition for
of the chapter and attested by its president. Cancellation of Registration - A petition for
cancellation of union registration shall not
suspend the proceedings for certification
ART. 242. [235] Action on Application. The election nor shall it prevent the filing of a petition
Bureau shall act on all applications for for certification election.
registration within thirty (30) days from filing.

MNI NOTES |70


In case of cancellation, nothing herein shall Section 1(a), (g), (i), (w), (cc), (ee), (ff),
restrict the right of the union to seek just and
(jj), (kk), (ll), (qq), (yy), (zz), (ccc),
equitable remedies in the appropriate courts.
Rule 1, Book 5 Omnibus
ART. 247. [239] Grounds for Cancellation of
Union Registration. - The following may a.) Affiliate – refers to an independent union
constitute grounds for cancellation of union affiliated with a federation, national union or a
registration: chartered local which was subsequently granted
(a) Misrepresentation, false independent registration but did not disaffiliate
statement or fraud in connection with the from its federation, reported to the Regional
adoption or ratification of the constitution Office and the Bureau in accordance with Rule
and by-laws or amendments thereto, the III, Section 6 and 7 of these Rules.
minutes of ratification, and the list of
members who took part in the ratification; g.) Cancellation Proceedings – refer to the legal
(b) process leading to the revocation of the
(c) Misrepresentation, false legitimate status of a union or workers’
statements or fraud in connection with the association.
election of officers, minutes of the election
of officers, and the list of voters; i.) Chartered Local – refers to a labor
(d) Voluntary dissolution by the
organization in the private sector operating at the
members.
enterprise level that acquired legal personality
ART. 248. [239-A] Voluntary Cancellation of through registration with the Regional Office in
Registration - The registration of a legitimate accordance with Rule III Section 2-E of these
labor organization may be cancelled by the Rules.
organization itself: Provided, that at least two-
thirds of its general membership votes, in a w.) Independent Union – refers to a labor
meeting duly called for that purpose to dissolve organization operating at the enterprise level that
the organization: Provided, further, That an required legal personality through independent
application to cancel registration is thereafter registration under Article 234 of Labor code and
submitted by the board of the organization, Rule III, Section 2-A of these Rules.
attested to by the president thereof.
cc.) Labor Organization – refers to any union or
ART. 249. [240] Equity of the Incumbent. - All association of employees in the private sector
existing federations and national unions which which exists in whole or in part for the purpose
meet the qualifications of a legitimate labor
of collective bargaining, mutual aid, interest,
organization and none of the grounds for
cooperation, protection, or other lawful purposes.
cancellation shall continue to maintain their
existing affiliates regardless of the nature of the ee.) Legitimate Labor Organization – refers to
industry and the location of the affiliates.
any labor organization in the private sector
registered or reported with the Department in

MNI NOTES |71


accordance with the Rules III and IV of these of a labor organization discharge the functions of
Rules. their office, unless a shorter period is stipulated
in the organization’s constitution and by-laws.
ff.) Legitimate Worker’s Association – refers
to an association of workers organized for mutual zz.) Union – refers to any labor organization in
aid and protection of its members or of any the private sector organized for collective
legitimate purpose other than collective bargaining and for other legitimate purposes.
bargaining registered with the Department in
accordance with Rule III, Sections 2-C and 2-D ccc.) Workers’ Association – refers to an
of these Rules. association of workers organized for the mutual
aid and protection of its members for any
jj.) Merger – refers to a process where a labor legitimate purpose other than collective
organization absorbs another resulting in the bargaining.
cessation of the absorbed labor organization’s
existence, and the continued existence of he Rule III to Rule V, Book 5, Omnibus
absorbing labor organization.

kk.) National Union or Federation – refers to a


group of legitimate labor organization for Rule XIV to XIV Book 5 Omnibus
collective bargaining or for dealing with
employers concerning terms and conditions of
employment for their member unions or for
participating in the formulation of social and 3.2.2 Required Readings
employment policies, standards, and programs,
G.R. No. L-41955 December 29, 1977
registered with the bureau in accordance with
Rule III, Section 2-Bof these rules. Elisco-Elirol Labor Union (NAFLU) and its
officers and members of The Board Of
ll.) Organized Establishment – refers to an Directors, petitioners vs. Carmelo Noriel, in
enterprise where there exists a recognized or his capacity as Director Of The Bureau Of
certified sole and exclusive bargaining agent. Labor Relations, Elizalde Steel Consolidated,
Inc. And National Federation Of Labor
qq.) Registration – refers to the process of Unions (Naflu), respondents.
determining whether the application for
FACTS:
registration of a union or workers’ association
and collective bargaining agreement complies Petitioner ELISCO-ELIROL LABOR UNION
with the documentary requirements for entered into a CBA with respondent Elizalde
registration prescribed in rules III, IV, and XVII Steel.
of these Rules. It was later discovered that the Elisco-Elirol
Labor Union was not registered with the
yy.) Term of Office – refers to the fixed period Bureau of Labor Relations of the Department
of 5 years during which the duly elected officers of Labor, and therefore not entitled to the

MNI NOTES |72


benefits and privileges embodied in the CBA. before the Bureau of Labor Relation against
Thus, the members of petitioner union later respondents Elizalde Steel and the National
decided in a resolution to register their union to Federation of Labor Unions be ordered to stop
protect and preserve the integrity of the CBA from presenting itself as the collective bargaining
between Elisco and Elizalde. agent.

By virtue of such resolution, petitioner union The members of petitioner union, who were then
applied for registration with the Bureau of Labor still affiliated with the mother union, negotiated
Relations. A Certification of Registration was and executed with respondent company a CBA.
later issued, and upon such issuance, the Later, the same members have formed
petitioner union acquired a personality separate themselves into an organization and applied for
and distinct from any other labor union. Through registration as a union.
its newly-acquired personality, petitioner
enforced its CBA with Elizalde as the principal A Certificate of Registration was later issued, and
party to the same, representing the workers through a resolution, the same members
covered by such CBA. disaffiliated from the mother union.

Sometime later, the general membership of ISSUE:


petitioner union, in a resolution, decided that Whether or not the petitioner union must be
their mother union, the National Federation of recognized as the sole and exclusive bargaining
Labor Unions, can no longer safeguard the rights representative, and not the mother
of its members and that the interests and welfare union, NAFLU. (YES)
of petitioner can be served best if it will stay
independent and disaffiliated from said mother HELD:
union. Respondent director correctly perceived in his
Resolution that "to grant to the former mother
Petitioner union, through its president, later union (NAFLU) the authority to administer and
informed respondents of said disaffiliation by enforce their collective bargaining agreement
means of a letter and subsequently requested without presumably any members in the
respondents to recognize petitioner as the sole bargaining unit is quite absurd" but fell unto the
and exclusive bargaining representatives grave error of holding that "When the employees
of the employees thereof. disaffiliated from the mother union and formed
Respondent, without justifiable reason refused themselves into a new union, their status as
to recognize petitioner union as the sole and employees was also terminated." by virtue of the
exclusive bargaining representative of its union security clause wherein it is a condition for
employees, and subsequently dismissed the continued employment in the company to
petitioner union’s officers and board members. maintain membership in the union.

A complaint for unfair labor practice was The employees and members of the local union
later filed by petitioners against respondents for did not form a new union but merely registered
the latter’s refusal to bargain collectively with the local union as was their right. Petitioner union
petitioner. Petitioner union also filed a petition was the principal party to the agreement. NAFLU

MNI NOTES |73


as the mother union merely acted as agent of the employees' interest in the existing bargaining
local union. agreement. The agent's interest never entered the
picture. In fact, the justification for said
In Liberty Cotton Mills Workers Union vs. doctrine was:
Liberty Cotton Mills, Inc., the Court expressly ... that the majority of the employees, as an entity
cited and affirmed the basic principle that "(T)he under the statute, is the true party in interest to
locals are separate and distinct units primarily the contract, holding rights through the agency
designed to secure and maintain an equality of of the union.
bargaining power between the employer and
their employee-members in the economic
G.R. No. 121241 December 10, 1997
struggle for the fruits of the joint productive
effort of labor and capital; and the association Furusawa Rubber Philippines,
of the locals into the national union (as Inc., petitioner, vs. Hon. Secretary of Labor
PAFLU) was in furtherance of the same end. And Employment And Furusawa Employees
These associations are consensual entities Union-Independent (FEU-IND), respondents.
capable of entering into such legal relations with
their members. The essential purpose was the
affirmation of the local unions into a common FACTS:
enterprise to increase by collective action the Mar. 8, 1995 - Furusawa Employees Union —
common bargaining power in respect of the Independent (FEU-IND) filed a petition for
terms and conditions of labor. Yet the locals certification election among the rank and file
remained the basic units of association, free to employees of Furusawa Rubber Philippines,
serve their own and the common interest of all, Inc. (domestic corporation engaged in the
subject to the restraints imposed by the manufacture of rubber and other related products
Constitution and By-Laws of the Association, and for export).
free also to renounce the affiliation for mutual
Apr. 3, 1995, Furusawa moved to dismiss the
welfare upon the terms laid down in the
petition for certification election on the ground
agreement which brought it into existence." that FEU-IND was not a legitimate labor
organization not having complied with all the
Corollarily, the "substitutionary" doctrine requisites of law.
likewise fully supports petitioner's stand.
Petitioner union to whom the employees owe Med-Arbiter ruled in the affirmative.
their allegiance has from the beginning expressly (photocopy of its certificate of registration
avowed that it "does not intend to change submitted by FEU-IND which has not been duly
and/or amend the provisions of the present authenticated and not supported by any other
collective bargaining agreement but only to be documentary evidence
given the chance to enforce the same since there constitutes conclusive proof that FEU-IND has
is a shift of allegiance in the majority of the acquired legitimate status and therefore entitled
to pursue its petition for certification election).
employees at respondent company."
• petitioner union is a legitimate labor
organization as evidenced by the attached
In formulating the "substitutionary" xerox copy of the certificate of
doctrine, the only consideration involved as the

MNI NOTES |74


registration, the instant petition therefore whether a photocopy of its certificate of
is hereby given due course. registration submitted by the FEU-IND which
• ordered that a certification election be has not been duly authenticated and not
conducted among the regular rank and supported by any other documentary evidence
file employees of Furusawa Rubber constitutes conclusive proof that FEU-IND has
Philippines Corporation. acquired legitimate status and therefore entitled
• The eligible voters shall be based on the to pursue its petition for certification election.
Company payroll three (3) months prior
to the filing of the petition. HELD:
• The representation Officer of the Office
is hereby directed to conduct the usual The court cannot sustain the petitioner, and
pre-election conference. agreed with the respondent - Secretary of Labor
The choices in the certification election are as and Employment that FEU-IND is a legitimate
follows: labor organization. As such, it enjoys all the
(1) Furusawa Employees Union — Independent rights and privileges recognized by law for the
(FEU-IND); and, following reasons:
(2) No union FEUIND has been issued Certificate of
Registration No. RO-400-9502-UR-003 by
FURUSAWA appealed to the Secretary of Regional Office No. 14 of the Department of
Labor but the latter affirmed the order of the Labor and Employment (DOLE) is sufficient
Med-Arbiter. proof of its legitimacy.
The presentation of the xerox copy of the
June 28, 1995 - petition for certiorari assails the certificate of registration to support its claim of
resolution of respondent Secretary of Labor and being a duly registered labor organization instead
of the submission of the original certificate is not
Employment which affirmed the order of the
a fatal defect organization conferred by its
Med Arbiter (Apr.3 1995) – allowed the registration with DOLE.
certification election to be conducted anong the The
regular rank and file EEs of Furusawa issuance of the certificate of registration
evidently shows that FEU-IND has complied
July 13, 1995 - FURUSAWA moved for with the requirements of Art. 234 of the Labor
reconsideration but the motion was again Code.
denied. The requirements for registration being
Main contention: FEU-IND is not a mandatory, they are complied with before any
legitimate labor organization so that, under the labor organization, association or group of
law, it could not file a petition for certification unions or workers acquires legal personality and
election. The basis of this argument is the failure be entitled to the rights and privileges granted by
of the petitioning union to submit an original law to legitimate labor organizations.
copy of its certificate of registration.
One of the rights of a legitimate labor
July 26, 1995 – subsequent order of Sec. of Labor organization is to represent its members in
denying Furusawa’s MR is challenged. collective bargaining agreements; also, to be
certified as the exclusive representative of all
employees in an appropriate unit for purposes of
ISSUE:
collective

MNI NOTES |75


Bargaining. legitimate labor organization for a
certification election.
Hence the petition of FEU-IND, as a legitimate
labor organization, for certification election may The issue on the legitimacy of the petitioning
rightfully be granted. union should be settled in its favor. The
FEU-IND filed a petition for certification submission of a xerox copy of the union's
election precisely to determine the will of the certificate of registration to prove its legitimacy
employees for purposes of collective bargaining. is sufficient, hence, the Med-Arbiter correctly
The only granted the petition for certification election.
exception is where the employer has to file a
petition for certification election so that it can In this regard, the factual findings of the Med-
bargain collectively as mandated by Art. 258 of Arbiter appear to be supported by substantial
the Labor Code. evidence, hence, we must accord them great
weight and respect.
In one case this court ruled that since the
petition for certification election was filed by a Under the premises, or at the very least, when
legitimate labor organization under Art. 258 conflicting interests of labor and capital are to be
of the Labor Code, the employer should not weighed on the scales of social justice, the
have involved itself in the process. heavier influence of the latter should be balanced
by sympathy and compassion which the law must
Section 3, Art. XIII, of the 1987 Constitution accord the underprivileged worker. This is only
underscores the right of the workers to organize in keeping with the constitutional mandate that
with others or to join any labor organization the
which he believes can assist and protect him in State shall afford full protection to labor.
the successful pursuit of his daily grind.
WHEREFORE, the instant petition is
this right to self organization is a fundamental DISMISSED. The assailed resolution and order
right — to give the workers the freedom to form dated 28 June 1995 and 26 July 1995,
or join any labor organization voluntarily without respectively, of respondent Secretary of Labor
fear of suppression or reprisal from management. and Employment are AFFIRMED
PD No. 828 encourages trade unionism to
supplement and strengthen the exercise of the G.R. No. 171153 September 12, 2007
workers' right to self-organization.
San Miguel Corporation Employees Union–
When FURUSAWA further argues that the Med- Philippine Transport And General Workers
Arbiter ignored the fact that FEU-IND does not Organization (SMCEU–PTGWO), petitioner,
represent at least 20% of the employees in the vs. San Miguel Packaging Products
bargaining unit which is seeks to represent, FEU- Employees Union–Pambansang Diwa Ng
IND has complied with the required 20% support Manggagawang Pilipino (SMPPEU–
signatures. Article 257 of the Labor Code PDMP), respondent
provides that in an unorganized establishment
where there is no certified bargaining agent, a Short Version: SMCEU-PTGWO challenges
certification election shall automatically be the legitimacy of SMPPEU-PDMP, a charter of
conducted by the Med-Arbiter upon filing of a PDMP, as a labor organization. The Court held
petition by a that PDMP cannot create a charter because it is

MNI NOTES |76


merely a trade union center. Trade union centers • Upon submission of its charter certificate
are not given by the Labor Code or any statute the and other documents, respondent was
power to create locals or charters therefore, issued Certificate of Creation of Local or
SMPPEU-PDMP must comply with the strict Chapter by the BLR on 6 July 1999.
requirements provided for in Art. 234, LC. • Respondent filed with the Med-Arbiter
of the DOLE Regional Officer in NCR
Nature: Review on Certiorari under Rule 45 of (DOLE-NCR), three separate petitions
the Revised Rules of Court, assailing CA for certification election to represent
decision affirming the decision of the petitioner SMPP, SMCSU, and SMBP. All three
Bureau of Labor Relations (BLR) of DOLE petitions were dismissed, on the ground
which upheld the Certificate of Registration of that the separate petitions fragmented a
respondent SAN MIGUEL PACKAGING single bargaining unit.
PRODUCTS EMPLOYEES UNION– • 17 August 1999: petitioner filed with the
PAMBANSANG DIWA NG DOLE-NCR a petition seeking the
MANGGAGAWANG PILIPINO (SMPPEU– cancellation of respondent's registration
and its dropping from the rolls of
PDMP); and its resolution denying petitioner’s
legitimate labor organizations, accusing
MR respondent of committing fraud and
Characters in the case falsification, and non-compliance with
registration requirements in obtaining its
-Petitioner(s): SMCEU-PTGWO is the certificate of registration.
incumbent bargaining agent for the bargaining o It alleged that respondent
unit comprised of the regular monthly-paid rank violated Articles 239 (a), (b) and
and file employees of the three divisions of San (c) and 234 (c) of the Labor
Miguel Corporation (SMC), namely, the San Code. Moreover, petitioner
Miguel Corporate Staff Unit (SMCSU), San claimed that PDMP is not a
Miguel Brewing Philippines (SMBP), and the legitimate labor organization, but
a trade union center, hence, it
San Miguel Packaging Products (SMPP), in all
cannot directly create a local or
offices and plants of SMC, including the Metal
chapter.
Closure and Lithography Plant in Laguna. It had
• 14 July 2000: DOLE-NCR Regional
been the certified bargaining agent for 20 years Director Maximo B. Lim issued an Order
(1987 to 1997). dismissing the allegations. He further
-Respondent(s): SMPPEU-PDMP is registered ruled that respondent is allowed to
directly create a local or chapter.
as a chapter PDMP.
However, he found that respondent did
Facts not comply with the 20% membership
requirement and, thus, ordered the
• PDMP issued a charter certificate to cancellation of its certificate of
respondent on 15 June 1999. In registration and removal from the rolls of
compliance with registration legitimate labor organizations.
requirements, respondent submitted the • Respondent appealed to the BLR who
requisite documents to the BLR for the granted the petition. The BLR ruled that
purpose of acquiring legal personality. as a chartered local union, respondent is
not required to submit the number of

MNI NOTES |77


employees and names of all its members Dispositive: Petition GRANTED. CA
comprising at least 20% of the employees REVERSED AND SET ASIDE.
in the bargaining unit where it seeks to
operate. Thus, the revocation of its Held:
registration based on non-compliance
• A legitimate labor organization is defined
with the 20% membership requirement
as "any labor organization duly registered
does not have any basis in the rules.
with the DOLE, and includes any branch
• The BLR also held that although or local thereof."
PDMP is considered as a trade union
• Why does the Labor Code demand strict
center, it is a holder of a Registration
compliance with the requirements on
Certificate issued by the BLR on 14
registration?
February 1991, which bestowed upon it
o Registration requirements are
the status of a legitimate labor
intended to afford a measure of
organization with all the rights and
protection to unsuspecting
privileges to act as representative of its
employees who may be lured into
members for purposes of collective
joining unscrupulous or fly-by-
bargaining agreement. On this basis,
night unions whose sole purpose
PDMP can charter or create a local, in
is to control union funds or use the
accordance with the provisions of
labor organization for illegitimate
Department Order No. 9.
ends.
• BLR denied petitioner’s appeal. CA o A legitimate labor organization is
affirmed BLR decision holding that entitled to specific rights under
Department Order No. 9 provides that a the Labor Code, 21 and are
registered federation or national union involved in activities directly
may directly create a local by submitting affecting matters of public
to the BLR copies of the charter interest. Legitimate labor
certificate, the local's constitution and by- organizations have exclusive
laws, the principal office address of the rights under the law which cannot
local, and the names of its officers and be exercised by non-legitimate
their addresses. Upon complying with the unions, one of which is the right
documentary requirements, the local shall to be certified as the exclusive
be issued a certificate and included in the representative of all the
roster of legitimate labor organizations. employees in an appropriate
Thus there is no need for SMPPEU to collective bargaining unit for
show a membership of 20% of the purposes of collective bargaining.
employees of the bargaining unit in order o The acquisition of rights by any
to be recognized as a legitimate labor union or labor organization,
union. particularly the right to file a
petition for certification election,
Issue: Whether or not respondent is a legitimate first and foremost, depends on
labor organization even if it failed to comply with whether or not the labor
the 20% requirement as provided in Art. 234, LC. organization has attained the
status of a legitimate labor
NO.
organization.

MNI NOTES |78


• Records show that respondent was o The Implementing Rules, as
chartered by PDMP. Article 234, LC amended by Department Order
provides that an independent labor No. 9, provide that only "a duly
organization acquires legitimacy only registered federation or national
upon its registration with the BLR. union" may directly create a local
However, the creation of a branch, or chapter.
local or chapter is treated differently. • DO 9 defines a "chartered local" as a
• Petitioners argue that PDMP is not a labor organization in the private sector
legitimate labor organization, thus cannot operating at the enterprise level that
form a charter. The Court held that the acquired legal personality through a
personality of a labor organization cannot charter certificate, issued by a duly
be attacked collaterally. It may be registered federation or national union
questioned only in an independent and reported to the Regional Office in
petition for cancellation in accordance accordance with Rule III, Section 2-E of
with Section 5 of Rule V, Book V of the these Rules (Sec. 1 (i), Rule 1, Book V of
Implementing Rules. the Implementing Rules, as amended by
DO No. 9)
• RA 9481 or "An Act Strengthening the
Here’s the twist: PDMP is a trade union center, Workers' Constitutional Right to Self-
THEREFORE IT CANNOT CREATE Organization, Amending for the
LOCALS OR CHARTERS. Purpose Presidential Decree No. 442,
As Amended, Otherwise Known as the
• “Trade union center” was never
Labor Code of the Philippines" lapsed
mentioned in the Labor Code. It first
into law on 25 May 2007 and became
appeared only in the Implementing Rules
effective on 14 June 2007. This law
of Department Order No. 9 which defined
further amends the Labor Code
a trade union center as any group of
provisions on Labor Relations, including
registered national unions or federations
trade union centers in Art. 234. However,
organized for the mutual aid and
it still makes no mention that such
protection of its members; for assisting
organizations can create a local or a
such members in collective bargaining; or
charter. [Expressio unius est exclusio
for participating in the formulation of
alterius, the expression of one thing is the
social and employment policies,
exclusion of another. Expressium facit
standards, and programs, and is duly
cessare tacitum. What is expressed puts
registered with the DOLE in accordance
an end to what is implied. Casus omissus
with Rule III, Section 2 of the
pro omisso habendus est. A person,
Implementing Rules.
object or thing omitted must have been
• While a "national union" or omitted intentionally. ]
"federation" is a labor organization o Therefore, since under the
with at least ten locals or chapters or pertinent status and applicable
affiliates, each of which must be a duly implementing rules, the power
certified or recognized collective granted to labor organizations to
bargaining agent, a trade union center, directly create a chapter or local
on the other hand, is composed of a through chartering is given to a
group of registered national unions or federation or national union, then
federations.

MNI NOTES |79


a trade union center is without This case is a resolution wherein Cirtek the
authority to charter directly. employer seeks reconsideration of a decision
WHY? To prevent circumvention of labor union wherein its MOA with the employee union had
requirements. As a legitimate labor organization been disregarded when the Sec of Labor imposed
is entitled to specific rights under the Labor Code a higher wage increase award. Cirtek also avers
and involved in activities directly affecting in this case that the Federation (FFW) does not
public interest, it is necessary that the law afford have personality to represent the union on the fact
utmost protection to the parties affected. (lori that the said union had already disaffiliated with
G.R. No. 190515 June 6, 2011 the federation. The court here held that the issue
of disaffiliation is an intra-union dispute and
Cirtek Employees Labor Union-Federation must be resolved between the parties in another
Of Free Workers Petitioner, vs. Cirtek forum and that the employer (Cirtek) does not
Electronics, Inc., Respondent. have personality to question it.
TOPIC: Registration & cancellation; c. FACTS:
Disaffiliation
SUMMARY OF PREVIOUS CASE: During a
PARTIES: bargaining deadlock between Cirtek and Cirtek
Union, the Secretary of Labor assumed
Employer: Cirtek Electronics Inc.
jurisdiction of the controversy. Before the
Federation: Cirtek Employees Labor Union- Secretary of Labor could rule on said controversy
Federation of Free workers the employer concluded with a MOA with the
remaining officers of the union on higher wage.
Representing: Cirtek Employees Labor Union
The Secretary of Labor then decided on the
controversy by awarding a wage increase higher
DOCTRINE:
than that of the MOA between the employer and
A local labor union is a separate and distinct unit union. The SC held that it was within the power
primarily designed to secure and maintain an of the Secretary of Labor to award a higher wage
equality of bargaining power between the increase even with the existence of the MOA.
employer and their employee-members. A local
• This case now resolves a motion for
union does not owe its existence to the federation
reconsideration filed by Cirtek.
with which it is affiliated. It is a separate and
• Cirtek avers:
distinct voluntary association owing its creation o that Cirtek Employees Labor
to the will of its members. The mere act of Union FFW, in filing the petition
affiliation does not divest the local union of its for certiorari under Rule 65,
own personality, neither does it give the mother availed of the wrong remedy.
federation the license to act independently of the o that the Court erred in resolving a
local union. It only gives rise to a contract of factual issue whether the
agency where the former acts in representation of Memorandum of Agreement
the latter. (MOA) was validly entered into,
which is not the office of a
RECIT-READY: petition for certiorari.

MNI NOTES |80


o that the MOA signed by the conflict between and among union members,
remaining officers of petitioner including grievances arising from any violation
Union and allegedly ratified by its of the rights and conditions of membership,
members should have been given violation of or disagreement over any provision
credence by the Court. of the unions constitution and by-laws, or
o that the Secretary of Labor cannot disputes arising from chartering or disaffiliation
insist on a ruling beyond the compromise
of the union.
agreement entered into by the parties; and that, as
early as February 5, 2010, petitioner Union had Sec 1 Rule XI of DO 40-03 provides that:
already filed with the Department of Labor and SECTION 1. Coverage. - Inter/intra-union
Employment (DOLE) a resolution of disputes shall include:
disaffiliation from the Federation of Free (a) cancellation of registration of a labor
Workers resulting in the latters lack of
organization filed by its members or by another
personality to represent the workers in the
labor organization;
present case.
(b) conduct of election of union and workers
association officers/nullification of election of
ISSUES: union and workers association officers;
(c) audit/accounts examination of union or
Whether or not since Cirtek Employees Labor
workers association funds;
Union FFW had already filed with the DOLE a
(d) deregistration of collective bargaining
resolution of disaffiliation from the Federation of
agreements;
Free Workers it lacked personality to represent
(e) validity/invalidity of union affiliation or
the workers in the present case. - NO
disaffiliation;
HELD: (f) validity/invalidity of acceptance/non-
acceptance for union membership;
ANSWER: No, because disaffiliation of the
(g) validity/invalidity of
union from FFW during the pendency of the case
impeachment/expulsion of union and workers
did not result in FFW losing its personality to
association officers and members;
represent the union. The same does not affect the
(h) validity/invalidity of voluntary recognition;
Courts upholding of the authority of the
(i) opposition to application for union and
Secretary of Labor to impose arbitral awards
CBA registration;
higher than what was supposedly agreed upon in
(j) violations of or disagreements over any
the MOA. The issue of disaffiliation bears no
provision in a union or workers association
significant legal repercussions to warrant the
constitution and by-laws;
reversal of the Courts Decision.
(k) disagreements over chartering or
The issue of disaffiliation is an intra- registration of labor organizations and collective
union dispute which must be resolved in a bargaining agreements;
different forum in an action at the instance of (l) violations of the rights and conditions of
either or both the FFW and the union or a rival union or workers association membership;
LO not the employer. (m) violations of the rights of legitimate labor
organizations, except interpretation of collective
bargaining agreements;
RULE: An intra-union dispute refers to any

MNI NOTES |81


(n) such other disputes or conflicts involving the signing of the MOA, is not for the Court, nor
the rights to self-organization, union membership for respondent to determine, but for the Union
and collective bargaining (1) between and and FFW to resolve on their own pursuant to their
among legitimate labor organizations; (2) principal-agent relationship.
between and among members of a union or
DISPOSITIVE PORTION / RULING:
workers association.
WHEREFORE, the motion for reconsideration of
IN THIS CASE:
this Courts Decision of November 15, 2010 is
• The alleged disaffiliation of the Union DENIED. SO ORDERED.
from the FFW was by virtue of a
Resolution signed on February 23, 2010
and submitted to the DOLE Laguna Field G.R. No. 127374 January 31, 2002
Office on March 5, 2010 two months after
the present petition was filed on Philippine Skylanders, Inc., Mariles C.
December 22, 2009, hence, it did not Romulo and Francisco Dakila, petitioners, vs.
affect FFW and its Legal Centers National Labor Relations Commission, Labor
standing to file the petition nor this Arbiter Emerson Tumanon, Philippine
Courts jurisdiction to resolve the same. Association Of Free Labor Unions (PAFLU)
• A local union may disaffiliate at any time September (now UNIFIED PAFLU) and
from its mother federation, absent any Serafin Ayroso, respondents.
showing that the same is prohibited under
its constitution or rule. Such, however, FACTS:
does not result in it losing its legal
personality altogether. • Nov 1993, the Philippine Skylanders
• A local labor union is a separate and Employees Association (PSEA), a local
distinct unit primarily designed to secure labor union affiliated with the Philippine
and maintain an equality of bargaining Association of Free Labor Unions
power between the employer and their (PAFLU) September (PAFLU), won in
employee-members. A local union does the certification election conducted
not owe its existence to the federation
among the rank and file employees of
with which it is affiliated. It is a separate
and distinct voluntary association owing Philippine Skylanders, Inc. (PSI). Its rival
its creation to the will of its members. The union, Philippine Skylanders Employees
mere act of affiliation does not divest the Association-WATU (PSEA-WATU)
local union of its own personality, neither immediately protested the result of the
does it give the mother federation the election before the Secretary of Labor.
license to act independently of the local
• Several months later, pending settlement
union. It only gives rise to a contract of
agency where the former acts in of the controversy, PSEA sent PAFLU a
representation of the latter. notice of disaffiliation citing as reason
PAFLU's supposed deliberate and
habitual dereliction of duty toward its
CONCLUSION: Whether then, FFW went
members. PSEA subsequently affiliated
against the will and wishes of its principal (the
member-employees) by pursuing the case despite itself with the National Congress of

MNI NOTES |82


Workers (NCW), changed its name to exclusive bargaining agent was pending
Philippine Skylanders Employees resolution before the Secretary of Labor,
Association - National Congress of PSEA could not validly separate from
Workers (PSEANCW), and allowed the PAFLU, join another national federation
former officers of PSEA-PAFLU to and subsequently enter into a collective
continue occupying their positions as bargaining agreement with its employer-
elected officers PSEA-NCW. company.
• On 17 March 1994 PSEA-NCW entered
ISSUE:
into a collective bargaining agreement
with PSI which was immediately Whether or not, PSEA, which is an independent
registered with DOLE. and separate local union, may validly disaffiliate
• PAFLU Secretary General Serafin from PAFLU pending the settlement of an
Ayroso wrote Mariles C. Romulo election protest questioning its status as the sole
requesting a copy of PSI's audited and exclusive bargaining agent of PSI's rank and
financial statement. Ayroso explained file employees. (the fundamental issue tapers
down to the legitimacy of PSEA's disaffiliation.)
that with the dismissal of PSEA-WATU's
election protestthe time was ripe for the HELD:
parties to enter into a collective
YES. The pendency of an election protest
bargaining agreement. PSI through its
involving both the mother federation and the
personnel manager Francisco Dakila local union did not constitute a bar to a valid
denied the request citing as reason disaffiliation.
PSEA's disaffiliation from PAFLU and
its subsequent affiliation with NCW. Reasoning In Liberty Cotton Mills Workers
PAFLU through Serafin Ayroso filed a Union vs. Liberty Cotton Mills, Inc. the SC
upheld the right of local unions to separate from
complaint for unfair labor practice
their mother federation on the ground that as
against PSI, its president Mariles Romulo
separate and voluntary associations, local unions
and personnel manager Francisco Dakila.
do not owe their creation and existence to the
PAFLU amended its complaint by national federation to which they are affiliated
including the elected officers of PSEA- but, instead, to the will of their members. The
PAFLU as additional party respondents. sole essence of affiliation is to increase, by
• Labor Arbiter declared PSEA's collective action, the common bargaining power
disaffiliation from PAFLU invalid and of local unions for the effective enhancement and
held PSI, PSEA-PAFLU and their protection of their interests.
respective officers guilty of unfair labor
Yet the local unions remain the basic units of
practice.
association, free to serve their own interests
• NLRC upheld the Decision of the Labor subject to the restraints imposed by the
Arbiter and conjectured that since an constitution and by-laws of the national
election protest questioning PSEA- federation, and free also to renounce the
PAFLU's certification as the sole and affiliation upon the terms laid down in the

MNI NOTES |83


agreement which brought such affiliation into over those of its members. If it were otherwise,
existence. instead of protection, there would be disregard
and neglect of the lowly workingmen.
There is nothing shown in the records nor is it
claimed by PAFLU that the local union was Disposition Petition is granted.
expressly forbidden to disaffiliate from the
WHEREFORE, the petitions of Philippine
federation nor were there any conditions imposed
Skylanders, Inc. and of Philippine Skylanders
for a valid breakaway.
and Workers Association-NCW, together with
their respective officers, are GRANTED. The
As such, the pendency of an election protest
Decision of the National Labor Relations
involving both the mother federation and the
Commission of 31 July 1996 affirming the
local union did not constitute a bar to a valid
Decision of the Labor Arbiter of 30 June 1995
disaffiliation.
holding petitioners Philippine Skylanders and
Workers Association-NCW, Philippine
Neither was it disputed by PAFLU that 111
Skylanders, Inc. and their respective officers,
signatories out of the 120 members of the local
guilty of unfair labor practice and ordering them
union, or an equivalent of 92.5% of the total
to pay damages to private respondent Philippine
union membership supported the claim of
Association of Free Labor Unions (PAFLU)
disaffiliation and had in fact disauthorized
September (now UNIFIED PAFLU) as well as
PAFLU from instituting any complaint in their
the Resolution of 31 October 1996 denying
behalf. Surely, this is not a case where one (1) or
reconsideration is REVERSED and SET ASIDE.
two (2) members of the local union decided to
No costs.
disaffiliate from the mother federation, but it is a
case where almost all local union members G.R. No. L-43495-99 January 20, 1990
decided to disaffiliate.
Tropical Hut Employees' Union-Cgw, Jose
It was entirely reasonable then for PSI to enter Encinas, et. al. petitioners, vs. Tropical Hut
into a collective bargaining agreement with Food Market, Inc., Estelita J. Que, Arturo
PSEA-NCW. As PSEA had validly severed itself Dilag, Marcelino Lontok Jr., National
from PAFLU, there would be no restrictions Association Of Trade Unions (natu), National
which could validly hinder it from subsequently Labor Relations Commission (nlrc), Hon.
affiliating with NCW and entering into a Diego Atienza, Geronimo Q. Quadra,
collective bargaining agreement in behalf of its Federico Borromeo, and Hon. Blas F.
members. Ople, respondents.

Policy considerations dictate that in FACTS:


weighing the claims of a local union as against 1. On January 2, 1968, the rank and file
those of a national federation, those of the former workers of the Tropical Hut Food Market
must be preferred. Parenthetically though, the Incorporated, organized a local union called the
desires of the mother federation to protect its Tropical Hut Employees Union (THEU) and
locals are not altogether to be shunned. It will elected their officers, and adopted their
however be to err greatly against the Constitution constitution and by-laws and immediately sought
if the desires of the federation would be favored

MNI NOTES |84


affiliation with the National Association of Trade previous union-shop security clause and the
Unions (NATU). attached check off authorization form.
2. On January 3, 1968, the NATU accepted 5. NATU received a letter dated December
the THEU application for affiliation. Following 15, 1973, jointly signed by the incumbent
such affiliation with NATU, Registration officers of the local union informing the NATU
Certificate No. 5544-IP was issued by the that THEU was disaffiliating from the NATU
Department of Labor in the name of the Tropical federation.
Hut Employees Union —NATU. It appears,
6. Secretary of the THEU, Nemesio Barro,
however, that NATU itself as a labor federation,
made an announcement in an open letter to the
was not registered with the Department of Labor.
general membership of the THEU, concerning
3. Collective Bargaining Agreement was the latter’s disaffiliation from the NATU and its
concluded between the parties on April 1, 1968, affiliation with the Confederation of General
the term of which expired on March 31, 1971. Workers (CGW). The letter was passed around
among the members of the THEU-NATU, to
Article I - Coverage and Effectivity
which around 137 signatures appeared as having
Sec. 1. The COMPANY recognizes the
given their consent to and acknowledgment of
UNION as the sole and exclusive collective
the decision to disaffiliate the THEU from the
bargaining agent for all its workers and
NATU.
employees in all matters concerning wages,
hours of work, and other terms and conditions of 7. The then so-called THEU-CGW held its
employment. annual election of officers, with Jose Encinas
elected as President. On January 3, 1974,
Article III - Union Membership and Union
Check-off Encinas, in his capacity as THEU-CGW
Sec. 1 —. . . Employees who are already President, informed the respondent company of
members of the UNION at the time of the signing the result of the elections. On January 9, 1974,
of this Agreement or who become so thereafter Pacifico Rosal, President of the Confederation of
shall be required to maintain their membership General Workers (CGW), wrote a letter in behalf
therein as a conditionof continued employment. of complainant THEU-CGWto the respondent
xxx company demanding the remittance of the union
dues collected by the Tropical Hut Food Mart,
Sec. 3—Any employee who is expelled Incorporated to the THEU-CGW, but this was
from the UNION for joining another federation refused by the respondent company.
or forming another union, or who fails or refuses
to maintain his membership therein as required, 8. A request made by the NATU federation
. . . shall, upon written request of the UNION be to the respondent company to dismiss him
discharged by the COMPANY. (Encinas) in view of his violation of Section 3 of
Article III of the Collective Bargaining
4. May 21, 1971, respondent company and Agreement.
THEU-NATU entered into a new Collective
Bargaining Agreement which ended on March 9. The respondent company applied for
31, 1974. This new CBA incorporated the clearance with the Secretary of Labor to dismiss
the other officers and members of THEU-CGW.

MNI NOTES |85


The company also suspended them effective that NLRC reversed the decision. Secretary of
day. Labor rendered a decision affirming the findings
of the Commission.
10. NLRC Case No. LR-2521 was filed by
THEU-CGW and individual complainants ISSUE:
against private respondents for unfair labor 1) Whether or not the petitioners failed to exhaust
practices. administrative remedies when they immediately
elevated the case to this Court without an appeal
• THEU-CGW asked the employees to
having been made to the Office of the President;
affirm their membership. Some did not
abidenso they were informed that they 2) Whether or not the disaffiliation of the local
will be dismissed under the CBA. union from the national federation was valid; and
• President/General Manager of respondent 3) Whether or not the dismissal of petitioner
company, upon Dilag’s request, employees resulting from their Unions’
suspended twenty four (24) workers on disaffiliation for the mother federation was
March 5, 1974, another thirty seven (37) illegal and constituted unfair labor practice on the
on March 8, 1974 and two (2) more on part of respondent company and federation
March 11, 1974, pending approval by the HELD:
Secretary of Labor of the application for
their dismissal. 1) The remedy of appeal from the Secretary of
• Labor Arbiter, Arbitrator Daniel Lucas Labor to the Office of the President is not a
issued an order dated March 21, 1974, mandatory requirement before resort to courts
can be had, but an optional relief provided by law
holding that the issues raised by the
to parties seeking expeditious disposition of their
parties became moot and academic with
labor disputes. Failure to avail of such relief shall
the issuance of NLRC Order dated not in any way served as an impediment to
February 25, 1974 in NLRC Case No. judicial intervention. And where the issue is lack
LR-2670, which directed the holding of a of power or arbitrary or improvident exercise
certification election among the rank and thereof, decisions of the Secretary of Labor may
file workers of the respondent company be questioned in a certiorari proceeding without
between the THEU-NATU and THE prior appeal to the President.
CGW.
2) The local union, being a separate and
He also ordered: voluntary association, is free to serve the interest
a) the reinstatement of all complainants; of all its members including the freedom to
b) for the respondent company to cease and desist disaffiliate when circumstances warrant. This
from committing further acts of dismissals right is consistent with the constitutional
without previous order from the NLRC and for guarantee of freedom of association.
the complainant Tropical Hut Employees All employees enjoy the right to self
UNION-CGW to file representation cases on a organization and to form and join labor
case to case basis during the freedom period organizations of their own choosing for the
provided for by the existing CBA between the purpose of collective bargaining and to engage in
parties. concerted activities for their mutual aid or
protection. This is a fundamental right of labor
that derives its existence from the Constitution.

MNI NOTES |86


The inclusion of the word NATU after the DHL Philippines v. Buklod ng mga
name of the local union THEU in the registration Manggawa ng DHL G.R. No. 152094
with the Department of Labor is merely to stress
that the THEU is NATU’s affiliate at the time of Doctrine: False statements made by union
the registration. It does not mean that the said officers before and during a certification election
local union cannot stand on its own. Neither can -- that the union is independent and not affiliated
it be interpreted to mean that it cannot pursue its with a national federation -- are material facts
own interests independently of the federation. likely to influence the election results. This
principle finds application in the present case in
A local union owes its creation and which the majority of the employees clearly
continued existence to the will of its members wanted an independent union to represent them.
and not to the federation to which it belongs. Thus, after the members learned of the
When the local union withdrew from the old misrepresentation, and after a majority of them
federation to join a new federation, it was merely disaffiliated themselves from the union and
exercising its primary right to labor organization formed another one, a new certification election
for the effective enhancement and protection of should be held to enable them to express their
common interests. In the absence of enforceable true will.
provisions in the federation’s constitution
preventing disaffiliation of a local union a local The late filing of the Petition for a new election
may sever its relationship with its parent. can be excused under the peculiar facts of this
case, considering that the employees concerned
Nothing in the constitution and by laws of did not sleep on their rights, but promptly acted
THEU NATU, prohibits the disaffiliation from to protect their prerogatives. Petitioner should not
NATU. Besides NATU is not even recognized as be permitted to use legal technicalities to
a national federation. perpetrate the betrayal foisted by its officers upon
3) When the THEU disaffiliated from its the majority of the employees. Procedural
mother federation, the former did not lose its technicalities should not be allowed to suppress
legal personality as the bargaining union under the welfare of labor.
the CBA. Moreover, the union security clause
embodied in the agreements cannot be used to FACTS:
justify the dismissals meted to petitioners since it
is not applicable to the circumstances obtaining a certification election was conducted among the
in this case. regular rank and file employees in the main office and
the regional branches of DHL Philippines
The CBA imposes dismissal only in case Corporation. The contending choices were petitioner
an employee is expelled from the union for and no union.
joining another federation or for forming another
union or who fails or refuses to maintain on the basis of the results of the certification election,
membership therein. The case at bar does not with petitioner receiving 546 votes and no union
involve the withdrawal of merely some garnering 348 votes, the election officer certified the
former as the sole and exclusive bargaining agent of
employees from the union but of the whole
the rank-and-file employees of the corporation.
THEU itself from its federation. Clearly, since
there is no violation of the union security Respondent Buklod ng Manggagawa ng DHL
provision in theCBA, there was no sufficient Philippines Corporation (BUKLOD) filed with the
ground to terminate the employment of Industrial Relations Division of the Department of
petitioners.

MNI NOTES |87


Labor and Employment (DOLE) a Petition for the A judgment is not confined to what appears on the
nullification of the certification election. face of the decision; it encompasses matters
necessarily included in or are necessary to such
• charged with committing fraud and deceit in judgment.
the election proceedings, particularly by
misrepresenting to the voter-employees that Second issue
it was an independent union, when it was in
fact an affiliate of the Federation of Free Under Section 13 of the Rules Implementing Book
Workers (FFW) V (Labor Relations) of the Labor Code, as amended,
• misrepresentation was supposedly the basis the election officer’s authority to certify the results of
for their selection of petitioner in the the election is limited to situations in which there has
certification election been no protest filed; or if there has been any, it has
not been perfected or formalized within five days
Med-Arbiter Tomas F. Falconitin nullified the from the close of the election proceedings.
November 25, 1997 certification election and
ordered the holding of another one with the Section 14 of the same Rules provides that when a
following contending choices: petitioner, respondent, protest has been perfected, only the med-arbiter can
and no choice proclaim and certify the winner. Clearly, this rule is
based on the election officers function, which is
DOLE Undersecretary Rosalinda Dimapilis- merely to conduct and supervise certification
Baldoz held on appeal that the issue of representation elections.
had already been settled with finality in favor of
petitioner, and that no petitions for certification It is the med-arbiter who is authorized to hear
election would be entertained within one year from and decide representation cases.
the time the election officer had issued the
Certification Order When the med-arbiter admitted and gave due course
to respondents Petition for nullification of the election
The CA held that the withdrawal of a great majority proceedings, the election officer should have deferred
of the members of petitioner -- 704 out of 894 of them issuing the Certification of the results thereof. Section
-- provided a compelling reason to conduct a 13 of the Implementing Rules cannot strictly be
certification election anew in order to determine, once applied to the present case.
and for all, which union reflected their choice.
The circumstances in the present case show that the
ISSUES: employees did not sleep on their rights. Hence, their
failure to follow strictly the procedural technicalities
1) the validity of the CA Decision and Resolution; and regarding the period for filing their protest should not
be taken against them. Mere technicalities should not
2) the validity of the certification election. be allowed to prevail over the welfare of the workers.

RULING: What is essential is that they be accorded an


opportunity to determine freely and intelligently
On the first issue which labor organization shall act on their behalf.

As correctly pointed out by respondent, in reversing A certification election may be set aside for
the undersecretary’s Resolution, the CA necessarily misstatements made during the campaign, where:
reinstated the med-arbiters earlier Decision to
conduct a new certification election. 1) a material fact has been misrepresented in the
campaign;

MNI NOTES |88


2) an opportunity for reply has been lacking; and THIGCI also alleged that some of the signatures in
the list of union members were secured through
3) the misrepresentation has had an impact on the free fraudulent and deceitful means, and submitted copies
choice of the employees participating in the election. of the handwritten denial and withdrawal of some of
its employees from participating in the petition.
A misrepresentation is likely to have an impact on THEU asserted that it had complied with all the
their free choice, if it comes from a party who has requirements for valid affiliation and inclusion in
special knowledge or is in an authoritative position to the roster of legitimate labor organizations pursuant
know the true facts. This principle holds true, to DOLE Department Order No. 9, series of 1997,
especially when the employees are unable to evaluate on account of which it was duly granted a
the truth or the falsity of the assertions. Certification of Affiliation by DOLE and that Section
5, Rule V of said Department Order provides that
The purpose of a certification election is precisely the legitimacy of its registration cannot be subject to
to ascertain the majority of the employees choice of collateral attack, and for as long as there is no final
an appropriate bargaining unit -- to be or not to be order of cancellation, it continues to enjoy the rights
represented by a labor organization and, in the accorded to a legitimate organization.
affirmative case, by which one.
THEU thus concluded in its Reply that under the
Once disaffiliation has been demonstrated beyond circumstances, the Med-Arbiter should, pursuant to
doubt, a certification election is the most expeditious Article 257 of the Labor Code and Section 11, Rule
way of determining which union should be the XI of DOLE Department Order No. 09,
exclusive bargaining representative of the employees. automatically order the conduct of a certification
election.
TAGAYTAY HIGHLAND’S INTERNATIONAL DOLE Med-Arbiter Anastacio Bactin ordered the
GOLF CLUB INCORPORATED, Petitioner, v. holding of a certification election among the rank-
TAGAYTAY HIGHLANDS EMPLOYEES and-file employees of THIGCI
UNION-PGTWO, Respondent.
In any event, the Med-Arbiter held that THIGCI
FACTS: failed to submit the job descriptions of the questioned
employees and other supporting documents to bolster
Tagaytay Highlands Employees Union (THEU) — its claim that they are disqualified from joining
Philippine Transport and General Workers THEU.
Organization (PTGWO), Local Chapter No. 776, a
legitimate labor organization said to represent ISSUE:
majority of the rank-and-file employees of THIGCI, CA erred IN AFFIRMING THE RESOLUTION
filed a petition for certification election before the DATED 12 NOVEMBER 1998 HOLDING THAT
DOLE Mediation-Arbitration Unit SUPERVISORY EMPLOYEES AND NON-
THIGCI, opposed THEU’s petition for certification EMPLOYEES COULD SIMPLY BE REMOVED
election on the ground that the list of union members FROM APPELLEES ROSTER OF RANK-AND-
submitted by it was defective and fatally flawed as it FILE MEMBERSHIP INSTEAD OF RESOLVING
included the names and signatures of supervisors, THE LEGITIMACY OF RESPONDENT UNION’S
resigned, terminated and absent without leave STATUS
(AWOL) employees, as well as employees of The THE COURT OF APPEALS GRIEVOUSLY
Country Club, Inc., a corporation distinct and separate ERRED IN AFFIRMING THE RESOLUTION
from THIGCI; and that out of the 192 signatories to DATED 12 NOVEMBER 1998 HOLDING THAT
the petition, only 71 were actual rank-and-file THE DISQUALIFIED EMPLOYEES’ STATUS
employees of THIGCI. COULD READILY BE RESOLVED DURING THE
INCLUSION AND EXCLUSION PROCEEDINGS

MNI NOTES |89


RULING: Sec. 5. Effect of registration. The labor
organization or workers’ association shall be
The statutory authority for the exclusion of deemed registered and vested with legal
supervisory employees in a rank-and-file union, and personality on the date of issuance of its
vice-versa, is Article 245 of the Labor Code, to wit certificate of registration. Such legal
Article 245. Ineligibility of managerial personality cannot thereafter be subject to
employees to join any labor organization; collateral attack, but may be questioned only
right of supervisory employees. — in an independent petition for cancellation in
Managerial employees are not eligible to accordance with these Rules.
join, assist or form any labor organization. The grounds for cancellation of union registration
Supervisory employees shall not be eligible are provided for under Article 239 of the Labor Code
for membership in a labor organization of the
rank-and-file employees but may join, assist (a) Misrepresentation, false statement or
or form separate labor organizations of their fraud in connection with the adoption or
own ratification of the constitution and by-laws or
amendments thereto, the minutes of
While above-quoted Article 245 expressly prohibits ratification, and the list of members who took
supervisory employees from joining a rank-and-file part in the ratification;
union, it does not provide what would be the effect if
a rank-and-file union counts supervisory employees b) Failure to submit the documents
among its members, or vice-versa mentioned in the preceding paragraph within
thirty (30) days from adoption or ratification
Citing Toyota which held that "a labor organization of the constitution and by-laws or
composed of both rank-and-file and supervisory amendments thereto;
employees is no labor organization at all,"
(c) Misrepresentation, false statements or
"Clearly, based on this provision Article 245, a labor fraud in connection with the election of
organization composed of both rank-and-file and officers, minutes of the election of officers,
supervisor employees is no labor organization at all. the list of voters, or failure to subject these
It cannot, for any guise or purpose, be a legitimate documents together with the list of the newly
labor organization. Not being one, an organization elected/appointed officers and their postal
which carries a mixture of rank-and-file and addresses within thirty (30) days from
supervisory employees cannot posses any of the election;
rights of a legitimate labor organization, including the (d) Failure to submit the annual financial
right to file a petition for certification election for the report to the Bureau within thirty (30) days
purpose of collective bargaining. after the losing of every fiscal year and
It becomes necessary, therefore, anterior to the misrepresentation, false entries or fraud in the
granting of an order allowing a certification election, preparation of the financial report itself;
to inquire into the composition of any labor (e) Acting as a labor contractor or engaging
organization whenever the status of the labor in the "cabo" system, or otherwise engaging
organization is challenged on the basis of Article 245 in any activity prohibited by law;
of the Labor Code."
f) Entering into collective bargaining
After a certificate of registration is issued to a union, agreements which provide terms and
its legal personality cannot be subject to collateral conditions of employment below minimum
attack. It may be questioned only in an independent standards established by law;
petition for cancellation in accordance with Section 5
of Rule V, Book IV of the "Rules to Implement the (g) Asking for or accepting attorney’s fees or
Labor Code" negotiation fees from employers

MNI NOTES |90


(h) Other than for mandatory activities under NUBE-PEC was certified as the sole and exclusive
this Code, checking off special assessments bargaining agent of the PNB rank-and-file employees. A
or any other fees without duly signed collective bargaining agreement (CBA) was
individual written authorizations of the subsequently signed between NUBE-PEC and PNB
members;
Pursuant to Article V on Check-off and Agency
(i) Failure to submit list of individual Fees of the CBA, PNB shall deduct the monthly
members to the Bureau once a year or membership fee and other assessments imposed
whenever required by the Bureau; and by the union from the salary of each union
(j) Failure to comply with the requirements member, and agency fee (equivalent to the
under Articles 237 and 238, monthly membership dues) from the salary of
while the procedure for cancellation of registration is the rank- and-file employees within the
provided for in Rule VIII, Book V of the bargaining unit who are not union members.
Implementing Rules.
Moreover, during the effectivity of the CBA,
The inclusion in a union of disqualified employees is NUBE, being the Federation union, agreed that
not among the grounds for cancellation, unless PNB shall remit ₱15.00 of the ₱65.00 union dues
such inclusion is due to misrepresentation, false
statement or fraud under the circumstances per month collected by PNB from every
enumerated in Sections (a) and (c) of Article 239 of employee, and that PNB shall directly credit the
above-quoted Article 239 of the Labor Code. amount to NUBE’s current account with PNB.
Following the expiration of the CBA, the Philnabank
Employees Association-FFW (PEMA-FFW) filed a
NATIONAL UNION OF BANK EMPLOYEES
(NUBE), PETITIONER, vs. PHILNABANK petition for certification election among the rank-and-
EMPLOYEES ASSOCIATION (PEMA) AND file employees of PNB. The petition sought the
PHILIPPINE NATIONAL conduct of a certification election to be participated in
BANK, RESPONDENTS. by PEMA-FFW and NUBE-PEC
FACTS: While the petition for certification election was still
pending, two significant events transpired – the
Respondent Philippine National Bank (PNB) used to
be a government-owned and controlled banking independent union registration of NUBE- PEC and its
institution established under Public Act 2612, as disaffiliation with NUBE.
amended by Executive Order No. 80
NUBE-PEC filed a Manifestation and Motion before
Its rank-and-file employees, being government the Med-Arbitration Unit of DOLE, praying that, in
personnel, were represented for collective negotiation view of its independent registration as a labor union
by the Philnabank Employees Association (PEMA), a and disaffiliation from NUBE, its name as appearing
public sector union. in the official ballots of the certification election be
the Securities and Exchange Commission approved changed to "Philnabank Employees Association
PNB’s new Articles of Incorporation and By-laws and (PEMA)" or, in the alternative, both parties be
its changed status as a private corporation. allowed to use the name "PEMA" but with PEMA-
PEMA affiliated with petitioner National Union of Bank FFW and NUBE-PEC be denominated as "PEMA-
Employees (NUBE), which is a labor federation Bustria Group" and "PEMA-Serrana Group,"
composed of unions in the banking industry, adopting respectively.
the name NUBE-PNB Employees Chapter (NUBE-
PEC). PEMA sent a letter to the PNB management
informing its disaffiliation from NUBE and

MNI NOTES |91


requesting to stop, effective immediately, the check- Thus, since only a minority of the members wanted
off of the ₱15.00 due for NUBE. disaffiliation as shown by the certification election, it
can be inferred that the majority of the members wanted
NUBE replied that: it remains as the exclusive the union to remain an affiliate of the NUBE.
bargaining representative of the PNB rank-and-
file employees; by signing the Resolution (on There being no justified disaffiliation that took place, the
disaffiliation), the chapter officers have abandoned bargaining agent’s right under the provision of the CBA
NUBE-PEC and joined another union; in abandoning on Check-Off is unaffected and still remained with the
NUBE-PEC, the chapter officers have abdicated their old NUBE-PNB Chapter.
respective positions and resigned as such; in joining While it is true that the obligation of an employee to pay
another union, the chapter officers committed an act union dues is co-terminus with his affiliation it is equally
of disloyalty to NUBE-PEC and the general tenable that when it is shown, as in this case, that the
membership; the circumstances clearly show that withdrawal from the mother union is not supported by
there is an emergency in NUBE-PEC necessitating its majority of the members, the disaffiliation is unjustified
placement under temporary trusteeship; and that PNB and the disaffiliated minority group has no authority to
should cease and desist from dealing with Serrana, represent the employees of the bargaining unit.
Roma, Latorre, Garcia, Medrano, and Magtibay, who
This is the import of the principle laid down in and the
are expelled from NUBE-PEC. inverse application of the Supreme Court decision in
ISSUE: regarding entitlement to the check-off provision of the
CBA.
W/n there was an effective disaffiliation
As a necessary consequence to our finding that no valid
RULING: disaffiliation took place, the right of NUBE to represent
its local chapter at the PNB, less those employees who
We find that indeed no effective disaffiliation took are no longer members of the latter, is beyond reproach.
place.
Prudence dictates the discontinuance of remittance of
It is well settled that [l]abor unions may disaffiliate union dues to NUBE under such circumstances was a
from their mother federations to form a local or legitimate exercise of management discretion
independent union only during the 60-day freedom apparently in order to protect the Bank’s business
period immediately preceding the expiration of the interest. The suspension of the check-off provision of
CBA. the CBA, at the instance of the latter made in good faith,
under the present circumstances cannot give rise to a
However, such disaffiliation must be effected by a
right of action.
majority of the members in the bargaining unit.
The power and freedom of a local union to disaffiliate
Further, even if for the sake of argument an attempt at from its mother union or federation is axiomatic.
disaffiliation occurred, the record is bereft of
substantial evidence to support a finding of effective As Volkschel vs. Bureau of Labor Relations [137 SCRA
disaffiliation. There might have been a mass 42] recognizes, a local union is, after all, a separate and
withdrawal of the union members from the NUBE- voluntary association that under the constitutional
PNB Chapter. The record shows, however, that only guarantee of freedom of expression is free to serve the
289 out of 3,742 members shifted their allegiance interests of its members.
from the mother union. Hence, they constituted a Such right and freedom invariably include the right to
small minority for which reason they could not have disaffiliate or declare its autonomy from the federation
successfully severed the local union’s affiliation with or mother union to which it belongs, subject to
NUBE. reasonable restrictions in the law or the federation’s
constitution.

MNI NOTES |92


The purpose of affiliation by a local union with a 3.3.1 Reference Materials
mother union [or] a federation

"x x x is to increase by collective action the bargaining Article 251-252 LC


power in respect of the terms and conditions of labor.
Yet the locals remained the basic units of association, ART. 251. [242] Rights of Legitimate Labor
free to serve their own and the common interest of all, Organizations. A legitimate labor
subject to the restraints imposed by the Constitution and organization shall have the right:
By-Laws of the Association, and free also to renounce
the affiliation for mutual welfare upon the terms laid
down in the agreement which brought it into existence." (a) To act as the representative of its
members for the purpose of collective
Thus, a local union which has affiliated itself with a
federation is free to sever such affiliation anytime and bargaining;
such disaffiliation cannot be considered disloyalty. In (b) To be certified as the exclusive
the absence of specific provisions in the federation's representative of all the employees in an
constitution prohibiting disaffiliation or the declaration appropriate bargaining unit for purposes of
of autonomy of a local union, a local may dissociate with collective bargaining;
its parent union.
(c) To be furnished by the employer, upon

Under the rules implementing the Labor Code, a written request, with its annual audited financial
chartered local union acquires legal personality through statements, including the balance sheet and the
the charter certificate issued by a duly registered profit and loss statement, within thirty (30)
federation or national union, and reported to the calendar days from the date of receipt of the
Regional Office in accordance with the rules request, after the union has been duly recognized
implementing the Labor Code. by the employer or certified as the sole and
exclusive bargaining representative of the
A local union does not owe its existence to the
federation with which it is affiliated. It is a separate and employees in the bargaining unit, or within sixty
distinct voluntary association owing its creation to the (60) calendar days before the expiration of the
will of its members. existing collective bargaining agreement, or
during the collective bargaining negotiation;
Mere affiliation does not divest the local union of its (d) To own property, real or personal, for the
own personality, neither does it give the mother
use and benefit of the labor organization and its
federation the license to act independently of the local
union.
members;
(e) To sue and be sued in its registered name;
It only gives rise to a contract of agency, where the and
former acts in representation of the latter. Hence, local (f) To undertake all other activities designed
unions are considered principals while the federation is to benefit the organization and its members,
deemed to be merely their agent.
including cooperative, housing, welfare and
As such principals, the unions are entitled to exercise the other projects not contrary to law.
rights and privileges of a legitimate labor organization,
including the right to seek certification as the sole and Notwithstanding any provision of a general or
exclusive bargaining agent in the appropriate employer special law to the contrary, the income and the
unit
properties of legitimate labor organizations,
including grants, endowments, gifts, donations
3.3 Reference Materials
and contributions they may receive from
fraternal and similar organizations, local or

MNI NOTES |93


foreign, which are actually, directly and G.R. No. L-38258 November 19, 1982
exclusively used for their lawful purposes, shall LAKAS NG MANGGAGAWANG
be free from taxes, duties and other assessments. MAKABAYAN (LAKAS), petitioner, vs.
The exemptions provided herein may be MARCELO ENTERPRISES and
withdrawn only by a special law expressly MARCELO TIRE & RUBBER CORP.,
repealing this provision. MARCELO RUBBER AND LATEX
PRODUCTS, MARCELO STEEL,
ART. 252. [242-A] Reportorial Requirements. CORPORATION, MARCELO CHEMICAL
The following are documents required to be & PIGMENT CORP., POLARIS
submitted to the Bureau by the legitimate labor MARKETING CORPORATION and THE
organization concerned: COURT OF INDUSTRIAL RELATIONS,
(a) Its constitution and by-laws, or respondents,
amendments thereto, the minutes of ratification,
FACTS:
and the list of members who took part in the
ratification of the constitution and by-laws
On May 23, 1967, the Lakas had existing CBAs
within thirty (30) days from adoption or
ratification of the constitution and by-laws or within the bargaining units in the respective
amendments thereto; companies comprising Marcelo Companies.

(b) Its list of officers, minutes of the election The said CBAs were entered into while they were
of officers, and list of voters within thirty (30) affiliated with a national federation, Phil Social
days from election; Security Labor Union.

(c)Its annual financial report within thirty Two of the CBAs were about to expire in May
(30) days after the close of every fiscal year; and and June 1967. The other one faced conflict as
there was a rival union.
(d) Its list of members at least once a year or
whenever required by the Bureau. On March 14, 1967, the management of Marcelo
Steel received a letter requesting negotiation of a
Failure to comply with the above requirements new CBA from PSSLU in behalf of UNWU.
shall not be a ground for cancellation of union There were also proposals from the unions in
registration but shall subject the erring officers Marcelo Tire and Marcelo Rubber as the existing
or members to suspension, expulsion from CBA was about to expire. Same day, the
membership, or any appropriate penalty. unionion Marcelo Tire disauthorized PSSLU as
their agent. Afterwards, the rival union submitted
its own proposals.
Rule XII, Book 5 Omnibus Another requests were received on May 3, 1967
and May 23, 1967 from two different unions.
3.3.2 Required Readings

MNI NOTES |94


As the management was confused as to which of Notices to return to work were posted and some
the union really represents the workers, the of the strikers started working again. The
president asked for the proof of authorization management required the workers to fill up forms
from the unions and they were informed of the so that they may be given a schedule. However,
conflicting claims and suggested that they file for the workers refused and insisted that they be
certification election and the decision of the court admitted without complying to the said
shall be followed and respected. requirement

PSSLU refused the suggestion of the Lakas then filed a ULP case based on the alleged
management and said that they will file ULP for fact of non readmission of striking members.
refusing to bargain with them. All of the unions
subsequently filed a Notice of Strike The trial court ruled that the Marcelo Companies
were not remiss in their obligation to bargain and
MUEWA was certified as the bargaining agent as that the strikes conducted were illegal. However,
it represents the majority of the workers in it was decided that there was ULP in not
Marcelo Tire and that there were no oppositions readmitting all the strikers.
from the other union or interested persons.
ISSUE:
Notices of Strike were withdrawn and the
management agreed to sit down in a conference 1) whether or not the complaint filed by LAKAS
for the bargaining. On the fourth conference, against the Marcelo Companies can be sustained,
Lakas declared a strike against Marcelo in view of the alleged fact that its authority to file
Companies. Acts of violence and vandalism and prosecute the same has been squarely raised
attended by picketing, the premises were in issue at the first instance before the respondent
blocked, windows of the plants were badly court; and
damaged.
(2) whether or not the Marcelo Companies are
Cases were filed against the strikers and a Return guilty of unfair labor practice, for which they
to Work order was agreed upon. Marcelo should be made liable for backwages and be
Companies resumed its operations and strikers obliged to reinstate the employees
went back to work.
RULING:
Marcelo Companies and Lakas resumed their
this Court rules in favor of the respondent
bargaining negotiations.
Marcelo Companies and consequently, the
On Oct. 13, 1967 the negotiations reached its appealed Decision is reversed
final stage. Then Lakas declared another strike
this Court has pored through the voluminuous
without filing a notice of strike resulting to
records of the case as well as after applying the
complete paralyzation of the business.
established jurisprudence and the law on the
matters raised.

MNI NOTES |95


We are not unmindful of the plight of the We find as a fact that the respondent Marcelo
employees in this case but We consider it Companies did not refuse to reinstate or re-
oppressive to grant their petition in G.R. No. employ the strikers, as a consequence of which
L38258 for not only is there no evidence which We overrule the finding of unfair labor practice
shows that the respondent Marcelo Companies against said companies based on the erroneous
were seeking for an opportunity to discharge conclusion )f the respondent court. It is clear
these employees for union activities, or to from the records that even before the
discriminate against them because of such unconditional offer to return to work contained in
activities, but there is affirmative evidence to , Exhibit "B" was made, the respondent Marcelo
establish the contrary conclusion. Companies had already posted notices for the
strikers to return back to work.
the respondent Marcelo Companies did not
ignore the demand for collective bargaining We are more impressed and are persuaded to
contained in its letter of June 20, 1967. Neither accept as true the contention of the respondent
did the companies refuse to bargain at all. What Marcelo Companies that the aforestated
it did was to apprise LAKAS of the existing requirement was only for purposes of proper
conflicting demands for recognition as the scheduling of the start of work for each returning
bargaining representative in the appropriate units striker.
involved, and suggested the settlement of the
issue by means of the filing of a petition for It must be noted that as a consequence of the two
certification election before the Court of strikes which were both attended by widespread
Industrial Relations. acts of violence and vandalism, the businesses of
the respondent companies were completely
.., in a situation like this where the issue of paralyzed.
legitimate representation in dispute is viewed for
not only by one legitimate labor organization but It would hardly be justiciable to demand of the
two or more, there is every equitable ground respondent companies to readmit all the returning
warranting the holding of a certification election. workers in one big force or as each demanded
In this way, the issue as to who is really the true readmission.
bargaining representative of all the employees
There were machines that were not in operating
may be firmly settled by the simple expedient of
condition because of long disuse during the
an election.
strikes. Some of the machines needed more than
It is the settled jurisprudence that it is an unfair one worker to operate them so that in the absence
labor practice for an employer not to reinstate, or of the needed team of workers, the start of work
refuse re-employment of members of union who by one without his teammates would necessarily
abandon their strike and make unconditional be useless, and the company would be paying for
offer to return to work. his time spent doing no work.

Finally, We take judicial cognizance of the fact


that companies whose businesses were

MNI NOTES |96


completely paralyzed by major strikes cannot union at all. A suit brought by another in
resume operations at once and in the same state representation of a real party in interest is
or force as before the strikes. defective." Under the uncontroverted facts
obtaining herein, the aforestated ruling is
We hold and rule that the requirement was an act applicable, the only difference being that, here, a
of self-preservation, designed to effect cost- labor federation seeks to represent members of a
savings as well as to insure peace and order registered local union never affiliated with it and
within their premises. members of registered local unions which, in the
course of the proceedings before the industrial
Firstly, LAKAS cannot bring any action for and
court, disaffiliated from it.
in behalf of the employees who were members of
MUEWA because, as intimated earlier in this G.R. No. 80685 March 16, 1989
Decision, the said local union was never an
affiliate of LAKAS. ALFREDO S. MARQUEZ, doing business
under the name and style of LITTLE FOLKS
What appears clearly from the records is that it SNACK MOBILE, petitioner, vs. HON.
was Augusto Carreon and his followers who SECRETARY OF LABOR AND KAISAHAN
joined LAKAS, but then Augusto Carreon was NG MANGGAGAWANG PILIPINO
not the recognized president of MUEWA and (KAMPIL-KATIPUNAN) AND IN BEHALF
OF ITS 79 MEMBERS, respondents.
neither he nor his followers can claim any
legitimate representation of MUEWA. FACTS:
Apparently, it is this split faction of MUEWA,
headed by Augusta Carreon, who is being sought Private respondent Kaisahan ng Manggagawang
to be represented by LAKAS. Pilipino (KAMPIL-KATIPUNAN) in behalf of
seventy nine (79) of its members who are
However, it cannot do so because the members employed at the Little Folks Snack Mobile
constituting this split faction of MUEWA were owned by petitioner, filed on July 16, 1986 with
still members of MUEWA which was on its own the Office of the Director of the National Capital
right a duly registered labor union. Hence, any Region, Department of Labor and Employment
suit to be brought for and in behalf of them can (DOLE) a complaint for underpayment of
minimum wage, non-payment of ECOLA, non-
be made only by MUEWA, and not LAKAS. It
payment of incentive leave benefits and non-
appearing then that Augusta Carreon and his
payment of overtime pay
cohorts did not disaffiliate from MUEWA nor
signed any individual affiliation with LAKAS, The complaint was later amended to include non-
LAKAS bears no legal interest in representing payment of holiday pay, non-payment of
MUEWA or any of its members premium pay on rest day, non-payment of
maternity leave benefits and illegal exaction
We ruled that, "(a) labor union would go beyond
After several hearings, both parties were required
the limits of its legitimate purposes if it is given to submit their respective position papers. While
the unrestrained liberty to prosecute any case the employees were able to submit a position
even for employees who are not members of any

MNI NOTES |97


paper, petitioner failed to do so. Hence, the case G.R. No. 172132, July 23, 2014
was submitted for resolution.
THE HERITAGE HOTEL MANILA,
Minerva Peran, the representative of the ACTING THROUGH ITS OWNER, GRAND
employees during the proceedings before the
PLAZA HOTEL CORPORATION,
hearing officer filed a motion to dismiss claiming
Petitioner, v. SECRETARY OF LABOR AND
that Samahan ng mga Manggagawa sa Little
EMPLOYMENT; MED-ARBITER TOMAS
Folks Snack Mobile (SAMAHAN) a local
chapter of respondent KAMPIL-KATIPUNAN, F. FALCONITIN; AND NATIONAL UNION
to which the seventy-nine (79) employees OF WORKERS IN THE HOTEL,
allegedly belong, and petitioner employer were RESTAURANT AND ALLIED
able to settle amicably their dispute through a INDUSTRIES–HERITAGE HOTEL
compromise agreement MANILA SUPERVISORS CHAPTER
(NUWHRAIN-HHMSC), Respondents
The employees opposed the motion on the
ground that Minerva Peran was not authorized to FACTS:
enter into the alleged compromise agreement and
much less to move for the dismissal of the * never-ending quest of the petitioner employer
complaint. to stop the conduct of the certification election on
The rule in this jurisdiction is that money claims the ground of the pendency of proceedings to
due to laborers cannot be the object of settlement cancel the labor organization's registration - on
or compromise effected by the union, union the ground that the membership of the labor
officers or counsel without the specific individual organization was a mixture of managerial and
consent of each laborer concerned supervisory employees with the rank-and-file
ISSUE: employees. (the pendency of the cancellation of
Who may award the money claims of the union registration brought against the labor
employees? organization applying for the certification
RULING: election should not prevent the conduct of the
In the case at bar, the various money claims of certification election)
the employees were never disputed by petitioner
during the proceedings before the Regional December 13, 2005 – CA dismissed its petition
Director and the Secretary of Labor. What was for certiorari to assail the resolutions of
sought was the reduction of petitioner's liability respondent Secretary of Labor and Employment
by entering into an amicable settlement with the sanctioning the conduct of the certification
representative of the employees who turned out election initiated by respondent labor
to be not authorized. Having failed in his attempt organization.
to reduce the claims of the employees, the ends
of justice and equity require that petitioner be not October 11, 1995, respondent National Union of
allowed to defeat the employees' right by the Workers in Hotel Restaurant and Allied
expedient of raising the issue of jurisdiction. Industries-Heritage Hotel Manila Supervisors

MNI NOTES |98


Chapter (NUWHRAIN-HHMSC) filed a petition certiorari - DOLE gravely abused its discretion in
for certification election – their OPPOSITiON – not suspending the certification election
but it was denied by Med-Arbiter Fernando on proceedings – CA dismissed for non-
Feb. 14, 1996 and issued ORDER FOR THE exhaustion of administrative remedies
CONDUCT OF THE CERTIFICATION
ELECTION; Petitioner appealed the order but Ruling of the Med-Arbiter
was denied again
January 26, 2001 - Falconitin issued an order,
Pre-Election Conference – Feb. 20, 1998, ruling that the petition for the cancellation
suspended until further notice due to non-
of union registration was not a bar to the
repeated non-appearance of UWHRAIN-
holding of the certification election.
HHMSC.
* premises considered, respondent
January 29, 2000, NUWHRAIN-HHMSC
employer/protestant’s protest with motion to
moved for the conduct of the pre-election
defer certification of results and winner is hereby
conference.
dismissed for lack of merit.
April 17, 2000 - petitioner filed a motion to
Petitioner timely appealed to the DOLE
dismiss
Secretary
raising the prolonged lack of interest of
(a) the membership of NUWHRAIN-HHMSC
NUWHRAIN-HHMSC to pursue its petition for
certification election. consisted of managerial, confidential, and rank-
and-file employees;
My 12, 2000 - petitioner filed a petition for the
cancellation of NUWHRAIN-HHMSC’s (b) NUWHRAIN-HHMSC failed to comply with
registration as a labor union for failing to submit
its annual financial reports and an updated list of the reportorial requirements; and
members as required by Article 238 and Article
239 of the Labor Code. (c) Med-Arbiter Falconitin simply brushed aside
serious questions on the illegitimacy of
Filed another motion on June 1, 2000 to seek NUWHRAINHHMSC.
either the dismissal or the suspension of the
proceedings on the basis of its pending petition Ruling of the DOLE Secretary
for the cancellation of union registration.
Augst 21, 2002, DOLE Secretary Patricia A. Sto.
June 23, 2000 - (DOLE) issued a notice Tomas issued a resolution denying the appeal,
scheduling the certification elections. and affirming the order of MedArbiter
Falconitin.
Dissatisfied, the petitioner commenced in the CA
on June 14, 2000 a special civil action for The petitioner moved for reconsideration.

MNI NOTES |99


October 21, 2002 – Motion Denied: Tagaytay Highlands proclaims Department
Order 9, that after a certificate of registration is
DOLE Secretary declared that: issued to a union, its legal personality cannot be
subject to a collateral attack
• the mixture or co-mingling of employees
in a union was not a ground for ** courts have held that a union cannot become a
dismissing a petition for the certification legitimate labor union if it shelters under its wing
election and; both types of employees. But there are elements
• appropriate remedy was to exclude the of an elliptical reasoning in the holding of these
ineligible employees from the bargaining two cases that a petition for certification election
unit during the inclusion-exclusion may not prosper until the composition of the
proceedings; union is settled therein.
• the dismissal of the petition for the
certification ** Article 245 prohibits supervisory employees
• election based on the legitimacy of the from joining a rank-and-file union, it does not
petitioning union would be inappropriate
provide what the effect is if a rank-and-fileunion
because it would effectively allow a
takes in supervisory employees as members, or
collateral attack against the union’s legal
vice versa.
personality; and
• that a collateral attack against the The fact that the cancellation proceeding has not
personality of the labor organization was yet been resolved makes it obvious that the legal
prohibited personality of the respondent union is still very
much in force – DOLE CAN STILL PROCEED
Petitioner elevated the matter to the CA by
WITH THE CERTIFICATION ELECTION
petition for certiorari.
AND NO GRAVE ABUSE OF DISCRETION
Ruling of the CA HAS BEEN COMMITTED - the right to be
certified as
Dec. 13, 2005 – CA dismissed.
collective bargaining agent is one of the
The petition for certiorari filed by the petitioner legitimate privileges of a registered union.
is, a continuation of the debate on the relevance
of the Toyota Motor, Dunlop Slazenger and ISSUES:
Progressive Development cases to the issues
• should the petition for the cancellation of
raised - anchored on the provisions of Article 245
union registration based on mixed
of the Labor Code which prohibit managerial
membership of supervisors and managers
employees from joining any labor union and
in a labor union, and the non-submission
permit supervisory employees to form a separate
of reportorial requirements to the DOLE
union of their own.
justify the suspension of the proceedings
for the certification elections or even the

MNI NOTES |100


denial of the petition for the certification Labor authorities should act with circumspection
election? - NO in treating petitions for cancellation of union
registration or be accused of interfering with
RULING: union activities.

Petition for Certiorari is denied In resolving the petition, consideration must be


taken of the fundamental rights guaranteed by
the petitioner lacked the legal personality to
Article XIII, Section 3 of the Constitution, i.e.,
assail the proceedings for the certification
the rights of all workers to self-organization,
election, and should stand aside and not oppose
the petition, or even appeal the Med-Arbiter’s collective bargaining and negotiations, and
orders relative to the conduct of the certification peaceful concerted activities.
election because the sole concern deemed an
intruder as far as the certification election is ART. 239. Grounds for Cancellation of Union
concerned. (Except when it is requested to Registration.--The following may constitute
bargain collectively) grounds for cancellation of union registration:

In that respect, only a final order for the (a) Misrepresentation, false statement or fraud in
cancellation of the registration would have connection with the adoption or ratification of
prevented NUWHRAINHHMSC from the
continuing to enjoy all the rights conferred on it
as a legitimate labor union, including the right to constitution and by-laws or amendments thereto,
the petition for the certification election. This the minutes of ratification, and the list of
rule is now enshrined in Article 238-A of the members who took part in the ratification;
Labor Code, as amended by Republic Act No.
(b) Misrepresentation, false statements or fraud
9481.
in connection with the election of officers,
Article 238-A. Effect of a Petition for minutes of the election of officers, and the list of
Cancellation of Registration. – A petition for voters;
cancellation of union registration
(c) Voluntary dissolution by the members.
shall not suspend the proceedings for
R.A. No. 9481 also inserted in the Labor Code
certification election nor shall it prevent the
filing of a petition for certification election. Article 242-A, which provides:
the Court declared that the dismissal of the ART. 242-A. Reportorial Requirements.--The
petition for the cancellation of the registration of following are documents required to be
NUWHRAIN-HHMSC was proper when viewed submitted to the Bureau by the legitimate labor
against the primordial right of the workers to self organization concerned:
organization, collective bargaining negotiations
and peaceful concerted actions.

MNI NOTES |101


(a) Its constitution and by-laws, or amendments December 13, 2005 by the Court of Appeals; and
thereto, the minutes of ratification, and the list of ORDERS the petitioner to pay the costs of suit
members who took part in the ratification of the
constitution and by-laws within thirty (30) days
from adoption or ratification of the constitution
and by-laws or amendments thereto;

(b) Its list of officers, minutesof the election of


officers, and list of voters within thirty (30) days
from election;

(c) Its annual financial report within thirty (30)


days after the close of every fiscal year; and

(d) Its list of members at least once a year or


whenever required by the Bureau.

Failure to comply with the above requirements


shall not be a ground for cancellation of union
registration but shall subject the erring officers
or members to suspension, expulsion from
membership, or any appropriate penalty.

Presently, then, the mixed membership does not


result in the illegitimacy of the registered labor
union unless the same was done through
misrepresentation, false statement or fraud
according to Article 239 of the Labor Code.

"What is important is that there is an


unmistakable intent of the members of [the]
union to exercise their right to organize. We
cannot impose rigorous restraints on such right if
we are to give meaning to the protection to labor
and social justice clauses of the Constitution."

the Court DENIES the petition for review on


certiorari; AFFIRMS the decision promulgated
on

MNI NOTES |102


MODULE 4 LO, in the establishment but they
represent different BU, then it may still be
possible that the applicant LO may still be
entitled to a SEBA Certification later on
4.1 ‘VOTE WISELY’: SELECTING A
BARGAINING REPRESENTATIVE b.) Unorganized Establishment and no CBA

4.1.1 WHO MAY VOTE -if there is a Cba, then there is a


recognized legitimate LO, or there is
4.1.2 MODES IN SELECTING A
already a recognized bargaining agent
BARGAINING AGENT/
REPRESENTATIVE c.) Majority of the employees belonging to the
bargaining unit support the certification
A. Seba Certification
-DOLE 40-I, enumerates the requirements
• Who may file
for the application
a.) any legitimate labor union
• Procedure
-not only refers to independent labor • Duties of Regional Director
union but also the chartered locals;
a.) Determine if the establishment is unorganized
because the chartered local, if the
chartered local is issued the charter -because it is a pre-requisite for the
certificate by the mother union or application and issuance of SEBA
federation, the purpose of the charter Certification
certificate is for filing a certification
election. b.) Confirm that there is only 1 legitimate labor
union, if not, call for certification/consent
-the charter certificate can also be used election
for filing a SEBA Certification
-if the RD will be able to determine that
b.) the federation or national union in the case of the establishment Is organized or there
local chapters are several LLU representing the same
BU, the RD will call for a certification
• Requirements
election or consent election. But before
a.) Only 1 legitimate labor union in the that conference muna, RD ill have to call
establishment all the representative of the LLU, and
then make them decide whether they want
-make sure that the legitimate labor union a certification election or consent election
represents a certain bargaining unit to be conducted in the establishment.
because if there are several legitimate
c.) Determine compliance with the requirements

MNI NOTES |103


d.) Schedule Validation Conference -because if there is a CBA, then it means
that the establishment is organized.
-after the RD determines that there is
compliance with the requirements or e.) Bargaining Deadlock Bar
documents
-because it means that there is a
-it will ask for the list of all the names of bargaining agent, only that a CBA has not
the employees who are members of the yet been executed because of a bargaining
BU in order to determine whether the deadlock
majority of them support the application
for SEBA Certification Once the SEBA Certification has ben issued in
favor of the LU, it will now be recognized as the
e.) Issue SEBA Certification BA, the BA can now enjoy all the rights and
privileges extended to a legitimate LO such that
• Effect of Issuance of Certification
o it can already initiate collective
*once the SEBA Certification is issued in favor bargaining with the employer,
of the applicant, the certification year bar will o assert its right to become the
apply. representative of a BU
o ask through a written request from the
a.) Employer as bystander rule applies during
employer a copy of its financial
validation conference
documents as a preliminary step in the
b.) Certified union enjoys all rights and privileges Collective Bargaining Procedure
of a legitimate labor union
Even if this is an application for SEBA, the
c.) Certification year bar applies employer as bystander rule will still apply.
Meaning that the employer cannot intervene
-Upon issuance of the SEBA during the application of the SEBA Certification,
Certification, this bar may apply by the LU, such that if the employer has
questions or challenges against the personality of
-For example, the SEBA Certification is the LU, it cannot intervene during the SEBA
issued on April 10, one year from now Certification procedure, what it can do under
there will be a bar against any applicant jurisprudence is to file a separate petition for
or any petitioner for a certification cancellation of the registration of the LU.
election. There cannot be any consent
election that may be done because of the Do not forget that the SEBA application/
1year bar or the certification year bar. certification will only be issued in favor of a
legitimate labor organization in an establishment
d.) Contract Bar that is not organized or there are no other
legitimate labor organizations representing the

MNI NOTES |104


bargaining unit that the SEBA certification In unorganized establishments, certification
applicant seeks to represent. election shall be "automatically" conducted upon
the filing of a petition for certification election by
a legitimate labor organization. However, it must
be emphasized that the petitioner-union should
B. Certification Election
have a valid certificate of registration; otherwise,
• Who may file it has no legal personality to file the petition for
certification election.
a.) any legitimate labor union, regardless if it is
independent or chartered local • Requirements

b.) the federation or national union in the case of a.) file a verified petition for certification election
local chapters
b.) if filed by employer: only one legitimate labor
c.) the employer union in the unorganized establishment, and
there is demand to collectively negotiate
-requirements:
c.) consent of at least 25% of members of
a.) the establishment is not organized bargaining unit

b.) the LU which is the sole legitimate Once the petition has been filed the Med-Arbiter
organization existing in the establishment has the following duties:
is demanding to enter into collective
bargaining with the employer a.) hear and resolve the petition

• Unorganized vs. Organized -“hearing” here means a summary hearing


Establishments based on the submissions of the parties it may
resolve the petition
The following are the requisites for certification
election in organized establishments. b.) determine the existence of a CBA, an
ongoing deadlock or certified SEBA
1. that a petition questioning the majority
status of the incumbent bargaining agent -if there has already been issued before a
is filed before the DOLE within the 60- SEBA certification, and the SEBA
day freedom period; certification is still valid because it is still
within the 1year prohibition
2. that such petition is verified; and
c.) resolve election protest
3. that the petition is supported by the
written consent of at least twenty-five d.) call a conference to determine
percent (25%) of all employees in the propriety of a consent election
bargaining unit.

MNI NOTES |105


-if in case there are several legitimate Second: majority of valid votes cast in favor of a
organizations in the establishment to union
determine whether they would want to
conduct a consent election instead of a C. Consent Election
certificate election
• Consent vs. Certification Election
e.) declare the election as valid o there is
Practically there is no difference.
failure of election
A consent election is one wherein the
• Grounds for Denial of the Med-Arbiter
representatives of the LU would come into an
a.) certified union enjoys all rights and privileges agreement that they will conduct or will hold an
of a legitimate labor union election to determine the BREP. If they do not
agree, then certification election.
b.) certification year bar applies
When it is a consent election, the representative
c.) contract bar of DOLE need not oversee the conduct of
election, as oppose to a certification election,
d.) bargaining deadlock bar they have to supervise the conduct of the election.

e.) negotiation bar • Who may vote? (in either of the two)
-ongoing negotiation a.) all members of the bargaining unit
All of these (certification, contract, bargaining, b.) probationary employees included
negotiation bar) would mean that there is already
a recognized bargaining agent for that BU, that is -there is no prohibition against
the reason why all of them would set as a bar to probationary employees from
the filing and even the conduct of a certification participating in the election
election.
-under the LC, they can exercise the right
When the bargaining agent would have to be to self-organization, even at the time of
declared or recognized as the winner in that their engagement / on day 1 they were
election, then the certified winner will enjoy all required to report for work
the rights and privileges accorded to a legitimate
labor organization. c.) separated employees who have existing
complaints for illegal dismissal during the
• Double Majority Rule conduct of the election

First: majority of members of bargaining unit -with regards to dismissed employees, they
voted; may be allowed in participating in the
election, if they contest the dismissal and the

MNI NOTES |106


complaint for illegal dismissal is still pending Did Union A win the election?
or there is no final and executory decision
with respect to the complaint for illegal Answer:
dismissal.
No, Union A did not win. Although the first
• Requirements to declare SEBA majority was satisfied, Union A did not receive
the majority votes out of the 70 votes. The
a.) double majority winning union should have received 36 votes.
Union A was one vote short.
b.) “no union” is always a choice
------
--------
D. Rerun Election
Sample Problem:
• Definition
A certification election was conducted in an
establishment comprising of 100 rank and file According to DOLE 40-I, it would refer to an
employees. A total of 75 employees cast their election conducted whenever there is a tie. Not
votes on the following available options: only referring a tie between 2 contending unions
because a tie may also be between the no union
Union A = 40 as a choice and another union.

Union B = 25 In a rerun election no union choice can be


present, but in a run-off election no union is not
Did Union A win the election?
a choice.
Answer:
Rerun election may also be conducted if there is
No. Because the “No Union” choice should a failure of elections. It will now be based on the
always be present in the consent or certification decision of the election officer or med-arbiter
election. So the election was not valid. whether a rerun election will be conducted in
case of a failure of election or just simply declare
A certification election was conducted in an a failure of election.
establishment comprising of 100 rank and file
employees. A total of 70 employees cast their • Requirements
votes on the following available options:
a.) there is a tie between 2 contending unions, or
Union A = 35 with the no union choice

Union B = 20 b.) there is a failure of elections

No Union = 15 • Procedure

MNI NOTES |107


Duties of election officer If a run-off election will be conducted in itself,
the ‘no union’ will no longer be a choice.
a.) post a notice within 5 days from the close of
the proceedings of the election It will only be between the 2 contending unions.

b.) conduct a rerun election within 10 days after Duties of the Election Officer
posting the notice.
During the election before the declaring the run-
There is a failure of election if there is: off election would be:

a.) no valid election a.) to rule on the segregated votes, if any

b.) if there is no winner in the election but b.) if segregated votes will not alter the
a run-off election cannot be conducted results of the election, post a notice of
run-off elections at least 5 days before
E. Runoff Election the actual date of run-off election

• Requirements c.) conduct the run-off election within 10


days from the close of the proceedings of
a.) 3 or more choices in the election; in the 3
the certification/consent election
choices, ‘no union’ is still part of the options
d.) declare the union garnering the
b.) No choice (No Union) receiving majority of
highest number of votes as SEBA
the valid votes cast
-if the basis of declaring the
c.) total number of votes of all contending unions
winner in a consent/certification
= at least 50% of the votes cast
election would be the double
d.) no objections or challenges which if majority, in the run-off election,
sustained, may substantially alter the result the basis of declaring a winner is
the labor union who were able to
-because if there are challenges in the gather the highest number of
elections, once the challenges have been votes, majority votes is not
decided upon or resolved, it may result in required.
the alteration of the total number of votes
cast in favor of the unions, and there may -----
be a winner already.
Sample problem:
e.) “No union” will no longer be a choice
1.)
• Procedure 100 rank and file employees; 70 employees cast
their votes;

MNI NOTES |108


Union A = 35 employees, similar also with probationary
employees. However, if the LU seeks to
Union B = 20 represent the rank-and-file employees, clearly
supervisors cannot be allowed to participate in
No union = 15
the election, because they are no longer members
Can there be a run-off election? of the BU that the labor union seeks to represent.

Answer: Is there a valid election?

Yes, because the contending unions received a A: Look at the First majority as well as the
total of 55 votes which is more than 50% of the choices. No union should always be a choice.
70 votes cast and there are no contested votes that First majority, you have to take into
may alter the results of the election. consideration the total number of the bargaining
unit or the total number of votes cast. First
2.) majority was satisfied in this case.

Total number of employees = 353 Was there a winning union?

Total votes cast = 346 A: To determine the Second majority, you have
to identify first, how many votes are cast in favor
Union A = 151 of the contending unions including no union.
Valid votes cast only, so deduct the spoiled
Union B = 169
ballots in the total votes cast as well as the
No union = 1 supervisors.

Spoiled = 3 What if Union A gathered all segregated


votes?
Segregated = 22
A: If all the segregated votes are in favor of
Dismissed employees with Union A, we will add 16 to the total number of
pending cases of illegal dismissal votes cast in favor of Union A. We will be able
= 11 to determine if we can conduct a run-off election,
or if there is already a winner in the election.
Supervisors at the time of election
=6 -------

Probationary employees = 5 F. Failure of Election

How will you rule on the segregated ballots? A failure of election, is when there are less than
majority of the eligible voters who had
A: We will allow or count the segregated ballots participated in the election. There are no
consisting of those cast by the dismissed challenged votes, if in case there are challenged

MNI NOTES |109


votes, it will alter the majority. The failure of
elections may be conducted within 6 months of
the declaration of a failure of elections. 4.1.3 Prohibitions/Bar to filing a petition or
conducting an election
Rerun/run-off election = within 10 days

There is a failure of election if there is:


4.1.4 Employer as Bystander
a.) no valid election
ART. 271. [258-A] Employer as Bystander.
b.) if there is no winner in the election but
a run-off election cannot be conducted In all cases, whether the petition for certification
election is filed by an employer or a legitimate
2 options of Election Officer: labor organization, the employer shall not be
considered a party thereto with a concomitant
If there is a failure of election, the election officer right to oppose a in such proceedings shall be
can declare it as a failure of election and then limited to:
declare a rerun election.
(1) being notified or informed of petitions
The election officer can simply declare the of such nature; and
election as a failure of election and not require
the conduct of a rerun election. (2) submitting the list of employees
during the pre-election conference should
• Requirements the Med-Arbiter act favorably on the
petition.
a.) less than a majority of the eligible voters had
voted in an election Article 238, 267-272 of the Labor Code

b.) there are no challenged votes ART. 238. [232] Prohibition on Certification
Election.
c.) shall not bar the filing for immediate holding
of certification or consent election within 6 The Bureau shall not entertain any petition for
months, except when the Election Officer/ med- certification election or any other action which
arbiter declares the conduct of a rerun election. may disturb the administration of duly registered
existing collective bargaining agreements
• Procedure affecting the parties except under Articles 253,
253-A and 256 of this Code.
Duties of election officer:
ART. 267. [255] Exclusive Bargaining
a.) declare a failure of election
Representation and Workers' Participation in
b.) direct the conduct of a rerun election Policy and Decision-Making.

MNI NOTES |110


The labor organization designated or selected by order an election by secret ballot when the
the majority of the employees in an appropriate verified petition is supported by the written
collective bargaining unit shall be the exclusive consent of at least twenty-five percent (25%) of
representative of the employees in such unit for all the employees in the bargaining unit to
the purpose of collective bargaining. However, ascertain the will of the employees in the
an individual employee or group of employees appropriate bargaining unit. To have a valid
shall have the right at any time to present election, at least a majority of all eligible voters
grievances to their employer. in the unit must have cast their votes. The labor
union receiving the majority of the valid votes
Any provision of law to the contrary cast shall be certified as the exclusive bargaining
notwithstanding, workers shall have the right, agent of all the workers in the unit. When an
subject to such rules and regulations as the election which provides for three or more choices
Secretary of Labor and Employment may results in no choice receiving a majority of the
promulgate, to participate in policy and decision- valid votes cast, a run-off election shall be
making processes of the establishment where conducted between the labor unions receiving the
they are employed insofar as said processes will two highest number of votes: Provided, That the
directly affect their rights, benefits and welfare. total number of votes for all contending unions is
For this purpose, workers and employers may at least fifty percent (50%) of the number of votes
form labor-management councils: Provided, That cast. In cases where the petition was filed by a
the representatives of the workers in such labor- national union or federation, it shall not be
management councils shall be elected by at least required to disclose the names of local chapter’s
the majority of all employees in said officers and members.
establishment.
At the expiration of the freedom period, the
ART. 268. [256] Representation Issue in employer shall continue to recognize the majority
Organized Establishments. status of the incumbent bargaining agent where
no petition for certification election is filed.
In organized establishments, when a verified
petition questioning the majority status of the ART. 269. [257] Petitions in Unorganized
incumbent bargaining agent is filed by any Establishments.
legitimate labor organization including a national
union or federation which has already issued a In any establishment where there is no certified
charter certificate to its local chapter bargaining agent, a certification election shall
participating in the certification election or a automatically be conducted by the Med-Arbiter
local chapter which has been issued a charter upon the filing of a petition by any legitimate
certificate by the national union or federation labor organization, including a national union or
before the Department of Labor and Employment federation which has already issued a charter
within the sixty (60)-day period before the certificate to its local/chapter participating in the
expiration of the collective bargaining certification election or a local/chapter which has
agreement, the Med-Arbiter shall automatically been issued a charter certificate by the national

MNI NOTES |111


union or federation. In cases where the petition Employment on the ground that the rules and
was filed by a national union or federation, it regulations or parts thereof established by the
shall not be required to disclose the names of the Secretary of Labor and Employment for the
local chapter’s officers and members. conduct of the election have been violated. Such
appeal shall be decided within fifteen (15)
ART. 270. [258] When an Employer May File calendar days.
Petition.
DOLE Department Order No. 40-I-2015
When requested to bargain collectively, an
employer may petition the Bureau for an election.
If there is no existing certified collective
bargaining agreement in the unit, the Bureau Ren Transport Corp. v. NLRC GR 188020 &
shall, after hearing, order a certification election. 188252, June 27, 2016

All certification cases shall be decided within FACTS:


twenty (20) working days.
Samahan ng Manggagawa sa Ren Transport
The Bureau shall conduct a certification election (SMART) is a registered union, which had a five-
within twenty (20) days in accordance with the year collective bargaining agreement (CBA) with
rules and regulations prescribed by the Secretary Ren Transport Corp. (Ren Transport) set to
of Labor. expire on 31 December 2004.

ART. 271. [258-A] Employer as Bystander. The 60-day freedom period of the CBA passed
without a challenge to SMART’s majority status
In all cases, whether the petition for certification as bargaining agent.
election is filed by an employer or a legitimate
labor organization, the employer shall not be SMART thereafter conveyed its willingness to
considered a party thereto with a concomitant bargain with Ren Transport, to which it sent
right to oppose a in such proceedings shall be bargaining proposals. Ren Transport, however,
limited to: (1) being notified or informed of failed to reply to the demand. Subsequently, two
petitions of such nature; and (2) submitting the members of SMART wrote to the Department of
list of employees during the pre-election Labor and Employment — National Capital
conference should the Med-Arbiter act favorably Region (DOLE-NCR).
on the petition.
The office was informed that a majority of the
ART. 272. [259] Appeal from Certification members of SMART had decided to disaffiliate
Election Orders. from their mother federation to form another
union, Ren Transport Employees Association
Any party to an election may appeal the order or (RTEA).
results of the election as determined by the Med-
Arbiter directly to the Secretary of Labor and SMART contested the alleged disaffiliation
through a letter dated 4 April 2005. During the

MNI NOTES |112


pendency of the disaffiliation dispute at the It bears stressing that Ren Transport had a duty
DOLE-NCR, Ren Transport stopped the to bargain collectively with SMART. Under
remittance to SMART of the union dues that had Article 263 in relation to Article 267 of the
been checked off from the salaries of union Labor Code, it is during the freedom period —
workers as provided under the CBA. or the last 60 days before the expiration of the
CBA — when another union may challenge
Further, on 19 April 2005, Ren Transport the majority status of the bargaining agent
voluntarily recognized RTEA as the sole and through the filing of a petition for a
exclusive bargaining agent of the rank-and-file certification election. If there is no such
employees of their company. On 6 July 2005, petition filed during the freedom period, then
SMART filed with the labor arbiter a complaint the employer “shall continue to recognize the
for unfair labor practice against Ren Transport. majority status of the incumbent bargaining
agent where no petition for certification
election is filed.”
HELD:
In the present case, the facts are not up for debate.
Ren Transport violated its duty to bargain
No petition for certification election challenging
collectively with SMART.
the majority status of SMART was filed during
Ren Transport concedes that it refused to bargain the freedom period, which was from November 1
collectively with SMART. It claims, though, that to December 31, 2004 — the 60-day period prior
the latter ceased to be the exclusive bargaining to the expiration of the five-year CBA.
agent of the rank-and-file employees because of
SMART therefore remained the exclusive
the disaffiliation of the majority of its members.
bargaining agent of the rank-and-file employees.
The argument deserves no consideration. Given that SMART continued to be the workers’
Violation of the duty to bargain collectively is exclusive bargaining agent, Ren Transport had
an unfair labor practice under Article 258 (g) the corresponding duty to bargain collectively
of the Labor Code. An instance of this practice with the former. Ren Transport’s refusal to do so
is the refusal to bargain collectively as held in constitutes an unfair labor practice.
General Milling Corp. v. CA. In that case, the
Consequently, Ren Transport cannot avail
employer anchored its refusal to bargain with and
itself of the defense that SMART no longer
recognize the union on several letters received by
represents the majority of the workers. The
the former regarding the withdrawal of the
fact that no petition for certification election was
workers’ membership from the union.
filed within the freedom period prevented Ren
We rejected the defense, saying that the employer Transport from challenging SMART’s existence
had devised a flimsy excuse by attacking the and membership.
existence of the union and the status of the
Moreover, it must be stressed that, according to
union’s membership to prevent any negotiation.
the labor arbiter, the purported disaffiliation from

MNI NOTES |113


SMART was nothing but a convenient, self- employees of the petitioner R. Transport
serving excuse. Corp.

This factual finding, having been affirmed by Med-Arbiter A. Dizon dismissed the petition on
both the CA and the NLRC, is now conclusive the ground that the bargaining unit sought to be
upon the Court. We do not see any patent error represented by respondent did not include all
that would take the instant case out of the general the eligible employees of petitioner but only the
rule. Ren Transport interfered with the exercise drivers, conductors and conductresses to the
of the employees’ right to self- organize. exclusion of the inspectors, inspectors,
Interference with the employees’ right to self- dispatchers, mechanics and washerboys.
organization is considered an unfair labor
practice under Article 258 (a) of the Labor Respondent. CLOP rectified its mistake and
Code. filed a second petition for certification election,
which included all the rank and file employees of
In this case, the labor arbiter found that the failure the company, who hold non-managerial. and
to remit the union dues to SMART and the non-supervisory positions.
voluntary recognition of RTEA were clear
indications of interference with the employees’ Med-Arbiter R. Parungo rendered a decision,
right to self-organization. It must be stressed that which ordered that a certification election
this finding was affirmed by the NLRC and the among the regular rank and file workers of the
CA; as such, it is binding on the Court, especially petitioner company be conducted.
when we consider that it is not tainted with any
The Associated Labor Unions (ALU-TUCP)
blatant error.
filed a motion for intervention and alleged that it
As aptly pointed out by the labor arbiter, these has members in the proposed bargaining unit.
acts were ill-timed in view of the existence of a Subsequently, the National Federation of
labor controversy over membership in the union. Labor Unions (NAFLU) filed a separate petition
Ren Transport also uses the supposed for certification election and a motion to
disaffiliation from SMART to justify the failure consolidate related cases to avoid confusion.
to remit union dues to the latter and the voluntary
Dissatisfied with the Decision dated July 3, 1991
recognition of RTEA.
rendered by Med-Arbiter R. Parungo, petitioner
Transport Corp v. Laguesma GR No. 106830, appealed to the DOLE Secretary, who, through
Nov. 16, 1993 Undersecretary Bienvenido E. Laguesma,
affirmed the Med-Arbiter in its Resolution dated
FACTS: July 22, 1992 calling for the conduct of the
certification election.
Respondent Christian Labor Organization of
the Philippines (CLOP), filed with the Med-
Arbitration Unit of the DOLE a petition for
certification election among the rank and file

MNI NOTES |114


Petitioner filed a Motion for Reconsideration, ISSUE:
again stressing the principle of res judicata.
Petitioner further argued that the second petition Whether the second certification election should
for a certification election by respondent CLOP, be suspended due to the pending resolution on the
NAFLU and ALU-TUCP were barred at least for question of the strike’s legality.
a period of one year from the time the first
petition of CLOP was dismissed pursuant to HELD:
Section Rule V, Book V of the Omnibus Rules
NO. As held in the case of Philippine Fruits and
Implementing the Labor Code as amended.
Vegetables Industries Inc. vs Torres:
On August 25, 1991, Undersecretary Laguesma
denied the motion for reconsideration. “At any rate, it is now well-settled that employees
who have been improperly laid-off but who have
On September 3, 1992, petitioner filed a Motion a present, unabandoned right to or expectation of
to Suspend Proceedings based on Prejudicial re-employment, are eligible to vote in
Questions as an Addendum to the Motion for certification elections (Rothenberg on Labor
Reconsideration filed on July 31, 1992. Relations, p. 548). Thus, and to repeat, if the
dismissal is under question, as in the case now at
Petitioner argued that the present case must be a bar where a case of illegal dismissal and/or
indefinitely suspended until the following cases unfair labor practices was filed, the employees
are resolved by the NLRC and the Supreme concerned could still qualify to vote in the
Court: a) NLRC-NCR Case No. 00-08-04708-91 elections.”
entitled "R". Transport Corporation v. Jose S.
Torregaza, et. al., wherein Labor Arbiter de Therefore, the employees of petitioner who
Castro declared the strike staged by respondent participated in the strike, legally remain as
CLOP illegal and ordered the strikers to pay such, until either the motion to declare their
petitioner the amount of P10,000.00 as employment status legally terminated or their
exemplary damages; b) NLRC-NCR Case No. complaint for illegal dismissal is resolved by
06-03415092 filed by respondent CLOP and its the NLRC.
members for illegal dismissal; and NLRC-NCR
Case No. 00-08-04389-92 filed by respondent It should be noted that it is the petitioner, the
CLOP in behalf of its affected members for employer, which has offered the most tenacious
illegal dismissal. resistance to the holding of a certification
election. This must not be so for the choice of a
On September 29, 1992, Undersecretary collective bargaining agent is the sole concern of
Laguesma in a resolution denied the motion to the employees. The employer has no right to
suspend the conduct of the certification election. interfere in the election and is merely
regarded as a bystander (Divine Word
University of Tacloban v. Secretary of Labor and
Employment, 213 SCRA 759 [1992]).

MNI NOTES |115


Republic Planters Bank General Services Med-Arbiter Bactin dismissed the petition for
Employees Union v. Laguesma GR No. 119675 certification election on the ground that there is
already a certified bargaining agent representing
FACTS: the appropriate bargaining unit within Republic
Planters Bank.
Petitioner (RPBEU) filed a petition for
certification election to determine the sole and Undersecretary Laguesma denied petitioner’s
exclusive bargaining representative of all regular Motion for Reconsideration.
employees outside the bargaining unit of
Republic Planters Bank. The proposed ISSUE:
bargaining unit is composed of clerks,
messengers, janitors, plumbers, telex operators, Whether USEC Laguesma committed grave
mailing and printing personnel, drivers, abuse of discretion when it allowed Republic
mechanics and computer personnel. Planters Bank to participate or intervene in the
certification election.
Allegedly, these employees are regular
employees but are considered contractual HELD:
employees by Republic Planters Bank. They are
NO. A petition for certification election may be
excluded from the existing collective bargaining
entertained if filed outside the sixty-day period
agreement between RPBEU, the duly certified
immediately before the expiration of the
bargaining representative of the regular
collective bargaining agreement.
employees of private respondent.
The purpose of the prohibition against the
Republic Planters Bank moved to dismiss the
filing of a petition for certification election
petition for certification election. Firstly, it
outside the so-called freedom period is to
contended that the petitioner union is composed
ensure industrial peace between the employer
of some thirty (30) employees of Superior
and its employees during the existence of the
Maintenance Services, Inc, (SMSI) who are
CBA. In this case, the petition for certification
assigned to the bank as messengers and janitors
election was filed on Jan. 21, 1991. The CBA
under a Contract of Services. The other
between the duly certified bargaining agent,
employees in the proposed bargaining unit are
Republic Planters Bank Employees Union, and
employed on “contractual basis” and are not
Republic Planters Bank was effective from June
members of the petitioner. Secondly, it stressed
30, 1988 to June 30, 1991. It is crystal clear that
the existence of a bargaining unit represented by
the filing of the petition for certification election
RPBEU. Thirdly, it alleged that the petition
was premature.
failed to state the number of employees in the
proposed bargaining unit and there is no prior In Singer Sewing Machine Company vs. Drilon,
determination that the members of petitioner are et al., where we ruled that if the union members
employees of private respondent. are not employees, no right to organize for
purposes of bargaining, nor to be certified as

MNI NOTES |116


bargaining agent can be recognized. Since the clear mandate of the aforequoted section is that a
persons involved are not employees of the petition for certification election may be filed at
company, we held that they are not entitled to the any time, in the absence of a CBA. Otherwise
constitutional right to join or form a labor put, the rule prohibits the filing of a petition for
organization for purposes of collective certification election in the following cases:
bargaining. Singer reiterated our earlier
pronouncement in La Suerte Cigar and Cigarette 1. During the existence of a CBA except
Factory v. Director of Labor Relations (123 w/in the freedom period;
SCRA 679 [1983]), thus:
2. w/in one year from the date of issuance of
The question of whether employer- declaration of a final certification election
employee relationship exists is a result; or
primordial consideration before
3. during the existence of a bargaining
extending labor benefits under the
deadlock to which had been submitted to
workmen's compensation, social security,
conciliation or arbitration or had become
medicare, termination pay and labor
the subject of a valid notice of strike or
relations law. It is important in the
lockout
determination of who shall be included in
the proposed bargaining unit because, it is The Deadlock Bar Rule simply provides that a
the sine qua non, the fundamental and petition for certification election can only be
essential condition that a bargaining unit entertained if there is no pending bargaining
be composed of employees. Failure to deadlock submitted to conciliation or arbitration
establish this juridical relationship or had become the subject of a valid notice of
between the union members and the strike or lockout. The principal purpose is to
employer affects the legality of the union ensure stability in the relationship of the workers
itself. It means the ineligibility of the and the management.
union members to present a petition for
certification election as well as to vote FACTS:
therein. Petitioner National Congress of Unions in the
Sugar Industry of the Philippines (NACUSIP)-
National Congress of Union in the Sugar
TUCP is the certified exclusive bargaining
Industry of the Phils. (NCUSIP)-TUCP v.
representative of the rank and file workers of
Trajano GR L-67485 April 10, 1992
Calinog Refinery Corporation.
DOCTRINE: Certification Election; A
representation may not be entertained if a Private respondent Federation of Unions of Rizal
bargaining deadlock to which an incumbent or (FUR)-TUCP is a labor organization duly
certified bargaining agent is a party had been registered with the Department of Labor and
submitted to conciliation or arbitration before Employment while private respondent Calinog
filing of a petition for certification election – The Refineries Employees Union (CREU)-

MNI NOTES |117


NACUSIP is the certified exclusive bargaining Office VI, Iloilo City for hearing and reception of
representative of the rank and file workers of the evidence.
private respondent Calinog Refinery Corporation
by virtue of the certification election held on ISSUE:
March 30, 1981. WON a petition for certification election may be
filed during the pendency of a bargaining
On June 21, 1982, petitioner union filed a petition deadlock submitted to arbitration or conciliation
for deadlock in collective bargaining with the
Ministry of Labor and Employment (now HELD:
Department of Labor and Employment).
NO. The law on the matter is Section 3, Book V,
Rule V of the Omnibus Rules Implementing the
On July 21, 1982, private respondent FUR-TUCP
Labor Code, to wit:
filed with the Regional Office No. VI, MOLE
(now DOLE), Iloilo City a petition for Sec. 3. When to file. — In the
certification election among the rank and file absence of a collective bargaining
employees of private respondent company, agreement duly registered in
alleging that: (1) about forty-five percent (45%) accordance with Article 231 of
of private respondent company's employees had the Code, a petition for
disaffiliated from petitioner union and joined certification election may be filed
private respondent union; (2) no election had at any time. However, no
been held for the past twelve (12) months; and (3) certification election may be held
while petitioner union had been certified as the within one year from the date of
sole collective bargaining agent, for over a year issuance of a final certification
it failed to conclude a collective bargaining election result. Neither may a
agreement with private respondent company. representation question be
entertained if, before the filing of
Petitioner union filed a motion to intervene in the a petition for certification
petition for certification election filed by private election, a bargaining deadlock to
respondent union. which an incumbent or certified
bargaining agent is a party had
Med-Arbiter ruled in favor of Petitioner
been submitted to conciliation or
dismissing the petition for certification for lack
arbitration or had become the
of merit since the petition is barred by bargaining
subject of valid notice or strike or
deadlock.
lockout.
The Bureau of Labor Relations through
If a collective bargaining
respondent Director Cresenciano B. Trajano
agreement has been duly
rendered a setting aside the order of the Acting
registered in accordance with
Med-Arbiter and remanding the case to Regional
Article 231 of the Code, a petition

MNI NOTES |118


for certification election or a alleged that it had 179 rank and file employees,
motion for intervention can only none of whom voted in the certification election.
be entertained within sixty (60)
days prior to the expiry date of St. James argued that those who voted were not
such agreement. its regular employees but construction workers of
In the case at bar, a bargaining deadlock was an independent contractor, Architect Conrado
already submitted to arbitration when private Bacoy ("Architect Bacoy"). Med-Arbiter Tomas
respondent FUR-TUCP filed a petition for F. Falconitin ("Med-Arbiter Falconitin") ruled
certification election. The same petition was that at the time of the certification election, the
dismissed for lack of merit by the Acting Med- 84 voters were no longer working at St. James.
Arbiter in an order dated July 23, 1982 on the sole
Med-Arbiter Falconitin supported his ruling
ground that the petition is barred by a pending
using the roster of rank-and-file employees
bargaining deadlock. However, the Respondent
submitted by St. James, which did not include the
Director set aside the same order and
names of the 84 voters. Med-Arbiter Falconitin
subsequently affirmed an order giving due course
also ruled that since the construction projects
to the petition for certification election and
have ceased, some of the workers were no longer
ordering that an election be held.
entitled to vote in the certification election.
The law demands that the petition for
certification election should fail in the presence Finally, Med-Arbiter Falconitin ruled that even if
of a then pending bargaining deadlock. the 84 workers were to be included in the 179
rank and file employees of St. James, the total
James School of Quezon Cityv. Samahang
number of voters would be 263. Thus, the 84
Manggagawa sa St. James School of Quezon
votes cast would not be sufficient to constitute a
City GR 151326 Nov. 23,2005
majority of all eligible voters to have a valid
FACTS: certification election.

The Samahang Manggagawa sa St. James School


of Quezon City ("Samahang Manggagawa") filed Samahang Manggagawa appealed to the
a petition for certification election to determine Secretary of Labor. According to the DOLE,
the collective bargaining representative of the Med-Arbiter Falconitin erred in including all the
motor pool, construction and transportation rank-and-file employees of St. James, whether
employees of St. James School of Quezon City teaching or non-teaching personnel, in the
("St. James"). computation of the total number of employees.
St. James filed a motion for reconsideration.
On 26 June 1999, the certification election was
held at the DOLE office in Intramuros, Manila. The DOLE denied the motion and St.
There were 149 eligible voters and 84 voters cast James filed a special civil action before the Court
their votes. St. James filed a certification election of Appeals. Court of Appeals dismissed the
protest challenging the 84 votes. St. James petition and ruled that the DOLE did not commit
grave abuse of discretion in reversing the ruling

MNI NOTES |119


of Med-Arbiter Falconitin. Hence, the petition Oriental Tin Can & Metal Sheet v. Laguesma
before the Court. GR 116779 Aug. 28, 1998

ISSUE: FACTS:

Whether or not there is quorum to warrant the On March 3, 1994, Oriental Tin Can and Metal
validity of the certification election Sheet Manufacturing Company, Inc. entered into
a collective bargaining agreement (CBA) with
RULING: petitioner Oriental Tin Can Labor Union
(OTCLU) as the existing CBA was due to expire
YES, the certification election was valid.
on April 15, 1994.
Section 2, Rule XII, Book V of the
Four days later, 248 of the company's rank-and-
Omnibus Rules provides:
file employees authorized the Federation of Free
“Section 2. Qualification of voters; Workers (FFW) to file a petition for certification
inclusion-exclusion proceedings. - All election. On March 10, 1994, however, this
employees who are members of the appropriate petition was repudiated via a written waiver by
bargaining unit sought to be represented by the 115 of the signatories, who, along with other
petitioner at the time of the certification or employees totalling 897, ratified the CBA on the
consent election shall be qualified to vote. A same date. respondent Oriental Tin Can Workers
dismissed employee whose dismissal is being Union - Federation of Free Workers (OTCWU-
contested in a pending case shall be allowed to FFW) filed a petition for certification election
vote in the election.” with the National Capital Region office of the
Department of Labor and Employment (DOLE).
In case of disagreement over the voters'
list or over the eligibility of voters, all contested Purporting to represent the regular rank-and-file
voters shall be allowed to vote. However, their employees of the company, the petition was
votes shall be segregated and sealed in individual accompanied by the "authentic signatures" of
envelopes in accordance with Section 9 of these 25% of the employees/workers in the bargaining
Rules. unit.

The motor pool, construction, and The OTCLU filed a manifestation and motio,
transportation employees of the Tandang Sora praying for the dismissal of the petition for
campus had 149 qualified voters at the time of the certification election on the ground that it was not
certification election. Hence, the 149 qualified endorsed by at least 25% of the employees of the
voters should be used to determine the existence bargaining unit.
of a quorum. Since a majority or 84 out of the 149
Some of the employees who initially signed the
qualified voters cast their votes, a quorum existed
petition had allegedly withdrawn in writing such
in the certification election.
support prior to the filing of the same. OTCWU-
FFW asserted that the petition had the required

MNI NOTES |120


support of more than 25% of all the employees in Med-Arbiter dismissed PCE and noted
the bargaining unit. the petition was filed after valid retractions were
made (115, thus the remaining 133 were clearly
The company then filed a comment less than 25% subscription requirement).
alleging that the new CBA was ratified by 897 OTCWU-FFW appealed to Labor Secretary, then
out of 1,020 employees in the bargaining unit. Undersecretary Laguesma (July 15, 1994
OTCLU argued that the petition did not garner resolution), issued a resolution ordering a
25% requirement and CBA was a bar to a certification election: he held that the petition
certification election. was filed during the freedom period and no
registered CBA with the company could be
OTCWU-FFW contended that the CBA was a
invoked to pose as a bar to the holding of a
sweetheart contract and concluded within
certification election and that the best forum for
freedom period, and that additional employees
the determination of whether there were
gave their support to the petition after the same
retractions is the certification election itself
had been filed. Company argued that the CBA
where workers can freely express their choice in
was ratified by 897 employees including the 245
a secret ballot.
workers who gave their consent to the filing of
the petition and that the benefits provided for are ISSUE:
being enjoyed by the workers—thus a
certification election would impair contract. Whether or not the 25% support requirement was
met
They also allege that officers of the OTCWU-
FFW were among those who ratified the CBA. RULING:
OTCWU failed to name the supervisors and
workers hired after the filing of the petition that Yes. The support requirement is a mere
were allegedly included in the list of rank-and- technicality which should be employed in
file employees. DOLE issued a certificate or determining the true will of the workers instead
registration of CBA—thus has the force and of frustrating it In line with the policy that
effect of law between parties Officers of holding of a CE is a certain and definitive mode
OTCWU-FFW walked out of their jobs, which of arriving at the choice of employees’
prompted the company to require them to explain bargaining representative, the administrative rule
in writing why no disciplinary action should be requiring the simultaneous submission of the
taken against them for walking out en masse. 25% consent signatures upon filing of the PCE
should not be strictly applied to frustrate the
The following day, the union filed a notice of determination of the legitimate representative of
strike due to the alleged dismissal of union the workers (PWUP v. Laguesma).
members/officers. The latter were directed by the
company to return to work within 48 hours but It is significant to note that this requirement is not
none of them did. found in Article 256, the law it seeks to
implement—thus, the regulation should be given
only a directory effect. Accordingly, the mere

MNI NOTES |121


filing of a petition for a certification election right to vote in a certification election cannot
within the freedom period is sufficient basis for thus be overemphasized.
the issuance of an order for the holding of a
certification election, subject to the submission FACTS:
of the consent signatures within a reasonable
A certification election was conducted on June
period after filing.
16, 2006 among the rank-and-file employees of
Thus, the doubts as to the number of respondent Holiday Inn Manila Pavilion Hotel
employees actually supporting the holding of a (the Hotel) with the following results:
certification election should be resolved,
EMPLOYEES IN VOTERS’ LIST = 353
therefore, by going through such procedure
because jurisprudence holds that it is the most TOTAL VOTES CAST = 346
effective and expeditious means of determining
the true representative for the working force in NUWHRAIN-MPHC = 151
the appropriate bargaining unit of the company.
If OTCLU wanted to remain as such, it should HIMPHLU = 169
have sought their vote, not engage in legal
NO UNION = 1
sophistry.
SPOILED = 3
NUWHRAIN v. Secretary of Labor GR
181531 July 31, 2009 SEGREGATED = 22
Doctrine: In a certification election, all rank- In view of the significant number of segregated
and-file employees in the appropriate bargaining votes, contending unions, petitioner,
unit, whether probationary or permanent are NUHWHRAIN-MPHC, and respondent
entitled to vote. Holiday Inn Manila Pavillion Hotel Labor
Union referred the case back to Med-Arbiter Ma.
Provision in the Collective Bargaining
Simonette Calabocal to decide which among
Agreement (CBA) disqualifying probationary
those votes would be opened and tallied.
employees from voting cannot override the
Constitutionally-protected right of workers to 11 votes were initially segregated because they
self-organization, as well as the provisions of were cast by dismissed employees (the legality
Labor Code and its Implementing Rules on of their dismissal was still pending before the
certification elections and jurisprudence thereon. Court of Appeals.)
A certification election is the process of 6 votes were segregated because the employees
determining the sole and exclusive bargaining who cast them were already occupying
agent of the employees in an appropriate supervisory positions at the time of the
bargaining unit for purposes of collective election.
bargaining; The significance of an employee’s

MNI NOTES |122


5 votes were segregated on the ground that they six probationary employees were not yet
were cast by probationary employees and, hired, hence, they could not vote.
pursuant to the existing Collective Bargaining
Agreement (CBA), such employees cannot vote. Votes cast by the 11 dismissed
however, that the vote of one Jose Gatbonton, a employees; they could be considered
probationary employee, was counted. since their dismissal was still pending
appeal.
Med-Arbiter Calabocal ruled for the opening of
17 out of the 22 segregated votes, specially those Votes cast by the 6 alleged supervisory
cast by the 11 dismissed employees and those employees; the SOLE held that their
cast by the six supposedly supervisory employees votes should be counted since their
of the Hotel. promotion took effect months after the
issuance
NUHWRAIN appealed to the SOLE, arguing
that the votes of probationary employees should ISSUE:
have been opened considering that probationary
whether employees on probationary status at the
employee Gatbonton’s vote was tallied, also
time of the certification elections should be
averred that respondent HIMPHLU, which
allowed to vote,
garnered 169 votes, should not be immediately
certified as the bargaining agent, as the opening whether HIMPHLU was able to obtain the
of the 17 segregated ballots would push the required majority for it to be certified as the
number of valid votes cast to 338 (151 + 169 + 1 exclusive bargaining agent
+ 17), hence, the 169 votes which HIMPHLU
garnered would be one vote short of the majority RULING:
which would then become 169.
1st Issue
SOLE affirmed the Med-Arbiter’s order, and
concluded that the certification election of Court rules in the affirmative.
HIMPHLU as the exclusive bargaining agent
The inclusion of Gatbonton’s vote was proper not
was proper.
because it was not questioned but because
held that pursuant to Section 5, Rule IX probationary employees have the right to vote
of the Omnibus Rules Implementing in a certification election.
the Labor Code on exclusion and
“In a certification election, all rank-and-file
inclusion of voters in a certification
employees in the appropriate bargaining unit,
election, the probationary employees
whether probationary or permanent are entitled to
cannot vote, as at the time the Med-
vote.
Arbiter issued on August 9, 2005 the
Order granting the petition for the This principle is clearly stated in Art. 255 of the
conduct of the certification election, the Labor Code which states that the “labor

MNI NOTES |123


organization designated or selected by the agent of all the workers in the appropriate
majority of the employees in an appropriate bargaining unit.
bargaining unit shall be the exclusive
representative of the employees in such unit for This majority is 50% + 1. Hence, 50% of 337 is
purposes of collective bargaining.” 168.5 + 1 or at least 170.

The Code makes no distinction as to their Clearly, HIMPHLU was not able to obtain a
employment status as basis for eligibility in majority vote.
supporting the petition for certification election.
It bears reiteration that the true importance of
The law refers to “all” the employees in the
ascertaining the number of valid votes cast is
bargaining unit. All they need to be eligible to
for it to serve as basis for computing the required
support the petition is to belong to the
majority, and not just to determine which union
“bargaining unit.”
won the elections.
For purposes of this section, any employee,
To be sure, the conduct of a certification election
whether employed for a definite period or not,
has a two-fold objective: to determine the
shall beginning on the first day of his/her service,
appropriate bargaining unit and to ascertain the
be eligible for membership in any labor
majority representation of the bargaining
organization.
representative, if the employees desire to be
the votes of the six supervisory employees must represented at all by anyone.
be excluded because at the time the certification
it follows that a run-off election must be held to
elections was conducted, they had ceased to be
determine which between HIMPHLU and
part of the rank and file,
petitioner should represent the rank-and-file
2nd issue employees.

Court rules in the negative. Capitol Medical Center Alliance of Concerned


Employees v. Laguesma GR 118915 Feb. 4,
It is well-settled that under the so-called “double 1997
majority rule,” for there to be a valid
certification election, majority of the bargaining Doctrine: Even if one year had lapsed since the
unit must have voted AND the winning union time of declaration of a final certification result,
must have garnered majority of the valid votes and there is no collective bargaining deadlock,
cast. there is no grave abuse of discretion committed
by the DOLE when it ruled against another
the number of valid votes cast would increase— certification election filed by another union
from 321 to 337. Under Art. 256 of the Labor where the delay in forging of the CBA could not
Code, the union obtaining the majority of the be attributed to the fault of the union who won
valid votes cast by the eligible voters shall be the earlier certification election.
certified as the sole and exclusive bargaining

MNI NOTES |124


FACTS: In spite of the refusal of respondent CMC,
respondent union still persisted in its demand
Med-Arbiter Rasidali C. Abdullah issued an for CBA negotiations, claiming that it has already
Order which granted respondent union’s been declared as the sole and exclusive
petition for certification election among the rank- bargaining agent of the rank-and-file employees
and-file employees of the Capitol Medical of the hospital.
Center.
ISSUE:
Respondent CMC appealed the Order to the
Office of the Secretary by questioning the legal whether or not public respondent committed
status of respondent union’s affiliation with the grave abuse of discretion in dismissing the
Alliance of Filipino Workers (AFW). respondent petition for certification election, and in directing
union registered itself independently and the hospital to negotiate a collective bargaining
withdrew the petition which had earlier been agreement with the said respondent union.
granted. Thereafter, it filed another petition for
certification election. RULING:

Med-Arbiter Manases T. Cruz issued an order We do not subscribe to petitioner’s contention.


granting the petition for certification election.
As correctly observed by the Solicitor General,
elections were finally held a total of 380 votes petitioner is merely “nit-picking, vainly trying to
cast. make a monumental issue out of a negligible
error of the public respondent.
respondent union = 204 votes
While it is true that, in the case at bench, one year
168 = no union had lapsed since the time of declaration of a final
certification result, and that there is no collective
8 = spoiled ballots bargaining deadlock, public respondent did not
commit grave abuse of discretion when it ruled
Med-Arbiter Cruz issued an Order certifying
in respondent union’s favor since the delay in
respondent union as the sole and exclusive
the forging of the CBA could not be attributed to
bargaining representative of the rank-and-file
the fault of the latter
employees at CMC.
CMC refused to negotiate with respondent union
Respondent CMC’s basic contention was the
and instead challenged the latter’s legal
supposed pendency of its petition for cancellation
personality through a petition for cancellation of
of respondent union’s certificate of registration,
the certificate of registration which eventually
wherein Med-Arbiter Adap issued an order
reached this Court.
declaring respondent union’s certificate of
registration as null and void. But was reversed by
OIC of the BLR.

MNI NOTES |125


There is a deadlock when there is a complete as sole bargaining representative in 1981, no
blocking or stoppage resulting from the action of collective bargaining agreement had been
equal and opposed forces. executed between it and VIRON.

Thus, we can only conclude that CMC was NAFLU appealed: It contended that:
unwilling to negotiate and reach an agreement
with respondent union. CMC has not at any 1. at the time the petition for certification election
instance shown willingness to discuss the was filed on April 11, 1985, it was in process of
economic proposals given by respondent union. collective bargaining with VIRON.

Kaisahan ng Manggagawang Pilipino v. 2. there was in fact a deadlock in the negotiations


Trajano GR 75810 Sept. 9, 1991 which had prompted it to file a notice of strike;
and that these circumstances constituted a bar to
Facts: the petition for election in accordance with
Section 3, Rule V, Book V of the Omnibus Rules
The propriety of holding a certification election Implementing the Labor Code.
is the issue in the special civil action of certiorari
at bar. SEC. 3. When to file. — In the
absence of a collective bargaining
February 27, 1981 – (By virtue of a Resolution of agreement submitted in accordance with
the Bureau of Labor Relations) the National Article 231 of the Code, a petition for
Federation of Labor Unions (NAFLU) was certification election may be filed at any
declared the exclusive bargaining time. However, no certification election
representative of all rank-and-file employees may be held within one year from the date
of Viron Garments Manufacturing Co., Inc. of issuance of declaration of a final
(VIRON). certification election result. Neither may
a representation question be entertained
April 11, 1985 - Kaisahan ng Manggagawang
if, before the filing of a petition for
Pilipino KAMPIL Katipunan filed with the
certification election, a bargaining
Bureau of Labor Relations a petition for
deadlock to which an incumbent or
certification election among the employees of
certified bargaining agent is a party had
VIRON. The petition allegedly counted with the
been submitted to conciliation or
support of more than thirty percent (30%) of the
arbitration or had become the subject of a
workers at VIRON.
valid notice of strike or lockout.
NAFLU opposed the petition.
If a collective bargaining
Med-Arbiter ordered that a certification agreement has been duly registered in
election be held at VIRON after ascertaining that accordance with Article 231 of the Code,
KAMPIL had complied with all the requirements a petition for certification election or a
of law and that since the certification of NAFLU motion for intervention can only be

MNI NOTES |126


entertained within sixty (60) days prior to KAMPIL moved for reconsideration but was
the expiry date of such agreement. denied. Court instituted the present certiorari
action.
Director of Labor Relations:
In this case, from February 27, 1981, the date of
• found merit in NAFLU’s appeal; the Resolution declaring NAFLU the exclusive
bargaining representative of rank-and-file
• rendered a Resolution on April 30, 1986;
workers of VIRON — can have no application to
• setting aside Med-Arb’s Order (June 14, the case at bar.
1985); and
That one-year period-known as the
• dismissing KAMPIL's petition for "certification year" during which the certified
certification election. union is required to negotiate with the
employer, and certification election is
Resolution: prohibited — has long since expired.

“While it may be true that the one-year period It is evident that the prohibition imposed by
(mentioned in Section 3 above quoted) has long law on the holding of a certification election
run its course since intervenor NAFLU was "within one year from the date of issuance of
certified on February 27, 1981, it could not be declaration of a final certification election
said, however, that NAFLU slept on its right to result.
bargain collectively with the employer. If a closer
look was made on the history of labor Issues:
management relations in the company, it could
whether or not KAMPIL's petition for
be readily seen that the delay in the negotiations
certification election is barred because, before its
for and conclusion of a collective agreement —
filing, a bargaining deadlock between VIRON
the object of the one-year period — could be
and NAFLU as the incumbent bargaining agent,
attributed first, on the exhaustion of all legal
had been submitted to conciliation or arbitration
remedies in the representation question twice
or had become the subject of a valid notice of
initiated in the company before the filing of the
strike or lockout, in accordance with Section 3,
present petition and second, to management who
Rule V, Book V of the Omnibus Rules - NO
had been resisting the representations of NAFLU
in collective bargaining. Held:

The one-year period therefore, should not be Prior to the filing of the petition for election in
applied literally to the present dispute, especially this case, there was no such "bargaining deadlock
considering that intervenor had to undergo a (which) had been submitted to conciliation or
strike to bring management to the negotiation arbitration or had become the subject of a valid
table.” notice of strike or lockout.

MNI NOTES |127


The prohibition imposed by law on the representative until the time KAMPIL filed its
holding of a certification election "within one petition for certification election, no CBA was
year from the date of issuance of declaration ever executed, and no deadlock ever arose from
of a final certification election result in this negotiations between NAFLU and VIRON
case, from the date of the Resolution declaring resulting in conciliation proceedings or the filing
NAFLU the exclusive bargaining of a valid strike notice.
representative can have no application here.
VIRON adverts to a strike declared by NAFLU
That one-year period known as the for its (VIRON) refusal to bargain and for
"certification year" during which the certified violation of terms and conditions of
union is required to negotiate with the employment, and to another strike staged in
employer, and certification election is connection with a claim of violation of said
prohibited has long since expired. agreement.

There are assertions by NAFLU that its attempts However, these activities took place after
to bring VIRON to the negotiation table had been KAMPIL initiated the certification election case,
unsuccessful because of the latter's recalcitrance and it was grave abuse of discretion to have
and unfulfilled promises to bargain collectively, regarded them as precluding the holding of the
but there is no proof that it had taken any certification election prayed for.
action to legally coerce VIRON to comply with
its statutory duty to bargain collectively. WHEREFORE, it being apparent that none of the
proscriptions to certification election set out in
• It could have charged VIRON with unfair the law exists in the case at bar, and it was in the
labor practice; but it did not. premises grave abuse of discretion to have ruled
otherwise, the contested Resolution of the
• It could have gone on a legitimate strike respondent Director of the Bureau of Labor
in protest against VIRON's refusal to Relations dated April 30, 1986 in BLR Case No.
bargain collectively and compel it to do A-7-139-85 (BZEO-CE-04-004-85) is
so; but it did not. NULLIFIED AND SET ASIDE. Costs against
private respondents.
There are assertions by NAFLU, too, that its
attempts to bargain collectively had been delayed Yokohama Tire Philippines v. Yokohama
by continuing challenges to the resolution Employees Union GR 159553 Dec. 10, 2007
pronouncing it the sole bargaining representative
in VIRON; but there is no adequate
substantiation thereof, or of how it did in fact Facts:
prevent initiation of the bargaining process
between it and VIRON. Petitioner Yokohama Tire Philippines, Inc.
(hereafter Yokohama, for brevity) assails the
The fact is that from the time that NAFLU was Decision1 dated April 9, 2003 of the Court of
proclaimed the exclusive bargaining Appeals in CA-G.R. SP No. 74273 and its

MNI NOTES |128


Resolution dated August 15, 2003, denying the alleged supervisor-
motion for reconsideration. trainees)

October 7, 1999 – (Upon appeal from the Med- --------


Arbiter's order dismissing the petition) YEU filed
a petition for certification election among the
rank-and-file employees of Yokohama.
TOTAL - 151
Sec. DOLE - ordered an election to be held on CHALLENGED
November 23, 2001 with: VOTES

(1) "Yokohama Employees' Union" and TOTAL VOTES - 401


CAST
(2) "No Union"

Votes result: Yokohama formalized its protest and raised as an


issue the eligibility to vote of the 78 dismissed
YOKOHAMA - 131 employees, while the Union submitted only a
EMPLOYEES handwritten manifestation during the election.
UNION
January 21, 2002 – Med-Arb resolved the
protests.
NO UNION - 117
2. The appreciation of the votes of the sixty-five
SPOILED - 2 (65) dismissed employees who contested their
dismissal before the National Labor Relations
--------- Commission shall be suspended until the final
disposition of their complaint for illegal
250 dismissal.

VOTES - 78 3. The votes of the sixty-eight (68) so-called


CHALLENGED BY “newly-regularized" rank-and-file employees
[YOKOHAMA] (Dismissed shall be appreciated in the final tabulation.
employees)
May 22, 2002 – Acting Sec. of DOLE:
VOTES - 73 • partial appeal of YTP is DENIED
CHALLENGED BY
[UNION] (68 - newly • appeal of the YEU is PARTIALLY
regularized rank-and- GRANTED
file employees and 5 -
• Order of the Med-Arbiter dated 21
January 2002 is MODIFIED

MNI NOTES |129


2. The votes of dismissed employees who October 16, 2003
contested their dismissal before the National
Labor Relations Commission (NLRC) shall be YTP attached a Resolution (April 25, 2003) of
appreciated in the final tabulation of the the Med-Arb, in a manifestation with motion to
certification election results. annul the DOLE Secretary's entry of judgment.

3. The votes of the sixty-eight (68) newly Resolution:


regularized rank-and-file employees shall be
• denied Yokohama's motion to suspend
excluded.
proceedings and cited the decision of the
CA: Court of Appeals.

Affirmed in toto he decision of the DOLE Acting • certified that the Union obtained a
Secretary: majority of 208 votes in the certification
election while "No Union" obtained 121
→ The 78 employees who contested their votes
dismissal were entitled to vote under Article 212
(f) of the Labor Code and Section 2, Rule XII of YTP attached an entry of judgment by DOLE
the rules implementing Book V of the Labor stating that the Resolution (April 25, 2003) of
Code. However, it disallowed the votes of the 68 the Med-Arbiter was affirmed by the DOLE
newly regularized employees since they were not Secretary's Office on July 29, 2003 and became
included in the voters' list submitted during the final on September 29, 2003.
July 12, 2001 pre-election conference.
October 21, 2003 - YTP deleted an allegation
→ Yokohama's insistence on their inclusion in its October 16, 2003 manifestation which was
lends suspicion that it wanted to create a included "through inadvertence and clerical
company union, and ruled that Yokohama had no mishap."
right to intervene in the certification election.
Allegation:
→ The union's handwritten manifestation during
Notably, the Resolution dated 29 July 2003 which
the election was substantial compliance with the
affirmed the Resolution dated 25 April 2003 is
rule on protest.
still not final and executory considering the
YTP appealed. timely filing of a motion for its reconsideration
on 15 August 2003 which until now has yet to be
September 15, 2003 – CA issued a TRO against resolved.
the implementation of the May 22, 2002 Decision
of the DOLE Acting Secretary and the October Issue:
15, 2002 Resolution of the DOLE Secretary,
1. Whether or not the court of appeals seriously
denying Yokohama's motion for reconsideration.
erred in disallowing the appreciation of the votes
of sixty-eight regular rank-and-file.

MNI NOTES |130


2. Whether or not the court of appeals seriously dismissed from work but has contested the
erred in upholding the DOLE Secretary’s legality of the dismissal in a forum of appropriate
declaration that the Union’s manifestation on the jurisdiction at the time of the issuance of the
day of the certification election was sufficient order for the conduct of a certification election
compliance with the rule on formalization of shall be considered a qualified voter, unless
protests. his/her dismissal was declared valid in a final
judgment at the time of the conduct of the
3. Whether or not the court of appeals seriously certification election.
erred in allowing the appreciation of votes of all
of its employees who were previously dismissed SC:
for serious misconduct and abandonment of work
which are causes unrelated to the certification → find no reversible error on the part of the
election. (Was it proper to appreciate the votes DOLE Acting Secretary and the Court of
of the dismissed employees?) YES Appeals in ordering the appreciation of the votes
of the dismissed employees.
Held:
→ the certification election is already a fait
1 and 2. accompli, and clearly petitioner's rank-and-file
employees had chosen respondent as their
Need not resolve the other issues for being moot. bargaining representative.
The 68 votes of the newly regularized rank-and-
file employees, even if counted in favor of "No WHEREFORE, the petition is DENIED for lack
Union," will not materially alter the result. There of merit. The assailed Decision dated April 9,
would still be 208 votes in favor of respondent 2003 of the Court of Appeals in CA-G.R. SP No.
and 189 votes in favor of "No Union." 74273 and the Resolution dated August 15, 2003
are AFFIRMED. The temporary restraining order
3. Was it proper to appreciate the votes of the issued on September 15, 2003 is hereby
dismissed employees? – YES DISSOLVED. No pronouncement as to costs.

Even the new rule has explicitly stated that


without a final judgment declaring the legality of
dismissal, dismissed employees are eligible or
qualified voters.

Rule IX

Conduct of Certification Election

Section 5. Qualification of voters; inclusion-


exclusion. - An employee who has been

MNI NOTES |131


MODULE 5 The Essence of the Duty to Bargain Collectively

Both the employer and the bargaining The duty to bargain collectively does not impose upon
representative shall observe the duty to bargain the employer the obligation to initiate contract
collectively. The duty to bargain requires both negotiation. Neither does it compel the parties to
agree to a proposal or to make any concession, much
parties to meet promptly and expeditiously in
less to reach an agreement.
good faith to negotiate an agreement respecting
wages, hours of work, and other conditions of All that is required is for the parties to approach the
employment. The parties are also expected to negotiations with an open mind and exert reasonable
respect an existing CBA unless there is a written effort to reach a common ground of agreement.
request to terminate or modify the terms within Proposals, if unacceptable, should be matched with
60 days before the CBA expires. counterproposals.

The CBA entered by the employer and the To offer the union a contract saying “Take it or leave
employees through the bargaining representative, it” is not in consonance with good faith bargaining.
Feigning negotiations through empty gestures is not
shall govern their relationship and is deemed as
bargaining in good faith.
the law of the plant. Before this agreement may
become valid, the law lays down several Pre-termination of a CBA
procedural and substantive requirements that
must be satisfied. GR: No termination or modification of the CBA
during its lifetime.
A. “Matinong Usapan:” Duty to Bargain
Collectively → To ask for modification of the CBA, can only
be during the 60-day period prior to its expiration
Duty to bargain collectively; Definition date
ART. 263. [252] Meaning of Duty to Bargain Refusal to enter into collective bargaining and
Collectively. negotiation
The duty to bargain collectively means the Unfair labor practice is committed when it is
performance of a mutual obligation to meet shown that the respondent employer, after having
and convene promptly and expeditiously in been served with a written bargaining proposal
good faith for the purpose of negotiating an by the petitioning Union, did not even bother to
agreement with respect to wages, hours of work submit an answer or reply to the said proposal
and all other terms and conditions of employment Unfair labor practice is committed when it is
including proposals for adjusting any grievances shown that the respondent employer, after having
or questions arising under such agreement and been served with a written bargaining proposal
executing a contract incorporating such by the petitioning Union, did not even bother to
agreements if requested by either party but such submit an answer or reply to the said proposal. –
duty does not compel any party to agree to a Kiok Loy v. NLRC
proposal or to make any concession.

MNI NOTES |132


Surface Bargaining vs. Blue-Sky Bargaining years. The relation between labor and
management should be undisturbed until the last
Surface Bargaining 60 days of the fifth year. Hence, it is indisputable
that when the union requested for a renegotiation
→ defined as going through the motions of
of the economic terms of the CBA on November
negotiating without any legal intent to reach an
29, 1991, it was still the certified collective
agreement.
bargaining agent of the workers, because it was
→ on the part of the management seeking said renegotiation within five (5) years
from the date of effectivity of the CBA on
→ There can be no surface bargaining, absent December 1, 1988. The union’s proposal was
any evidence that management had done acts, also submitted within the prescribed 3-year
both at and away from the bargaining table, period from the date of effectivity of the CBA,
which tend to show that it did not want to reach albeit just before the last day of said period. It
an agreement with the union or to settle the was obvious that GMC had no valid reason to
differences between it and the union. refuse to negotiate in good faith with the
union. For refusing to send a counter-proposal to
Blue-Sky Bargaining the union and to bargain anew on the economic
terms of the CBA, the company committed an
→ means making exaggerated or unreasonable
unfair labor practice under Article 248 of the
proposals
Labor Code.”
→ on the part of the union
“Similarly, in the earlier 2000 case of Colegio de
→ In order to be considered as unfair labor San Juan de Letran vs. Association of Employees
practice, there must be proof that the demands and Faculty of Letran, [G. R. No. 141471,
made by the union were exaggerated or September 18, 2000], the petitioner school was
unreasonable. declared guilty of unfair labor practice when it
failed to make a timely reply to the proposals of
------ the union more than a month after the same were
submitted by the union. In explaining its failure
In the 2004 case of General Milling to reply, the school merely offered the feeble
Corporation vs. CA, [G. R. No. 146728, excuse that its Board of Trustees had not yet
February 11, 2004], the Supreme Court declared convened to discuss the matter. Clearly, its
that the petitioner is guilty of unfair labor actuation showed a lack of sincere desire to
practice under Article 248 [g] for refusing to negotiate rendering it guilty of unfair labor
send a counter-proposal to the union and to practice.
bargain anew on the economic terms of the
CBA. It ruled: “Surface bargaining” on the part of
management.
“The law mandates that the representation
provision of a CBA should last for five

MNI NOTES |133


“Surface bargaining” is defined as “going “Blue-sky bargaining” on the part of union.
through the motions of negotiating” without any
legal intent to reach an agreement. (Standard “Blue-sky bargaining” means making
Chartered Bank Employees Union [NUBE] vs. exaggerated or unreasonable proposals. (Arthur
Confesor, G. R. No. 114974, June 16, 2004). A. Sloane and Fred Witney, Labor Relations, 7th
Edition 1991, p. 195).
The resolution of surface bargaining allegations
never presents an easy issue. The determination In order to be considered as unfair labor practice,
of whether a party has engaged in unlawful there must be proof that the demands made by
surface bargaining is usually a difficult one the union were exaggerated or
because it involves, at bottom, a question of the unreasonable. In the same 2004 case of
intent of the party in question, and usually such Standard Chartered Bank [supra], the minutes of
intent can only be inferred from the totality of the the meeting show that the union based its
challenged party’s conduct both at and away economic proposals on data of rank-and-file
from the bargaining table. (Luck Limousine, 312 employees and the prevailing economic benefits
NLRB 770, 789 [1993]). received by bank employees from other foreign
banks doing business in the Philippines and other
According to Standard Chartered Bank branches of the bank in the Asian region. Hence,
Employees Union [NUBE] vs. Confesor, [G. R. it cannot be said that the union was guilty of ULP
No. 114974, June 16, 2004], surface bargaining for blue-sky bargaining.
involves the question of whether an employer’s
conduct demonstrates an unwillingness to Article 237, 261-266 LC
bargain in good faith or is merely hard
ART. 237. [231] Registry of Unions and File of
bargaining. There can be no surface bargaining,
Collective Bargaining Agreements.
absent any evidence that management had done
acts, both at and away from the bargaining table, The Bureau shall keep a registry of legitimate
which tend to show that it did not want to reach labor organizations. The Bureau shall also
an agreement with the union or to settle the maintain a file of all collective bargaining
differences between it and the union. Here, agreements and other related agreements and
admittedly, the parties were not able to agree and records of settlement of labor disputes and copies
reached a deadlock. However, it must be of orders and decisions of voluntary arbitrators or
emphasized that the duty to bargain “does not panel of voluntary arbitrators. The file shall be
compel either party to agree to a proposal or open and accessible to interested parties under
require the making of a concession.” Hence, conditions prescribed by the Secretary of
the parties’ failure to agree does not amount to Labor and Employment, provided that no
ULP under Article 248 [g] for violation of the specific information submitted in confidence
duty to bargain. (See also National Union of shall be disclosed unless authorized by the
Restaurant Workers [PTUC] vs. CIR, 10 SCRA Secretary, or when it is at issue in any judicial
843 [1964]). litigation, or when public interest or national
security so requires.

MNI NOTES |134


Within thirty (30) days from the execution of a (a) When a party desires to negotiate an
Collective Bargaining Agreement, the parties agreement, it shall serve a written notice upon
shall submit copies of the same directly to the the other party with a statement of its
Bureau or the Regional Offices of the proposals. The other party shall make a reply
Department of Labor and Employment for thereto not later than ten (10) calendar days from
registration accompanied with verified proofs of receipt of such notice;
its posting in two conspicuous places in the place
of work and ratification by the majority of all the (b) Should differences arise on the basis of such
workers in the bargaining unit. The Bureau or notice and reply, either party may request for a
Regional Offices shall act upon the application conference which shall begin not later than ten
for registration of such Collective Bargaining (10) calendar days from the date of request.
Agreement within five (5) calendar days from
(c) If the dispute is not settled, the Board shall
receipt thereof. The Regional Offices shall
intervene upon request of either or both parties or
furnish the Bureau with a copy of the Collective
at its own initiative and immediately call the
Bargaining Agreement within five (5) days from
parties to conciliation meetings. The Board shall
its submission.
have the power to issue subpoenas requiring the
The Bureau or Regional Office shall assess the attendance of the parties to such meetings. It shall
employer for every Collective Bargaining be the duty of the parties to participate fully and
Agreement a registration fee of not less than promptly in the conciliation meetings the Board
one thousand pesos (P1,000.00) or in any other may call;
amount as may be deemed appropriate and
(d) During the conciliation proceedings in the
necessary by the Secretary of Labor and
Board, the parties are prohibited from doing any
Employment for the effective and efficient
act which may disrupt or impede the early
administration of the Voluntary Arbitration
settlement of the disputes; and
Program. Any amount collected under this
provision shall accrue to the Special Voluntary (e) The Board shall exert all efforts to settle
Arbitration Fund. disputes amicably and encourage the parties to
submit their case to a voluntary arbitrator.
The Bureau shall also maintain a file, and shall
undertake or assist in the publication of all final ART. 262. [251] Duty to Bargain Collectively
decisions, orders and awards of the Secretary of in the Absence of Collective Bargaining
Labor and Employment, Regional Directors and Agreements.
the Commission.
In the absence of an agreement or other
ART. 261. [250] Procedure in Collective voluntary arrangement providing for a more
Bargaining expeditious manner of collective bargaining, it
shall be the duty of employer and the
The following procedures shall be observed in
representatives of the employees to bargain
collective bargaining:

MNI NOTES |135


collectively in accordance with the provisions of representation aspect is concerned, be for a term
this Code. of five (5) years. No petition questioning the
majority status of the incumbent bargaining agent
ART. 263. [252] Meaning of Duty to Bargain shall be entertained and no certification election
Collectively. shall be conducted by the Department of Labor
and Employment outside of the sixty-day period
The duty to bargain collectively means the
immediately before the date of expiry of such
performance of a mutual obligation to meet
five-year term of the Collective Bargaining
and convene promptly and expeditiously in good
Agreement.
faith for the purpose of negotiating an
agreement with respect to wages, hours of All other provisions of the Collective
work and all other terms and conditions of Bargaining Agreement shall be renegotiated
employment including proposals for adjusting not later than three (3) years after its
any grievances or questions arising under such execution. Any agreement on such other
agreement and executing a contract incorporating provisions of the Collective Bargaining
such agreements if requested by either party but Agreement entered into within six (6) months
such duty does not compel any party to agree to from the date of expiry of the term of such other
a proposal or to make any concession. provisions as fixed in such Collective Bargaining
Agreement, shall retroact to the day immediately
ART. 264. [253] Duty to Bargain Collectively
following such date. If any such agreement is
When There Exists a Collective Bargaining
entered into beyond six months, the parties shall
Agreement.
agree on the duration of retroactivity thereof.
When there is a collective bargaining
In case of a deadlock in the renegotiation of the
agreement, the duty to bargain collectively shall
Collective Bargaining Agreement, the parties
also mean that neither party shall terminate nor
may exercise their rights under this Code.
modify such agreement during its lifetime.
However, either party can serve a written ART. 266. [254] Injunction Prohibited.
notice to terminate or modify the agreement at
least sixty (60) days prior to its expiration date. No temporary or permanent injunction or
It shall be the duty of both parties to keep the restraining order in any case involving or
status quo and to continue in full force and effect growing out of labor disputes shall be issued by
the terms and conditions of the existing any court or other entity, except as otherwise
agreement during the 60-day period and/or until provided in Articles 218 and 264 of this Code.
a new agreement is reached by the parties.

ART. 265. [253-A] Terms of a Collective


Bargaining Agreement. Section 1(j), (t), and (ss), rule 1 Book 5,
Omnibus Rules Implementing the LC
Any Collective Bargaining Agreement that the
parties may enter into shall, insofar as the

MNI NOTES |136


(j) Collective Bargaining Agreement or CBA Article 249, par. (g) of the Labor Code
refers to the contract between a legitimate labor makes it an unfair labor practice for an
union and the employer concerning wages, hours employer to refuse “to meet and convene
of work, and all other terms and conditions of promptly and expeditiously in good faith for the
employment in a bargaining unit. purpose of negotiating an agreement with respect
to wages, hours of work, and all other
(t) Exclusive Bargaining Representative refers terms and conditions of employment
to a legitimate labor union duly recognized or including proposals for adjusting any
certified as the sole and exclusive bargaining grievance or question arising under such an
representative or agent of all the employees in a agreement and executing a contract
bargaining unit. incorporating such agreement, if requested by
either party.
(ss) Run-off Election refers to an election
between the labor unions receiving the two (2) NATURE: Petition for certiorari to annul the
highest number of votes in a certification or decision of the NLRC dated July 20, 1979 which
consent election with three (3) or more choices, found petitioner Sweden Ice Cream guilty of
where such a certified or consent results in none unfair labor practice for unjustified refusal to
of the three (3) or more choices receiving the bargain, in violation of par. (g) of Article 249 of
majority of the valid vote cast; Provided, That is the New Labor Code, and declared the draft
the total number of votes for all contending proposal of the Union for a collective bargaining
unions is at least fifty percent (50%) of the agreement as the governing collective bargaining
number of votes cast. agreement between the employees and the
management
Kiok Loy v. NLRC GR 54334, January 22,
1986 Facts:

DOCTRINE: Refusal to bargain - Unfair labor In a certification election held on October 3,


practice is committed when it is shown that the 1978, the Pambansang Kilusang Paggawa, a
respondent employer, after having been served legitimate late labor federation, won and was
with a written bargaining proposal by the subsequently certified in a resolution by the
petitioning Union, did not even bother to submit Bureau of Labor Relations as the sole and
an answer or reply to the said proposal. exclusive bargaining agent of the rank and-file
employees of Sweden Ice Cream Plant. The
A Company’s refusal to make counter proposal if
Company's MR of the said resolution was denied.
considered in relation to the entire bargaining
process, may indicate bad faith and this is Thereafter, the Union furnished the Company
specially true where the Union’s request for a with two copies of its proposed CBA. At the
counter proposal is left unanswered same time, it requested the Company for its
counter proposals. Eliciting no response to the
Unfair Labor Practice
aforesaid request, the Union again wrote the

MNI NOTES |137


Company reiterating its request for collective further evidence in support of its stand and when
bargaining negotiations. Both requests were its request for further postponement was denied.
ignored and remained unacted upon by the
Company. • that the NLRC's finding of unfair labor practice
for refusal to bargain is not supported by law and
Left with no other alternative in its attempt to the evidence considering that it was only on May
bring the Company to the bargaining table, the 24, 1979 when the Union furnished them with a
Union, on February 14, 1979, filed a "Notice of copy of the proposed CBA and it was only then
Strike", with the Bureau of Labor Relations that they came to know of the Union's demands;
(BLR) on ground of unresolved economic issues and finally,
in collective bargaining.
• that the Collective Bargaining Agreement
Conciliation proceedings then followed during approved and adopted by the NLRC is
the thirty-day statutory cooling-off period. But unreasonable and lacks legal basis.
all attempts towards an amicable settlement
failed, prompting the Bureau of Labor Relations Issue:
to certify the case to the NLRC for compulsory
Whether the Company is guilty of unfair labor
arbitration pursuant to Presidential Decree No.
practice for refusal to bargain
823, as amended. The Company did not submit
its position paper, and instead requested for a Held:
resetting which was granted.
Yes. Petition dismissed for lack of merit.
The case was reset many times upon the request
of defendant’s counsel. When the company’s Collective bargaining is one of the democratic
representative failed to appear in a scheduled frameworks under the New Labor Code,
hearing and defendant’s counsel again asked for designed to stabilize the relation between labor
a postponement, the labor arbiter denied the and management and to create a climate of sound
request. He also ruled that the Company has and stable industrial peace. It is a mutual
waived its right to present further evidence and, responsibility of the employer and the Union and
therefore, considered the case submitted for is characterized as a legal obligation.
resolution.
Article 249, par. (g) of the Labor Code makes it
The LA submitted its report to the NLRC, which an unfair labor practice for an employer to
ruled in favor of the union. refuse "to meet and convene promptly and
expeditiously in good faith for the purpose of
Petitioner went to the SC assailing the NLRC negotiating an agreement with respect to wages,
order and alleging the following: hours of work, and all other terms and conditions
of employment including proposals for adjusting
• that its right to procedural due process has been
any grievance or question arising under such an
violated when it was precluded from presenting

MNI NOTES |138


agreement and executing a contract incorporating PETITIONER’S ACTS SHOW
such agreement, if requested by either party. DISINTEREST TO BARGAIN IN GOOD
FAITH
CBA, DEFINED
From the over-all conduct of petitioner company
Collective bargaining which is defined as in relation to the task of negotiation, there can be
negotiations towards a collective agreement, is no doubt that the Union has a valid cause to
one of the democratic frameworks under the New complain against its Company's attitude, the
Labor Code, designed to stabilize the relation totality of which is indicative of the latter's
between labor and management and to create a disregard of, and failure to live up to, what is
climate of sound and stable industrial peace. It is enjoined by the Labor Code — to bargain in good
a mutual responsibility of the employer and the faith.
Union and is characterized as a legal obligation.
So much so that Article 249, par. (g) of the Labor Petitioner Company is GUILTY of unfair
Code makes it an unfair labor practice for an labor practice. It has been indubitably
employer to refuse "to meet and convene established that:
promptly and expeditiously in good faith for the
purpose of negotiating an agreement with respect (1) respondent Union was a duly certified
to wages, hours of work, and all other terms and bargaining agent;
conditions of employment including proposals
(2) it made a definite request to bargain,
for adjusting any grievance or question arising
accompanied with a copy of the proposed
under such an agreement and executing a
Collective Bargaining Agreement, to the
contract incorporating such agreement, if
Company not only once but twice which were left
requested by either party.
unanswered and unacted upon; and (3) the
The mechanics of collective bargaining are set in Company made no counter proposal whatsoever
motion only when the following jurisdictional all of which conclusively indicate lack of a
preconditions are present, namely, sincere desire to negotiate.

o (1) possession of the status of majority A Company's refusal to make counter proposal if
representation of the employees' representative in considered in relation to the entire bargaining
accordance with any of the means of selection or process, may indicate bad faith since the Union's
designation provided for by the Labor Code; request for a counter proposal is left unanswered.
Besides, petitioner Company's approach and
o (2) proof of majority representation; and attitude-stalling the negotiation by a series of
postponements, non-appearance at the hearing
o (3) a demand to bargain under Article 251, conducted, and undue delay in submitting its
par. (a) of the New Labor Code. financial statements, lead to no other conclusion
except that it is unwilling to negotiate and reach
an agreement with the Union.

MNI NOTES |139


Petitioner's aforesaid submittal that it has been In its two plants located at Cebu City and Lapu-
denied due process failed to impress the SC. Lapu City, petitioner General Milling
Considering the various postponements granted Corporation (GMC) employed 190 workers.
in its behalf, the claimed denial of due process They were all members of private respondent
appeared totally bereft of any legal and factual General Milling Corporation Independent Labor
support. The moves and overall behavior of Union. On April 28, 1989, GMC and the union
petitioner-company were in total derogation of concluded a collective bargaining agreement
the policy enshrined in the New Labor Code (CBA) which included the issue of representation
which is aimed towards expediting settlement of effective for a term of three years. The day before
economic disputes. the expiration of the CBA, the union sent GMC a
proposed CBA, with a request that a counter-
It is not obligatory upon either side of a labor proposal be submitted within ten (10) days.
controversy to precipitately accept or agree to the
proposals of the other. But an erring party should However, GMC had received collective and
not be tolerated and allowed with impunity to individual letters from workers who stated that
resort to schemes feigning negotiations by going they had withdrawn from their union
through empty gestures. More so, as in the instant membership, on grounds of religious affiliation
case, where the intervention of the NLRC was and personal differences. Believing that the
properly sought for after conciliation efforts union no longer had standing to negotiate a CBA,
undertaken by the BLR failed. GMC did not send any counter-proposal.

The instant case being a certified one, it must be On December 16, 1991, GMC wrote a letter to
resolved by the NLRC pursuant to the mandate the union’s officers, Rito Mangubat and Victor
of P.D. 873, as amended, which authorizes the Lastimoso.
said body to determine the reasonableness of the
terms and conditions of employment embodied in The letter stated that it felt there was no basis to
any CBA. To that extent, utmost deference to its negotiate with a union which no longer existed,
findings of reasonableness of any CBA as the but that management was nonetheless always
governing agreement by the employees and willing to dialogue with them on matters of
management must be accorded due respect by common concern and was open to suggestions on
this Court. how the company may improve its operations. In
answer, the union officers wrote a letter dated
San Pedro Hospital of Digos v. Secretary of December 19, 1991 disclaiming any massive
Labor GR 104624, October 11, 2006 disaffiliation or resignation from the union and
submitted a manifesto, signed by its members,
stating that they had not withdrawn from the
union.
General Mining Corp. v. CA GR 146728
NLRC held that the action of GMC in not
FACTS:
negotiating was ULP.

MNI NOTES |140


ISSUE: ART. 253-A. Terms of a collective bargaining
agreement. – Any Collective Bargaining
1. WON the company (GMC) should have Agreement that the parties may enter into
entered into collective bargaining with the union shall, insofar as the representation aspect is
concerned, be for a term of five (5) years. No
2. Whether or not GMC is guilty of unfair labor
petition questioning the majority status of the
practice for violating its duty to bargain
incumbent bargaining agent shall be entertained
collectively and/or for interfering with the right
and no certification election shall be conducted
of its employees to self-organization.- YES
by the Department of Labor and Employment
HELD: outside of the sixty-day period immediately
before the date of expiry of such five year term
The law mandates that the representation of the Collective Bargaining Agreement. All
provision of a CBA should last for five other provisions of the Collective Bargaining
years. The relation between labor and Agreement shall be renegotiated not later than
management should be undisturbed until the three (3) years after its execution.
last 60 days of the fifth year.
2. YES
Hence, it is indisputable that when the union
requested for a renegotiation of the economic GMC is guilty of unfair labor practice when it
terms of the CBA on November 29, 1991, it was refused to negotiate with the union upon its
still the certified collective bargaining agent of request for the renegotiation of the economic
the workers, because it was seeking said terms of the CBA on November 29, 1991. the
renegotiation within five (5) years from the union’s proposal was submitted within the
date of effectivity of the CBA on December 1, prescribed 3-year period from the date of
1988. effectivity of the CBA. It was obvious that GMC
had no valid reason to refuse to negotiate in good
The union’s proposal was also submitted faith with the union. The refusal to send counter
within the prescribed 3-year period from the proposal to the union and to bargain anew on the
date of effectivity of the CBA, albeit just economic terms of the CBA is tantamount to an
before the last day of said period. It was unfair labor practice under Article 248 of the
obvious that GMC had no valid reason to Labor Code.
refuse to negotiate in good faith with the
union. ART. 248. Unfair labor practices of
employers. – It shall be unlawful for an
For refusing to send a counter-proposal to the employer to commit any of the following unfair
union and to bargain anew on the economic terms labor practice:
of the CBA, the company committed an unfair
labor practice under Article 248 of the Labor (g) To violate the duty to bargain collectively as
Code. prescribed by this Code;

MNI NOTES |141


Under Article 252 abovecited, both parties are in its duty under Article 252. What it did was to
required to perform their mutual obligation to devise a flimsy excuse, by questioning the
meet and convene promptly and expeditiously in existence of the union and the status of its
good faith for the purpose of negotiating an membership to prevent any negotiation. It bears
agreement. The union lived up to this obligation stressing that the procedure in collective
when it presented proposals for a new CBA to bargaining prescribed by the Code is mandatory
GMC within three (3) years from the effectivity because of the basic interest of the state in
of the original CBA. But GMC failed in its duty ensuring lasting industrial peace.
under
The Court of Appeals found that the letters
ART. 250. Procedure in collective between February to June, 1993 by 13 union
bargaining. – The following procedures shall be members signifying their resignation from the
observed in collective bargaining: union clearly indicated that GMC exerted
pressure on the employees. We agree with the
(a) When a party desires to negotiate an Court of Appeals’ conclusion that the ill-timed
agreement, it shall serve a written notice upon the letters of resignation from the union members
other party with a statement of its proposals. The indicate that GMC interfered with the right of its
other party shall make a reply thereto not later employee to self-organization.
than ten (10) calendar days from receipt of such
notice.

GMC’s failure to make a timely reply to the Sundowner Development Corporation v.


proposals presented by the union is indicative of Drilon GR 82341, December 06, 1989
its utter lack of interest in bargaining with the
union. Its excuse that it felt the union no longer
represented the workers, was mainly dilatory as
BPI v. BPI employees GR 164301 August 10,
it turned out to be utterly baseless.
2010
Failing to comply with the mandatory obligation
FACTS:
to submit a reply to the union’s proposals, GMC
violated its duty to bargain collectively, making Bangko Sentral ng Pilipinas approved the
it liable for unfair labor practice. Articles of Merger executed by and between BPI,
herein petitioner, and Far East Bank and Trust
Under Article 252 of the Labor Code, both
Company (FEBTC) and was approved by the
parties are required to perform their mutual
Securities and Exchange Commission.
obligation to meet and convene promptly and
expeditiously in good faith for the purpose of The Articles of Merger and Plan of Merger
negotiating an agreement. The union lived up to did not contain any specific stipulation with
this obligation when it presented proposals for a respect to the employment contracts of existing
new CBA to GMC within 3 years from the
effectivity of the original CBA. But GMC failed

MNI NOTES |142


personnel of the non-surviving entity which is [The] Court cannot uphold the reasoning that the
FEBTC. general stipulation regarding transfer of FEBTC
assets and liabilities to BPI as set forth in the
Pursuant to the said Article and Plan of Merger, Articles of Merger necessarily includes the
all the assets and liabilities of FEBTC were transfer of all FEBTC employees into the employ
transferred to and absorbed by BPI as the of BPI and neither BPI nor the FEBTC
surviving corporation. FEBTC employees, employees allegedly could do anything about
including those in its different branches across it. Even if it is so, it does not follow that the
the country, were hired by petitioner as its own absorbed employees should not be subject to the
employees, with their status and tenure terms and conditions of employment obtaining in
recognized and salaries and benefits maintained. the surviving corporation.

ISSUE: Furthermore, [the] Court believes that it is


Whether or not employees are ipso jure absorbed in a contrary to public policy to declare the former
merger of the two corporations.
FEBTC employees as forming part of the assets
RULING: or liabilities of FEBTC that were transferred and
absorbed by BPI in the Articles of
NO. [H]uman beings are never embraced in the
Merger. Assets and liabilities, in this instance,
term “assets and liabilities.”Moreover, BPI’s
should be deemed to refer only to property rights
absorption of former FEBTC employees was
and obligations of FEBTC and do not include the
neither by operation of law nor by legal
employment contracts of its personnel.
consequence of contract.
A corporation cannot unilaterally transfer its
There was no government regulation or law that
employees to another employer like
compelled the merger of the two banks or the
chattel. Certainly, if BPI as an employer had the
absorption of the employees of the dissolved
right to choose who to retain among FEBTC’s
corporation by the surviving corporation.
employees, FEBTC employees had the
Had there been such law or regulation, the concomitant right to choose not to be absorbed
absorption of employees of the non-surviving by BPI.
entities of the merger would have been
Even though FEBTC employees had no choice or
mandatory on the surviving corporation.
control over the merger of their employer with
In the present case, the merger was voluntarily BPI, they had a choice whether or not they would
entered into by both banks presumably for some allow themselves to be absorbed by
mutually acceptable consideration. In fact, the BPI. Certainly nothing prevented the FEBTC’s
Corporation Code does not also mandate the employees from resigning or retiring and seeking
absorption of the employees of the non-surviving employment elsewhere instead of going along
corporation by the surviving corporation in the with the proposed absorption.
case of a merger.

MNI NOTES |143


Employment is a personal consensual contract In 2001, the Bangko Sentral ng Pilipinas (BSP)
and absorption by BPI of a former FEBTC issued the Manual of Regulations for Banks
employee without the consent of the employee is (MoRB). Section X338 which states that Banks
in violation of an individual’s freedom to may provide financial assistance to their officers
contract. and employees, as part of their fringe benefits
program, to meet housing, transportation,
It would have been a different matter if there was household and personal needs of their officers
an express provision in the articles of merger that and employees. Financing plans and amendments
as a condition for the merger, BPI was being thereto shall be with prior approval of the BSP.
required to assume all the employment contracts
of all existing FEBTC employees with the Pursuant to the above-cited provision, respondent
conformity of the employees. In the absence of Hongkong and Shanghai Banking Corporation
such a provision in the articles of merger, then Limited (HSBC), submitted its Financial
BPI clearly had the business management Assistance Plan (Plan) to the BSP for approval.
decision as to whether or not employ FEBTC’s The Plan allegedly contained a credit checking
employees. FEBTC employees likewise retained proviso stating that "repayment defaults on
the prerogative to allow themselves to be existing loans and adverse information on outside
absorbed or not; otherwise, that would be loans will be considered in the evaluation of loan
tantamount to involuntary servitude. applications."

Meanwhile, petitioner Hongkong Bank


Hongkong Bank Independent Labor Union v. Independent Labor Union (HBILU), the
Hongkong and Shanghai Bank GR 218930 incumbent bargaining agent of HSBC's rank-and-
February 28, 2018 file employees, entered into a CBA with the bank
covering the period from April 1, 2010 to March
DOCTRINE: 31, 2012.
Unilateral amendments to the CBA violate
When the CBA was about to expire, the parties
Article 253 of the Labor Code. The provisions of
started negotiations for a new one to cover the
the CBA must be respected since its terms and
period from April 1, 2012 to March 31, 2017.
conditions constitute the law between the parties. During the said negotiations, HSBC proposed
And until a new CBA is executed by and between
amendments to provisions in Salary Loan
the parties, they are duty-bound to keep the status
allegedly to align the wordings of the CBA with
quo and to continue in full force and effect the
its BSP-approved Plan. HBILU vigorously
terms and conditions of the existing agreement. objected to the proposed amendments, claiming
This finds basis under Article 253 of the Labor
that their insertions would curtail its members'
Code, which provides duty to bargain
availment of salary loans. This, according to the
collectively when there exists a collective
Union, violates the existing exceptions set forth
bargaining agreement. in BSP Circular 423, Series of 2004, and Section
FACTS: X338.3 of the MoRB. In view of HBILU's

MNI NOTES |144


objection, HSBC withdrew its proposed It ruled in favor of HSBC. It held that respondent,
amendments and, consequently, Article XI as an employer, has the right to issue and
remained unchanged. implement guidelines for the availment of loan
accommodations under the CBA as part of its
Despite the withdrawal of the proposal, HSBC management prerogative.
sent an e-mail to its employees on April 20, 2012
concerning the enforcement of the Plan, CA:
including the Credit Checking provisions thereof.
Thereafter, HBILU member Vince Mananghaya It sustained the findings and conclusions of the
(Mananghaya) applied for a loan under the NCMB-PVA in toto on the ratiocination that
provisions of Article XI of the CBA. His first HSBC was merely complying with Section X338
loan application in March 2012 was approved, of the MoRB when it submitted the Plan to BSP.
but adverse findings from the external checks on When BSP, in turn, approved the said Plan,
his credit background resulted in the denial of his HSBC became legally bound to enforce its
September application. provisions, including the conduct of external
credit checks on its loan applicants. The appellate
HBILU then raised the denial as a grievance issue court further ruled that the Plan should be deemed
with the National Conciliation Mediation Board incorporated in the CBA because it is a regulatory
(NCMB). It argued that the imposition of an requirement of BSP without which the salary
additional requirement — the external credit loan provisions of the CBA are rendered
checking prior to approval of any loan inoperative.
application under Article XI of the CBA — is not
sanctioned under the CBA. The Union ISSUE:
emphasized that under the terms of Article XI,
Whether or not the HSBC could validly enforce
there is no such requirement and that it cannot,
the credit-checking requirement under its BSP-
therefore, be unilaterally imposed by HSBC.
approved Plan in processing the salary loan
Justifying its denial of the loan application,
applications of covered employees even when the
HSBC countered that the external credit check
said requirement is not recognized under the
conducted in line with Mananghaya's loan
CBA.
application was merely an implementation of the
BSP-approved Plan. The adoption of the Plan, RULING:
HSBC stressed, is a condition sine qua non for
any loan grant under Section X338 of the MoRB. No. The basic law of the land guarantees the
Moreover, the Credit Check policy has been in rights of workers to collective bargaining and
place since 2003, and is a sound practice in the negotiations as well as to participate in policy and
banking industry to protect the interests of the decision-making processes affecting their rights
public and preserve confidence in banks. and benefits. We deem it necessary to remind
HSBC of the basic and well-entrenched rule that
NCMB: although jurisprudence recognizes the validity of
the exercise by an employer of its management

MNI NOTES |145


prerogative and will ordinarily not interfere with negotiated for its inclusion thereon as early as the
such, this prerogative is not absolute and is April 1, 2010 to March 31, 2012 CBA which it
subject to limitations imposed by law, collective entered into with HBILU. However, the express
bargaining agreement, and general principles of provisions of said CBA inked by the parties
fair play and justice. Indeed, being a product of clearly make no reference to the Plan.
said constitutionally-guaranteed right to
participate, the CBA is, therefore, the law It appears that, based on its actuations, HSBC
between the parties and they are obliged to never intended to apply the credit checking item
comply with its provisions. under the Plan to salary loans under the CBA.
Otherwise, it would have enforced such
Unilateral amendments to the CBA violate requirement from the moment the salary loans
Article 253 of the Labor Code. The provisions of provisions under the old CBA were implemented,
the CBA must be respected since its terms and which it did not. It may be that said requirement
conditions constitute the law between the parties. was being applied to other types of loans under
And until a new CBA is executed by and between the Plan, but based on the evidence presented,
the parties, they are duty-bound to keep the status this court cannot say the same for salary loans
quo and to continue in full force and effect the under the CBA.
terms and conditions of the existing agreement.
This finds basis under Article 253 of the Labor The Court is of the view that tolerating HSBC's
Code, which provides duty to bargain conduct would be tantamount to allowing a
collectively when there exists a collective blatant circumvention of Article 253 of the Labor
bargaining agreement. Code. It would contravene the express
prohibition against the unilateral modification of
In the present controversy, it is clear from the a CBA during its subsistence and even thereafter
arguments and evidence submitted that the Plan until a new agreement is reached. It would unduly
was never made part of the CBA. The subsequent license HSBC to add, modify, and ultimately
implementation of the Plan's external credit further restrict the grant of Salary Loans beyond
check provisions in relation to employee loan the terms of the CBA by simply adding stringent
applications under Article XI of the CBA was requirements in its Plan, and having the said Plan
then an imposition solely by HSBC. approved by BSP in the guise of compliance with
the MoRB.
Thus, no other conclusion can be had in this
factual milieu other than the fact that HSBC's SONEDCO Workers Free Labor Union v.
enforcement of credit checking on salary loans Universal Robina Corporation GR 220383
under the CBA invalidly modified the latter's October 5, 2016
provisions thereon through the imposition of
additional requirements which cannot be found Doctrine: An employer who refuses to bargain
anywhere in the CBA. If it were true that said with the union and tries to restrict its bargaining
credit checking under the Plan covers salary power is guilty of unfair labor practice. In
loans under the CBA, then the bank should have determining whether an employer has not

MNI NOTES |146


bargained in good faith, the totality of all the acts receive the benefits, employees had to sign a
of the employer at the time of negotiations must waiver stating that the new CBA shall only be
be taken into account. effective January 1, 2008. Several SONEDCO
Workers Free Labor Union members refused to
FACTS: sign the 2007 waiver. Hence, they did not receive
the benefits given to other members of the
In 2002, Universal Robina Corporation Sugar bargaining unit who had done so.
Division-Southern Negros Development
Corporation (URC-SONEDCO) and Philippine In 2008, URC-SONEDCO offered the same
Agricultural Commercial and Industrial Workers arrangement. It extended an additional
Union (PACIWU-TUCP), then the exclusive ₱16.00/day wage increase to employees who
bargaining representative of URC-SONEDCO's would agree that any CBA negotiated for that
rank-and-file employees, entered into a year would only be effective on January 1, 2009.
Collective Bargaining Agreement (2002 CBA). Several members of SONEDCO Workers Free
Under the 2002 CBA, rank-and-file employees Labor Union again refused to waive their rights.
were entitled to a wage increase of P14.00/day Consequently, they did not receive the wage
for 2002 and P12.00/day for the succeeding years increase which already amounted to a total of
until 2006. ₱32.00/day, beginning 2009.

Days after the 2002 CBA was signed, a On July 2, 2009, SONEDCO Workers Free
certification election was conducted. SONEDCO Labor Union and its members who refused to
Workers Free Labor Union won and replaced sign the 2007 and 2008 waivers filed a complaint
PACIWU-TUCP as the exclusive bargaining for unfair labor practices against URC-
representative. SONEDCO. They argued that the requirement of
a waiver prior to the release of the wage increase
URC-SONEDCO consistently refused to constituted interference to the employees' right to
negotiate a new CBA with SONEDCO Workers self-organization, collective bargaining, and
Free Labor Union, despite several demands from concerted action. They asked that they be granted
SONEDCO Workers Free Labor Union. a ₱16.00/day wage increase for 2007 and an
additional ₱l6.00/day wage increase for 2008.8
In 2006, the 2002 CBA expired with no new SONEDCO Workers Free Labor Union also
collective bargaining agreement being signed. demanded a continuing wage increase of
Respondent repeatedly refused to meet and ₱32.00/day "from January 1, 2009 onwards."
bargain with SONEDCO Workers Free Labor
Union, the exclusive bargaining agent of its rank- Both the NLRC and the CA found URC-
and-file employees. SONEDCO not guilty of unfair labor practice.
Nonetheless, they ordered URC-SONEDCO to
In 2007, while there was no CBA in effect, URC- give petitioners the same benefits their co-
SONEDCO offered, among other benefits, a workers received in 2007 and 2008. However,
₱16.00/day wage increase to their employees. To SONEDCO Workers Free Labor Union's claim

MNI NOTES |147


for the 2009 wage increase was denied. Since a ARTICLE 259. [248]Unfair
new CBA was already in effect by 2009, this Labor Practices of
CBA governed the relationship between the Employers. — It shall be
management and the union. unlawful for an employer to
commit any of the following
In their Motion for Partial Reconsideration, unfair labor practices:
petitioners ask for four (4) awards: 1) a (a) To interfere with, restrain
P16.00/day wage increase for 2007; 2) another or coerce employees in
P16.00/day wage increase for 2008; 3) the 2009 the exercise of their right to
wage increase, which is a "continuing wage self-organization;
increase," 21 of P32.00/day from January 1, 2009 xxx xxx xxx
to present, and 4) attorney's fees. (e) To discriminate in regard
to wages, hours of work and
The Court already granted the wage increases for other terms and conditions of
2007 and 2008 in its October 5, 2016 Decision. employment in order to
encourage or discourage
This 2017 Resolution stems from the Motion for membership in any labor
Partial Reconsideration filed by SONEDCO organization.
Workers Free Labor Union. The concerned xxx xxx xxx
SONEDCO Workers Free Labor Union members (g) To violate the duty to
are asking that the wage increase given to their bargain collectively as
fellow employees be awarded to them as well. prescribed by this Code;
Their co-workers of the same rank are allegedly xxx xxx xxx
earning P32.00/day more than they are receiving.
Under this provision, an employer is guilty of
ISSUE:
unfair labor practice when it fails in its duty to
bargain in good faith.
w/n respondent is guilty of ULP

RULING: In ruling that respondent did not commit unfair


labor practice, the National Labor Relations
Yes, respondent is guilty of unfair labor Commission and the Court of Appeals failed to
practice. consider the totality of respondent's acts, which
The Court of Appeals failed to take into showed that it violated its duty to bargain
account that unfair labor practice not only collectively. This constitutes unfair labor
involves acts that violate the right to self- practice under Article 259 (g) of the Labor
organization but also covers several acts Code.
enumerated in Article 259 of the Labor Code,
thus: When petitioners held a conference on May 26,
2003, respondent refused to attend. Because
respondent failed to appear in the conference,

MNI NOTES |148


petitioners wrote their demands in a letter B. “Nakasaad sa Kontrata”: The Collective
sometime in July 2003. Bargaining Agreement

The letter included, among others, a wage Substitution/ Absorption Doctrine


increase of P50.00/day from September 2003 to
2006. Instead of explaining its non-attendance to In cases where the contract-bar principle is not
the conference or making a counter-offer, applicable (as when the CBA was not duly
respondent replied on August 15, 2003 registered) the EEs may change their CBAgent,
acknowledging the receipt and contents of the but the CBA continues to bind them up to the
July 2003 letter but invoking the 2002 Collective expiration date.
Bargaining Agreement as an excuse not to
The doctrine: EEs cannot revoke a validly
answer petitioners' demands to negotiate. This is
executed CBA by the simple expedient of
contrary to Article 261 of the Labor Code, which
changing their bargaining agent.
requires the other party to reply within 10 days
from receipt of the written demand: The new agent is obliged to respect the
ARTICLE 261. [250] Procedure in CBA, although it could negotiate for the
Collective Bargaining. — The following shortening of the life of the said
procedures shall be observed in collective agreement.
bargaining:
(a) When a party desires to negotiate an What is the effect of cancellation during the
agreement, it shall serve a written notice pendency of a case?
upon the other party with a statement of
its proposals. The other party shall make In case cancellation of a union registration is
a reply thereto not later than ten (10) made during the pendency of a case, the labor
calendar days from receipt of such organization whose registration is cancelled may
notice[.] still continue to be a party to the case without
necessity for substitution.
This was not respondent's only violation of
Whatever decision, however, may be rendered
Article 261. Respondent likewise failed to reply
therein shall only be binding on those members
to the collective bargaining agreement proposal
of the union who have not signified their desire
sent by petitioners on August 21, 2007. The
to withdraw from the case before its trial and
September 22, 2007 letter, sent with the
decision on the merits. (Itogon-Suyoc Mines, Inc.
agreement proposal, also went unheeded.
vs. Sangilo-Itogon Workers Union, 24 SCRA
Confederation for Unity, Recognition and 873).
Advancement of Government Employees v.
Automatic Incorporation Clause
Abad GR 200418 November 10, 2020

MNI NOTES |149


Automatic Renewal Clause execution. Any agreement on such other
provisions of the Collective Bargaining
This Agreement shall be automatically renewed Agreement entered into within six (6) months
for two (2) successive two (2) year periods, from the date of expiry of the term of such other
unless one Party provides the other Party with provisions as fixed in such Collective Bargaining
written notice of its desire not to renew this Agreement, shall retroact to the day immediately
Agreement at least ninety (90) days prior to the following such date. If any such agreement is
commencement of each such two-year period. entered into beyond six months, the parties shall
agree on the duration of retroactivity thereof.
CBA; Ratification
In case of a deadlock in the renegotiation of the
The CBA will be deemed ratified if it is
Collective Bargaining Agreement, the parties
approved by the majority of the employees
may exercise their rights under this Code.
covered by the bargaining unit. Without
ratification the CA cannot be registered. Mandatory provisions

Ratification of a CBA does not validate a void Effectivity


election of union officers because what the
membership ratified were the terms of the new The CBA is effective through its lifetime.
CBA and not the issue of union leadership.
Freedom period
Terms
The freedom period is the 60-day period prior
ART. 265. [253-A] Terms of a Collective to the expiration of the collective bargaining
Bargaining Agreement. agreement.

Any Collective Bargaining Agreement that the It is called the freedom period because it is the
parties may enter into shall, insofar as the time when the bargaining agent can validly serve
representation aspect is concerned, be for a term notice to renegotiate the existing CBA.
of five (5) years. No petition questioning the
majority status of the incumbent bargaining agent It is the time when a union member can validly
shall be entertained and no certification election resign from the union and the time for a local
shall be conducted by the Department of Labor union to disaffiliate from its mother federation
and Employment outside of the sixty-day period without being subjected to sanctions.
immediately before the date of expiry of such
It is also the time for challenging the majority
five-year term of the Collective Bargaining
status of the incumbent collective bargaining
Agreement.
agent through a petition for certification election.
All other provisions of the Collective
Bargaining Agreement shall be renegotiated
not later than three (3) years after its

MNI NOTES |150


Bargaining deadlock specific information submitted in confidence
shall be disclosed unless authorized by the
No representation question may be entertained if, Secretary, or when it is at issue in any judicial
before the filing of a petition for certification election,
litigation, or when public interest or national
a bargaining deadlock to which an incumbent or
security so requires.
certified bargaining agent is a party, had been
submitted to conciliation or arbitration or had become
Within thirty (30) days from the execution of a
the subject of a valid notice of strike or lockout.
Collective Bargaining Agreement, the parties
o Purpose: To ensure stability in the shall submit copies of the same directly to the
relationship of the workers and the Bureau or the Regional Offices of the
management. Department of Labor and Employment for
registration accompanied with verified proofs of
Remedies in Case of Deadlock its posting in two conspicuous places in the place
of work and ratification by the majority of all the
(a) Call upon the National Conciliation and
workers in the bargaining unit. The Bureau or
Mediation Board to assist them in arriving at an
Regional Offices shall act upon the application
amicable settlement;
for registration of such Collective Bargaining
(b) Submit the matter for compulsory arbitration Agreement within five (5) calendar days from
by filing a complaint with the National Labor receipt thereof. The Regional Offices shall
Relations Commission; furnish the Bureau with a copy of the Collective
Bargaining Agreement within five (5) days from
(c) Submit the matter for resolution by a its submission.
voluntary arbitrator; or
The Bureau or Regional Office shall assess the
(d) Declare a strike or lockout. employer for every Collective Bargaining
Agreement a registration fee of not less than
Article 237 and 265, LC one thousand pesos (P1,000.00) or in any other
amount as may be deemed appropriate and
ART. 237. [231] Registry of Unions and File of
necessary by the Secretary of Labor and
Collective Bargaining Agreements.
Employment for the effective and efficient
The Bureau shall keep a registry of legitimate administration of the Voluntary Arbitration
labor organizations. The Bureau shall also Program. Any amount collected under this
maintain a file of all collective bargaining provision shall accrue to the Special Voluntary
agreements and other related agreements and Arbitration Fund.
records of settlement of labor disputes and copies
The Bureau shall also maintain a file, and shall
of orders and decisions of voluntary arbitrators or
undertake or assist in the publication of all final
panel of voluntary arbitrators. The file shall be
decisions, orders and awards of the Secretary of
open and accessible to interested parties under
Labor and Employment, Regional Directors and
conditions prescribed by the Secretary of
the Commission.
Labor and Employment, provided that no

MNI NOTES |151


ART. 265. [253-A] Terms of a Collective filing a complaint with the Regional Branch of
Bargaining Agreement. the Commission.

Any Collective Bargaining Agreement that the


parties may enter into shall, insofar as the
representation aspect is concerned, be for a term MERALCO v. Quisumbing GR 127598
of five (5) years. No petition questioning the January 27, 1999
majority status of the incumbent bargaining agent
shall be entertained and no certification election
shall be conducted by the Department of Labor Benguet Consolidated, Inc. v. BCI Employees
and Employment outside of the sixty-day period and Worker’s Union GR L-24711 April 30,
immediately before the date of expiry of such 1968
five-year term of the Collective Bargaining
Agreement.

All other provisions of the Collective Capitol Medical Center Alliance of Concerned
Bargaining Agreement shall be renegotiated Employees v. Laguesma GR 118915 February
not later than three (3) years after its 4, 1997
execution. Any agreement on such other
provisions of the Collective Bargaining Doctrine: Even if one year had lapsed since the
Agreement entered into within six (6) months time of declaration of a final certification result,
from the date of expiry of the term of such other and there is no collective bargaining deadlock,
provisions as fixed in such Collective Bargaining there is no grave abuse of discretion committed
Agreement, shall retroact to the day immediately by the DOLE when it ruled against another
following such date. If any such agreement is certification election filed by another union
entered into beyond six months, the parties shall where the delay in forging of the CBA could not
agree on the duration of retroactivity thereof. be attributed to the fault of the union who won
the earlier certification election.
In case of a deadlock in the renegotiation of the
Collective Bargaining Agreement, the parties FACTS:
may exercise their rights under this Code.
Med-Arbiter Rasidali C. Abdullah issued an
Section 7, Rule XIV; Rule XV, Book 5 Order which granted respondent union’s
Omnibus Rules Implementing the LC petition for certification election among the rank-
and-file employees of the Capitol Medical
Section 7, Rule XIV Center.

SECTION 7. Right to contest dismissal. — Respondent CMC appealed the Order to the
Any decision taken by the employer shall be Office of the Secretary by questioning the legal
without prejudice to the right of the worker to status of respondent union’s affiliation with the
contest the validity or legality of his dismissal by Alliance of Filipino Workers (AFW). respondent

MNI NOTES |152


union registered itself independently and the hospital to negotiate a collective bargaining
withdrew the petition which had earlier been agreement with the said respondent union.
granted. Thereafter, it filed another petition for
certification election. RULING:

Med-Arbiter Manases T. Cruz issued an order We do not subscribe to petitioner’s contention.


granting the petition for certification election.
As correctly observed by the Solicitor General,
elections were finally held a total of 380 votes petitioner is merely “nit-picking, vainly trying to
cast. make a monumental issue out of a negligible
error of the public respondent.
respondent union = 204 votes
While it is true that, in the case at bench, one year
168 = no union had lapsed since the time of declaration of a final
certification result, and that there is no collective
8 = spoiled ballots bargaining deadlock, public respondent did not
commit grave abuse of discretion when it ruled
Med-Arbiter Cruz issued an Order certifying
in respondent union’s favor since the delay in
respondent union as the sole and exclusive
the forging of the CBA could not be attributed to
bargaining representative of the rank-and-file
the fault of the latter
employees at CMC.
CMC refused to negotiate with respondent union
Respondent CMC’s basic contention was the
and instead challenged the latter’s legal
supposed pendency of its petition for cancellation
personality through a petition for cancellation of
of respondent union’s certificate of registration,
the certificate of registration which eventually
wherein Med-Arbiter Adap issued an order
reached this Court.
declaring respondent union’s certificate of
registration as null and void. But was reversed by There is a deadlock when there is a complete
OIC of the BLR. blocking or stoppage resulting from the action of
equal and opposed forces.
In spite of the refusal of respondent CMC,
respondent union still persisted in its demand Thus, we can only conclude that CMC was
for CBA negotiations, claiming that it has already unwilling to negotiate and reach an agreement
been declared as the sole and exclusive with respondent union. CMC has not at any
bargaining agent of the rank-and-file employees instance shown willingness to discuss the
of the hospital. economic proposals given by respondent union.

ISSUE:

whether or not public respondent committed Caltex Refinery Employees Association v.


grave abuse of discretion in dismissing the Brillantes GR 123782 September 16, 1997
petition for certification election, and in directing

MNI NOTES |153


University of the East v. Pepanio GR 193897
January 13, 2013

FVC-Labor Union-PTGWO v. SANAMA-


FVC-SIGLO GR 176249 Nov. 27, 2009

Social Housing Employees Association, Inc. v.


Social Housing Finance Corp. GR 237729
October 14, 2020

MNI NOTES |154


MODULE 6
hearing and resolution of all cases involving unfair
labor practices. They shall resolve such cases within
Once a party fails to observe good faith in the thirty (30) calendar days from the time they are
establishment at any stage (from creation of a submitted for decision.
labor union until collective bargaining), the
Recovery of civil liability in the administrative
entire relationship and transactions are affected,
proceedings shall bar recovery under the Civil Code.
and industrial disputes arise. When industrial
conflicts occur, both parties have remedies in No criminal prosecution under this Title may be
order to correct if not, restore some semblance of instituted without a final judgment finding that an
harmony in the establishment. unfair labor practice was committed, having been first
obtained in the preceding paragraph. During the
A. “Unfair Ka!”: Unfair Labor Practices in pendency of such administrative proceeding, the
the Workplace running of the period of prescription of the criminal
offense herein penalized shall be considered
Article 258-260 LC interrupted: Provided, however, That the final
judgment in the administrative proceedings shall not
ART. 258. [247] Concept of Unfair Labor Practice be binding in the criminal case nor be considered as
and Procedure for Prosecution Thereof. evidence of guilt but merely as proof of compliance
of the requirements therein set forth.
Unfair labor practices violate the constitutional right
of workers and employees to self-organization, are ART. 259. [248] Unfair Labor Practices of
inimical to the legitimate interests of both labor and Employers.
management, including their right to bargain
collectively and otherwise deal with each other in an It shall be unlawful for an employer to commit any of
atmosphere of freedom and mutual respect, disrupt the following unfair labor practices:
industrial peace and hinder the promotion of healthy
and stable labor-management relations.
(a) To interfere with, restrain or coerce employees in
Consequently, unfair labor practices are not only the exercise of their right to self-organization;
violations of the civil rights of both labor and
management but are also criminal offenses against the (b) To require as a condition of employment that a
State which shall be subject to prosecution and person or an employee shall not join a labor
punishment as herein provided. organization or shall withdraw from one to which he
belongs;
Subject to the exercise by the President or by the
Secretary of Labor and Employment of the powers (c) To contract out services or functions being
vested in them by Articles 263 and 264 of this performed by union members when such will
Code,201 the civil aspects of all cases involving interfere with, restrain or coerce employees in the
unfair labor practices, which may include claims for exercise of their right to self-organization;
actual, moral, exemplary, and other forms of
damages, attorney’s fees, and other affirmative relief, (d) To initiate, dominate, assist or otherwise interfere
shall be under the jurisdiction of the Labor Arbiter. with the formation or administration of any labor
The Labor Arbiters shall give utmost priority to the

MNI NOTES |155


organization, including the giving of financial or ART. 260. [249] Unfair Labor Practices of Labor
other support to it or its organizers or supporters; Organizations.

(e) To discriminate in regard to wages, hours of work It shall be unfair labor practice for a labor
and other terms and conditions of employment in organization, its officers, agents or representatives:
order to encourage or discourage membership in any
labor organization. Nothing in this Code or in any (a) To restrain or coerce employees in the exercise of
other law shall stop the parties from requiring their right to self-organization. However, a labor
membership in a recognized collective bargaining organization shall have the right to prescribe its own
agent as a condition for employment, except those rules with respect to the acquisition or retention of
employees who are already members of another union membership;
at the time of the signing of the collective bargaining
agreement. (b) To cause or attempt to cause an employer to
discriminate against an employee, including
Employees of an appropriate bargaining unit who are discrimination against an employee with respect to
not members of the recognized collective bargaining whom membership in such organization has been
agent may be assessed a reasonable fee equivalent to denied or to terminate an employee on any ground
the dues and other fees paid by members of the other than the usual terms and conditions under which
recognized collective bargaining agent, if such non- membership or continuation of membership is made
union members accept the benefits under the available to other members;
collective bargaining agreement: Provided, That the
individual authorization required under Article 242, (c) To violate the duty, or refuse to bargain
paragraph (o) of this Code204 shall not apply to the collectively with the employer, provided it is the
non-members of the recognized collective bargaining representative of the employees;
agent;
(d) To cause or attempt to cause an employer to pay
(f) To dismiss, discharge or otherwise prejudice or or deliver or agree to pay or deliver any money or
discriminate against an employee for having given or other things of value, in the nature of an exaction, for
being about to give testimony under this Code; services which are not performed or not to be
performed, including the demand for fee for union
(g) To violate the duty to bargain collectively as negotiations;
prescribed by this Code;
(e) To ask for or accept negotiation or attorney’s fees
(h) To pay negotiation or attorney’s fees to the union from employers as part of the settlement of any issue
or its officers or agents as part of the settlement of any in collective bargaining or any other dispute; or
issue in collective bargaining or any other dispute; or
(f) To violate a collective bargaining agreement.
(i) To violate a collective bargaining agreement. The
provisions of the preceding paragraph The provisions of the preceding paragraph
notwithstanding, only the officers and agents of notwithstanding, only the officers, members of
corporations, associations or partnerships who have governing boards, representatives or agents or
actually participated in, authorized or ratified unfair members of labor associations or organizations who
labor practices shall be held criminally liable. have actually participated in, authorized or ratified
unfair labor practices shall be held criminally liable.

MNI NOTES |156


Insular Life Assurance Employees Asso v. Unions filed on January 27, 1958 a notice of strike
Insular Life, GR No. L-25291, Jan. 30, 1971 for “deadlock on collective bargaining.” The issue
was dropped subsequently (in short, nagkasundo).
Doctrine:
But, the parties negotiated on the labor demands but
FACTS:
with no satisfactory result due to a stalemate on the
The Insular Life Assurance Co., Ltd., Employees matter of salary increases.
Association-NATU, FGU Insurance Group
Workers & Employees Association-NATU, and Meanwhile, 87 unionists were reclassified as
Insular Life Building Employees Association- supervisors without increase in salary nor in
NATU (hereinafter referred to as the Unions), while responsibility while negotiations were going on in
still members of the Federation of Free Workers the Department of Labor after the notice to strike
(FFW), entered into separate CBAs with the Insular was served on the Companies. These employees
Life Assurance Co., Ltd. and the FGU Insurance resigned from the Unions.
Group (hereinafter referred to as the Companies).
On May 21, 1958 the Companies through their
Two of the lawyers of the Unions then were Felipe acting manager and president, sent to each of the
Enaje and Ramon Garcia; the latter was formerly the strikers a letter (exhibit A) quoted verbatim as
secretary-treasurer of the FFW and acting president of follows:
the Insular Life/FGU unions and the Insular Life
Building Employees Association. Garcia, as such We recognize it is your privilege both to
acting president, in a circular issued in his name and strike and to conduct picketing.
signed by him, tried to dissuade the members of the
Unions from disaffiliating with the FFW and joining However, if any of you would like to come
the National Association of Trade Unions (NATU), to back to work voluntarily, you may:
no avail.

Enaje and Garcia soon left the FFW and secured 1. Advise the nearest police officer
employment with the Anti-Dummy Board of the or security guard of your
Department of Justice. intention to do so.

Thereafter, the Companies hired Garcia in the latter 2. Take your meals within the
part of 1956 as assistant corporate secretary and legal office.
assistant in their Legal Department. Enaje was hired
as personnel manager of the Companies, and was 3. Make a choice whether to go
likewise made chairman of the negotiating panel for home at the end of the day or to
the Companies in the collective bargaining with the sleep nights at the office where
Unions. comfortable cots have been
prepared.
Unions jointly submitted proposals to the Companies;
negotiations were conducted on the Union’s 4. Enjoy free coffee and occasional
proposals, but these were snagged by a deadlock on movies.
the issue of union shop, as a result of which the

MNI NOTES |157


5. Be paid overtime for work continued to operate and will continue to
performed in excess of eight do so with or without you.
hours. • If you are still interested in continuing in
the employ of the Group Companies, and
6. Be sure arrangements will be if there are no criminal charges pending
made for your families. against you, we are giving you until 2
June 1958 to report for work at the home
7. The decision to make is yours —
office. If by this date you have not yet
whether you still believe in the
reported, we may be forced to obtain your
motives of the strike or in the
replacement.
fairness of the Management.
• Before, the decisions was yours to make.
• So it is now.

Unions, however, continued on strike, with the Incidentally, all of the more than 120 criminal
exception of a few unionists who were convinced to charges filed against the members of the Unions,
desist by the aforesaid letter except 3, were dismissed by the fiscal’s office and
by the courts. These three cases involved “slight
From the date the strike was called on May 21, 1958, physical injuries” against one striker and “light
until it was called off on May 31, 1958, some coercion” against two others.
management men tried to break thru the Unions’
picket lines xxx succeeded in penetrating the At any rate, because of the issuance of the writ of
picket lines in front of the Insular Life Building, preliminary injunction against them as well as the
thus causing injuries to the picketers and also to ultimatum of the Companies giving them until June 2,
the strike-breakers due to the resistance offered by 1958 to return to their jobs or else be replaced, the
some picketers. striking employees decided to call off their strike
and to report back to work on June 2, 1958.
Alleging that some non-strikers were injured and
with the use of photographs as evidence, the However, before readmitting the strikers, the
Companies then filed criminal charges against the Companies required them not only to secure
strikers with the City Fiscal’s Office of Manila. clearances from the City Fiscal’s Office of Manila
but also to be screened by a management
Another letter was sent by the company to the
committee among the members of which were Enage
individual strikers:
and Garcia.

• The first day of the strike was last 21


The screening committee initially rejected 83 strikers
May 1958. with pending criminal charges. However, all non-
• Our position remains unchanged and the strikers with pending criminal charges which arose
strike has made us even more convinced from the breakthrough incident were readmitted
of our decision. immediately by the Companies without being
• We do not know how long you intend to required to secure clearances from the fiscal’s office.
stay out, but we cannot hold your
positions open for long. We have

MNI NOTES |158


Subsequently, when practically all the strikers had ISSUE:
secured clearances from the fiscal’s office, the
Companies readmitted only some but adamantly
Whether the respondent company is guilty of unfair
refused readmission to 34 officials and members of
labor practice - YES
the Unions who were most active in the strike, on
the ground that they committed “acts inimical to
the interest of the respondents,” without however RULING:
stating the specific acts allegedly committed.
The act of an employer in notifying absent employees
Some 24 of the above number were ultimately individually during a strike following unproductive
notified months later that they were being dismissed efforts at collective bargaining that the plant would be
retroactively as of June 2, 1958 and given separation operated the next day and that their jobs were open
pay checks computed under Rep. Act 1787, while for them should they want to come in has been held
others (ten in number) up to now have not been to be an unfair labor practice, as an active
readmitted although there have been no formal interference with the right of collective bargaining
dismissal notices given to them. through dealing with the employees individually
instead of through their collective bargaining
representatives.
CIR prosecutor filed a complaint for unfair labor
practice against the Companies under Republic Act Although the union is on strike, the employer is still
875. under obligation to bargain with the union as the
employees’ bargaining representative.
The complaint specifically charged the Companies Individual solicitation of the employees or visiting
with their homes, with the employer or his representative
urging the employees to cease union activity or cease
(1) interfering with the members of the striking, constitutes unfair labor practice.
Unions in the exercise of their right to
concerted action, by sending out individual All the above-detailed activities are unfair labor
letters to them urging them to abandon their practices because they tend to undermine the
strike and return to work, with a promise of concerted activity of the employees, an activity to
comfortable cots, free coffee and movies, and which they are entitled free from the employer’s
paid overtime, and, subsequently, by warning molestation.
them that if they did not return to work on or
before June 2, 1958, they might be replaced; Indeed, when the respondents offered reinstatement
and and attempted to “bribe” the strikers with
“comfortable cots,” “free coffee and occasional
movies,” “overtime” pay for “work performed in
(2) discriminating against the members of excess of eight hours,” and “arrangements” for their
the Unions as regards readmission to work families, so they would abandon the strike and return
after the strike on the basis of their union to work, they were guilty of strike-breaking and/or
membership and degree of participation in union-busting and, consequently, of unfair labor
the strike. practice.

MNI NOTES |159


It is equivalent to an attempt to break a strike for an the active from the less active unionists on the basis
employer to offer reinstatement to striking employees of their militancy, or lack of it, on the picket lines.
individually, when they are represented by a union, Unionists belonging to the first category were refused
since the employees thus offered reinstatement are readmission even after they were able to secure
unable to determine what the consequences of clearances from the competent authorities with
returning to work would be. respect to the criminal charges filed against them.

ULP also: It is noteworthy that — perhaps in an anticipatory


effort to exculpate themselves from charges of
Hiring of Enage and Garcia with attractive discrimination in the readmission of strikers returning
compensations; respondents reclassified 87 to work — the respondents delegated the power to
employees as supervisors without increase in salary readmit to a committee.
or in responsibility, in effect compelling these
employees to resign from their unions; respondents, III. Anent the third assignment of error, the record
thru their president and manager, respondent Jose M. shows that not a single dismissed striker was given
Olbes, brought three truckloads of non-strikers and the opportunity to defend himself against the
others, escorted by armed men, who, despite the supposed charges against him.
presence of eight entrances to the three buildings
occupied by the Companies, entered thru only one As earlier mentioned, when the striking employees
gate less than two meters wide and in the process, reported back for work on June 2, 1958, the
crashed thru the picket line posted in front of the respondents refused to readmit them unless they first
premises of the Insular Life Building. secured the necessary clearances; but when all, except
three, were able to secure and subsequently present
This resulted in injuries on the part of the picketers the required clearances, the respondents still refused
and the strike-breakers; respondents brought against to take them back.
the picketers criminal charges, only three of which
were not dismissed, and these three only for slight Indeed, the individual cases of dismissed officers
misdemeanors. As a result of these criminal actions, and members of the striking unions do not indicate
the respondents were able to obtain an injunction sufficient basis for dismissal.
from the court of first instance restraining the strikers
from stopping, impeding, obstructing, etc. the free
and peaceful use of the Companies’ gates, entrance
Foodbev International v. Ferrer GR No.
and driveway and the free movement of persons and
vehicles to and from, out and in, of the Companies’ 206795, September 16,2019
buildings.
FACTS:
Verily, the above actuations of the respondents
Petitioner Foodbev International (Foodbev) is a
before and after the issuance of the letters, exhibit A
partnership engaged in the food service industry by
and B, yield the clear inference that the said letters
providing after-sales support for specialized
formed of the respondents scheme to preclude if not
equipment, like hot and cold dispensers and displays.
destroy unionism within them.
Respondent Foodbev rank and file employees and
II. The respondents did not merely discriminate
members of Samahan ng Nagkakaisang
against all the strikers in general. They separated

MNI NOTES |160


Manggagawa ng Foodbev International Central for the dismissal of the case before her, and to pursue
(Samahan), a labor union established on May 31, the action filed before LA Que.
2008.
On August 13, 2008, respondents filed a Notice of
Respondent Bernadette Belardo (Bernadette) is a Dismissal or Withdrawal of Complaint without
managerial employee and spouse of respondent Prejudice.
Jever. She filed a complaint for illegal dismissal,
which was consolidated with the other cases. Foodbev drew up a written offer of wage, sack of rice,
and canned corned beef to the 13 union members in
From July 3 to 9, 2008, meetings were held between exchange for a waiver.
the union members, Foodbev managers, and
petitioner Lucila Dela Cruz (Lucila), Foodbev Lucila instructed Eroles to take the day off the next
president. day to convince the 13 respondents to accept their
offer.
Lucila asked their grievances and reasons in
establishing a union, and threatened to close Foodbev On August 21, 2008, Eroles informed Foodbev that
if the union activities persist. their offer was rejected. On July 16, 2009, LA Que
rendered a decision dismissing the four consolidated
Lucila reiterated to stop union activities and to complaints for violation of the rule against forum
withdraw from the union for the sake of their jobs. shopping.

Most of the union members did not resign, so The labor arbiter explained that while the filing of
Foodbev castigated them by conducting a written consolidated cases before his branch initially
examination exclusively for union members. involved dissimilar causes of action from the cases
filed before LA Azarraga, the subsequent amendment
It was only after Galela complained that other non- of the complaints to include unfair labor practice, and
union-member employees were made to take the the failure to inform his branch of the status of the
examination. pending complaints was a violation of the rule against
forum shopping.
Those who failed the examination were considered
guilty of violating Article VI, Section C4 of The NLRC established that respondents failed to
Foodbev's Code of Discipline on slowing down, disclose in their verification that there were other
dragging or limiting out. pending cases before LA Azarraga, which is a
violation of the rule against forum shopping.
On July 28, 2008, the five ice cream machine
technicians filed a complaint for illegal dismissal and The NLRC affirmed the dismissal of the four
money claims with the NLRC. complaints. The CA affirmed the labor tribunal's
finding that respondents committed forum shopping.
On August 12, 2008, Eroles returned from Isabela and
reported for work at Foodbev's office in Makati. He However, it deemed appropriate to resolve the
requested that his absence on August 11, 2008 be substantial issues presented as a dismissal on pure
counted against his leave credits. technicalities was frowned upon. On the claim of
unfair labor practice, the CA determined that Foodbev
During the hearing, LA Azarraga advised the was discouraging the formation of a union, and
respondents to secure the services of a lawyer, move

MNI NOTES |161


committed acts constituting unfair labor practice Therefore, the CA was correct in setting aside
based on the following evidence: the union's technical rules on forum shopping to give way to the
application for registration, the minutes of the more important Constitutional and statutory rights of
meeting between Foodbev's president and/or respondent workers. It is settled that a valid dismissal
managers and union members, the affidavits of mandates compliance with substantive and
Aquino and Pario, the acts of Carpio and Brosas, the procedural requirements.
blotter report, the transfer of the union president to
Isabela, the show cause memo, and the notices of The records reveal that Ferrer, Aquino, Trapago, and
termination. Pario were individually served with a show cause
memo notifying them of their violation of company
The CA ruled that the NLRC arbitrarily pronounced rules in order to explain in writing.
that there was no unfair labor practice despite the lack
of factual and legal bases. The Court observes several flaws in the show cause
memo.
The CA resolved that the burden to prove the validity
of the dismissal rests on the employer, and the proof First, the memo contains a general statement
must be based on substantial evidence. The CA found that a cockroach was found in the ice cream
that there was a dearth of evidence to prove that machine that respondents installed at Don
respondents refused to follow instructions for their Bosco Makati. It does not indicate when and
transfer to EMI. how the pest was discovered, and/or in which
part of the machine was it found.
It was further revealed that nine of the 11 employees
transferred to EMI were union members, which led Second, the memo draws a conclusion that
the CA to believe that the transfer was made to the fault lies on the respondents in the
prevent them from conducting union activities. absence of a proper investigation.

ISSUE: In sum, the memo does not comply with the


requirements laid down by the King of
Whether the CA committed a reversible error Kings Transport case. The respondents'
dismissal due to gross negligence resulting to
RULING: loss and damage to the company's reputation
and image, lacks factual foundation and
Unfair labor practices violate the constitutional disregards due process
rights of workers and employees to self-organization,
are inimical to the legitimate interests of both labor The memo stated that the three respondents were
and management, including their right to bargain absent on July 21, 2008. Ferrer and Aquino's memo
collectively and otherwise deal with each other in an stated their accumulated number of absences without
atmosphere of freedom and mutual respect; and indicating specific dates to establish habitual absence.
disrupt industrial peace and hinder the promotion The respondents' dismissal on the ground of habitual
of healthy and stable labor-management relations. absence lacks factual basis and violates procedural
requirements.
As the conscience of the government, it is the Court's
sworn duty to ensure that none trifles with labor However, this memo applies only to the charge of
rights. gross negligence, and does not include the charge of

MNI NOTES |162


habitual absence, serious misconduct, and willful The records reveal several instances to support unfair
disobedience. Since respondents were not formally labor practice, specifically union busting, the
charged of serious misconduct, fraud, and willful Minutes of the Meetings disclose that as early as July
breach of trust and confidence causing serious 2008, Lucila and Espeña had been discouraging the
damage and prejudice to the company, they were employees from joining the union and in participating
unable to defend their side and present evidence on in union activities. These evidence on record belie
their behalf. The termination notice clearly violates Foodbev's claim of ignorance on the existence of the
respondents' rights to due process. union. The fact that the examination was at first
limited to union members is in itself an unfair labor
Article 297 of the Labor Code listed gross and practice because it is discriminatory.
habitual neglect of duties by the employee as a
ground for termination of his/her services. Article 298 of the Labor Code mandates the
payment of separation pay to an employee
Respondents did not exhibit acts constituting gross terminated from the service.
negligence, nor did Foodbev cite other instances
when respondents failed to perform assigned tasks, Here, Foodbev's offer does not include separation
signifying habitual negligence. There was no pay, which is contrary to law. The discussions above
showing that respondents had deliberate or demonstrate Foodbev's unfair labor practices, which
thoughtless disregard for the cleaning procedure. create an unpleasant working atmosphere for
If at all, respondents are liable of simple negligence respondent union members and officers.
for failing to use robby vapor in sanitizing the
machine. They were targeted to take part in a written
examination, or prone to being transferred to another
The Court finds that respondent's dismissal from company or to another branch. They were urged to
employment is illegal due to several violations of file for resignation and accept a measly compensation
procedural and substantive requirements of the and goods, instead of full benefits under the law. If
Labor Code and its Implementing Rules. these will not work, their employment will be
terminated in order to dissolve the union.
Bernadette's verbal termination from employment
is a violation of her right to security of tenure, and was The facts undeniably point to interference and
done without just cause and due process under restraining respondents' right to self-organization,
Articles 294 and 297 of the Labor Code. and discriminate their terms and conditions of
employment, as enumerated in paragraphs (a) and
Eroles is susceptible to being transferred to another (e) of Article 259 of the Labor Code.
branch or company in the guise of training or
company practice, or verbal harassment similar to his This further supports respondents' allegation that they
dismissed co-workers. The insinuations to resign were targeted because of their union membership, and
and the successive termination from employment confirms that Foodbev is liable for union busting.
of union members had created a hostile working
environment, which convinced him to sacrifice his Mendoza v. Officers of Manila Water
employment and tantamount to constructive Employees Union, GR No. 201595, Jan. 25,
dismissal. 2016

MNI NOTES |163


Doctrine: In essence, unfair labor practice relates In 2008, during the freedom period and
to the commission of acts that transgress the negotiations for a new CBA with
workers’ right to organize. All the prohibited acts MWC, Mendoza joined another union,the Worker
constituting unfair labor practice in essence relate to sAssociation for Transparency, Empowerment and R
the workers’ right to self-organization. The term eform, All-Filipino Workers Confederation
unfair labor practice refers to that gamut of offenses (WATER-AFWC) and was elected as
defined in the Labor Code which, at their core, union President.
violates the constitutional right of workers and
employees to self-organization. Other MWEU members were inclined to join
WATER-AFWC, but MWEU director Torres
FACTS: threatened that they would not get benefits from
the new CBA.
Petitioner Mendoza was a member of MWEU, a
DOLE-registered labor organization consisting of The MWEU leadership submitted a proposed
rank-and-file employees within Manila Water CBA which contained provisions to the effect that
Company (MWC) while the respondents were
MWEU officers. (1) in the event of retrenchment, non-
MWEU members shall be removed first,
In the April 2007 letter, Mendoza was warned of his and that upon the signing of the CBA,
failure to pay union dues (since he failed to authorize
the December 2006 checkoff). (2) only MWEU members shall receive a
signing bonus.
In the May 2007 letter, Mendoza was charged for
violation of CBL and was referred to MWEU October 2008, Mendoza filed a Complaint against
grievance committee for investigation. respondents for ULP, damages, and attorney’s fees
and accused the respondents of
On June 2007, Mendoza was, after notice and
hearing, suspended for 30 days through a unanimous (1) illegal termination (removal) from
approval due to the non-payment of union MWEU in connection with the events relative
dues. Mendoza’s appeal for the convening of to his non-payment of union dues;
General Membership Assembly had been repeatedly
(2) unlawful interference, coercion, and
denied (either by denial or by ignoring the appeal).
violation of the rights of MWC employees
Meanwhile, MWEU scheduled an election of to self-organization in connection with the
officers. Petitioner filed his certificate proposed CBA submitted by MWEU
of candidacy for Vice-President, but he was leadership,
disqualified for not being a member in
which petitioner claims contained provisions
good standing on account of his suspension.
that discriminated against non-MWEU members.
In addition, petitioner was charged with non-
Petitioner claims that he was suspended and expelled
payment of union dues for the third time was then on
from MWEU illegally as a result of the denial of his
expelled from the union per unanimous approval of
right to appeal his case to the general membership
Executive Board. His appeals to the General
assembly in accordance with the union’s CBL.
Membership Assembly were again unheeded.

MNI NOTES |164


On the other hand, respondents claimed that LA has When an MWEU member is suspended, he is
no jurisdiction, which is intra-union in nature and given the right to appeal such suspension within
BLR was the proper venue and that they were not three working days from the date of notice of said
guilty of ULP, discrimination, coercion or restraint. suspension, which appeal the MWEU Executive
LA: dismissed the complaint still premature: exhaust
Board is obligated to act upon by a simple
first all available remedies before resorting to majority vote.
compulsory arbitration) and referred back to Union
for General Assembly.
When the penalty imposed is expulsion, the
expelled member is given seven days from
notice of said dismissal and/or expulsion to
NLRC: reversed decision of LA but still
dismissed the complaint (lack of jurisdiction: intra- appeal to the Executive Board, which is required
union dispute) to act by a simple majority vote of its members.

CA: dismissed certiorari (lack of merit: no The Board’s decision shall then be
jurisdiction. Jurisdiction of BLR on union matters) approved/disapproved by a majority vote of the
general membership assembly in a meeting duly
ISSUE: called for the purpose. (pertinent provisions of
CBL were Article VI, Sections 2(a) and(d),
Whether the respondents were guilty of unfair labor
practice under Art. 249 (a) and (b) of the LC Section 3, and Article X, Sections 4 and 5)

RULING: The documentary evidence is clear that when


Mendoza received the suspension letter, he
LOWER COURTS IGNORED ULP immediately and timely filed a written
CHARGES (RE: DENIAL OF RIGHT TO APPEAL appeal. However, the Executive Board did not
UNDER CBL) act thereon. Then again, when he was charged
for the 3rd time and expelled from MWEU, his
• Unfair labor practices may be committed
both by the employer under Article 248
timely appeal was again not acted upon by
and by labor organizations under Article said board.
249 of the Labor Code. Petitioner contends
that respondents committed acts Thus, contrary to respondents’ argument that
constituting unfair labor practices – which petitioner lost his right to appeal when he failed
charge was particularly laid out in his to petition to convene the general assembly
pleadings, but that the Labor Arbiter, the through the required signature of 30% of the
NLRC, and the CA ignored it and simply union membership in good standing pursuant to
dismissed his complaint on the ground that Article VI, Section 2(a) of MWEU’s CBL or by
his causes of action were intra- or inter- a petition of the majority of the general
union in nature. membership in good standing under Article VI,
Section 3, this Court finds that petitioner was
MENDOZA WAS ILLEGALLY
illegally suspended for the second time and
SUSPENDED AND EXPELLED = ULP (LA’s
thereafter unlawfully expelled from MWEU
jurisdiction)

MNI NOTES |165


due to respondents failure to act on his written Article 248(a) declares it to be an ULP for an
appeals. Because respondents did not act on employer, among others, to “interfere with, restrain or
his two appeals, petitioner was coerce employees in the exercise of their right to self-
unceremoniously suspended, disqualified and organizaion”.
deprived of his right to run for the position of Article 249(a) makes it an unfair labor practice for a
MWEU Vice President in the September 14, labor organization to “restrain or coerce employees in
2007 election of officers, expelled from the exercise of their rights to self-organization…”
MWEU, and forced to join another union,
WATER-AFWC. The right of self-organization includes the right to
organize or affiliate with a labor union or determine
For these, respondents are guilty of ULP under which of two or more unions in an establishment to
Article 249(a) and (b) – that is, violation of join, and to engage in concerted activities with
petitioner’s right to self-organization, coworkers for purposes of collective bargaining
unlawful discrimination, and illegal through representatives of their own choosing, or for
their mutual aid and protection, i.e., the protection,
termination of his union membership– which
promotion, or enhancement of their rights and
case falls within the original and exclusive
interests.
jurisdiction of the Labor Arbiters, in
accordance with Article 217 of the Labor Code.

NATURE OF ULP (RELATES TO THE Cathay Pacific Steel v. CA, GR No.164561,


VIOLATION OF WORKER’S RIGHT TO August 30, 2006
ORGANIZE)
Doctrine: Supervisors can join unions. Article
In essence, unfair labor practice relates to the 212(m) of the Labor Code, as amended,
commission of acts that transgress the workers’
differentiates supervisory employees from
right to organize. managerial employees, to wit: supervisory employees
are those who, in the interest of the employer,
All the prohibited acts constituting unfair labor
effectively recommend such managerial actions, if
practice in essence relate to the workers’ right to self-
the exercise of such authority is not merely routinary
organization. The term unfair labor practice refers
or clerical in nature but requires the use of
to that gamut of offenses defined in the Labor Code
independent judgment; whereas, managerial
which, at their core, violates the constitutional right
employees are those who are vested with powers or
of workers and employees to self-organization.
prerogatives to lay down and execute management
Guaranteed to all employees or workers is the ‘right policies and/or hire, transfer, suspend, lay off, recall,
to self-organization and to form, join, or assist labor discharge, assign or discipline employees.
organizations of their own choosing for purposes of
collective bargaining.’ This is made plain by no less
FACTS:
than three provisions of the Labor Code of the
Petitioner Cathay Pacific Steel Corporation
Philippines. Article 243 of Labor Code provides for
(CAPASCO) hired private respondent
the coverage of employees’ right to self-organization.
Tamondong as Assistant to the Personnel Manager
for its Cainta Plant on 16 February 1990. Thereafter,

MNI NOTES |166


he was promoted to the position of organization of the union of supervisory personnel in
Personnel/Administrative Officer, and later to that of CAPASCO.
Personnel Superintendent.
Though private respondent Tamondong admitted
Sometime in June 1996, the supervisory personnel of
his active role in the formation of a union composed
CAPASCO launched a move to organize a union
of supervisory personnel in the company, he claimed
among their ranks, later known as private
that such was not a valid ground to terminate his
respondent CUSE.
employment because it was a legitimate exercise of
his constitutionally guaranteed right to self-
Private respondent Tamondong actively involved organization.
himself in the formation of the union and was even
elected as one of its officers after its creation.
In contrast, petitioner CAPASCO contended that by
virtue of private respondent Tamondong’s position
Consequently, petitioner CAPASCO sent a memo to as Personnel Superintendent and the functions
private respondent Tamondong requiring him to actually performed by him in the company, he was
explain and to discontinue from his union activities, considered as a managerial employee, thus, under the
with a warning that a continuance thereof shall law he was prohibited from joining a union as well as
adversely affect his employment in the company. from being elected as one of its officers.

Private respondent Tamondong ignored said Accordingly, petitioners maintained their argument
warning and made a reply letter invoking his right as that the dismissal of private respondent
a supervisory employee to join and organize a labor Tamondong was perfectly valid based on loss of trust
union. and confidence because of the latter’s active
participation in the affairs of the union.
In view of that, on 6 February 1997, petitioner
CAPASCO through a memo terminated the ISSUE:
employment of private respondent Tamondong on
the ground of loss of trust and confidence, citing his
Whether the dismissal of Tamondong constitutes
union activities as acts constituting serious disloyalty
unfair labor practice
to the company.

RULING:
Private respondent Tamondong challenged his
dismissal for being illegal and as an act involving
unfair labor practice by filing a Complaint for Tamondong may have possessed enormous powers
Illegal Dismissal and Unfair Labor Practice before the and was performing important functions that goes
NLRC, Regional Arbitration Branch IV. with the position of Personnel Superintendent,
nevertheless, there was no clear showing that he is at
liberty, by using his own discretion and disposition,
According to him, there was no just cause for his
to lay down and execute major business and
dismissal and it was anchored solely on his
operational policies for and in behalf of CAPASCO.
involvement and active participation in the

MNI NOTES |167


CAPASCO miserably failed to establish that Cainta Catholic School v. Cainta Catholic
Tamondong was authorized to act in the interest School Employees union, GR No.15102, May
of the company using his independent judgment. x 04, 2006

Tamondong may have been exercising certain FACTS:


important powers, such as control and supervision
On 6 March 1986, a CBA was entered into between
over erring rank-and-file employees, however, he
does not possess the power to hire, transfer, terminate, Cainta Catholic School (School) and the Cainta
or discipline erring employees of the company. Catholic School Employees Union (Union)
effective 1 January 1986 to 31 May 1989.

At the most, the record merely showed that Petitioner Balbago was appointed School Director in
Tamondong informed and warned rank-and-file April 1987. From this time, the Union became
employees with respect to their violations of inactive.
CAPASCO’s rules and regulations.
It was only after about 7 years (September 1993) that
the Union held an election of officers, with Mrs.
[Also, the functions performed by private
Llagas (President); Javier (Vice-President); Villegas
respondent such as] issuance of warning to
(Treasurer); and Santos (Secretary). Llagas was then
employees with irregular attendance and
the Dean of the Student Affairs while Villegas and
unauthorized leave of absences and requiring
Santos were Year-Level Chairmen.
employees to explain regarding charges of
abandonment of work, are normally performed by On October 1993, the School retired Llagas and
a mere supervisor, and not by a manager. Javier, who had rendered more than 20 years of
continuous service, pursuant to Section 2, Article
Being a supervisory employee of CAPASCO, he X of the CBA, to wit:
cannot be prohibited from joining or participating in
the union activities of private respondent CUSE, and An employee may be retired, either
in making such a conclusion, the Court of Appeals did upon application by the employee
not act whimsically, capriciously or in a despotic himself or by the decision of the
manner, rather, it was guided by the evidence Director of the School, upon
submitted before it. reaching the age of sixty
(60) or after having rendered at
least twenty (20) years of service to
Thus, given the foregoing findings of the Court of the School the last three (3) years of
Appeals that private respondent is a supervisory which must be continuous.
employee, it is indeed an unfair labor practice on
the part of petitioner CAPASCO to dismiss him on Three days later, the Union filed a notice of strike
account of his union activities, thereby curtailing with the National Conciliation and Mediation Board
his constitutionally guaranteed right to self- (NCMB).
organization.
Later, the Union struck and picketed the School’s
entrances.

MNI NOTES |168


On November 1993, then Secretary of Labor The appellate court concluded that the retirement of
Confesor issued an Order certifying the labor dispute the two (2) union officers was clearly to bust the
to the NLRC. On December 1993, the School filed a reactivated union.
petition with the NLRC to declare the strike illegal.
Having established that the School committed unfair
On July 1994, the Union filed a complaint for labor practice, the Court of Appeals declared that the
unfair labor practice before the NLRC Arbitration "no-strike, no-lockout clause" in the CBA was not
(due to mandatory retirement of Llagas and Javier). violated when the union members staged a strike
Upon motion, then LA Lorenzo ordered from 8 to 12 November 1993. It further held that
the consolidation of this unfair labor practice case minor disorders or isolated incidents of perceived
with the case regarding the illegality of strike. coercion attending the strike do not categorize it
as illegal:
ISSUE:
Nonetheless, the premise warrants considering
Whether the school is guilty of unfair labor practice
whether management may be precluded from retiring
by invoking the compulsory retirement pursuant to
an employee whom it is entitled to retire upon a
CBA upon the attempt to reactivate the union
determination that the true cause for compulsory
RULING: retirement is the employee's union activities.

Yes. The law and this Court frowns upon unfair labor
practices by management, including so-called union-
It is not difficult to see the anti-union bias of the busting.
school.
Such illegal practices will not be sustained by the
One of the first acts of private respondent Msgr. Court, even if guised under ostensibly legal premises.
Balbago immediately after his assumption of office
as school director was to ask for a moratorium on all But with respect to an active unionized employee who
claims having lost his/her job for union activities,
union activities.
there are different considerations presented if the
With the union in inactive status, the school felt termination is justified under just or authorized cause
secure and comfortable but when the union under the Labor Code; and if separation from service
reactivated, the school became apprehensive and is effected through the exercise of a duly accorded
reacted by retiring the union's two topmost officers by management prerogative to retire an employee.
invoking the provisions of the CBA.
There is perhaps a greater imperative to recognize the
When the union furnished the school, through management prerogative on retirement than the
counsel, a copy of a proposed CBA on 3 November prerogative to dismiss employees for just or
1993, the school in a cavalier fashion ignored it on the authorized causes.
pretext that the union no longer enjoyed the majority
status among the employees For one, there is a greater subjectivity, not to mention
factual dispute, attached to the concepts of just or
authorized cause than retirement which normally

MNI NOTES |169


contemplates merely the attainment of a certain age which included among others a PULO
or a certain number of years in the service. representative.

It would be easier for management desirous to While Purefoods formally acknowledged


eliminate pesky union members to abuse the receipt of the union’s proposals, it refused to
prerogative of termination for such purpose since the recognize PULO and its participation, even as a
determination of just or authorized cause is rarely a mere observer, in the negotiation.
simplistic question, but involves facts highly prone to
dispute and subjective interpretation. In the meantime, STFWU and PGFWU also
submitted their respective proposals for CBA
Purefoods v. Nagkakaisang Samahang
renewal, and their general membership
Manggagawa ng Purefoods GR No. 150896,
resolutions which, among others, affirmed the
Aug. 28, 2008
two organizations’ affiliation with PULO.
FACTS:
Consistent with its stance, Purefoods refused to
3 labor organizations and a federation are negotiate with the unions should a PULO
respondents in this case–Nagkakaisang representative be in the panel. The parties then
Samahang Manggagawa Ng Purefoods Rank- agreed to postpone the negotiations indefinitely.
And-File (NAGSAMA-Purefoods), the
However, the petitioner company concluded a
exclusive bargaining agent of the rank-and-file
new CBA with another union in its farm in
workers of Purefoods’ meat division throughout
Malvar, Batangas. 5 days thereafter, 4 company
Luzon; St. Thomas Free Workers Union
employees facilitated the transfer of around
(STFWU), of those in the farm in Sto. Tomas,
23,000 chickens from the poultry farm in Sto.
Batangas; and Purefoods Grandparent Farm
Tomas, Batangas (where STFWU was the
Workers Union (PGFWU), of those in the
exclusive bargaining agent) to that in Malvar.
poultry farm in Sta. Rosa, Laguna.
The following day, the regular rank-and-file
These organizations were affiliates of the
workers in the Sto. Tomas farm were refused
respondent federation, Purefoods Unified
entry in the company premises; and later, 22
Labor Organization (PULO).
STFWU members were terminated from
NAGSAMA-Purefoods manifested to employment.
petitioner corporation its desire to re-negotiate
The farm manager, supervisors and electrical
the CBA then due to expire.
workers of the Sto. Tomas farm, who were
Together with its demands and proposal, the members of another union, were nevertheless
organization submitted to the company its retained by the company in its employ.
General Membership Resolution approving and
Aggrieved by these developments, the 4
supporting the union’s affiliation with PULO,
respondent labor organizations jointly
adopting the draft CBA proposals of the
instituted a complaint for ULP, illegal
federation, and authorizing a negotiating panel

MNI NOTES |170


lockout/dismissal and damages with the Labor We hold that the sudden termination of the
Arbitration Branch of the NLRC. STFWU members is tainted with ULP because
it was done to interfere with, restrain or coerce
ISSUE: employees in the exercise of their right to self-
organization.
Whether there was an unfair labor practice

RULING:
BPI Employees Union-Davao City-FUBU v.
It is crystal clear that the closure of the Sto.
BPI, GR No. 174912, Jul. 24, 2013
Tomas farm was made in bad faith.
Doctrine: : It is to be emphasized that
Badges of bad faith are evident from the
contracting out of services is not illegal per se. It
following acts of the petitioner:
is an exercise of business judgment or
• it unjustifiably refused to recognize the management prerogative. Absent proof that the
STFWU’s and the other unions’ management acted in a malicious or arbitrary
affiliation with PULO; manner, the Court will not interfere with the
exercise of judgment by an employer.
• it concluded a new CBA with another
union in another farm during the agreed FACTS:
indefinite suspension of the collective BOMC, which was primarily engaged in
bargaining negotiations; providing and/or handling support services for
banks and other financial institutions, is a
• it surreptitiously transferred and
subsidiary of the Bank of Philippine Islands
continued its business in a less hostile
operating and functioning as an entirely separate
environment; and
and distinct entity.
• it suddenly terminated the STFWU
A service agreement between BPI and BOMC
members, but retained and brought the
was initially implemented in BPI's Metro Manila
non-members to the Malvar farm.
branches.
Petitioner presented no evidence to support the
In this agreement, BOMC undertook to provide
contention that it was incurring losses or that the
services such as check clearing, delivery of bank
subject farm’s lease agreement was pre-
statements, fund transfers, card production,
terminated.
operations accounting and control, and cash
Ineluctably, the closure of the Sto. Tomas farm servicing, conformably with BSP Circular No.
circumvented the labor organization’s right to 1388.
collective bargaining and violated the members’
Not a single BPI employee was displaced and
right to security of tenure.
those performing the functions, which were

MNI NOTES |171


transferred to BOMC, were given other personnel who, by virtue of the merger, would
assignments. have formed part of the bargaining unit
represented by the Union pursuant to its union
The Manila chapter of BPI Employees Union shop provision in the CBA.
then filed a complaint for unfair labor practice.
The Union then filed a formal protest addressed
The Labor Arbiter decided the case in favor of to BPI Vice Presidents Claro M. Reyes and
the union. The decision was, however, reversed Cecil Conanan reiterating its objection.
on appeal by the NLRC.
It requested the BPI management to submit the
BPIEU-Metro Manila-FUBU filed a petition BOMC issue to the grievance procedure under
for certiorari before the CA which denied it, the CBA, but BPI did not consider it as
holding that BPI transferred the employees in the "grievable." Instead, BPI proposed a Labor
affected departments in the pursuit of its Management Conference between the parties.
legitimate business.
During the LMC, BPI invoked management
The employees were neither demoted nor were prerogative stating that the creation of the
their salaries, benefits and other privileges BOMC was to preserve more jobs and to
diminished. designate it as an agency to place employees
where they were most needed.
The service agreement was likewise
implemented in Davao City. On the other hand, the Union charged that
BOMC undermined the existence of the union
Later, a merger between BPI and Far East Bank
since it reduced or divided the bargaining unit.
and Trust Company took effect with BPI as the
surviving corporation. While BOMC employees perform BPI functions,
they were beyond the bargaining unit's coverage.
Thereafter, BPI's cashiering function and
FEBTC's cashiering, distribution and In contracting out FEBTC functions to BOMC,
bookkeeping functions were handled by BOMC. BPI effectively deprived the union of the
membership of employees handling said
Consequently, twelve (12) former FEBTC
functions as well as curtailed the right of those
employees were transferred to BOMC to
employees to join the union.
complete the latter's service complement.
Thereafter, the Union demanded that the matter
BPI Davao's rank and file collective bargaining
be submitted to the grievance machinery as the
agent, BPI Employees UnionDavao City-
resort to the LMC was unsuccessful. As BPI
FUBU, objected to the transfer of the functions
allegedly ignored the demand, the Union filed a
and the twelve (12) personnel to BOMC
notice of strike before the National Conciliation
contending that the functions rightfully belonged
and Mediation Board.
to the BPI employees and that the Union was
deprived of membership of former FEBTC

MNI NOTES |172


BPI then filed a petition for assumption of Clearly, only gross violations of the economic
jurisdiction/certification with the Secretary of the provisions of the CBA are treated as ULP.
Department of Labor and Employment, who Otherwise, they are mere grievances.
subsequently issued an order certifying the labor
dispute to the NLRC for compulsory arbitration. In the present case, the alleged violation of the
union shop agreement in the CBA, even
ISSUE: assuming it was malicious and flagrant, is not a
violation of an economic provision in the
Whether the act of BPI to outsource the agreement.
cashiering, distribution, and bookkeeping
functions to BOMC is unfair labor practice – No. The provisions relied upon by the Union were
those articles referring to the recognition of the
RULING: union as the sole and exclusive bargaining
representative of all rank and-file employees, as
The Union claims that a union shop agreement is
well as the articles on union security,
stipulated in the existing CBA, citing the case of
specifically, the maintenance of membership in
Shell Oil Workers' Union v. Shell Company of
good standing as a condition for continued
the Philippines, Ltd. ; however, the Union's
employment and the union shop clause.
reliance on the case is misplaced. The rule now is
covered by Article 261 of the Labor Code, which It failed to take into consideration its recognition
took effect on November 1, 1974. Article 261 of the bank's exclusive rights and prerogatives,
provides: likewise provided in the CBA, which included
the hiring of employees, promotion, transfers,
ART. 261. Jurisdiction- of Voluntary
and dismissals for just cause and the maintenance
Arbitrators or panel of Voluntary
of order, discipline and efficiency in its
Arbitrators. — . . . Accordingly,
operations.
violations of a Collective Bargaining
Agreement, except those which are gross The Union, however, insists that jobs being
in character, shall no longer be treated outsourced to BOMC were included in the
as unfair labor practice and shall be existing bargaining unit, thus, resulting in a
resolved as grievances under the reduction of a number of positions in such unit.
Collective Bargaining Agreement. For
purposes of this article, gross The reduction interfered with the employees'
violations of Collective Bargaining right to self-organization because the power of a
Agreement shall mean flagrant and/or union primarily depends on its strength in
malicious refusal to comply with the number.
economic provisions of such
agreement. It is incomprehensible how the "reduction of
positions in the collective bargaining unit"
interferes with the employees' right to self-
organization because the employees themselves

MNI NOTES |173


were neither transferred nor dismissed from Monetary Board of the then Central Bank of the
the service. Philippines.

As the NLRC clearly stated: In the case at hand, CEPALCO v. CEPALCO Employee’s Labor
the union has not presented even an iota of Union, GR No. 211015, Jun. 20, 2016
evidence that petitioner bank has started to
terminate certain employees, members of the Doctrine: Under Article 106 of the Labor Code,
union. as amended, labor-only contracting is an
arrangement where the contractor, who does not
In fact, what appears is that the Bank has exerted have substantial capital or investment in the form
utmost diligence, care and effort to see to it that of tools, equipment, machineries, work premises,
no union member has been terminated. among others, supplies workers to an employer
and the workers recruited are performing
In the process of the consolidation or merger of activities which are directly related to the
the two banks which resulted in increased principal business of such employer. There are
diversification of functions, some of these non- two elements;
banking functions were merely transferred to the
BOMC without affecting the union membership. 1. contractor does not have substantial
capital or investment which relates to the
BPI also stresses that not a single employee or job, work, or service to be performed and
union member was or would be dislocated or the employees recruited, supplied, or
terminated from their employment as a result of placed by such contractor or
the Service Agreement. subcontractor are performing activities
which are directly related to the main
Neither had it resulted in any diminution of
business of the principal
salaries and benefits nor led to any reduction of
union membership. 2. contractor does not exercise the right to
control over the performance of the work
It is to be emphasized that contracting out of
of the contractual employee.
services is not illegal per se.
FACTS:
It is an exercise of business judgment or
management prerogative. Respondent is the duly certified bargaining
representative of CEPALCO’s regular rank-and-
Absent proof that the management acted in a
file employees. On the other hand, CEPALCO is
malicious or arbitrary manner, the Court will not
a domestic corporation engaged in electric
interfere with the exercise of judgment by an
distribution in CDO and other municipalities in
employer.
Misamis Oriental; while CESCO is a business
In this case, bad faith cannot be attributed to entity engaged in trading and services.
BPI because its actions were authorized by CBP
Circular No. 1388, Series of 1993 issued by the

MNI NOTES |174


CEPALCO and CESCO (petitioners) entered seek that the latter be declared as CEPALCO’s
into a Contract for Meter Reading Work where regular employees.
CESCO undertook to perform CEPALCO’s
meter reading activities. As a result, several LA dismissed the case. It held that CESCO
employees and union members of CEPALCO carries on an independent business of contracting
were relieved, assigned in Floating positions, and services and that CESCO has an authorized
replaced with CESCO workers, prompting capital stock of 100M pesos, as well as
respondents to file a complaint for ULP against equipment and materials necessary to carry out
petitioners. its business. As an independent contractor,
CESCO is the statutory employer of the workers
Respondents alleged that it was CEPALCO’s it supplied to CEPALCO, pursuant to their
intention to evade its responsibilities under the contract. Thus, there is no factual basis to say that
CBA and labor laws, and that it would ultimately CEPALCO committed unfair labor practice as
result in the dissipation of respondent’s there can be no splitting or erosion of the existing
membership in CEPALCO. rank-and-file bargaining unit that negates
interference with the exercise of CEPALCO
Respondent claimed that CEPALCO’s act of worker’s right to self-organize.
contracting out services, which used to be part of
the functions of the regular union members, is NLRC affirmed the decision of the LA.
violative of Article 259 ( c ) of the Labor Code,
as amended, governing unfair labor practice of CA found no substantial evidence that
employers. CEPALCO was engaged in ULP, there being no
showing that CEPALCO was motivated by ill
It further averred that for engaging in LOC, the will, bad faith, or malice, or that it was aimed at
workers placed by CESCO must be deemed interfering with its employees’ right to self-
regular rank-and-file employees of CEPALCO, organize.
and that the contract for meter reading work be
declared null and void. ISSUE:

Petitioners averred that CESCO is an Whether the contracting out of activities or


independent job contractor and that contracting services being performed by union members
out of the meter-reading services did not interfere constitute ULP? – No.
with CEPALCO’s regular worker’s right to self-
RULING:
organize, denying that none of the respondent’s
members was put on Floating status. Under Article 106 of the Labor Code, as
amended, labor-only contracting is an
Moreover, they argued that the case is only a
arrangement where the contractor, who does not
labor standard issue and that respondents is not
have substantial capital or investment in the form
the proper party to raise the issue regarding the
of tools, equipment, machineries, work premises,
status of CESCO’s employees and, hence cannot
among others, supplies workers to an employer

MNI NOTES |175


and the workers recruited are performing There is no available document to show
activities which are directly related to the that CESCO’s authorized capital stock at
principal business of such employer. There are the time of the contracting out of
two elements; CEPALCO’s meter reading activities to
CESCO
1. contractor does not have substantial
capital or investment which relates to the The list of CESCO’s office equipment,
job, work, or service to be performed and furniture and fixtures, and vehicle offered
the employees recruited, supplied, or in evidence by petitioners does not satisfy
placed by such contractor or the requirement that they could have been
subcontractor are performing activities used in the performance of the specific
which are directly related to the main work contracted out
business of the principal
The Court, similar to the CA and the labor
2. contractor does not exercise the right to tribunals, finds that CEPALCO’s contracting
control over the performance of the work arrangements with CESCO did not amount to
of the contractual employee. unfair labor practice.

Labor- only contracting is considered as a form This is because respondent was not able to
of unfair Labor Practice when the same is present any evidence to show that such
devised by the employer to “interfere with arrangements violated CEPALCO’s workers’
restrain or coerce employees in the exercise of right to self-organization, which as above-
their rights to self-organization”. mentioned, constitutes the core of unfair labor
practice.
In Banking, Inc. v. NLRC; the Court ruled that
the prohibited acts considered as ULP relate to Records do not show that this finding was further
the worker’s right to self-organization and to appealed by respondent. Thus, the complaints
the observance of a CBA. It refers to “acts that filed by respondent, should be dismissed with
violate the workers’ right to organize.” finality.
Without such element, the acts, even if unfair are
nor ULP. The Court, also observes that while respondent
did ask for the nullification of the subject
In these cases, the Court agrees with the CA that contracts between he petitioners, and even sought
CEPALCO was engaged in LOC. that the employees provided by CESCO to
CEPALCO be declares as the latter’s own
Petitioners failed to show that CESCO employees, petitioners correctly argue that
has substantial capital or investment respondent is not a real party-in-interest and
which relates to the job, work, or service hence, had no legal standing insofar as these
to be performed matters are concerned.

MNI NOTES |176


This is because respondent failed to demonstrate New Labor Code, shall give notice of
how it stands to be benefitted or injure by a termination of services of any employee
judgement on the same, or that any personal or who shall fail to fulfill the condition
direct injury would be sustained by it if these provided in Section 6.1 and 6.2 of this
reliefs were not granted. Article

PICOP v. Taneca, GR No. 160828, Aug. 9,


Atty. Fuentes sent a letter to the management of
2010
PRI demanding the termination of employees
FACTS: who allegedly campaigned for, supported and
signed the Petition for Certification Election of
Respondents were regular rank-and-file the Federation of Free Workers Union (FFW)
employees of PRI and bona fide members during the effectivity of the CBA. NAMAPRI-
of Nagkahiusang Mamumuo sa PRI Southern SPFL considered said act of campaigning for and
Philippines Federation of Labor (NAMAPRI- signing the petition for certification election of
SPFL), which is the collective bargaining agent FFW as an act of disloyalty and a valid basis for
for the rank-and-file employees of petitioner PRI. termination for a cause in accordance with its
Constitution and By-Laws, and the terms and
PRI has a CBA with NAMAPRI-SPFL. The conditions of the CBA, specifically Article II,
CBA contained the following union security Sections 6.1 and 6.2 on Union Security Clause.
provisions:
On October 16, 2000, PRI served notices of
Article II- Union Security and Check- termination for causes to employees whom
Off NAMAPRIL-SPFL sought to be terminated on
the ground of “acts of disloyalty” committed
against it when respondents allegedly supported
Section 6. Maintenance of membership.
and signed the Petition for Certification Election
of FFW before the “freedom period” during the
6.1 All employees within the appropriate effectivity of the CBA. A Notice dated October
bargaining unit who are members of the 21, 2000 was also served on the DOLE, Caraga
UNION at the time of the signing of this Region.
AGREEMENT shall, as a condition of
continued employment by the
Respondents then accused PRI of ULP.
COMPANY, maintain their membership
in the UNION in good standing during
the effectivity of this AGREEMENT. ISSUE:

6.3 The COMPANY, upon the written Whether the termination of the employees
request of the UNION and after constitute unfair labor practice
compliance with the requirements of the

MNI NOTES |177


RULING: (1) the union security clause is
applicable;
“Union security” is a generic term, which is
(2) the union is requesting for the
applied to and comprehends “closed shop,”
enforcement of the union security
“union shop,” “maintenance of membership,” or
provision in the CBA; and
any other form of agreement which imposes
upon employees the obligation to acquire or (3) there is sufficient evidence to support
retain union membership as a condition the decision of the union to expel the
affecting employment. employee from the union.

There is union shop when all new regular These requisites constitute just cause for
employees are required to join the union within a terminating an employee based on the union
certain period as a condition for their continued security provision of the CBA.
employment.
As to the first requisite, there is no question that
There is maintenance of membership the CBA between PRI and respondents included
shop when employees, who are union members a union security clause.
as of the effective date of the agreement, or who
thereafter become members, must maintain union Secondly, it is likewise undisputed that
membership as a condition for continued NAMAPRI-SPFL, in two (2) occasions
employment until they are promoted or demanded from PRI, in their letters dated May 16
transferred out of the bargaining unit, or the and 23, 2000, to terminate the employment of
agreement is terminated. respondents due to their acts of disloyalty to the
Union.
A closed shop may be defined as an enterprise in
which, by agreement between the employer and
However, as to the third requisite, we find that
his employees or their representatives, no person
there is no sufficient evidence to support the
may be employed in any or certain agreed
decision of PRI to terminate the employment of
departments of the enterprise unless he or she is,
the respondents.
becomes, and, for the duration of the agreement,
remains a member in good standing of a union
entirely comprised of or of which the employees The mere signing of the authorization in
in interest are a part. support of the Petition for Certification Election
of FFW on March 19, 20 and 21, or before the
However, in terminating the employment of an “freedom period,” is not sufficient ground to
employee by enforcing the union security clause, terminate the employment of respondents
the employer needs to determine and prove inasmuch as the petition itself was actually filed
that: during the freedom period.

MNI NOTES |178


Nothing in the records would show that Bargaining Agreement were to be effective
respondents failed to maintain their membership for five years or until 30 June 2005; and the
in good standing in the Union. Respondents did economic provisions of the same were to be
not resign or withdraw their membership from effective for three years or until 30 June 2003.
the Union to which they belong. Respondents
continued to pay their union dues and never The parties subsequently re-negotiated the
joined the FFW. economic provisions of the Collective
Bargaining Agreement and extended the term of
NUHWRAIN v. NLRC, GR No. 160828, Sept. their effectivity for another two years or until 30
30, 2008 June 2005.

FACTS:
During the 60-day freedom period which
This is a Petition for Review on Certiorari under preceded the expiration of the Collective
Rule 45, assailing the Decision rendered Bargaining Agreement, starting on 1 May 2005
by the Court of Appeals, which affirmed the and ending on 30 June 2005, the Hotel and
Resolution of the National Labor Relations HIMPHLU negotiated the extension of the
Commission, dismissing for lack of merit the provisions of the existing Collective Bargaining
complaint for unfair labor practice filed by Agreement for two years, effective 1 July 2005
petitioner National Union of Workers in Hotels, to 30 June 2007. The parties signed the
Restaurants and Allied Industries-Manila Memorandum of Agreement on 20 May 2005
Pavilion Hotel (NUWHRAIN) against Manila and the employees ratified it on 27 May
Pavilion Hotel (the Hotel). 2005.6

Petitioner NUWHRAIN is a legitimate labor On 21 June 2005, NUWHRAIN was accorded


organization composed of rank-and-file by the Labor Relations Division of
employees of the Hotel, while respondent the Department of Labor and Employment
Acesite Philippines Hotel Corporation is the (DOLE) the status of a legitimate labor
owner and operator of said Hotel. organization.

The Hotel entered into a Collective Bargaining Thereafter, NUWHRAIN exercised the right to
Agreement with HI-MANILA PAVILION challenge the majority
HOTEL LABOR UNION (HIMPHLU), the status of the incumbent union, HIMPHLU, by
exclusive bargaining agent of the rank-and-file filing a Petition for Certification Election
employees of the Hotel. on 28 June 2005.

Both parties consented that the representation On 5 July 2007, the Industrial Relations Division
aspect and other of the DOLE allowed the registration of
non-economic provisions of the Collective the Memorandum of Agreement executed

MNI NOTES |179


between HIMPHLU and the Hotel, extending the Resolution dated 30 June 2006.20 Thus,
effectivity of the existing Collective Bargaining NUWHRAIN filed a Petition for Certiorari
Agreement for another two years.9 before the Court of Appeals, docketed as C.A.
G.R. SP No. 96171.
After the lapse of the 60-day freedom period, but
pending the disposition of the In the meantime, on 16 June 2006, the
Petition for Certification Election filed by Certification Election for regular rank and file
NUWHRAIN, HIMPHLU served the Hotel employees of the Hotel was held, which
with a written demand dated 28 July 200510 for HIMPHLU won. It was accordingly certified
the dismissal of 36 employees as the exclusive bargaining agent for rank-and-
following their expulsion from HIMPHLU for file employees of the Hotel.21
alleged acts of disloyalty and violation
of its Constitution and by-laws. On 30 May 2007, the Court of Appeals
promulgated its Decision upholding the
An Investigation Report was attached to the said Resolution of the NLRC. It declared that the
written demand, stating that the 36 employees, Hotel had acted prudently when it issued the
who were members of HIMPHLU, Notices to the 36 employees after HIMPHLU
joined NUWHRAIN, in violation of Section 2, demanded their dismissal. It clarified that these
Article IV of the Collective Bargaining Notices did not amount to the termination of the
Agreement, which provided for a union security employees concerned but merely sought their
clause. explanation on why the union security clause
should not be applied to them.
On 1 August 2005, the Hotel issued Disciplinary
Action Notices (Notices) to the 36 The appellate court also gave credence to the
employees identified in the written demand of denial by the officers of the respondent and the
HIMPHLU. Hotel that they made statements favoring
HIMPHLU over NUWHRAIN during the
The Notices directed the 36 reconciliatory conferences.
employees to submit a written explanation for
their alleged acts of disloyalty and violation of The Court of Appeals further noted that the
the union security clause for which HIMPHLU unhampered organization and registration of
sought their dismissal. NUWHRAIN negated its allegation that the
Hotel required its employees not to join a labor
NLRC decided that there was no unfair labor organization as a condition for their employment.
practice. NUWHRAIN filed a Motion for
Reconsideration of the foregoing NLRC NUWHRAIN’s Motion for Reconsideration of
Resolution. It was denied by the NLRC in the aforementioned Decision of the Court of
another

MNI NOTES |180


Appeals was denied by the same court in a acquire or retain union membership
Resolution. as a condition affecting employment.

Hence, the present Petition, in which Article 248(e) of the Labor Code recognizes the
NUWHRAIN makes the following assignment of effectivity of a union shop clause:
errors:
The law allows stipulations for “union
ISSUE: shop” and “closed shop” as a means of
encouraging workers to join and support
Whether the dismissal of the employees in the union of their choice in the protection
accordance with the CBA’s Union Security of their rights and interests vis-à-vis the
Clause deemed unfair labor practice employer.

RULING:
By thus promoting unionism, workers are
The instant Petition lacks merit. able to negotiate with management on an
even playing field and with more
NUWHRAIN maintains that the respondent persuasiveness than if they were to
committed unfair labor practice when individually and separately bargain with
the employer. In Villar v. Inciong, this
Court held that employees have the right
(1) the Hotel issued the Notices to the 36
to disaffiliate from their union and form a
employees, former members of
new organization of their own; however,
HIMPHLU, who switched allegiance to
they must suffer the consequences of their
NUWHRAIN; and
separation from the union under the
(2) the officers of the respondent and the
security clause of the Collective
Hotel allegedly uttered statements during
Bargaining Agreement.
the reconciliatory conferences indicating
their preference for HIMPHLU and their
disapproval of NUWHRAIN. In the present case, the Collective Bargaining
Agreement includes a union security
provision
This argument is specious.

To avoid the clear possibility of liability for


“Union security” is a generic term which is
breaching the union security clause
applied to and comprehends “closed
of the Collective Bargaining Agreement and to
shop,” “union shop,” “maintenance of
protect its own interests, the only sensible
membership” or any other form of agreement
option left to the Hotel, upon its receipt of the
which imposes upon employees the obligation to
demand of HIMPHLU for the dismissal of the
36 employees, was to conduct its own inquiry so

MNI NOTES |181


as to make its own findings on whether for unfair labor practice where respondent did not
there was sufficient ground to dismiss the said dismiss the 36 employees, despite the
employees who defected from HIMPHLU. insistence of HIMPHLU, the sole bargaining
agent for the rank-and-file employees of the
Hotel, on the basis of the union security clause of
The issuance by the respondent of the Notices the Collective Bargaining Agreement.
requiring the 36 employees to submit their
explanations to the charges against them was the Theonly act attributed to the respondent is its
reasonable and logical first step in a fair issuance of the Notices which, contrary to being
investigation. an unfair labor practice, even afforded the
employees involved a chance to be heard.
It is important to note that the Hotel did not take
further steps to terminate the In all, respondent had not committed any act
36 employees. Instead, it arranged for which would constitute unfair labor practice.
reconciliatory conferences between the
contending unions in order to avert the possibility IN VIEW OF THE FOREGOING, the instant
of dismissing the 36 employees for violation of Petition is DENIED. The assailed Decision
the union security clause of the Collective dated 30 May 2007 of the Court of Appeals in
Bargaining Agreement. CA-G.R. SP No. 96171 is hereby AFFIRMED.
Costs against petitioner NUWHRAIN.
This Court, in Malayang Samahan ng
Manggagawa sa M. Greenfield v. Ramos29 BPI v. BPI Employees Union-Davao Chapter,
clearly stated the general rule: the dismissal of GR No. 164301, Aug. 10, 2010
an employee by the company pursuant to a
labor union’s demand in accordance with a FACTS:
union security agreement does not constitute
unfair labor practice. Pursuant to the Article and Plan of Merger, all
the assets and liabilities of FEBTC were
transferred to and absorbed by BPI as the
An employer is not considered guilty of unfair
surviving corporation.
labor practice if it merely
complied in good faith with the request of the FEBTC employees, including those in its
certified union for the dismissal of employees different branches across the country, were hired
expelled from the union pursuant to the union by petitioner as its own employees, with their
security clause in the Collective Bargaining status and tenure recognized and salaries and
Agreement. benefits maintained.
Respondent BPI Employees Union-Davao
In the case at bar, there is even less possibility of
Chapter - Federation of Unions in BPI
sustaining a finding of guilt
Unibank (hereinafter the Union, for brevity)

MNI NOTES |182


is the exclusive bargaining agent of BPIs rank 2001, ruled in favor of petitioner BPIs
and file employees in Davao City. The former interpretation that the former FEBTC employees
FEBTC rank-and-file employees in Davao City were not covered by the Union Security Clause
did not belong to any labor union at the time of of the CBA between the Union and the Bank on
the merger the ground that the said employees were not new
employees who were hired and subsequently
Prior to the effectivity of the merger,
regularized, but were absorbed employees by
respondent Union invited said FEBTC
operation of law because the former
employees to a meeting regarding the Union
employees of FEBTC can be considered assets
Shop Clause
and liabilities of the absorbed
Section 2. Union Shop - New corporation. The Voluntary Arbitrator
employees falling within the concluded that the former FEBTC employees
bargaining unit as defined in could not be compelled to join the Union, as it
Article I of this Agreement, who was their constitutional right to join or not to join
may hereafter be regularly any organization.
employed by the Bank shall,
within thirty (30) days after they ISSUE:
become regular employees, join
whether or not the former FEBTC employees that
the Union as a condition of their
were absorbed by petitioner upon the merger
continued employment. It is
between FEBTC and BPI should be covered by
understood that membership in
the Union Shop Clause found in the existing
good standing in the Union is a
CBA between petitioner and respondent Union
condition of their continued
employment with the RULING:
Bank.[8] (Emphases supplied.) Section 2, Article II of the CBA is silent
Respondent Union then sent notices to as to how one becomes a regular employee of the
the former FEBTC employees who BPI for the first time.There is nothing in the
refused to join, as well as those who said provision which requires that a new
retracted their membership, and called regular employee first undergo a temporary
them to a hearing regarding the matter. or probationary status before being deemed as
such under the union shop clause of the CBA.
When these former FEBTC employees
refused to attend the hearing, the Union security is a generic term which is applied
president of the Union requested BPI to to and comprehends closed shop, union shop,
implement the Union Shop Clause of the maintenance of membership or any other form of
CBA and to terminate their employment agreement which imposes upon employees the
pursuant thereto. obligation to acquire or retain union membership
as a condition affecting employment.
Voluntary Arbitrator Rosalina Letrondo-
Montejo, in a Decision] dated November 23,

MNI NOTES |183


There is union shop when all new regular of employees of the non-surviving entities of the
employees are required to join the union within a merger would have been mandatory on the
certain period for their continued employment. surviving corporation.
There is maintenance of membership shop In the present case, the merger was voluntarily
when employees, who are union members as of entered into by both banks presumably for some
the effective date of the agreement, or who mutually acceptable consideration. In fact, the
thereafter become members, must maintain union Corporation Code does not also mandate the
membership as a condition for continued absorption of the employees of the non-
employment until they are promoted or surviving corporation by the surviving
transferred out of the bargaining unit or the corporation in the case of a merger.
agreement is terminated.
A closed-shop, on the other hand, may be Proper Appreciation of the Term New
defined as an enterprise in which, by agreement Employees Under the CBA
between the employer and his employees or their In any event, it is of no moment that the
representatives, no person may be employed in former FEBTC employees retained the regular
any or certain agreed departments of the status that they possessed while working for their
enterprise unless he or she is, becomes, and, for former employer upon their absorption by
the duration of the agreement, remains a member petitioner. This fact would not remove them from
in good standing of a union entirely comprised of the scope of the phrase new employees as
or of which the employees in interest are a part. contemplated in the Union Shop Clause of the
In other words, the purpose of a union CBA, contrary to petitioners insistence that the
shop or other union security arrangement is to term new employees only refers to those who are
guarantee the continued existence of the union initially hired as non-regular employees for
through enforced membership for the benefit of possible regular employment.
the workers. The Union Shop Clause in the CBA
Absorbed FEBTC Employees are Neither simply states that new employees who during the
Assets nor Liabilities effectivity of the CBA may be regularly
employed by the Bank must join the union within
In legal parlance, however, human beings are thirty (30) days from their regularization. There
never embraced in the term assets and is nothing in the said clause that limits its
liabilities. Moreover, BPIs absorption of former application to only new employees who possess
FEBTC employees was neither by operation of non-regular status, meaning probationary
law nor by legal consequence of contract. There status, at the start of their employment. Petitioner
was no government regulation or law that likewise failed to point to any provision in the
compelled the merger of the two banks or the CBA expressly excluding from the Union Shop
absorption of the employees of the dissolved Clause new employees who are absorbed as
corporation by the surviving corporation. Had regular employees from the beginning of their
there been such law or regulation, the absorption employment. What is indubitable from the Union

MNI NOTES |184


Shop Clause is that upon the effectivity of the c) whose number will affect the number
CBA, petitioners new regular employees of members of the collective bargaining unit will
(regardless of the manner by which they be compelled to join the union.
became employees of BPI) are required to join
• Such compulsion has legal effect,
the Union as a condition of their continued
precisely because the employer by
employment.
voluntarily entering in to a union
No Substantial Distinction Under the CBA shop clause in a CBA with the
Between Regular Employees Hired After certified bargaining agent takes
Probationary Status and Regular Employees on the responsibility of
Hired After the Merger They both enjoy dismissing the new regular
benefits that the Union was able to secure for employee who does not join the
them under the CBA. When they both entered union.
the employ of BPI, the CBA and the Union
Shop Clause therein were already in effect and
Right of an Employee not to Join a
neither of them had the opportunity to express
Union is not Absolute and Must Give Way to the
their preference for unionism or not. We see
Collective Good of All Members of the
no cogent reason why the Union Shop Clause
Bargaining Unit Time and again, this Court has
should not be applied equally to these two
ruled that the individual employees right not to
types of new employees, for they are
join a union may be validly restricted by a union
undeniably similarly situated.
security clause in a CBA[49] and such union
PURPOSE OF UNION SECURITY security clause is not a violation of the employees
CLAUSE constitutional right to freedom of
association.[50] Laws and jurisprudence promote
Indeed, a union security clause in a CBA
unionism and afford certain protections to the
should be interpreted to give meaning and effect
certified bargaining agent in a unionized
to its purpose, which is to afford protection to the
company because a strong and effective union
certified bargaining agent and ensure that the
presumably benefits all employees in the
employer is dealing with a union that represents
bargaining unit since such a union would be in
the interests of the legally mandated percentage
a better position to demand improved benefits
of the members of the bargaining unit.
and conditions of work from the employer.
The union shop clause offers protection to
settled jurisprudence has already swung the
the certified bargaining agent by ensuring that
balance in favor of unionism, in recognition that
future regular employees who
ultimately the individual employee will be
(a) enter the employ of the company benefited by that policy. In the hierarchy of
during the life of the CBA; constitutional values, this Court has repeatedly
held that the right to abstain from joining a labor
(b) are deemed part of the collective
organization is subordinate to the policy of
bargaining unit; and (
encouraging unionism as an instrument of social

MNI NOTES |185


justice.

CONCLUSION
Union Shop Clause of the CBA covers
the former FEBTC employees who were
hired/employed by BPI during the effectivity of
the CBA in a manner which petitioner describes
as absorption.

Montealegre v. De Vera, GR No. 208920, Jul.


10, 2019

MNI NOTES |186


MODULE 7 (d) The notice must be in accordance with such
implementing rules and regulations as the Minister of
Labor and Employment may promulgate.

B. “Makibaka!”: Resorting to Lawful (e) During the cooling-off period, it shall be the duty
Concerted Activities of the Ministry to exert all efforts at mediation and
conciliation to effect a voluntary settlement. Should
Article 278-281, LC the dispute remain unsettled until the lapse of the
requisite number of days from the mandatory filing of
ART. 278. [263] Strikes, Picketing, and Lockouts. the notice, the labor union may strike or the employer
may declare a lockout.
(a) It is the policy of the State to encourage free trade
unionism and free collective bargaining. (f) A decision to declare a strike must be approved by
a majority of the total union membership in the
(b) Workers shall have the right to engage in bargaining unit concerned, obtained by secret ballot
concerted activities for purposes of collective in meetings or referenda called for that purpose.
bargaining or for their mutual benefit and protection.
The right of legitimate labor organizations to strike A decision to declare a lockout must be approved by
and picket and of employers to lockout, consistent a majority of the board of directors of the corporation
with the national interest, shall continue to be or association or of the partners in a partnership,
recognized and respected. However, no labor union obtained by secret ballot in a meeting called for that
may strike and no employer may declare a lockout on purpose. The decision shall be valid for the duration
grounds involving inter-union and intra-union of the dispute based on substantially the same grounds
disputes. considered when the strike or lockout vote was taken.
The Ministry may, at its own initiative or upon the
(c) In cases of bargaining deadlocks, the duly certified request of any affected party, supervise the conduct of
or recognized bargaining agent may file a notice of the secret balloting. In every case, the union or the
strike or the employer may file a notice of lockout employer shall furnish the Ministry the results of the
with the Ministry at least 30 days before the intended voting at least seven days before the intended strike
date thereof. In cases of unfair labor practice, the or lockout, subject to the cooling-off period herein
period of notice shall be 15 days and in the absence provided.
of a duly certified or recognized bargaining agent, the
notice of strike may be filed by any legitimate labor (g) When, in his opinion, there exists a labor dispute
organization in behalf of its members. causing or likely to cause a strike or lockout in an
industry indispensable to the national interest, the
However, in case of dismissal from employment of Secretary of Labor and Employment may assume
union officers duly elected in accordance with the jurisdiction over the dispute and decide it or certify
union constitution and by-laws, which may constitute the same to the Commission for compulsory
union busting where the existence of the union is arbitration. Such assumption or certification shall
threatened, the 15-day cooling-off period shall not have the effect of automatically enjoining the
apply and the union may take action immediately. intended or impending strike or lockout as specified
in the assumption or certification order.

MNI NOTES |187


If one has already taken place at the time of Labor and Employment or the Commission, under
assumption or certification, all striking or locked out pain of immediate disciplinary action, including
employees shall immediately return to work and the dismissal or loss of employment status or payment by
employer shall immediately resume operations and the locking-out employer of backwages, damages and
readmit all workers under the same terms and other affirmative relief, even criminal prosecution
conditions prevailing before the strike or lockout. The against either or both of them.
Secretary of Labor and Employment or the
Commission may seek the assistance of law The foregoing notwithstanding, the President of the
enforcement agencies to ensure compliance with this Philippines shall not be precluded from determining
provision as well as with such orders as he may issue the industries that, in his opinion, are indispensable to
to enforce the same. the national interest, and from intervening at any time
and assuming jurisdiction over any such labor dispute
In line with the national concern for and the highest in order to settle or terminate the same.
respect accorded to the right of patients to life and
health, strikes and lockouts in hospitals, clinics and (h) Before or at any stage of the compulsory
similar medical institutions shall, to every extent arbitration process, the parties may opt to submit their
possible, be avoided, and all serious efforts, not only dispute to voluntary arbitration.
by labor and management but government as well, be
exhausted to substantially minimize, if not prevent, (i) The Secretary of Labor and Employment, the
their adverse effects on such life and health, through Commission or the voluntary arbitrator or panel of
the exercise, however legitimate, by labor of its right voluntary arbitrators shall decide or resolve the
to strike and by management to lockout. dispute within thirty (30) calendar days from the date
of the assumption of jurisdiction or the certification
In labor disputes adversely affecting the continued or submission of the dispute, as the case may be. The
operation of such hospitals, clinics or medical decision of the President, the Secretary of Labor and
institutions, it shall be the duty of the striking union Employment, the Commission or the voluntary
or locking-out employer to provide and maintain an arbitrator shall be final and executory ten (10)
effective skeletal workforce of medical and other calendar days after receipt thereof by the parties.
health personnel, whose movement and services shall
be unhampered and unrestricted, as are necessary to ART. 279. [264] Prohibited activities.
insure the proper and adequate protection of the life
(a) No labor organization or employer shall declare a
and health of its patients, most especially emergency
strike or lockout without first having bargained
cases, for the duration of the strike or lockout.
collectively in accordance with Title VII of this Book
In such cases, therefore, the Secretary of Labor and or without first having filed the notice required in the
Employment may immediately assume, within twenty preceding Article or without the necessary strike or
four (24) hours from knowledge of the occurrence of lockout vote first having been obtained and reported
such a strike or lockout, jurisdiction over the same or to the Ministry.
certify it to the Commission for compulsory
No strike or lockout shall be declared after
arbitration.
assumption of jurisdiction by the President or the
For this purpose, the contending parties are strictly Minister or after certification or submission of the
enjoined to comply with such orders, prohibitions dispute to compulsory or voluntary arbitration or
and/or injunctions as are issued by the Secretary of

MNI NOTES |188


during the pendency of cases involving the same premises for lawful purposes, or obstruct public
grounds for the strike or lockout. thoroughfares.

Any worker whose employment has been terminated ART. 280. [265] Improved Offer Balloting.
as a consequence of any unlawful lockout shall be
entitled to reinstatement with full backwages. Any In an effort to settle a strike, the Department of Labor
union officer who knowingly participates in an illegal and Employment shall conduct a referendum by
strike and any worker or union officer who knowingly secret balloting on the improved offer of the employer
participates in the commission of illegal acts during a on or before the 30th day of the strike. When at least
strike may be declared to have lost his employment a majority of the union members vote to accept the
status: Provided, That mere participation of a worker improved offer the striking workers shall immediately
in a lawful strike shall not constitute sufficient ground return to work and the employer shall thereupon
for termination of his employment, even if a readmit them upon the signing of the agreement.
replacement had been hired by the employer during
such lawful strike. In case of a lockout, the Department of Labor and
Employment shall also conduct a referendum by
(b) No person shall obstruct, impede, or interfere with secret balloting on the reduced offer of the union on
by force, violence, coercion, threats or intimidation, or before the 30th day of the lockout. When at least a
any peaceful picketing by employees during any labor majority of the board of directors or trustees or the
controversy or in the exercise of the right to self- partners holding the controlling interest in the case of
organization or collective bargaining, or shall aid or a partnership vote to accept the reduced offer, the
abet such obstruction or interference. workers shall immediately return to work and the
employer shall thereupon readmit them upon the
(c) No employer shall use or employ any strike- signing of the agreement.
breaker, nor shall any person be employed as a strike-
breaker. ART. 281. [266] Requirement for Arrest and
Detention.
(d) No public official or employee, including officers
and personnel of the New Armed Forces of the Except on grounds of national security and public
Philippines or the Integrated National Police, or peace or in case of commission of a crime, no union
armed person, shall bring in, introduce or escort in members or union organizers may be arrested or
any manner, any individual who seeks to replace detained for union activities without previous
strikers in entering or leaving the premises of a strike consultations with the Secretary of Labor.
area, or work in place of the strikers. The police force
shall keep out of the picket lines unless actual Section 1-14; Rule XXII, Book V, Omnibus
violence or other criminal acts occur therein: Rules Implementing the Labor Code
Provided, That nothing herein shall be interpreted to
prevent any public officer from taking any measure Section 1. Conciliation of Labor-Management
necessary to maintain peace and order, protect life and Disputes.
property, and/or enforce the law and legal orders.
The Board may, upon request of either of both
(e) No person engaged in picketing shall commit any parties or upon its own initiative, provide
act of violence, coercion or intimidation or obstruct conciliation-mediation services to labor disputes
the free ingress to or egress from the employer’s other than notices of strikes or lockouts.

MNI NOTES |189


Conciliation cases which are not subjects of Section 5. grounds for Strike or Lockout
notices of strike or lockout shall be docketed as
preventive mediation cases. A strike or lockout may be declared in cases of
bargaining deadlocks and unfair labor practices.
Section 2. privileged Communication Violations of collective bargaining agreements,
except flagrant and malicious refusal to comply
Information and statements given in confidence with its economic provisions, shall not be
at conciliation proceedings shall be treated as considered unfair labor practice and shall not be
privileged communications. Conciliators and strikeable. No strike or lockout may be declared
similar officials shall not testify in any court or on grounds involving inter-union and intra-union
body regarding any matter taken up at disputes or without first having filed a notice to
conciliation proceedings conducted by them. strike or lockout or without the necessary strike
or lockout vote having been obtained and
Section3. issuances of Subpoena
reported to the Board. Neither will a strike be
The Board shall have the power to require the declared after assumption of jurisdiction by the
appearance of any parties at conciliation Secretary or after certification or submission of
meetings. the dispute to compulsory or voluntary
arbitration or during the pendency of cases
Section 4. compromise Agreement involving the same grounds for the strike or
lockout.
Any compromise settlement, including those
involving labor standard laws, voluntarily agreed Section 6. Who may Declare a Strike or
upon by the parties with the assistance of the Lockout?
Board and its regional branches shall be final and
binding upon the parties. The National Labor Any certified or duly recognized bargaining
Relations Commission or any court shall not representative ay declare a strike in cases of
assume jurisdiction over issues involved therein bargaining deadlocks and unfair labor practices.
except in case of non-compliance thereof or if The employer may declare a lockout in the same
there is prima facie evidence that the settlement cases. In the absence of a certified or duly
was obtained through fraud, misrepresentation, recognized bargaining representative, any
or coercion. Upon motion of any interested party, legitimate labor organization in the establishment
the Labor Arbiter in the region where the may declare a strike but only grounds of unfair
agreement was reached may issue a writ of labor practice.
execution requiring a sheriff of the Commission
Section 7. Notice of Strike or Lockout
or the courts to enforce the terms of the
agreement. In bargaining deadlocks, a notice of strike or
lockout shall be filed with the regional branch of
the Board at least 30 dyas before the intended
date thereof, a copy of said notice having been
served on the other party concerned. In cases of

MNI NOTES |190


unfair labor practice, the period shall be 15 days. Section 9. Action on Notice
However, incase of unfair labor practice
involving the dismissal from employment of any Upon receipt of the notice, the regional branch of
union officer duly elected in accordance with the the Board shall exert all efforts at mediation and
union constitution and by-laws which may conciliation to enable the parties to settle dispute
constitute union-busting where the existence of amicably. The regional branch of the Board may,
the union is threatened, the 15 day cooling-off upon agreement of the parties, treat a notice as a
period shall not apply and the union may take preventive mediation case. It shall also
action immediately after the strike vote is encourage the parties to submit the dispute to
conducted and the results thereof submitted to the voluntary arbitration.
appropriate regional branch of the Board.
During the proceedings, the parties, shall not do
Section 8. Contents of Notice any act which may disrupt or impede the early
settlement of the dispute. They are obliged, as
The notice shall state, among others, the names part of their duty to bargain collectively in good
and addresses of the employer and the union faith and to participate fully and promptly in the
involved, the nature of the industry to which the conciliation meetings called by the regional
employer belongs, the number of union members branch of the Board.
and of the workers in the bargaining unit, and
such other relevant data as may facilitate the A notice, upon agreement of the parties, may be
settlement of the dispute, such as a brief referred to alternative modes of dispute
statement or enumeration of all pending labor resolution, including voluntary arbitration.
disputes involving the same parties.
Section 10. Strike or Lockout Vote
In cases of bargaining deadlocks, the notice shall
A decision to declare a strike must be approved
as far as practicable, further state the unresolved
by a majority of the total union membership in
issues in the bargaining negotiations and be
the bargaining unit concerned obtained by secret
accomplished by the written proposals of the
ballot in the meetings or referenda called for the
union the counter-proposals of the employer and
purpose. A decision to declare a lockout must be
the proof of a request for conference to settle the
approved by a majority of the Board of Directors
differences. In cases of unfair labor practices, the
of the employer, corporation or association or the
notice shall, as far as practicable, state the acts
partner in a partnership obtained by a secret
complained of and the efforts taken to resolve the
ballot in a meeting called for the purpose.
dispute amicably.
The regional branch of the Board may, at its own
In case a notice does not conform with the
initiative or upon request of any affected party,
requirements, of this and the foregoing Section/s,
supervise the conduct of the secret balloting. In
the regional branch of the Board shall inform the
every case, the union or the employer shall
concerned party of such fact.
furnish the regional branch of the Board and the
notice of meetings referred to in the preceding

MNI NOTES |191


paragraph at least 24 hours before such meetings Section 13. Peaceful Picketing
as well as the results of the voting at least 7 days
before the intended strike or lockout, subject to Workers shall have the right to peaceful
the cooling-off period provided in this Rule. picketing. No person engaged in picketing, shall
commit any act of violence, coercion or
Section 11. Declaration of Strike or Lockout intimidation, or obstruct the free ingress to or
egress from the employer’s premises for lawful
Should the dispute remain unsettled after the purposes, or obstruct public thoroughfares.
lapse of the requisite number of days from the
filing notice of strike or lockout and of the results No person shall obstruct, impede or interfere
of the election required in the preceding Section, with, by force, violence, coercion, threats or
the labor union may strike or the employer may intimidation, any peaceful picketing by workers
lockout workers. The regional branch of the during any labor controversy or in the exercise of
Board shall continue mediating and conciliating. the right to self-organization or collective
bargaining or shall aid or abet such obstruction or
Section 12. Improved Offer Balloting interference. No employer shall use or employ
any person to commit such acts nor shall any
In case of a strike, the regional branch of the
person be employed for such purpose.
Board shall, at its own initiative or upon the
request of any affected party, conduct a Section 14. Injunctions
referendum by secret balloting on the improved
offer of the employer on or before 30th day of No court or entity shall enjoin any picketing
strike. When at least a majority of the union strike, or lockout, except as provided in Article
members vote o accept the improved offer, the 218 and [269] of the Labor Code.
striking workers shall immediately return to work
and the employer shall thereupon re-admit them The commission shall have the power to issue
upon the signing of the agreement. temporary restraining orders in such cases but
only after due notice and hearing and in
In case of a lockout, the regional branch of the accordance with its rules. The reception of
Board shall also conduct a referendum by secret evidence for the application of a writ of
balloting on the reduced offer of the union on or injunction may be delegated by the Commission
before 0th day of the lockout. When at least to any Labor Arbiter who shall submit his
majority of the board of directors or trustees or recommendations to the Commission for its
the partners holding the controlling interest in the consideration and resolution.
case of partnership vote to accept the reduced
offer, the workers shall immediately return to Any ex-parte restraining order issued by the
work and the employer shall thereupon readmit commission, or its Chairman or vice-Chairman
them upon the signing of the agreement. where the Commission is not in session and as
prescribed by its rules, shall be valid for a period
not exceeding 20 days.

MNI NOTES |192


Interphil Laboratories Employees Union v. In March 1993, Ocampo and Clemente again
Interphil Laboratories, GR No. 142824, Dec. approached Salazar. They inquired once more
19, 2001 about the CBA status and received the same reply
from Salazar.
Doctrine
In April 1993, Ocampo requested for a meeting
The Labor Secretary’s authority to assume to discuss the duration and effectivity of the
jurisdiction over a labor dispute must include and CBA. Salazar acceded and a meeting was held on
extend to all questions and controversies arising 15 April 1993 where the union officers asked
therefrom, including cases over which the labor whether Salazar would be amenable to make the
arbiter has exclusive jurisdiction. new CBA effective for two (2) years, starting 01
August 1993. Salazar, however, declared that it
An “overtime boycott” or “work slowdown” by
would still be premature to discuss the matter and
the employees constitutes a violation of the CBA
that the company could not make a decision at the
which prohibits the union or employee, during
moment.
the existence of the CBA, to stage a strike or
engage in slowdown or interruption of work. The next day, all the rank-and-file employees of
the company refused to follow their regular two-
FACTS:
shift work schedule of from 6:00 a.m. to 6:00
Interphill Laboratories Employees Union is the p.m., and from 6:00 p.m. to 6:00 a.m. At 2:00
sole and exclusive bargaining agent of the p.m. and 2:00 a.m., respectively, the employees
ranked-and-file employees of Interphil stopped working and left their workplace without
Laboratories Inc. a company engaged in the sealing the containers and securing the raw
business of manufacturing and packing materials they were working on.
pharmaceutical products. They had a CBA
When Salazar inquired about the reason for their
effective from Aug.1,1990 to July 1993.
refusal to follow their normal work schedule, the
Prior to the expiration of the CBA, Allesandro G. employees told him to “ask the union officers. To
Salazar Vice-President-Human Resources minimize the damage, the overtime boycott was
Department of respondent company, was causing the company, Salazar immediately asked
approached by Nestor Ocampo, the union for a meeting with the union officers.
president, and Hernando Clemente, a union
In the meeting, Enrico Gonzales, a union
director. The two union officers inquired about
director, told Salazar that the employees would
the stand of the company regarding the duration
only return to their normal work schedule if the
of the CBA which was set to expire in a few
company would agree to their demands as to the
months. Salazar told the union officers that the
effectivity and duration of the new CBA. Salazar
matter could be best discussed during the formal
again told the union officers that the matter could
negotiations which would start soon.
be better discussed during the formal
renegotiations of the CBA.

MNI NOTES |193


Since the union was apparently unsatisfied with conditions prevailing prior to the strike, and to
the answer of the company, the overtime boycott pay all the unpaid accrued year end benefits of its
continued. In addition, the employees started to employees in 1993. On the other hand, petitioner
engage in a work slowdown campaign during the union was directed to “strictly and immediately
time they were working, thus substantially comply with the return-to-work orders issued by
delaying the production of the company. the office. The same order pronounced that all
pending cases which are direct offshoots of the
On 14 May 1993, petitioner union submitted with instant labor dispute are hereby subsumed
respondent company its CBA proposal, and the herewith.
latter filed its counter-proposal.
In the interim, the case before Labor Arbiter
On 03 September 1993, respondent company Caday continued. Petitioner union filed an
filed with the NLRC a petition to declare illegal “Urgent Manifestation and Motion to
petitioner union’s “overtime boycott” and “work Consolidate the Instant Case and to Suspend
slowdown” which, according to respondent Proceedings” seeking the consolidation of the
company, amounted to illegal strike. case with the labor dispute pending before the
Secretary of Labor. Despite objection by
On 22 October 1993, respondent company filed
respondent company, Labor Arbiter Caday held
with the NCMB an urgent request for preventive
in abeyance the proceedings before him.
mediation aimed to help the parties in their CBA
However, on 06 June 1994, Acting Labor
negotiations. The parties, however, failed to
Secretary Jose S. Brillantes, after finding that the
arrive at an agreement and on 15 November
issues raised would require a formal hearing and
1993, respondent company filed with the Office
the presentation of evidentiary matters, directed
of the Secretary of Labor and Employment a
Labor Arbiters Caday and M. Sol del Rosario to
petition for assumption of jurisdiction.
proceed with the hearing of the cases before them
On 24 January 1994, petitioner union filed with and to thereafter submit their report and
the NCMB a Notice of Strike citing unfair labor recommendation to his office.
practice allegedly committed by respondent
Ruling of LA: Labor Arbiter Caday submitted
company.
his recommendation to the then Secretary of
On 12 February 1994, the union staged a strike. Labor Leonardo A. Quisumbing. Then Secretary
Quisumbing approved and adopted the report in
On 14 February 1994, Secretary of Labor Nieves his Order declaring the ‘overtime boycott’ and
Confesor issued an assumption order4 over the ‘work slowdown’ as illegal strike.
labor dispute. On 02 March 1994, Secretary
Confesor issued an order directing respondent Petitioners contentions:
company to “immediately accept all striking
- Secretary of Labor and Employment has
workers, including the fifty-three (53) terminated
no jurisdiction over the petition to declare
union officers, shop stewards and union members
back to work under the same terms and

MNI NOTES |194


strike illegal which had long been filed participated in the proceedings, knowing fully
and pending before the LA. well that there was also a directive for Labor
- That the ‘overtime boycott’ and ‘work Arbiter Caday to thereafter submit his report and
slowdown’ does not amount to illegal recommendation to the Secretary. As the
strike. appellate court pointed out, the subsequent
participation of petitioner union in the
ISSUE/S: continuation of the hearing was in effect an
affirmation of the jurisdiction of the Secretary of
1. Whether the Secretary of Labor and
Labor.
Employment has jurisdiction over the
petition to declare strike illegal which had The appellate court also correctly held that the
long been filed and pending before the labor question of the Secretary of Labor and
arbiter. YES. Employment’s jurisdiction over labor and labor-
2. Whether the ‘overtime boycott’ and ‘work related disputes was already settled in
slowdown’ committed by the petitioners International Pharmaceutical, Inc. vs. Hon.
amounted to illegal strike. YES. Secretary of Labor and Associated Labor Union
(ALU) where the Court declared: “In the present
HELD:
case, the Secretary was explicitly granted by
1. Yes. On the matter of the authority and Article 263(g) of the Labor Code the authority to
jurisdiction of the Secretary of Labor and assume jurisdiction over a labor dispute causing
Employment to rule on the illegal strike or likely to cause a strike or lockout in an industry
committed by petitioner union, it is undisputed indispensable to the national interest, and decide
that the petition to declare the strike illegal the same accordingly. Necessarily, this authority
before Labor Arbiter Caday was filed long to assume jurisdiction over the said labor dispute
before the Secretary of Labor and must include and extend to all questions and
Employment issued the assumption order on controversies arising therefrom, including cases
14 February 1994. However, it cannot be over which the labor arbiter has exclusive
denied that the issues of “overtime boycott” jurisdiction.”
and “work slowdown” amounting to illegal
strike before Labor Arbiter Caday are
2. YES. Petitioner union maintained that the
intertwined with the labor dispute before the
Labor Arbiter and the appellate court
Labor Secretary.
disregarded the “parol evidence rule” when
In fact, on 16 March 1994, petitioner union even they upheld the allegation of respondent
asked Labor Arbiter Caday to suspend the company that the work scheduleof its
proceedings before him and consolidate the same employees was from 6:00 a.m. to 6:00 p.m.
with the case before the Secretary of Labor. and from 6:00 p.m. to 6:00 a.m. According to
When Acting Labor Secretary Brillantes ordered petitioner union, the provisions of their CBA
Labor Arbiter Caday to continue with the hearing on working hours clearly stated that the
of the illegal strike case, the parties acceded and normal working hours were “from 7:30 a.m.

MNI NOTES |195


to 4:30 p.m. Petitioner union underscored that no knowledge of some of the reports. Even then,
the regular work hours for the company was the comparison is undeniably of no moment. The
only eight (8) hours. It further contended that boycott and work slowdown resulted in financial
the Labor Arbiter as well as the Court of loss and damage to its reputation.
Appeals should not have admitted any other
evidence contrary to what was stated in the The SC classified the acts by the union as a
CBA. ‘strike on installment basis.’ The overtime
boycott or work slowdown by the employees
The reliance on the parol evidence rule is constituted a violation of their CBA, which
misplaced. In labor cases pending before the prohibits the union or employee, during the
Commission or the Labor Arbiter, the rules of existence of the CBA, to stage a strike or engage
evidence prevailing in courts of law or equity are in slowdown or interruption of work. In Ilaw at
not controlling. Rules of procedure and evidence Buklod ng Manggagawa vs. NLRC, the court
are not applied in a very rigid and technical sense ruled that Slowdown is an inherently illegal
in labor cases. Hence, the Labor Arbiter is not activity wherein the workers purposely remain at
precluded from accepting and evaluating their positions and accept wages, but at the same
evidence other than, and even contrary to, what time, select what parts of their allotted task they
is stated in the CBA. want to perform, and refuse to do other work.

In any case, the CBA said that the work hours Disposiiton: WHEREFORE, the petition is
could change at the company’s discretion. Also, DENIED DUE COURSE and the 29
the employees were aware, and in fact complied December 1999 decision of the Court of
with the 12 hours shifts. Their own witnesses Appeals is AFFIRMED.
show this.

As stated in the facts, it’s clear that the actions


taken by the union amounted to illegal strike. NUWHRAIN-APL-IUF Dusit Hotel Nikko
They also cited a testimony of a union member Chapter v. CA, GR No. 163942, Nov. 11, 2008
where he said that the union officers called for a
FACTS:
stop to the overtime activities. When he
disregarded it and actually went to work, he was On October 24, 2000, the Union submitted its
branded a traitor by the union officers and was Collective Bargaining Agreement (CBA)
shouted at. negotiation proposals to the Hotel. As
negotiations ensued, the parties failed to arrive at
The union also claimed that they had no hand in
mutually acceptable terms and conditions. Due to
the work slowdown – there was no change in
the bargaining deadlock, the Union,
performance/efficiency for the year 1993. This
on December 20, 2001, filed a Notice of Strike
was rebutted by their own witness. She could not
on the ground of the bargaining deadlock with the
answer how she prepared the productivity
National Conciliation and Mediation Board
reports, because she was on union leave. She had
(NCMB)

MNI NOTES |196


Soon thereafter, in the afternoon of January 17, sixty-one (61) members; and suspended eighty-
2002, the Union held a general assembly at its one (81) employees for 30 days, forty-eight (48)
office located in the Hotels basement, where employees for 15 days, four (4) employees for 10
some members sported closely cropped hair or days, and three (3) employees for five days. On
cleanly shaven heads. The next day, or on the same day, the Union declared a strike.
January 18, 2002, more male Union members Starting that day, the Union engaged in picketing
came to work sporting the same hair style. The the premises of the Hotel. During the picket, the
Hotel prevented these workers from entering the Union officials and members unlawfully blocked
premises claiming that they violated the Hotels the ingress and egress of the Hotel premises.
Grooming Standards.
Consequently, on January 31, 2002, the Union
In view of the Hotels action, the Union staged a filed its third Notice of Strike with the NCMB
picket outside the Hotel premises. Later, other which was docketed as NCMB-NCR-NS-01-
workers were also prevented from entering the 050-02, this time on the ground of unfair labor
Hotel causing them to join the picket. For this practice and union-busting.
reason the Hotel experienced a severe lack of
manpower which forced them to temporarily On the same day, the Secretary, through her
cease operations in three restaurants January 31, 2002 Order, assumed jurisdiction
over the labor dispute and certified the case to the
Subsequently, on January 20, 2002, the Hotel NLRC for compulsory arbitration
issued notices to Union members, preventively
suspending them and charging them with the Pursuant to the Secretarys Order, the Hotel, on
following offenses: (1) violation of the duty to February 1, 2002, issued an Inter-Office
bargain in good faith; (2) illegal picket; (3) unfair Memorandum,[9] directing some of the
labor practice; (4) violation of the Hotels employees to return to work, while advising
Grooming Standards; (5) illegal strike; and (6) others not to do so, as they were placed under
commission of illegal acts during the illegal payroll reinstatement.
strike. The next day, the Union filed with the
Unhappy with the Secretarys January 31, 2002
NCMB a second Notice of Strike on the ground
Order, the Union moved for reconsideration, but
of unfair labor practice and violation of Article
the same was denied per the Secretarys
248(a) of the Labor Code on illegal lockout,
subsequent March 15, 2002 Order. Affronted by
which was docketed as NCMB-NCR-NS-01-
the Secretarys January 31, 2002 and March 15,
019-02. In the meantime, the Union officers and
2002 Orders, the Union filed a Petition for
members submitted their explanations to the
Certiorari with the CA
charges alleged by the Hotel, while they
continued to stage a picket just inside the Hotels Meanwhile, after due proceedings, the NLRC
compound issued its October 9, 2002 Decision in NLRC
NCR CC No. 000215-02, in which it ordered the
On January 26, 2002, the Hotel terminated the
Hotel and the Union to execute a CBA within 30
services of twenty-nine (29) Union officers and
days from the receipt of the decision

MNI NOTES |197


The Union then filed a Motion for (3) [when it] is declared for an unlawful
Reconsideration of the NLRCs Decision which purpose, such as inducing the employer to
was denied in the February 7, 2003 NLRC commit an unfair labor practice against non-
Resolution. Unfazed, the Union filed a Petition union employees; or
for Certiorari under Rule 65 with the CA,
docketed as CA-G.R. SP No. 76568, and assailed (4) [when it] employs unlawful means in the
both the October 9, 2002 Decision and the pursuit of its objective, such as a widespread
February 7, 2003 Resolution of the NLRC. terrorism of non-strikers [for example, prohibited
acts under Art. 264(e) of the Labor Code]; or
ISSUE/S:
(5) [when it] is declared in violation of an
Whether Or Not The Union, The 29 Union existing injunction[, such as injunction,
Officers And 61 Members May Be Adjudged prohibition, or order issued by the DOLE
Guilty Of Staging An Illegal Strike On January Secretary and the NLRC under Art. 263 of the
18, 2002 Despite Respondents Admission That Labor Code]; or
They Prevented Said Officers And Members
From Reporting For Work For Alleged Violation (6) [when it] is contrary to an existing
Of The Hotels Grooming Standards. - YES agreement, such as a no-strike clause or
conclusive arbitration clause.[18]
HELD:
With the foregoing parameters as guide and the
Art. 212(o) of the Labor Code defines a strike as following grounds as basis, we hold that the
any temporary stoppage of work by the concerted Union is liable for conducting an illegal strike for
action of employees as a result of an industrial or the following reasons
labor dispute.
First, the Unions violation of the Hotels
In Toyota Motor Phils. Corp. Workers Grooming Standards was clearly a deliberate and
Association (TMPCWA) v. National Labor concerted action to undermine the authority of
Relations Commission, we cited the various and to embarrass the Hotel and was, therefore,
categories of an illegal strike, to wit: not a protected action. The appearances of the
Hotel employees directly reflect the character
Noted authority on labor law, Ludwig Teller, lists and well-being of the Hotel, being a five-star
six (6) categories of an illegal strike, viz.: hotel that provides service to top-notch clients.
Being bald or having cropped hair per se does not
(1) [when it] is contrary to a specific prohibition
evoke negative or unpleasant feelings
of law, such as strike by employees performing
governmental functions; or Second, the Unions concerted action which
disrupted the Hotels operations clearly violated
(2) [when it] violates a specific requirement of
the CBAs No Strike, No Lockout provision,
law[, such as Article 263 of the Labor Code on
which reads:
the requisites of a valid strike]; or

MNI NOTES |198


ARTICLE XXII NO STRIKE/WORK strike was illegal since, as shown by the
STOPPAGE AND LOCKOUT pictures[21] presented by the Hotel, the Union
officers and members formed human barricades
SECTION 1. No Strikes and obstructed the driveway of the Hotel.

The Union agrees that there shall be no strikes, Furthermore, this Court, not being a trier of
walkouts, stoppage or slow-down of work, facts, finds no reason to alter or disturb the
boycott, refusal to handle accounts, picketing, NLRC findings on this matter, these findings
sit-down strikes, sympathy strikes or any other being based on substantial evidence and
form of interference and/or interruptions with affirmed by the CA. Factual findings of labor
any of the normal operations of the HOTEL officials, who are deemed to have acquired
during the life of this Agreement. expertise in matters within their respective
jurisdictions, are generally accorded not only
The facts are clear that the strike arose out of
respect but even finality, and bind us when
a bargaining deadlock in the CBA
supported by substantial evidence.[23] Likewise,
negotiations with the Hotel. The concerted
we are not duty-bound to delve into the accuracy
action is an economic strike upon which the
of the factual findings of the NLRC in the
afore-quoted no strike/work stoppage and
absence of clear showing that these were arrived
lockout prohibition is squarely applicable and
at arbitrarily and/or bereft of any rational basis
legally binding
Clearly, the 29 Union officers may be dismissed
Third, the Union officers and members
pursuant to Art. 264(a), par. 3 of the Labor Code
concerted action to shave their heads and crop
which imposes the penalty of dismissal on any
their hair not only violated the Hotels Grooming
union officer who knowingly participates in an
Standards but also violated the Unions duty and
illegal strike. We, however, are of the opinion
responsibility to bargain in good faith. By
that there is room for leniency with respect to the
shaving their heads and cropping their hair, the
Union members. It is pertinent to note that the
Union officers and members violated then
Hotel was able to prove before the NLRC that the
Section 6, Rule XIII of the Implementing Rules
strikers blocked the ingress to and egress from
of Book V of the Labor Code.
the Hotel. But it is quite apparent that the Hotel
Fourth, the Union failed to observe the failed to specifically point out the participation of
mandatory 30-day cooling-off period and the each of the Union members in the commission of
seven-day strike ban before it conducted the illegal acts during the picket and the strike.
strike on January 18, 2002. The NLRC correctly
For this lapse in judgment or diligence, we are
held that the Union failed to observe the
constrained to reinstate the 61 Union member.
mandatory periods before conducting or holding
a strike

Last, the Union committed illegal acts in the


conduct of its strike. The NLRC ruled that the

MNI NOTES |199


Club Filipino v. Bautista, GR No. 200487, Meanwhile, the union conducted a strike vote
Mar. 6, 2019 under the supervision of the DOLE.

FACTS: In response to the company’s counter-proposal,


the union sent the company its improved
Petitioner Club Filipino, Inc. (the company) is a proposal, but the company refused to improve on
non-stock, non profit corporation duly formed, its offer. This prompted the union to stage a strike
organized and existing under Philippine laws. on May 26, 2001 on the ground of a CBA
bargaining deadlock.
Respondents on the other hand, were former
officers and members of the Club Filipino The company filed before the National Labor
Employees Association (the union). Relations Commission (NLRC) a petition to
declare the strike illegal, and prayed that all union
The union and the company had a collective
officers who participated in the illegal strike be
bargaining agreement. Prior to the expiration of
considered separated from the service.
the CBA and within the freedom period, the
union made several demands for negotiation but The labor arbiter declared the strike procedurally
the company replied that it could not muster a [infirm] and therefore illegal. The labor arbiter
quorum, thus no CBA negotiations could be held. noted that the union failed to attach its written
CBA proposal and the company’s counter-
The union submitted its formal CBA proposal to
proposal to the notice of strike and to provide
the company’s negotiating panel and repeatedly
proof of a request for a conference to settle the
asked for the start of negotiations. No
dispute.
negotiations, however, took place for various
reasons proffered by the company, among them Thus, the notice to strike was deemed not to have
the illness of the chairman of the management been filed and the strike illegal. All the officers
panel. of the union were deemed terminated from
service. However, they were entitled to
In order to compel the company to negotiate,
separation pay equivalent to that granted to
respondents, as officers of the union, filed a
employees affected by the retrenchment program
request for preventive mediation with the
which the company had earlier launched.
National Conciliation and Mediation Board
(NCMB). However, meeting concluded with a Respondents appealed but the NLRC affirmed
declaration by both parties of a deadlock in their the labor arbiter. The NLRC did not see fit to pass
negotiations. upon the issues raised by respondents because, by
the time they appealed, they had either resigned
The union filed a notice of strike with the NCMB
from the company or were no longer part of the
on the grounds of bargaining deadlock and
union because of the election of new set of
failure to bargain. The company formally
officers.
responded to the demands of the union when it
submitted its economic counter-proposal.

MNI NOTES |200


The CA set aside the rulings of the NLRC and the proposal. This was almost a year after the
labor arbiter. expiration of the CBA sought to be renewed.

ISSUE/S: The Implementing Rules use the words as far as


practicable.
Whether or not the strike was legal. - NO
In this case, attaching the counter-
HELD: proposal of the company to the notice of strike of
the union was not practicable. It was absurd to
Rule XXII, Section 4 of the Omnibus Rules
expect the union to produce the companys
Implementing the Labor Code states:
counter-proposal which it did not have. One
In cases of bargaining deadlocks, the cannot give what one does not have. Indeed,
notice shall, as far as practicable, further state compliance with the requirement was impossible
the unresolved issues in the bargaining because no counter-proposal existed at the time
negotiations and be accompanied by the written the union filed a notice of strike. The law does
proposals of the union, the counter-proposals of not exact compliance with the impossible. Nemo
the employer and the proof of a request for tenetur ad impossibile.
conference to settle differences. In cases of unfair
Another error committed by the labor arbiter was
labor practices, the notice shall, as far as
his declaration that respondents, as union
practicable, state the acts complained of, and
officers, automatically severed their employment
efforts taken to resolve the dispute amicably.
with the company due to the alleged illegal strike.
Any notice which does not conform with In the first place, there was no illegal strike.
the requirements of this and the foregoing section Moreover, it is hornbook doctrine that a mere
shall be deemed as not having been filed and the finding of the illegality of the strike should not be
party concerned shall be so informed by the automatically followed by the wholesale
regional branch of the Board. dismissal of the strikers from employment.

In the instant case, the union cannot be faulted The law is clear:
for its omission. The union could not have
Any union officer who knowingly participates in
attached the counter-proposal of the company in
an illegal strike and any worker or union officer
the notice of strike it submitted to the NCMB as
who knowingly participates in the commission of
there was no such counter-proposal. To recall,
illegal acts during a strike may be declared to
the union filed a notice of strike after several
have lost his employment status.
requests to start negotiations proved futile. It was
only after two weeks when the company formally Note that the verb participates is preceded by the
responded to the union by submitting the first adverb knowingly. This reflects the intent of the
part of its counter-proposal. Worse, it took the legislature to require knowledge as a
company another three weeks to complete it by condition sine qua non before a union officer can
submitting the second part of its counter-

MNI NOTES |201


be dismissed from employment for participating obtain the “strike vote” from its members.
in an illegal strike. According to Bigg’s, the union filed a Notice of
Strike with the NCMB on the same day to
The provision is worded in such a way as to make conceal the illegality of the sit-down strike.
it very difficult for employers to circumvent the Bigg’s issued a memorandum placing the striking
law by arbitrarily dismissing employees in the members in preventive suspension and requiring
guise of exercising management prerogative. them to explain their actions within 24 hours
This is but one aspect of the States constitutional from notice. The union members did not comply
and statutory mandate to protect the rights of with the company’s order so they were sent
employees to self-organization. employment termination letters on February 19,
1996.
Nowhere in the ruling of the labor arbiter can we
find any discussion of how respondents, as union On the other hand, the union members
officers, knowingly participated in the alleged accuse Bigg’s of interfering with union activities.
illegal strike. Thus, even assuming arguendo that They allege that on February 1996, union
the strike was illegal, their automatic dismissal members were asked to withdraw their
had no basis. membership under threat of losing their
employment. On the day of the alleged sit-down
strike, union president Boncacas and other union
Bigg’s Inc. v. Boncacas, GR No. 200487, Mar. members were prevented from entering the
6, 2019 premises of Bigg’s. On the same day, they filed a
Notice of Strike with the NCMB. The following
FACTS: day they were informed, through a memoranda,
of their suspension from work for participation in
Bigg’s Inc. (Bigg’s) operates a chain of a sit-down strike. Some union members tried to
restaurants. Its employees formed a labor union talk with the Bigg’s management, but they were
named Bigg’s Employees Union (Union) which told not to report for work the next day.
was issued a Certificate of Registration by the
Department of Employment. The union members filed a complaint
before NCMB for unfair labor practice, illegal
Both parties have contrasting versions of dismissal, and damages. Bigg’s also filed a
the incidents leading to the conflict between the complaint before NCMB for illegal strike against
Bigg’s management and the union members. the union members.

Bigg’s alleges that on February 16, 1996, On the issue of the illegality of strike The
around 50 union members staged an illegal “sit- Labor Arbiter ruled in favor of Bigg’s. Under the
down strike” in Bigg’s restaurant. The union did provisions of Article 263 of the Labor Code and
not comply with the requirements of sending a its implementing rules, for a strike to enjoy the
Notice of Strike to the National Conciliation and protection of the law, the union must observe
Mediation Board (NCMB). Neither did the union certain procedural requirements. The LA held

MNI NOTES |202


that the strike conducted by the union members action of employees as a result of an industrial or
on February 16, 1996 was illegal for failure to labor dispute.
furnish Bigg’s a Notice of Strike and for not
observing the cooling-off period. Under Article 278 (formerly Article 263)
of the Labor Code, there are different procedural
The NLRC initially reversed the LA requirements depending on the ground of the
Decision ruling that the strike was valid because strike:
it was grounded on unfair labor practices
committed by Bigg’s, however, on motion for (c) In cases of bargaining deadlocks, the
reconsideration, the NLRC reversed its own duly certified or recognized bargaining agent
ruling and held that the two strikes staged by the may file a notice of strike or the employer may
union were illegal. file a notice of lockout with the Ministry at least
30 days before the intended date thereof. In cases
Both parties elevated the case to the Court of unfair labor practice, the period of notice shall
of Appeals. be 15 days and in the absence of a duly certified
or recognized bargaining agent, the notice of
The CA overturned the findings of the strike may be filed by any legitimate labor
NLRC and held that Bigg’s failed to adduce organization in behalf of its members. However,
substantial evidence showing that the union in case of dismissal from employment of union
conducted a sit-down strike on February 16, officers duly elected in accordance with the
1996. union constitution and by-laws, which may
constitute union busting where the existence of
Both parties filed their respective petition
the union is threatened, the 15-day cooling-off
for review on certiorari before the Supreme
period shall not apply and the union may take
Court.
action immediately.
ISSUE/S: Whether or not the strike held on
(d) The notice must be in accordance with
February 16, 1996 was illegal for failure to
such implementing rules and regulations as the
observe the cooling-off period? - YES
Minister of Labor and Employment may
HELD: promulgate.

YES, the strike held on February 16, 1996 was (e) During the cooling-off period, it shall
illegal. be the duty of the Ministry to exert all efforts at
mediation and conciliation to effect a voluntary
settlement. Should the dispute remain unsettled
until the lapse of the requisite number of days
REQUIREMENTS OF A VALID STRIKE from the mandatory filing of the notice, the labor
union may strike or the employer may declare a
As defined under Article 219 (formerly
lockout.
Article 212) (o) of the Labor Code, a strike means
any temporary stoppage of work by the concerted

MNI NOTES |203


(f) A decision to declare a strike must be Only a certified or duly recognized
approved by a majority of the total union bargaining representative may declare a strike in
membership in the bargaining unit concerned, case of a bargaining deadlock. However, in cases
obtained by secret ballot in meetings or of unfair labor practices, the strike may be
referenda called for that purpose. declared by any legitimate labor organization.

A decision to declare a lockout must be In both instances, the union must conduct
approved by a majority of the board of directors a "strike vote" which requires that the actual
of the corporation or association or of the strike is approved by majority of the total union
partners in a partnership, obtained by secret membership in the bargaining unit concerned.
ballot in a meeting called for that purpose. The union is required to notify the regional
branch of the NCMB of the conduct of the strike
The decision shall be valid for the vote at least 24 hours before the conduct of the
duration of the dispute based on substantially the voting. Thereafter, the union must furnish the
same grounds considered when the strike or NCMB with the results of the voting at least
lockout vote was taken. seven days before the intended strike or
lockout.[50] This seven-day period has been
The Ministry may, at its own initiative or
referred to as the "seven-day strike ban" or
upon the request of any affected party, supervise
"seven-day waiting period."
the conduct of the secret balloting.
In Lapanday Workers Union v. National
In every case, the union or the employer
Labor Relations Commission, the Court reasoned
shall furnish the Ministry the results of the voting
that the period is intended to give the NCMB an
at least seven days before the intended strike or
opportunity to verify whether the projected strike
lockout, subject to the cooling-off period herein
really carries the imprimatur of the majority of
provided.”
the union members. In a strike due to bargaining
This provision was further implemented deadlocks, the union must file a notice of strike
by Department Order (DO) Order No. 40-03, or lockout with the regional branch of the NCMB
Amending the Implementing Rules of Book V of at least 30 days before the intended date of the
the Labor Code of the Philippines (IRR) and DO strike and serve a copy of the notice on the
40-A-03[47] which amended Section 5, Rule employer. This is the so-called "cooling-
XXII of the IRR. off period" when the parties may enter into
compromise agreements to prevent the strike. In
The Labor Code and the IRR limit the case of unfair labor practice, the period of notice
grounds for a valid strike to: (1) a bargaining is shortened to 15 days; in case of union busting,
deadlock in the course of collective bargaining, the "cooling-off period" does not apply and the
or (2) the conduct of unfair labor practices by the union may immediately conduct the strike after
employer. the strike vote and after submitting the results
thereof to the regional arbitration branch of the

MNI NOTES |204


NCMB at least seven days before the intended Moreover, the Court affirms the findings
strike. of the labor tribunals that the union failed to
prove with substantial evidence that Bigg's was
Thus, in a strike grounded on unfair guilty of unfair labor practice as defined under
labor practice, the following are the Article 259 of the Labor Code to allow the union,
requirements: (1) the strike may be declared by a non-certified bargaining agent to initiate the
the duly certified bargaining agent or legitimate strike. Likewise, the union failed to prove that
labor organization; (2) the conduct of the strike there was union busting to exempt compliance
vote in accordance with the notice and reportorial with the cooling-off period. The union did not
requirements to the NCMB and subject to the present any substantial evidence to prove its
seven-day waiting period; (3) notice of strike allegations that union members were actually
filed with the NCMB and copy furnished to the dismissed or threatened with dismissal for their
employer, subject to the 15-day cooling- union membership.
off period. In cases of union busting, the 15-
day cooling-off period shall not apply. In fine, the union's failure to comply with
the mandatory requirements rendered the strike
The Court reinstates and affirms the on February 16, 1996 illegal.
ruling of the NLRC, which had, for its part,
affirmed the findings of the LA that the union
conducted an illegal sit-down strike on February
16, 1996, for failure of the union to comply with Santa Rosa Coca Cola Plant Employees Union
the pre-requisites for a valid strike. v. Coca-Cola Bottlers, GR No. 164301-03, Jan.
24, 2007
The union did not file the requisite
Notice of Strike and failed to observe the FACTS:
cooling-off period. In an effort to legitimize
Petitioner Union is the sole and exclusive
the strike on February 16, 1996, the union
bargaining representative of the regular daily
filed a Notice of Strike on the same day. This
paid workers and the monthly paid non -
cannot be considered as compliance with the
commission earning employees of the respondent
requirement, as the cooling-off period is
Company. The individual petitioners are Union
mandatory. The cooling-off period is not
officers, directors, and shop stewards.
merely a period during which the union and
the employer must simply wait. The purpose On Aug. 30, 1999, the petitioners filed a “Notice
of the cooling-off period is to allow the parties of Strike” with the NCMB Regional Office in
to negotiate and seek a peaceful settlement of Imus, Cavite. The petitioners relied on two
their dispute to prevent the actual conduct of grounds:
the strike. In other words, there must be
genuine efforts to amicably resolve the a.) deadlock on CBA ground rules; and
dispute. b.) ULP arising from the company’s refusal to
bargain.

MNI NOTES |205


The respondent filed a Motion to Dismiss constituted ULP; and the officers who knowingly
alleging that the reasons cited by the Union were participated in the commission of illegal acts
not valid grounds for a strike. during the strike should be declared to have lost
their employment status.
The Union then filed an Amended Notice of
Strike on Sept. 17, 1999 on the ff. grounds: The Union, in its Answer with a Motion to
Dismiss, alleged that the mass action conducted
a.) ULP for the company’s refusal to bargain by its officers and members was not a strike but
in good faith; and just a valid exercise of their right to picket.
b.) interference with the exercise of their right
to self-organization. The Labor Arbiter granted the petition of the
respondent Company. NLRC affirmed. CA
On Sept. 15, 1999, the Union decided to affirmed the NLRC’s decision. Hence this
participate in a mass action organized by petition.
the Alyansa ng mga Unyon sa Coca-Cola in front
of the Company’s premises set for Sept. 21, 1999. ISSUE/S:
The petitioners filed for leave of absence for the
latter date. a.) Whether the Sept. 21, 1999 mass action
staged by the Union was a strike; YES
The Company disapproved all leave applications b.) if, in the affirmative, whether it was legal; NO
and notified the applicants accordingly. A day c.) Whether the individual officers and shop
before the mass action, some Union members stewards of petitioner Union should be dismissed
wore gears, red tag cloths stating “YES KAMI from their employment. – may be terminated
SA STRIKE” as headgears and on the different from employment
parts of their uniform, shoulder and chests.
HELD:
The office of the Mayor issued a permit to the
Union, allowing it to conduct a mass protest. a.) YES. The ruling of the CA that petitioners
Thus, the Union officers and members held a staged a strike on Sept. 21, 1999, and not merely
picket on the Company’s premises. a picket is correct. The term “strike”
encompasses not only concerted work stoppages,
The Company filed a “petition to declare strike but also slowdowns, mass leaves, sit-downs,
illegal” on the ground that the petitioners staged attempts to damage, destroy or sabotage plant
a strike at the premises of the Company without equipment and facilities, and similar activities.
observing the requirements mandated by law.
Moreover, because of the slowdown in the work, Petitioners notified the respondent of
the Company suffered losses. their intention to stage a strike, and not merely to
picket. Petitioners’ insistence to stage a strike is
The Union also violated the provision of the CBA evident in the fact that an amended notice to
on the grievance machinery; there being a direct strike was filed even as respondent moved to
violation of the CBA, the Union’s action dismiss the first notice.

MNI NOTES |206


The bare fact that petitioners were given during a strike, the law provides that a union
a Mayor’s permit is not conclusive evidence that officer may be terminated from employment.
their action/activity did not amount to strike. The
Mayor’s description of what activities petitioners Petitioners who are shop stewards are considered
were allowed to conduct is inconsequential. union officers. In this case, instead of playing the
role of “peacemakers” and grievance solvers, the
What is definitive of whether the action staged by petitioners-shop stewards participated in the
petitioners is a strike and not merely a picket is strike. Thus, like the officers and directors of
the totality of the circumstances surrounding the petitioner Union who joined the strike,
situation. petitioners-shop stewards also deserve the
b.) NO. For a strike to be valid, the ff. procedural penalty of dismissal from their employment.
requisites provided by the Labor Code must be
observed: The petition is denied for lack of merit.
• A notice of strike filed with the DOLE 30 days
before the intended date thereof, or 15 days in
case of ULP; Hongkong & Shanghai Banking Corp. v.
• Strike vote approved by a majority of the total NLRX, GR No. 156335, Jan. 11, 2016
union membership in the bargaining unit
concerned obtained by secret ballot in a meeting
called for that purpose;
• Notice given to the DOLE of the results of the Ramirez v. Polyson Industries, Inc. GR No.
voting at least 7 days before the intended strike. 207898, Oct. 19, 2016

These requirements are mandatory and the failure


of a union to comply therewith renders strike
illegal. It is clear in this case that petitioners
totally ignored the statutory requirements and
embarked on their illegal strike.

c.) The law makes a distinction between union


members and union officers.

A worker merely participating in an illegal strike


may not be terminated from employment. It is
only when he commits illegal acts during a strike
that he may be declared to have lost employment
status.

For knowingly participating in an illegal strike or


participates in the commission of illegal acts

MNI NOTES |207


C. “Usap Muna Tayo”: Grievance <achinery, health personnel, whose movement and services shall
Arbitration, and Assumption of Jurisdiction be unhampered and unrestricted, as are necessary to
insure the proper and adequate protection of the life
Article 278(g), LC and health of its patients, most especially emergency
cases, for the duration of the strike or lockout.
(g) When, in his opinion, there exists a labor dispute
causing or likely to cause a strike or lockout in an In such cases, therefore, the Secretary of Labor and
industry indispensable to the national interest, the Employment may immediately assume, within twenty
Secretary of Labor and Employment may assume four (24) hours from knowledge of the occurrence of
jurisdiction over the dispute and decide it or certify such a strike or lockout, jurisdiction over the same or
the same to the Commission for compulsory certify it to the Commission for compulsory
arbitration. Such assumption or certification shall arbitration.
have the effect of automatically enjoining the
intended or impending strike or lockout as specified For this purpose, the contending parties are strictly
in the assumption or certification order. enjoined to comply with such orders, prohibitions
and/or injunctions as are issued by the Secretary of
If one has already taken place at the time of Labor and Employment or the Commission, under
assumption or certification, all striking or locked out pain of immediate disciplinary action, including
employees shall immediately return to work and the dismissal or loss of employment status or payment by
employer shall immediately resume operations and the locking-out employer of backwages, damages and
readmit all workers under the same terms and other affirmative relief, even criminal prosecution
conditions prevailing before the strike or lockout. The against either or both of them.
Secretary of Labor and Employment or the
Commission may seek the assistance of law The foregoing notwithstanding, the President of the
enforcement agencies to ensure compliance with this Philippines shall not be precluded from determining
provision as well as with such orders as he may issue the industries that, in his opinion, are indispensable to
to enforce the same. the national interest, and from intervening at any time
and assuming jurisdiction over any such labor dispute
In line with the national concern for and the highest in order to settle or terminate the same.
respect accorded to the right of patients to life and
health, strikes and lockouts in hospitals, clinics and DOLE Order No. 40-H-2013
similar medical institutions shall, to every extent
possible, be avoided, and all serious efforts, not only
by labor and management but government as well, be
exhausted to substantially minimize, if not prevent,
San Fernando Cocal-Cola Rank-and-File
their adverse effects on such life and health, through Union v. CCBPI, GR No. 200499, Oct. 4, 2017
the exercise, however legitimate, by labor of its right
to strike and by management to lockout.

In labor disputes adversely affecting the continued University of the Immaculate Concepcion v.
operation of such hospitals, clinics or medical Office of the Secretary of Labor and
institutions, it shall be the duty of the striking union Employment, GR No. 178085-178086, Sept.
or locking-out employer to provide and maintain an 14, 2015
effective skeletal workforce of medical and other

MNI NOTES |208


Octavio v. PLDT, GR No. 175492, Feb. 27,
2013

Goya Inc. v. Goya Employee’s Union, GR No.


170054, Jan. 21, 2013

Honda Cars v. honda Cars technical


Specialist, GR. No. 204102, Nov. 19, 2014

Ace Navigation Co,. Inc. v. Fernandez, GR No.


197309, Oct. 10, 2012

Albay Electric Cooperative, Inc. v. ALECO


Labor Employees Organization, GR No.
241437, Sept. 14, 2020

MNI NOTES |209


MODULE 8 ONE MONTH NOTICE

→ An employee who intends to voluntarily resign


from his employment should give his employer a
A. Resignation v. Constructive Dismissal; Just written notice (resignation letter) at least one (1)
v. Authorized Causes month in advance

DOLE Memorandum Order No. 147-2015 → WITHOUT 1-MONTH NOTICE – employer can
hold him liable for damages
VOLUNTARY RESIGNATION
→ The employer cannot compel him to render
→ Formal renouncement or relinquishment of an service during the period as it amounts to
office involuntary servitude.

→ Voluntary act of severing an employment relation PURPOSE OF THE ONE-MONTH NOTICE


at the initiative of the employee who finds himself in
a situation where he believes that personal reasons → To enable the employer to look for a replacement
cannot be sacrificed in favor of the exigency of and therefore, prevent a disruption of work
service that he has no other choice but to dissociate
himself from his employment WAIVER OF THE ONE-MONTH RULE

→ Must be unconditional and WITH INTENT to → The one-month notice may be waived by the
operate as such employer 

→ There must be an INTENTION TO RELINQUISH → The rule requiring an employee to stay or complete
a portion of the terms of the office accompanied by an the 30-day period prior to the effectivity of his
act of relinquishment resignation is discretionary on the part of the
employer
→ Can be inferred from the wordings of the letter or
memorandum EFFECT OF ACCEPTANCE OF
RESIGNATION
→Inferred from the actuations of the employee
→ Resignation may not be withdrawn without the
RESIGNATION AS ALTERNATIVE TO consent of the employer.
DISMISSAL
→ The moment an employee resigns and his
→ The voluntariness of resignation is not negated by resignation is accepted, he no longer has any right to
the fact that the employer persuades an employee to the job
resign instead of being dismissed for cause
→ IF EMPLOYEE CHANGES HIS MIND – he
→If a result of reorganization, the employee is given must ask for approval of the withdrawal of his
the option to resign or be terminated with separation resignation from his employer
pay, and the employee chooses to resign, the
resignation is still voluntary o EMPLOYER ACCEPTS – employee
retains his job

MNI NOTES |210


o EMPLOYER DOES NOT ACCEPT – → When an employee his employment, there is
employee cannot claim illegal dismissal constructive resignation

 REASON: Employer has the right → Mere absence does not by itself indicate
to determine who his employees will abandonment of employment. There must be overt
be acts unerringly pointing to the fact that the employee
does not want to work anymore.
→ This is in recognition of the contractual nature of
employment which requires mutuality of consent Intent can be inferred from the following:
between the parties. An employment contract is
CONSENSUAL and VOLUNTARY a.) Failure of the employee to comply with notices or
directives for him to report for work;
→ A resigned employee who desires to take his job
back has to re-apply therefor, and he shall have the b.) Failure to report for work within a reasonable time
status of a stranger who cannot unilaterally demand after expiration of leave of absence without pay;
an appointment.
c.) Failure to report for work despite disapproval of
ENTITLEMENT TO SEPARATION PAY application for indefinite leave of absence;

GENERAL RULE: An employee who voluntarily d.) Prolonged absences without justifiable reason
resigns from his employment is not entitled to
General Rule: Intent to abandon is negated by the
separation
immediate filing of a complaint for illegal dismissal.
EXCEPTION:
Exception: When the complaint for illegal dismissal
1. Stipulation in the employment contract does not pray for reinstatement, but only for
separation pay.
2. Collective bargaining agreement
Abandonment and Absence Without Leave
3. Sanctioned by established employer (AWOL)
practice or policy

CONSTRUCTIVE RESIGNATION Abandonment AWOL


(Abandonment of Employment)
No intention to return There is intent to return
→ Deliberate, unjustified refusal of an employee to to work. to work.
resume his work
The issue of whether or not an employee abandoned
→ Voluntary act of the employee akin to voluntary his employment is a question of fact. The burden is on
recognition the employer to show clear and deliberate intent on
the part of the employee to discontinue employment
→ Employee just quits his employment without
without intention of returning.
notice

MNI NOTES |211


An employee can still be sanctioned for absence → One-Month Notice Not Required
without leave, in the event abandonment is not
proven. -The employee can leave his employment
immediately.
Involuntary Resignation (Constructive
Dismissal) Relief for Constructive Dismissal

Involuntary resignation is a situation where an The appropriate relief is separation pay plus
employee is constrained to quit his job because indemnities in the form of nominal damages or back
continued employment is rendered impossible, wages. Reinstatement is not a proper relief because of
unreasonable or unlikely; when there is a demotion in strained relations between the parties.
rank, diminution in pay or when a clear
discrimination, insensibility or disdain by an Retrenchment
employer becomes unbearable to the employee.
→Reduction of personnel due to actual or anticipated
Grounds Under Art 285(b) losses, lack of work, or reduction in the volume of
business
a.) Serious insult by the employer or his
→“Retrenchment to prevent losses”
representative on the honor and person of the
employee;
o Art. 298 (283), Labor Code
- Where an employee who quits his
Closure of Establishment and
employment after being demoted without just
Reduction of Personnel.
cause.
The employer may also terminate the
b.) Inhuman and unbearable treatment accorded the
employment of any employee due to
employee by the employer or his representative;
the installation of labor-saving
- Where an employee quits his employment devices, redundancy, retrenchment
to prevent losses or the closing or
because of a legitimate desire for self-
cessation of operation of the
preservation.
establishment or undertaking unless
c.) Commission of a crime or offense by the employer the closing is for the purpose of
or his representative against the person of the circumventing the provisions of this
employee or any of the immediate members of his Title, by serving a written notice on
family; and the workers and the Ministry of
Labor and Employment at least one
- Where the employer or his representative (1) month before the intended date
commits rape, physical injuries, mutilation, thereof. In case of termination due to
abortion, infanticide, homicide, murder, the installation of labor-saving
parricide, etc. against the employee or the devices or redundancy, the worker
immediate members of his family. affected thereby shall be entitled to
a separation pay equivalent to at
d.) Other causes analogous to any of the foregoing least his one (1) month pay or to at

MNI NOTES |212


least one (1) month pay for every means, have been tried and found wanting or
year of service, whichever is higher. insufficient
In case of retrenchment to prevent
losses and in cases of closures or 4. The alleged losses already realized and the
cessation of operations of expected imminent losses sought to be forestalled,
establishment or undertaking not due must be proven by sufficient and convincing evidence
to serious business losses or financial
reverses, the separation pay shall be REQUISITES OF VALID
equivalent to one (1) month pay or RETRENCHMENT
at least one-half (1/2) month pay
for every year of service, 1. That the retrenchment is reasonably necessary and
whichever is higher. A fraction of at likely to prevent losses which, if already incurred, are
least six (6) months shall be not merely de minimis but substantial, serious, actual
considered one (1) whole year. and real, or if only expected, are reasonably imminent
as perceived objectively and in good faith by the
o An employer can adopt retrenchment employer
measures even before the anticipated losses
are actually sustained 2. That the employer exercises its prerogative to
retrench employees in good faith for the advancement
o Resorted to by an employer primarily to of its interest and not to defeat or circumvent the
avoid or minimize business losses employees’ right to security of tenure

o The lawmaker did not intend that the losses 3. That the employer used fair and reasonable criteria
shall have in fact materialized before in ascertaining who would be dismissed and who
adopting retrenchment measures would be retained among the employees, such as:

→Potential losses that are speculative cannot justify a. Less preferred status, i.e., whether they are
retrenchment temporary, casual, regular, or managerial
employees
THE FOUR STANDARDS OF
RETRENCHMENT (Substantive b. Efficiency
Requirements of Retrenchment) c. Seniority
1. The expected losses should be substantial and not d. Physical fitness
merely de minimis in extent
e. Age
2. That substantial loss apprehended must be
reasonably imminent, as such imminence can be f. financial hardship for certain workers
perceived objectively and in good faith by the
employer 4. That the employer served written notice both to the
employee and the Department of Labor and
3. It must be reasonably necessary and likely to Employment at least one month prior to the intended
effectively prevent the expected loss. It must be date of retrenchment
resorted to as a means of last resort, after less drastic

MNI NOTES |213


5. That the employer pays the retrenched employees (d) petitioner BEMI would reimburse
separation pay ADMU the costs of electrical, water,
telephone, and other utility charges,
→Must be proved by clear and convincing evidence
(e) the agreement would be valid for a period
Blue Eagle Management v. Naval, GR 192488, of three years
April 19, 2016
Petitioner BEMI was able to conduct its businesses
Doctrine: For the resignation of an employee to be at MLSC from January 2, 2005 to September 30, 2006
a viable defense in an action for illegal dismissal, an under a draft MOA, which was basically the same as
employer must prove that the resignation was the final MOA.
voluntary, and its evidence thereon must be clear,
positive, and convincing. When petitioner BEMI took over the operations of
MLSC on January 2, 2005, it also agreed to absorb
The employee’s filing of the complaint for illegal all the employees of the previous operator.
dismissal by itself is not sufficient to disprove that
said employee voluntarily resigned. Respondent Naval was hired on January 15, 2005 by
petitioner BEMI as a member of its maintenance staff.
FACTS:
During the first year of its operation, petitioner
Petitioner BEMI is a domestic corporation, with the BEMI suffered financial losses, and in attempt to
primary purpose of establishing, owning, operating, reduce its financial losses, the Management of
or managing a sports complex, and performing any petitioner BEMI (Management) resolved to
and all acts necessary and incidental to carrying out decrease the operational expenses of the company.
the same.
one of the measures the Management intended to
ADMU and petitioner BEMI entered into a MOA. implement was the downsizing of its workforce.

Under the MOA, ADMU and petitioner BEMI • The management evaluated and
agreed, among other terms and conditions, that identified several employees who could
be the subject of retrenchment
(a) petitioner BEMI would operate the proceedings, taking into consideration
businesses on its own account and employ its the employees’ positions and tenures
own employees, secure the necessary • identified five employees for
business licenses and permits under its name, retrenchment, namely, Arvin A. Aluad,
and pay all taxes related to its operations Alghie B. Domdom, Randell S. Esureña,
under its name; Edmund T. Tugay, and respondent
Naval.
(b) profits or losses from operations would be • Respondent was included in the list
for the account of petitioner BEMI; because she was one of the employees
with the shortest tenure
(c) petitioner BEMI would be responsible
• Petitioners head of management,
for the costs of maintaining MLSC in the
separately met with each of the five
same condition as it was when turned over by
aforementioned employees and
ADMU excluding ordinary wear and tear;

MNI NOTES |214


presented to them the option of business losses that would have justified
resigning instead. retrenchment of its employees.
• employees who would choose to resign • no factual or legal basis for giving respondent
would no longer be required to report for the option to resign in lieu of the alleged
work after their resignation but would retrenchment to be implemented by
still be paid their full salary and their petitioners.
prorated 13th month pay, plus financial • mere fact that respondent protested her act
assistance in the amount of one-month of signing a resignation letter by immediately
salary for every year of service at filing a complaint for illegal dismissal against
petitioner BEMI. petitioners negated the allegation that
respondent voluntarily resigned.
respondent returned to petitioner Dela Rama’s office
and informed petitioner Dela Rama that she would NLRC reversed the LA’s decision, and ruled in favor
voluntarily resign. respondent then executed a of the petitioners.
resignation letter in her own handwriting.
Respondent’s resignation letter was forwarded to and CA ruled in favor of the respondent.
approved by petitioner Bonoan on the same day.
• whether it be by redundancy or retrenchment
Since all the five employees identified for or any of the other authorized causes, no
retrenchment decided to voluntarily resign instead employee may be dismissed without
and avail themselves of the financial package offered observance of the fundamentals of good faith.
by petitioner BEMI, there was no more need for the • it is still incumbent upon the [petitioners] to
company to initiate retrenchment proceedings. prove that the employee voluntarily resigned.
• Well-entrenched is the rule that resignation
the resigned employees, except for respondent, is inconsistent with the filing of a
appeared at the premises of petitioner BEMI, complaint for illegal dismissal. To be valid,
completed their exit procedures, received the amounts the resignation must be unconditional, with
due them, and executed release waivers and the intent to operate as such; there must be a
quitclaims in favor of petitioner BEMI. clear intention to relinquish the position.
Because respondent was finding it difficult to find ISSUE:
new employment, she asked if it was possible for her
to return to work for petitioner BEMI, but she was Whether the respondent was illegally dismissed;
refused due to the financial situation of the company.
whether or not respondent’s resignation was
respondent filed a complaint for illegal dismissal voluntary.
against petitioners before the NLRC.
RULING:
The Labor Arbiter, rendered a decision in favor of
the respondent, that she was illegally dismissed. Resignation is the voluntary act of an employee who
is in a situation where one believes that personal
• petitioners were not able to prove that reasons cannot be sacrificed for the favor of
petitioner BEMI was suffering from serious employment, and opts to leave rather than stay
employed.

MNI NOTES |215


formal pronouncement or relinquishment of (4) that the employer exercises its
an office, with the intention of relinquishing prerogative to retrench employees in good
the office accompanied by the act of faith for the advancement of its interest and
relinquishment. not to defeat or circumvent the employees’
right to security of tenure; and
For the resignation of an employee to be a viable
defense in an action for illegal dismissal, an employer (5) that the employer used fair and
must prove that the resignation was voluntary, and reasonable criteria in ascertaining who
its evidence thereon must be clear, positive, and would be dismissed and who would be
convincing. retained among the employees, such as status,
efficiency, seniority, physical fitness, age,
In this case, petitioners, as employers, were able to and financial hardship for certain workers.
present sufficient evidence to establish that
respondent’s resignation was voluntary. Proof of financial losses becomes the determining
factor in proving the legitimacy of retrenchment.
Under Article 283 of the Labor Code,
retrenchment is one of the authorized causes for The foregoing circumstances persuade the Court that
termination of employment which the law accords no fraud or deception was employed upon
an employer who is not making good in its operations respondent to resign because petitioner BEMI was
in order to cut back on expenses for salaries and indeed about to implement in good faith a
wages by laying off some employees. retrenchment of its employees in order to advance its
interest and not merely to defeat or circumvent the
The requirements for valid retrenchment which respondent’s right to security of tenure.
must be proved by clear and convincing evidence are:
employee’s filing of the complaint for illegal
(1) reasonably necessary and likely to prevent dismissal by itself is not sufficient to disprove that
business losses which, if already incurred, are said employee voluntarily resigned.
not merely de minimis, but substantial,
serious, actual and real, or if only expected, FASAP v. PAL GR 178083, March 13, 2018
are reasonably imminent as perceived
objectively and in good faith by the Doctrine: In determining the validity of a
employer; retrenchment, judicial notice may be taken of the
financial losses incurred by an employer undergoing
(2) that the employer served written notice corporate rehabilitation.
both to the employees and to the
Department of Labor and Employment at FACTS:
least one month prior to the intended date of
retrenchment; PAL retrenched cabin crew personnel in a
retrenchment and demotion scheme.
(3) that the employer pays the retrenched
employees separation pay equivalent to The Labor Arbiter’s found the retrenchment illegal.
one-month pay or at least 1/2-month pay for However, the NLRC set aside the LA’s findings of
every year of service, whichever is higher; illegal retrenchment.

The CA affirmed the decision of the NLRC.

MNI NOTES |216


FASAP appealed to the Supreme Court. Resolving The Court en Banc issued a resolution assuming
the appeal of FASAP, the 3rd Division of the SC jurisdiction over the petition.
promulgated its decision on July 22, 2008 reversing
the decision promulgated on August 23, 2006 by the FASAP contends that PAL failed to prove that it
CA and entering a new one finding PAL guilty of had complied with the requirements for a valid
unlawful retrenchment. retrenchment by not submitting its audited financial
statements; and that PAL had immediately
The 3rd division of the SC, disbelieved the veracity of terminated the employees without prior resort to less
PAL’s claim of severe financial losses, and drastic measures; and that PAL did not observe any
concluded that PAL had not established its severe criteria in selecting the employees to be retrenched.
financial losses because of its non-presentation of
audited financial statements. ISSUE:

It further concluded that PAL had implemented the Whether the retrenchment was valid, and whether the
retrenchment program in bad faith, and had not used submission of audited financial statements is still
fair and reasonable criteria in selecting the employees required when complainants admitted the loss
to be retrenched.
RULING:
After Pal filed its MR, the Court upon motion held
oral arguments. Upon conclusion of the oral Retrenchment or downsizing is a mode of
arguments, the Court directed the parties to explore a terminating employment initiated by the employer
possible settlement and to submit their respective through no fault of the employee and without
memoranda. The parties did not reach any settlement, prejudice to the latter, resorted to by management
hence the SC through the Special Third Division, during periods of business recession, industrial
resolved the issues on the merits through the depression or seasonal fluctuations or during lulls
resolution of October 2, 2009 denying PAL’s MR. over shortage of materials. It is a reduction in
manpower, a measure utilized by an employer to
The SC Special 3rd Division was unconvinced by minimize business losses incurred in the operation of
PAL’s change of theory in urging the ALPAP pilot’s its business.
strike as the reason behind the immediate
retrenchment of; and observed that the strike was a Accordingly, the employer may resort to
temporary occurrence that did not require the retrenchment in order to avert serious business losses.
immediate and sweeping retrenchment of around To justify such retrenchment, the following
1,400 cabin crew. conditions must be present, namely:

Upon the SC’s subsequent reorganization, the petition 1. The retrenchment must be reasonably
was transferred to the 1st Division. Justice Velasco, necessary and likely to prevent business
inhibited himself from the case due to personal losses;
reasons.
2. The losses, if already incurred, are not
On September 7, 2011, the 2nd Division denied with merely de minimis, but substantial, serious,
finality PAL’s second MR. actual and real, or, if only expected, are
reasonably imminent;

MNI NOTES |217


3. The expected or actual losses must be inferred from PAL's conduct before, during and after
proved by sufficient and convincing the implementation of the retrenchment plan.
evidence;

4. The retrenchment must be in good faith for


the advancement of its interest and not to Closure of Business
defeat or circumvent the employees' right to
security of tenure; and →Permanent closure

5. There must be fair and reasonable criteria →Temporary closure – legal effect is governed by
in ascertaining who would be dismissed and Art. 286 of the Labor Code
who would be retained among the employees,
such as status, efficiency, seniority, physical →The right to close the entire establishment carries
fitness, age, and financial hardship for certain with it the right to close a part thereof, hence,
workers. closure may be TOTAL or PARTIAL

Upon a critical review of the records, we are →Can be exercised even if the employer is not
convinced that PAL had met all the standards in suffering from serious business losses or financial
effecting a valid retrenchment. reverses

We emphasize, too, that the presentation of the →Must be done in good faith or with no intent to
audited financial statements should not the sole lockout its employees as a means to coercing them to
means by which to establish the employer's serious its demands
financial losses. The presentation of audited financial
REQUISITES OF VALID CLOSURE
statements, although convenient in proving the
unilateral claim of financial losses, is not required for
1. The closure of business must be bona fide in
all cases of retrenchment. The evidence required for
character
each case of retrenchment really depends on the
particular circumstances obtaining. 2. A written notice must be served upon the
employees and the DOLE at least one month before
At any rate, even assuming that serious business
the intended date of closure
losses had not been proved by PAL, it would still be
justified under Article 298 of the Labor Code to 3. The employer must give separation pay to the
retrench employees to prevent the occurrence of employees, if the closure was not due to serious
losses or its closing of the business, provided that the business losses
projected losses were not merely de minimis, but
substantial, serious, actual, and real, or, if only →Relocation of plant may amount to closure
expected, were reasonably imminent as perceived
objectively and in good faith by the employer. PROCEDURAL REQUIREMENT

PAL could not have been motivated by ill will or bad →The employer should serve a written notice at least
faith when it decided to terminate FASAP's affected once (1) month in advance to the:
members. On the contrary, good faith could be justly
1. Affected employees

MNI NOTES |218


2. Department of Labor and Employment  pay for every year of service, computed on their basic
Must be served personally upon the employee monthly pay, in addition to the commutation to cash
concerned  The mere posting of the notice of their unused vacation and sick “(a) (b) (c) leaves.
of termination of employment on the
employees’ bulletin board does not a complaint was filed with respondent Labor
substantially comply with the statutory Arbiter by respondent Wilfredo Guillema and 271
requirement other separated employees for:

→“One month” = thirty (30) days (1) additional separation pay of 17.5 days for
every year of service;
→FAILURE TO COMPLY – Subject the employer
to sanction in the nature of indemnification or penalty, (2) back wages equivalent to two days a
the amount of which will depend on the facts of each month;
case and the gravity of the omission committed by the
(3) transportation allowance;
employer
(4) hazard pay;
North Davao Mining v. NLRC, 254 SCRA 721
(5) housing allowance;
Doctrine: Art. 283 of the Labor Code does not
obligate an employer to pay separation benefits when (6) food allowance;
the closure is due to losses.
(7) post-employment medical clearance; and
The law in protecting the rights of the laborer,
authorizes neither oppression nor self-destruction of (8) future medical allowance
the employer.
The Labor Arbiter, rendered a decision ordering
FACTS: petitioner North Davao to pay the complainants.
NLRC, affirmed the decision of the LA.
Petitioner North Davao Mining Corporation
(North Davao) was incorporated as a 100% privately- ISSUE:
owned company.
Whether or not an employer whose business
Respondent Wilfredo Guillema is one among operations ceased due to serious business losses or
several employees of North Davao who were financial reverses is obliged to pay separation pay to
separated by reason of the company’s closure on May its employees separated by reason of such closure.
31, 1992, and who were the complainants in the cases
before the respondent labor arbiter. RULING:

On May 31, 1992, petitioner North Davao Art. 283 of the Labor Code,
completely ceased operations due to serious business
reverses. “Art. 283. Closure of establishment and
reduction of personnel.— The employer may
When it ceased operations, its remaining employees also terminate the employment of any
were separated and given the equivalent of 12.5 days’ employee due to the installation of labor
saving devices, redundancy, retrenchment to

MNI NOTES |219


prevent losses or the closing or cessation of benefits were granted when the company finally met
operation of the establishment or undertaking its business death cannot be characterized as
unless the closing is for the purpose of discrimination.
circumventing the provisions of this Title, by
serving a written notice on the workers and Art. 283 of the Labor Code does not obligate an
the Ministry of Labor and Employment at employer to pay separation benefits when the closure
least one (1) month before the intended date is due to losses.
thereof. In case of termination due to the
installation of labor saving devices or
redundancy, the worker affected thereby shall
Benson Industries Employees Union v. Benson
be entitled to a separation pay equivalent to
at least his one (1) month pay or to at least Employees, GR 200746, August 6, 2014
one (1) month pay for every year of service,
Doctrine: Closure of business may be considered as
whichever is higher. In case of retrenchment
a reversal of an employer’s fortune whereby there is
to prevent losses and in cases of closures or
a complete cessation of business operations and/or an
cessation of operations of establishment or
actual locking-up of the doors of the establishment,
undertaking not due to serious business
usually due to financial losses.
losses or financial reverses, the separation
pay shall be equivalent to one (1) month pay
It is only in instances of retrenchment to prevent
or at least one-half (1/2) month pay for every
losses and in cases of closures or cessation of
year of service, whichever is higher. A
operations of establishment or undertaking not due to
fraction of at least six (6) months shall be
serious business losses or financial reverses that
considered one (1) whole year.”
employees whose employment has been terminated as
a result are entitled to separation pay.
The underscored portion of Art. 283 governs the
grant of separation benefits “in case of closures or
When the obligation to pay separation benefits,
cessation of operation” of business establishments
however, is not sourced from law (particularly,
“NOT due to serious business losses or financial
Article 297 of the Labor Code), but from contract,
reverses.
such as an existing collective bargaining agreement
(CBA) between the employer and its employees, an
Where, however, the closure was due to business
examination of the latter’s provisions becomes
losses—as in the instant case, —the Labor Code
necessary in order to determine the governing
does not impose any obligation upon the employer to
parameters for the said obligation.
pay separation benefits, for obvious reasons.
FACTS:
In the instant case however, the company’s practice
of giving one month’s pay for every year of service
Respondent Benson Industries, Inc. (Benson) is a
could no longer be continued precisely because the
domestic corporation engaged in the manufacturing
company could not afford it anymore.
of green coils with the brand name Lion-Tiger
Mosquito Killer.
when a business enterprise completely ceases
operations, i.e., upon its death as a going business
Benson sent its employees, including herein
concern, its vital lifeblood— its cashflow—literally
petitioners, a notice informing them of their intended
dries up. Therefore, the fact that less separation

MNI NOTES |220


termination from employment, on the ground of Under the Labor Code, it is treated as an
closure and/or cessation of business operations. authorized cause for termination, aimed at
preventing further financial drain upon an employer
In consequence, the majority of Benson’s employees who cannot anymore pay its employees since
resigned. business has already stopped.

petitioners, through Benson Industries Employees While serious business losses generally exempt the
Union-ALU-TUCP (Union), filed a notice of strike, employer from paying separation benefits, it must be
claiming that the company’s supposed closure was pointed that the exemption only pertains to the
merely a ploy to replace the union members with obligation of the employer under Article 297 of the
lower paid workers, and, as a result, increase its profit Labor Code.
at their expense.
This is because of the law’ s express parameter that
due to the parties’ amicable settlement during the mandates payment of separation benefits “in case of
conciliation proceedings before the NCMB, closures or cessation of operations of establishment or
petitioners accepted Benson’s payment of separation undertaking not due to serious business losses or
pay, computed at 15 days for every year of service, as financial reverses.”
per the parties’ Memorandum of Agreement
Article 283 [(now, Article 297)] of the Labor Code
Reaching an impasse on the conflict, the parties does not obligate an employer to pay separation
referred the issue to voluntary arbitration, wherein the benefits when the closure is due to serious losses. To
validity of Benson’s closure was brought up as well. require an employer to be generous when it is no
longer in a position to do so, in our view, would be
The VA ruled in favor of the petitioners. unduly oppressive, unjust, and unfair to the employer.
the CA reversed and set aside the VA’s ruling, and When the obligation to pay separation benefits,
accordingly deleted the award of additional however, is not sourced from law (particularly,
separation benefits equivalent to four (4) days of work Article 297 of the Labor Code), but from contract,
for every year of service. such as an existing collective bar gaining agreement
between the employer and its employees, an
ISSUE:
examination of the latter’s provisions becomes
whether or not the CA correctly deleted the award to necessary in order to determine the governing
petitioners of additional separation benefits parameters for the said obligation.
equivalent to four (4) days of work for every year of
For a similar exemption to obtain against a contract,
service.
such as a CBA, the tenor of the parties’ agreement
RULING: ought to be similar to the law’s tenor.

Closure of business may be considered as a reversal When the parties, however, agree to deviate
of an employer’s fortune whereby there is a complete therefrom, and unqualifiedly covenant the payment of
cessation of business operations and/or an actual separation benefits irrespective of the employer’s
lockingup of the doors of the establishment, usually financial position, then the obligatory force of that
due to financial losses. contract prevails and its terms should be carried out
to its full effect.

MNI NOTES |221


Hence, if the terms of a CBA are clear and there is no
doubt as to the intention of the contracting parties, the
literal meaning of its stipulations shall prevail.

It is equally undisputed that Benson agreed to and was


thus obligated under the CBA to pay its employees
who had been terminated without any fault
attributable to them separation benefits at the rate of
19 days for every year of service.

Thus, in view of the foregoing, the Court disagrees


with the CA in negating Benson’s obligation to pay
petitioners their full separation benefits under the said
agreement.

MNI NOTES |222


B. Procedural Requirements for Terminating MECO Manning & Crewing Services v.
an Employee Cuyos GR 222939, July 3, 2019

DOLE Memorandum Order No. 147-2015 Doctrine: In termination cases, the burden of proof
rests upon the employer to show that the dismissal is
Twin Notice Requirement for a just and valid cause; For this purpose, the
employer must present substantial evidence to prove
PROCEDURAL REQUIREMENT the legality of an employee’s dismissal.

→The employer should serve a written notice at least In termination proceedings, it is settled that for the
once (1) month in advance to the: manner of dismissal to be valid, the employer must
comply with the employee’s right to procedural due
1. Affected employees
process by furnishing him with two (2) written notices
before the termination of his employment.
2. Department of Labor and Employment 
Must be served personally upon the employee
FACTS:
concerned  The mere posting of the notice
of termination of employment on the Cuyos filed a complaint for illegal dismissal and
employees’ bulletin board does not claims for salaries and other benefits for the
substantially comply with the statutory unexpired portion of his employment contract,
requirement damages, and attorney’s fees against International
Crew Services, Ltd. (ICS), and petitioners Meco
→“One month” = thirty (30) days
Manning & Crewing Services, Inc. (MECO) and
Captain Igmedio G. Sorrera (Capt. Sorrera) before the
→FAILURE TO COMPLY – Subject the employer
Regional Arbitration Branch of the NLRC in
to sanction in the nature of indemnification or penalty,
Cebu City.
the amount of which will depend on the facts of each
case and the gravity of the omission committed by the
Constantino alleged that MECO, for and on behalf
employer
of its principal, ICS, hired him as the Second Marine
Engineer of the vessel “M/V Crown Princess.” The
PURPOSE OF NOTICE
employment was for a period of eight months.
→To obviate abrupt and arbitrary dismissal and to
Constantino claimed that the ship’s Chief Engineer,
enable the employee to survive while he is looking for
Francisco G. Vera, Jr. (Vera), mistreated him during
another job
his short stay onboard the “M/V Crown Princess.”
→EMPLOYEE – to give him some to prepare for the
Constantino was shocked when the Third Mate of the
eventual loss of his job
vessel handed to him an electronic plane ticket and
→DOLE – opportunity to ascertain the veracity of informed him that he must disembark at Cristobal,
the alleged cause for termination Panama, where a reliever would take his place.

Capt. Sorrera informed him that he was dismissed


because he challenged Vera to a fight. Constantino

MNI NOTES |223


denied the allegation and claimed that it was Vera the Court concurs with the appellate court’s
who was very rude to him. conclusion that the petitioners failed to establish the
validity of Constantino’s dismissal by substantial
However, they claimed that Constantino’s evidence
dismissal was valid. Petitioners claimed that
Constantino’s dismissal was necessitated by as aptly observed by the appellate court, the non-
reason of his unsatisfactory performance presentation of the ship’s logbook or copies of the
evaluation, violation of his contract of employment pertinent pages thereof raises doubts as to the
as he violated the provisions on insubordination and occurrence of Constantino’s alleged infractions.
inefficiency, his angry and provocative utterances and
his attempt to physically assault his superior. In fine, the pieces of evidence presented by the
petitioners to establish the validity of the dismissal
Labor Arbiter dismissed the complaint for lack of are either unreliable or plainly insufficient to
merit. It ratiocinated that the pieces of evidence prove that Constantino is guilty of insubordination
presented by the petitioners clearly showed that and serious misconduct. Thus, the appellate court
Constantino defied the lawful orders of his superior correctly reversed the NLRC’s and Labor Arbiter’s
officer. decisions considering that they were not duly
supported by substantial evidence.
NLRC affirmed Labor Arbiter’s Decision.
In termination proceedings, it is settled that for the
CA reversed and set aside, ruled that the petitioners manner of dismissal to be valid, the employer must
failed to present substantial evidence to prove that comply with the employee’s right to procedural due
Constantino’s dismissal was made for a valid and process by furnishing him with two written notices
justifiable cause. before the termination of his employment.

ISSUE: Constantino was illegally dismissed from his


employment, thus he is entitled to his salaries
WHETHER THE COURT OF APPEALS ERRED corresponding to the unexpired portion of his contract
WHEN IT RULED THAT CONSTANTINO R. which is six months.
CUYOS WAS ILLEGALLY DISMISSED FROM
EMPLOYMENT. J’Marketing Corp. v. Iguiz, GR 211522,
September 4, 2019
RULING:

The petition lacks merit.


Doctrine: For substantive due process, the
dismissal must be for a just and authorized cause as
It is settled that in termination cases, the burden of provided under Articles 282, 283, and 284 of the
proof rests upon the employer to show that the Labor Code; and for procedural due process, the
dismissal is for a just and valid cause. opportunity to be heard and to defend oneself must be
observed.
Failure to do so would necessarily mean that the
dismissal was illegal. The law and the rules provide that the employer must
furnish the employee with two (2) written notices
the employer must present substantial evidence to before dismissal from employment:
prove the legality of an employee’s dismissal.

MNI NOTES |224


(1) notice to apprise the employee of the On 12 March 2007, Iguiz received the
particular acts or omissions for which the memorandum of termination. Aggrieved, Iguiz
dismissal is sought, and filed a Complaint for illegal dismissal with money
claims with the National Labor Relations
(2) subsequent notice to inform him of the Commission (NLRC) Sub-Regional Arbitration
employer’s decision to dismiss him. Branch No. VI in Kalibo, Aklan.

FACTS: LA dismissed the complaint. NLRC reversed the


situation.
Respondent Fernando S. Iguiz (Iguiz) was hired as
a driver by petitioner J’ Marketing Corporation ISSUE:
(JMC).
Whether or not the appellate court committed
JMC is a company engaged in the business of selling reversible error in upholding the finding of the NLRC
appliances to the general public and has several that Iguiz was illegally dismissed from his
branches in the Visayas region. employment and is entitled to backwages, separation
pay, damages and attorney’s fees.
Iguiz submitted a Daily Cash Collection Report, and
remitted his collections for the week, JMC found that RULING:
Iguiz was short in his remittance collection in the
amount of P5,811.00. The petition lacks merit.

petitioner Pepito P. Estrellan, JMC’s Kalibo Branch Under the Labor Code, the dismissal of an employee
Manager, directed Iguiz to explain within 24 hours has a two-fold due process requirement: one is
the reason for the P5,811.00 shortage and suspended substantive and the other, procedural. For
Iguiz from his position as collector/credit substantive due process, the dismissal must be for a
investigator. just and authorized cause as provided under Articles
282, 283, and 284 of the Labor Code; and for
Iguiz said that his shortage of remittance was because procedural due process, the opportunity to be heard
the money was lost due to the typhoon which affected and to defend oneself must be observed.
his home.
An employer may terminate the services of an
Estrellan issued a memorandum to Iguiz asking him employee for just causes under Article 282 of the
to explain within 24 hours why he should not be Labor Code which provides:
reprimanded for loss of trust and confidence for
receiving payments of P15,300.00 and $29 without Art. 282. Termination by employer.—An
issuing official receipts, as per Sonio’s audit report. employer may terminate an employment for
any of the following causes:
before Iguiz could file an explanation for the
memorandum, he received another memorandum a. Serious misconduct or willful disobedience
from Estrellan, asking him to sign the administrative by the employee of the lawful orders of his
investigation report within 12 hours; otherwise it employer or representative in connection
would mean that Iguiz is waiving his right to be heard with his work;
and JMC would be constrained to evaluate his case
based on the evidence on hand.

MNI NOTES |225


b. Gross and habitual neglect by the (1) notice to apprise the employee of the
employee of his duties; particular acts or omissions for which the
dismissal is sought, and
c. Fraud or willful breach by the employee of
the trust reposed in him by his employer or (2) subsequent notice to inform him of the
duly authorized representative; employer’s decision to dismiss him.

d. Commission of a crime or offense by the to the notices, the employer must set a hearing or
employee against the person of his employer conference to give the employee an opportunity to
or any immediate member of his family or his present evidence and rebut the charges against
duly authorized representatives; and him. The requirement of two notices and a hearing is
mandatory; otherwise the order of dismissal is void.
e. Other causes analogous to the foregoing.
Under the implementing rule of Article 277, an
In the present case, JMC terminated the employment employee should be given “reasonable opportunity”
of Iguiz due to dishonesty and fraud or willful breach to file a response to the notice.
of the trust reposed in him as provided under Article
282(c). “reasonable opportunity” should be a period of at
least five calendar days from receipt of the notice
we held that the language of Article 282(c) of the
Labor Code states that the loss of trust and Accordingly, given the illegality of Iguiz’s dismissal
confidence must be based on willful breach of the without just cause and the nonobservance of
trust reposed in the employee by his employer. procedural due process, Iguiz is entitled to
Ordinary breach will not suffice; it must be willful. reinstatement and backwages as provided in Article
Such breach is willful if it is done intentionally, 279 of the Labor Code,
knowingly, and purposely, without justifiable excuse,
as distinguished from an act done carelessly, thought
lessly, heedlessly or inadvertently. Stated otherwise,
it must be based on substantial evidence. Failure to Observe Due Process

We agree with the appellate court that JMC failed to In termination proceedings, it is settled that for the
prove by substantial evidence the loss of trust and manner of dismissal to be valid, the employer must
confidence in Iguiz based on willful breach of trust. comply with the employee’s right to procedural due
process by furnishing him with two written notices
It bears stressing that in illegal dismissal cases, the before the termination of his employment.
employer bears the burden of showing that the
dismissal was for a just or authorized cause. Not only For substantive due process, the dismissal must be
must the reasons for dismissing an employee be for a just and authorized cause as provided under
substantiated, the manner of his dismissal must be in Articles 282, 283, and 284 of the Labor Code; and for
accordance with governing rules and regulations. procedural due process, the opportunity to be heard
and to defend oneself must be observed.
The law and the rules provide that the employer must
furnish the employee with two written notices before Failure by the employer to discharge this burden
dismissal from employment: would necessarily mean that the dismissal is not

MNI NOTES |226


justified, and therefore illegal. This means that the member of his family or his duly authorized
requirements of due process must be observed. representative; and
(e) other causes analogous to the foregoing.
The law and the rules provide that the employer must
furnish the employee with two (2) written notices Book VI, Rule I, Section 2(d) of the Omnibus Rules
before dismissal from employment: Implementing the Labor Code:

(1) notice to apprise the employee of the Standards of due process: requirements of notice. – In
particular acts or omissions for which the all cases of termination of employment, the following
standards of due process shall be substantially
dismissal is sought, and
observed:
(2) subsequent notice to inform him of the I. For termination of employment based on just causes
employer’s decision to dismiss him. as defined in Article 282 of the Code:

In addition to the notices, the employer must set a (a) A written notice served on the employee
hearing or conference to give the employee an specifying the ground or grounds for termination, and
opportunity to present evidence and rebut the charges giving to said employee reasonable opportunity
within which to explain his side;
against him. The requirement of two notices and a
hearing is mandatory; otherwise the order of (b) A hearing or conference during which the
dismissal is void. employee concerned, with the assistance of counsel if
the employee so desires, is given opportunity to
Failure by the employer to discharge this burden respond to the charge, present his evidence or rebut
would necessarily mean that the dismissal is not the evidence presented against him; and
justified, and therefore illegal
(c) A written notice of termination served on the
employee indicating that upon due consideration of
Agabon v. NLRC, GR 158693, November 17,
all the circumstances, grounds have been established
2004 to justify his termination.
Doctrine: Article 282 of the Labor Code
enumerates the just causes for termination by the In case of termination, the foregoing notices shall be
employer: served on the employee's last known address.

(a) serious misconduct or willful disobedience by the ART. 294. [279] Security of Tenure. - In cases of
employee of the lawful orders of his employer or the regular employment, the employer shall not terminate
latter's representative in connection with the the services of an employee except for a just cause or
employee's work; when authorized by this Title. An employee who is
unjustly dismissed from work shall be entitled to
(b) gross and habitual neglect by the employee of his reinstatement without loss of seniority rights and
duties; other privileges and to his full backwages, inclusive
of allowances, and to his other benefits or their
(c) fraud or willful breach by the employee of the trust
monetary equivalent computed from the time his
reposed in him by his employer or his duly authorized compensation was withheld from him up to the time
representative; of his actual reinstatement.
(d) commission of a crime or offense by the employee ART. 297. [282] Termination by Employer - An
against the person of his employer or any immediate employer may terminate an employment for any of
the following causes:

MNI NOTES |227


(a) Serious misconduct or willful disobedience by the suffering from any disease and whose continued
employee of the lawful orders of his employer or employment is prohibited by law or is prejudicial to
representative in connection with his work; his health as well as to the health of his co-employees:
Provided, That he is paid separation pay equivalent to
(b) Gross and habitual neglect by the employee of his
at least one (1) month salary or to one-half (1/2)
duties;
month salary for every year of service, whichever is
(c) Fraud or willful breach by the employee of the greater, a fraction of at least six (6) months being
trust reposed in him by his employer or duly considered as one (1) whole year.
authorized representative;
Wenphil Doctrine - where the employer had a valid
(d) Commission of a crime or offense by the reason to dismiss an employee but did not follow the
employee against the person of his employer or any due process requirement, the dismissal may be upheld
immediate member of his family or his duly but the employer will be penalized to pay an
authorized representatives; and indemnity to the employee. (Wenphil or Belated Due
Process Rule)
(e) Other causes analogous to the foregoing.
Serrano Ruling - The sanction should be in the
ART. 298. [283] Closure of Establishment and nature of indemnification or penalty and should
Reduction of Personnel. - The employer may also depend on the facts of each case, taking into special
terminate the employment of any employee due to the consideration the gravity of the due process violation
installation of labor-saving devices, redundancy, of the employer. The violation of petitioners’ right to
retrenchment to prevent losses or the closing or statutory due process by the private respondent
cessation of operation of the establishment or warrants the payment of indemnity in the form of
undertaking unless the closing is for the purpose of nominal damages.
circumventing the provisions of this Title, by serving
a written notice on the workers and the Ministry of Procedural due process requirements for
Labor and Employment at least one (1) month before dismissal are found in the Implementing Rules of
the intended date thereof. P.D. 442, as amended ,otherwise known as the Labor
Code of the Philippines in Book VI, Rule I, Sec. 2, as
In case of termination due to the installation of labor- amended by Department Order Nos. 9 and 10
saving devices or redundancy, the worker affected
thereby shall be entitled to a separation pay equivalent Presidential Decree No. 851 - evident intention is to
to at least his one (1) month pay or to at least one (1) grant an additional income in the form of the 13th
month pay for every year of service, whichever is month pay to employees not already receiving the
higher. In case of retrenchment to prevent losses and same so as "to further protect the level of real wages
in cases of closures or cessation of operations of from the ravages of world-wide inflation".
establishment or undertaking not due to serious
Article 97(f) of the Labor Code
business losses or financial reverses, the separation
pay shall be equivalent to one (1) month pay or at least (f) "Wage" paid to any employee shall mean the
one-half (1/2) month pay for every year of service, remuneration or earnings, however designated,
whichever is higher. capable of being expressed in terms of money
whether fixed or ascertained on a time, task, piece , or
A fraction of at least six (6) months shall be
commission basis, or other method of calculating the
considered one (1) whole year.
same, which is payable by an employer to an
ART. 299. [284] Disease as Ground for employee under a written or unwritten contract of
Termination. - An employer may terminate the employment for work done or to be done, or for
services of an employee who has been found to be services rendered or to be rendered and includes the

MNI NOTES |228


fair and reasonable value, as determined by the Private respondents are ordered to pay petitioners
Secretary of Labor, of board, lodging, or other holiday pay for four (4) regular holidays in 1996,
facilities customarily furnished by the employer to the 1997, and 1998, as well as their service incentive
employee…" leave pay for said years, and to pay the balance of
petitioner Virgilio Agabon's 13th month pay for 1998
FACTS: in the amount of P2,150.00.
Private respondent Riviera Home Improvements, Inc.
is engaged in the business of selling and installing Petitioners:
ornamental and construction materials. It employed - assert that they were dismissed because the private
petitioners Virgilio Agabon and Jenny Agabon as respondent refused to give them assignments unless
gypsum board and cornic installers on January 2, they agreed to work on a "pakyaw" basis when they
1992 until February 23, 1999 when they were reported for duty on February 23, 1999. --> They did
dismissed for abandonment of work. not agree on this arrangement because it would mean
losing benefits as Social Security System (SSS)
Agabons filed a complaint for illegal dismissal and members.
payment of money claims. - also claim that private respondent did not comply
with the twin requirements of notice and hearing.
Labor Arbiter rendered a decision declaring the
dismissals illegal and ordered private respondent: Respondents:
1. to pay the monetary claims of P56, 231.93 each - maintained that petitioners were not dismissed but
and, had abandoned their work.
2. in lieu of reinstatement to pay them their separation --> sent two letters to the last known addresses of the
pay of one (1) month for every year of service from petitioners advising them to report for work and the
date of hiring up to November 29, 1999. manager even talked to petitioner Virgilio Agabon by
3. to pay their holiday pay and service incentive leave telephone sometime in June 1999 to tell him about the
pay for the years 1996, 1997 and 1998 as well as their new assignment at Pacific Plaza Towers involving
premium pay for holidays and rest days and Virgilio 40,000 square meters of cornice installation work.
Agabon's 13th month pay differential amounting to - petitioners did not report for work because they had
P2,150 or the aggregate amount of P121,678.93 - subcontracted to perform installation work for
Jenny Agabon and P123,828.93 for Virgilio Agabon another company

NLRC reversed the Labor Arbiter: - demanded for an increase in their wage to P280.00
it found that the petitioners had abandoned their work, per day. When this was not granted, petitioners
and were not entitled to backwages and separation stopped reporting for work and filed the illegal
pay. The other money claims awarded by the Labor dismissal case.
Arbiter were also denied for lack of evidence
ISSUE:
Petitioners filed a petition for certiorari with the Court Whether or not the Agabons were illegally dismissed
of Appeals. from their employment. - NO

CA ruled that the dismissal of the petitioners was not Factual issue: Whether or not an employee has
illegal because they had abandoned their employment abandoned employment. – YES
but ordered the payment of money claims.
RULING:

MNI NOTES |229


SC Ruling on just cause: Terminations were for a
just and valid cause Dismissals based on just causes contemplate acts or
omissions attributable to the employee while
To dismiss an employee, the law requires not only the dismissals based on authorized causes involve
existence of a just and valid cause but also enjoins the grounds under the Labor Code which allow the
employer to give the employee the opportunity to be employer to terminate employees.
heard and to defend himself.
A termination for an authorized cause requires
Article 282 of the Labor Code enumerates the just payment of separation pay. When the termination of
causes for termination by the employer. employment is declared illegal, reinstatement and full
Abandonment is the deliberate and unjustified refusal backwages are mandated under Article 279. If
of an employee to resume his employment. It is a form reinstatement is no longer possible where the
of neglect of duty, hence, a just cause for termination dismissal was unjust, separation pay may be granted.
of employment by the employer.
Procedurally:
Factors in validly finding of abandonment: (1) if the dismissal is based on a just cause under
(1) the failure to report for work or absence without Article 282, the employer must give the employee two
valid or justifiable reason; and written notices and a hearing or opportunity to be
(2) a clear intention to sever employer-employee heard if requested by the employee before terminating
relationship, with the second as the more the employment: a notice specifying the grounds for
determinative factor which is manifested by overt acts which dismissal is sought a hearing or an opportunity
from which it may be deduced that the employees has to be heard and after hearing or opportunity to be
no more intention to work. heard, a notice of the decision to dismiss; and
(2) if the dismissal is based on authorized causes
Subcontracting for another company clearly showed under Articles 283 and 284, the employer must give
the intention to sever the employer-employee the employee and the Department of Labor and
relationship with private respondent. Employment written notices 30 days prior to the
Despite the warnings from the employer, the effectivity of his separation.
employees disregarded the warning.
Four possible situations may be derived:
An employee who deliberately absented from work
without leave or permission from his employer, for
(1) the dismissal is for a just cause under Article 282
the purpose of looking for a job elsewhere, is
of the Labor Code, for an authorized cause under
considered to have abandoned his job. (Sandoval
Article 283, or for health reasons under Article 284,
Shipyard v. Clave)
and due process was observed; (the dismissal is
undoubtedly valid and the employer will not suffer
SC Ruling on Due Process: if the procedures for
any liability.)
dismissal were observed - No
(2) the dismissal is without just or authorized cause
The procedure for terminating an employee is found
but due process was observed; and
in Book VI, Rule I, Section 2(d) of the Omnibus
Rules
(3) the dismissal is without just or authorized cause
Implementing the Labor Code.
and there was no due process; (where the dismissals

MNI NOTES |230


are illegal, Article 279 mandates that the employee is Statutory due process should be differentiated from
entitled to reinstatement without loss of seniority failure to comply with constitutional due process:
rights and other privileges and full backwages, Constitutional due process protects the individual
inclusive of allowances, and other benefits or their from the government and assures him of his rights in
monetary equivalent computed from the time the criminal, civil or administrative proceedings;
compensation was not paid up to the time of actual statutory due process found in the Labor Code and
reinstatement.) Implementing Rules protects employees from being
unjustly terminated without just cause after notice and
(4) the dismissal is for just or authorized cause but due hearing.
process was not observed. (the dismissal should be
upheld) The unfairness of declaring illegal or ineffectual
dismissals for valid or authorized causes but not
Here, the present case falls under the fourth situation. complying with statutory due process may have far-
The dismissal should be upheld because it was reaching consequences. --> Invalidating the dismissal
established that the petitioners abandoned their jobs would not serve public interest. It could also
to work for another company. discourage investments that can generate
employment in the local economy.
While the procedural infirmity cannot be cured, it
should not invalidate the dismissal. However, the The constitutional policy to provide full protection to
employer should be held liable for non-compliance labor is not meant to be a sword to oppress employers
with the procedural requirements of due process. --> an employer should not be compelled to pay
- did not follow the notice requirements and instead employees for work not actually performed and in
argued that sending notices to the last known fact abandoned.
addresses would have been useless because they did
not reside there anymore. --> this is not a valid excuse The employer should not be compelled to continue
because the law mandates the twin notice employing a person who is admittedly guilty of
requirements to the employee's last known address. misfeasance or malfeasance and whose continued
employment is patently inimical to the employer --
Due process under the Labor Code, like >The law protecting the rights of the laborer
Constitutional due process, has two aspects: authorizes neither oppression nor self-destruction of
1. substantive, i.e., the valid and authorized causes of the employer.
employment termination under the Labor Code; and
It must be stressed that in the present case, the
2. procedural, i.e., the manner of dismissal petitioners committed a grave offense, i.e.,
abandonment, which, if the requirements of due
Procedural due process requirements for dismissal process were complied with, would undoubtedly
are found in the Implementing Rules of P.D. 442, as result in a valid dismissal.
amended, otherwise known as the Labor Code of the
An employee who is clearly guilty of conduct
Philippines in Book VI, Rule I, Sec. 2, as amended by
violative of Article 282 should not be protected by the
Department Order Nos. 9 and 10.27 Breaches of these
Social Justice Clause of the Constitution - Social
due process requirements violate the Labor Code.
justice should be used only to correct an injustice and
must be founded on the recognition of the necessity

MNI NOTES |231


of interdependence among diverse units of a society of P3,255.00 and the balance of Virgilio Agabon's
and of the protection that should be equally and thirteenth month pay for 1998 in the amount of
evenly extended to all groups as a combined force in P2,150.00.
our social and economic life, consistent with the
fundamental and paramount objective of the state of Jaka Food Processing v. Pacot, GR 151378,
promoting the health, comfort, and quiet of all March 28, 2005
persons, and of bringing about "the greatest good to Doctrine:
the greatest number"
There is a difference between dismissal due to just
Where the dismissal is for a just cause, as in the and authorized causes. In a dismissal due to just
instant case, the lack of statutory due process should causes, the employee commits some violation against
not nullify the dismissal, or render it illegal, or the employer. In authorized causes, dismissal process
ineffectual. is initiated by the employer’s exercise of his
management prerogative i.e. when the employer opts
- The sanction should be in the nature of
to install labor saving devices, when he decides to
indemnification or penalty and should depend on the
cease business operations or when, as in this case, he
facts of each case, taking into special consideration
undertakes to implement a retrenchment program.
the gravity of the due process violation of the
employer. Because of this distinction, as a general rule,
separation pay is not needed in dismissal for just
- Under the Civil Code, nominal damages is
causes, while it is required in dismissal for authorized
adjudicated in order that a right of the plaintiff, which
causes. Additionally for failure to comply with the
has been violated or invaded by the defendant, may
notice requirement should be tempered under just
be vindicated or recognized, and not for the purpose
causes, as it is the employee who in truth is the root
of indemnifying the plaintiff for any loss suffered by
cause for the dismissal, while the penalty for failure
him.
to comply should be stiffer under authorized causes,
- The violation of the petitioners' right to statutory due as it is caused by the employer in the exercise of their
process by the private respondent warrants the management prerogative.
payment of indemnity in the form of nominal
ART. 297. [282] Termination by Employer - An
damages. The amount of such damages is addressed
employer may terminate an employment for any of
to the sound discretion of the court, taking into
the following causes:
account the relevant circumstances.
(a) Serious misconduct or willful disobedience by the
As a general rule, one who pleads payment has the
employee of the lawful orders of his employer or
burden of proving it. Even where the employee must
representative in connection with his work;
allege nonpayment, the general rule is that the burden
rests on the employer to prove payment, rather than (b) Gross and habitual neglect by the employee of his
on the employee to prove non-payment. duties;
The Court of Appeals properly reinstated the (c) Fraud or willful breach by the employee of the
monetary claims awarded by the Labor Arbiter trust reposed in him by his employer or duly
ordering the private respondent to pay each of the authorized representative;
petitioners holiday pay for four regular holidays from
1996 to 1998, in the amount of 6,520.00, service (d) Commission of a crime or offense by the
incentive leave pay for the same period in the amount employee against the person of his employer or any

MNI NOTES |232


immediate member of his family or his duly of service incentive leave and 13th month pay against
authorized representatives; and JAKA and its HRD Manager, Rosana Castelo. After
due proceedings, the Labor Arbiter rendered a
(e) Other causes analogous to the foregoing.
decision 3 declaring the termination illegal and
ART. 298. [283] Closure of Establishment and ordering JAKA and its HRD Manager to reinstate
Reduction of Personnel. - The employer may also respondents with full backwages, and separation pay
terminate the employment of any employee due to the if reinstatement is not possible.
installation of labor-saving devices, redundancy,
retrenchment to prevent losses or the closing or JAKA went on appeal to the NLRC, which affirmed
cessation of operation of the establishment or the decision of the Labor Arbiter. Their motion for
undertaking unless the closing is for the purpose of reconsideration having been denied by the NLRC,
circumventing the provisions of this Title, by serving respondents went to the Court of Appeals via a
a written notice on the workers and the Ministry of petition for certiorari. The Court of Appeals reversed
Labor and Employment at least one (1) month before said decision and ordered respondent JAKA to pay
the intended date thereof. petitioners separation pay equivalent to one (1) month
salary, the proportionate 13th month pay and, in
Serrano Ruling - The sanction should be in the nature addition, full backwages from the time their
of indemnification or penalty and should depend on employment was terminated. JAKA moved for a
the facts of each case, taking into special reconsideration but its motion was denied by the
consideration the gravity of the due process violation appellate court. Hence, this petition.
of the employer. The violation of petitioners’ right to
statutory due process by the private respondent ISSUE:
warrants the payment of indemnity in the form of
nominal damages. What are the legal implications of a situation where
an employee is dismissed for cause but such dismissal
FACTS: was effected without the employer’s compliance with
Respondents Darwin Pacot, Robert Parohinog, David the notice requirement under the Labor Code. - Must
Bisnar, Marlon Domingo, Rhoel Lescano and pay Nominal Damages.
Jonathan Cagabcab were earlier hired by petitioner
JAKA Foods Processing Corporation (JAKA, for RULING:
short) until the latter terminated their employment on In the very recent case of Agabon vs. NLRC, we had
August 29, 1997 because the corporation was "in dire the opportunity to resolve a similar question. Therein,
financial straits". It is not disputed, however, that the we found that the employees committed a grave
termination was effected without JAKA complying offense, i.e., abandonment, which is a form of a
with the requirement under Article 283 of the Labor neglect of duty which, in turn, is one of the just causes
Code regarding the service of a written notice upon enumerated under Article 282 of the Labor Code. In
the employees and the Department of Labor and said case, we upheld the validity of the dismissal
Employment at least one (1) month before the despite non-compliance with the notice requirement
intended date of termination. of the Labor Code. However, we required the
employer to pay the dismissed employees the amount
In time, respondents separately filed with the of P30,000.00, representing nominal damages for
Regional Arbitration Branch of the National Labor non-compliance with statutory due process.
Relations Commission (NLRC) complaints for illegal
dismissal, underpayment of wages and nonpayment

MNI NOTES |233


The difference between Agabon and the instant case stiffer because the dismissal, process was initiated by
is that in the former, the dismissal was based on a just the employer's exercise of his management
cause under Article 282 of the Labor Code while in prerogative.
the present case, respondents were dismissed due to
The records before us reveal that, indeed, JAKA was
retrenchment, which is one of the authorized causes
suffering from serious business losses at the time it
under Article 283 of the same Code.
terminated respondents’ employment.
A dismissal for just cause under Article 282 implies
It is, therefore, established that there was ground for
that the employee concerned has committed, or is
respondents' dismissal, i.e., retrenchment, which is
guilty of, some violation against the employer, i.e. the
one of the authorized causes enumerated under
employee has committed some serious misconduct, is
Article 283 of the Labor Code. Likewise, it is
guilty of some fraud against the employer, or, as in
established that JAKA failed to comply with the
Agabon, he has neglected his duties. Thus, it can be
notice requirement under the same Article.
said that the employee himself initiated the dismissal
Considering the factual circumstances in the instant
process.
case and the above ratiocination, we, therefore, deem
On another breath, a dismissal for an authorized cause it proper to fix the indemnity at P50,000.00.
under Article 283 does not necessarily imply
"The rule, therefore, is that in all cases of business
delinquency or culpability on the part of the
closure or cessation of operation or undertaking of the
employee. Instead, the dismissal process is initiated
employer, the affected employee is entitled to
by the employer’s exercise of his management
separation pay. This is consistent with the state policy
prerogative, i.e. when the employer opts to install
of treating labor as a primary social economic force,
labor saving devices, when he decides to cease
affording full protection to its rights as well as its
business operations or when, as in this case, he welfare. The exception is when the closure of
undertakes to implement a retrenchment program. business or cessation of operations is due to serious
business losses or financial reverses; duly proved, in
The clear-cut distinction between a dismissal for just
which case, the right of affected employees to
cause under Article 282 and a dismissal for authorized
separation pay is lost for obvious reasons.
cause under Article 283 is further reinforced by the
fact that in the first, payment of separation pay, as a
rule, is not required, while in the second, the law
requires payment of separation pay Deoferio v. Intel Technology, GR 202996,
Accordingly, it is wise to hold that: June 18, 2014
Doctrine: Art. 284. Disease as ground for
(1) if the dismissal is based on a just cause under termination. – An employer may terminate the
Article 282 but the employer failed to comply with services of an employee who has been found to be
the notice requirement, the sanction to be imposed suffering from any disease and whose continued
upon him should be tempered because the dismissal employment is prohibited by law or is prejudicial to
process was, in effect, initiated by an act imputable to his health as well as to the health of his co-employees:
the employee; and Provided, That he is paid separation pay equivalent to
(2) if the dismissal is based on an authorized cause at least one (1) month salary or to one-half (1/2)
under Article 283 but the employer failed to comply month salary for every year of service, whichever is
with the notice requirement, the sanction should be

MNI NOTES |234


greater, a fraction of at least six (6) months being On August 8, 2005, Dr. Paul Lee, a consultant
considered as one (1) whole year. psychiatrist... of the Philippine General Hospital,
concluded that Deoferio was suffering from
Article 113(c) of the Labor Code
schizophrenia. After several consultations, Dr. Lee
ART. 113. Wage Deduction. - No employer, in his issued a psychiatric report dated January 17, 2006
own behalf or in behalf of any person, shall make any concluding and stating that Deoferio's psychotic
deduction from the wages of his employees, except: symptoms are not curable within a period of six...
months and "will negatively affect his work and social
(a) In cases where the worker is insured with his relation with his co-workers."Pursuant to these
consent by the employer, and the deduction is to findings, Intel issued Deoferio a notice of termination
recompense the employer for the amount paid by him on March 10, 2006.
as premium on the insurance;
Deoferio responded to his termination of employment
(b) For union dues, in cases where the right of the by filing a complaint for illegal dismissal with prayer
worker or his union to checkoff has been recognized for money claims against respondents Intel and Mike
by the employer or authorized in writing by the Wentling (respondents). He denied that he ever had
individual worker concerned; and mental illness and insisted that he satisfactorily
performed his duties as a product engineer. He argued
(c) In cases where the employer is authorized by law
that Intel violated his statutory right to procedural due
or regulations issued by the Secretary of Labor and
process when it summarily issued a notice of
Employment.
termination.
FACTS: In defense, the respondents argued that Deoferio's
On February 1, 1996, respondent Intel Technology dismissal was based on Dr. Lee's certification that:
Philippines, Inc. (Intel) employed Deoferio as a
product quality and reliability engineer with a (1) his schizophrenia was not curable within a period
monthly salary of P9,000.00. In July 2001, Intel of six months even with proper medical treatment;
assigned him to the United States as a validation and
engineer for an agreed... period of two years and with (2) his continued employment would be prejudicial to
a monthly salary of US$3,000.00. On January 27, his and to the other employees' health.
2002, Deoferio was repatriated to the Philippines after
being confined at Providence St. Vincent Medical The respondents also insisted that Deoferio's presence
Center for major depression with psychosis.[4] In the at Intel's premises would pose an actual harm to his
Philippines, he... worked as a product engineer with a co-employees as shown by his previous acts. On May
monthly salary of P23,000.00. 8, 2003, Deoferio emailed an Intel employee with this
message: "All soul's day back to work Monday" On
Deoferio underwent a series of medical and January 18, 2005, he cut the mouse cables, stepped on
psychiatric treatment at Intel's expense after his the keyboards, and disarranged the desks of his co-
confinement in the United States. In 2002, Dr. employees.
Elizabeth Rondain of Makati Medical Center
diagnosed him to be suffering from mood disorder, The respondents also highlighted that Deoferio
major depression, and auditory... hallucination.He incurred numerous absences from work due to his
was also referred to Dr. Norieta Balderrama, Intel's mental condition, specifically, from January 31, 2002
forensic psychologist, and to a certain Dr. Cynthia until February 28, 2002, from August 2002 until
Leynes who both confirmed his mental condition. September 2002, and from May 2003 until July 2003.

MNI NOTES |235


Deoferio also took an administrative leave with pay from schizophrenia is belied by his subsequent
from January 2005 until December 2005. employment at Maxim Philippines Operating Corp.
and Philips Semiconductors Corp., which both
The respondents further asserted that the twin-notice
offered him higher compensations.
requirement in dismissals does not apply to
terminations under Article 284 of the Labor Code. He also asserts that the Labor Code does not exempt
They emphasized that the Labor Code's implementing the employer from complying with the twin-notice
rules (IRR) only requires a competent public health requirement in terminations due to disease
authority's certification to effectively terminate the
services of an employee. ISSUE:
LA RULING: the Labor Arbiter (LA) ruled that 1) Whether Deoferio was suffering from
Deoferio had been validly dismissed. The LA gave schizophrenia (YES) and whether his continued
weight to Dr. Lee's certification that Deoferio had employment was prejudicial to his health, as well as
been suffering from schizophrenia and was not fit for to the health of his co-employees (YES);
employment. The evidence on record shows that (2) Whether the twin-notice requirement in dismissals
Deoferio's continued employment at Intel would pose applies to terminations due to disease; and (YES,
a threat to the health of his co-employees. The LA DUE PROCESS)
further held that the Labor Code and its IRR do not
require the employer to comply with the twin-notice As part of the second issue, the following issues are
requirement in dismissals due to disease. raised:

The LA also found unmeritorious Deoferio's money (a) Whether Deoferio is entitled to nominal damages
claims against Intel. for violation of his right to statutory procedural due
process (YES); and
NLRC RULING: On appeal by Deoferio, the
National Labor Relations Commission (NLRC) (b) Whether the respondents are solidarily liable to
wholly affirmed the LA's ruling. Deoferio for nominal damages (No,Intel is solely
liable; Wentling is not personally liable for the
CA RULING: the CA affirmed the NLRC decision. It satisfaction of nominal damages in favor of Deoferio,
agreed with the lower tribunals' findings that Deoferio being a corporate officer, cannot be held liable for
was suffering from schizophrenia and that his acts done in his official capacity because a
continued employment at Intel would be prejudicial corporation, by legal fiction, has a personality
to his health and to those of his co-employees. It ruled separate and distinct from its officers, stockholders,
that the only procedural requirement under the IRR is and members.) - Piercing the veil of corporate fiction
the certification by a competent public health because Wentling acted in good faith and merely
authority on the non-curability of the disease within a relied on Dr. Lee’s psychiatric report in carrying out
period of six months even with proper medical the dismissal.
treatment. It also concurred with the lower tribunals
that Intel was justified in not paying Deoferio (3) Whether Deoferio is entitled to salary differential,
separation pay as required by Article 284 of the Labor backwages, separation pay, moral and exemplary
Code because this obligation had already been offset damages, as well as attorney’s fees (NO).
by the matured car loan that Deoferio owed Intel.
RULING:
In the present petition before the Court, Deoferio We find the petition partly meritorious.
argues that the uniform finding that he was suffering

MNI NOTES |236


Intel had an authorized cause to dismiss Deoferio The third element substantiates the contention that the
from employment. employee has indeed been suffering from a disease
that:
The present case involves termination due to disease
an authorized cause for dismissal under Article 284 of (1) is prejudicial to his health as well as to the health
the Labor Code. As substantive requirements, the of his co-employees; and
Labor Code and its IRR require the presence of the
(2) cannot be cured within a period of six months even
following elements:
with proper medical treatment. Without the medical
(1) An employer has been found to be suffering from certificate, there can be no authorized cause for the
any disease. employee's dismissal. The absence of this element
thus renders the dismissal void and illegal.
(2) His continued employment is prohibited by law or
prejudicial to his health, as well as to the health of his Simply stated, this requirement is not merely a
co-employees. procedural requirement, but a substantive one.

(3) A competent public health authority certifies that The certification from a competent public health
the disease is of such nature or at such a stage that it authority is precisely the substantial evidence
cannot be cured within a period of six months even required by law to prove the existence of the disease
with proper medical treatment. itself, its non-curability within a period of six months
even with proper medical treatment, and the prejudice
With respect to the first and second elements, the
that it would cause to the health of the sick employee
Court liberally construed the phrase "prejudicial to his
and to those of his co-employees.
health as well as to the health of his co-employees" to
mean "prejudicial to his health or to the health of his In the current case, we agree with the CA that Dr.
co-employees." We did not limit the scope of this Lee's psychiatric report substantially proves that
phrase to contagious diseases for the reason that this Deoferio was suffering from schizophrenia, that his
phrase is preceded by the phrase "any disease" under disease was not curable within a period of six months
Article 284 of the Labor Code, to wit: even with proper medical treatment, and that his
continued employment would be prejudicial to his
Art. 284. Disease as ground for termination.
mental health. This conclusion is further substantiated
An employer may terminate the services of an by the unusual and bizarre acts that Deoferio
employee who has been found to be suffering from committed while at Intel's employ.
any disease and whose continued employment is
The twin-notice requirement applies to terminations
prohibited by law or is prejudicial to his... health as
under Article 284 of the Labor Code.
well as to the health of his co-employees: Provided,
That he is paid separation pay equivalent to at least The Labor Code and its IRR are silent on the
one (1) month salary or to one-half (1/2) month salary procedural due process required in terminations due
for every year of service, whichever is greater, a to disease. Despite the seeming gap in the law,
fraction of at least six (6) months being... considered Section 2, Rule 1, Book VI of the IRR expressly states
as one (1) whole year. that the employee should be afforded procedural due
process in all cases of dismissals.
Consistent with this construction, we applied this
provision in resolving illegal dismissal cases due to In Sy v. Court of Appeals and Manly Express, Inc. v.
non-contagious diseases such as stroke, heart attack, Payong, Jr., promulgated in 2003 and 2005,
osteoarthritis, and eye cataract, among others. respectively, the Court finally pronounced the rule

MNI NOTES |237


that the employer must furnish the employee two (3) the employer's grant of other termination benefits
written notices... in terminations due to disease, in favor of the employee; and (4) whether there was a
namely: bona fide attempt on the part of the employer to
comply with the twin-notice requirement as opposed
(1) the notice to apprise the employee of the ground
to giving no notice at all.
for which his dismissal is sought; and
We award Deoferio the sum of P30,000.00 as nominal
(2) the notice informing the employee of his
damages for violation of his statutory right to
dismissal, to be issued after the employee has been
procedural due process. In so ruling, we take into
given reasonable opportunity to answer and to be
account Intel's faithful compliance with Article 284
heard on his defense.
of the Labor Code and Section 8, Rule 1, Book 6 of
From these perspectives, the CA erred in not finding the IRR. We also note that Deoferio's separation pay
that the NLRC gravely abused its discretion when it equivalent to one-half month salary for every year of
ruled that the twin-notice requirement does not apply service was validly offset by his matured car loan.
to Article 284 of the Labor Code. Under Article 1278 of the Civil Code, in relation to
Article 1706 of the Civil Code and
Deoferio is entitled to nominal damages for violation
of his right to statutory procedural due process Article 113(c) of the Labor Code, compensation shall
take place when two persons are creditors and debtors
Intel's violation of Deoferio's right to statutory of each other in their own right. We likewise consider
procedural due process warrants the payment of the fact that Intel exhibited real concern to Deoferio
indemnity in the form of nominal damages. when it financed his medical expenses for more than
With respect to Article 284 of the Labor Code, four years. Furthermore, prior to his termination, Intel
terminations due to disease do not entail any liberally allowed Deoferio to take lengthy leave of
wrongdoing on the part of the employee. It also does absences to allow him to attend to his medical needs.
not purely involve the employer's willful and
voluntary exercise of management prerogative a
function associated with the employer's inherent right
to control and effectively manage its enterprise.
Clemente v. Eso Nice Transport, GR 228231,
Rather, terminations due to disease are occasioned by August 28, 2019
matters generally beyond the worker and the DOCTRINE: ART. 297. [282] Termination by
employer's control. Employer - An employer may terminate an
employment for any of the following causes:
In fixing the amount of nominal damages whose
determination is addressed to our sound discretion, (a) Serious misconduct or willful disobedience by the
the Court should take into account several factors employee of the lawful orders of his employer or
surrounding the case, such as: representative in connection with his work;

(1) the employer's financial, medical, and/or moral (b) Gross and habitual neglect by the employee of his
assistance to the sick employee; duties;

(2) the flexibility and leeway that the employer (c) Fraud or willful breach by the employee of the
allowed the sick employee in performing his duties trust reposed in him by his employer or duly
while attending to his medical needs; authorized representative;

MNI NOTES |238


(d) Commission of a crime or offense by the greater, a fraction of at least six (6) months being
employee against the person of his employer or any considered as one (1) whole year.
immediate member of his family or his duly
Book VI, Rule I, Section 2(d) of the Omnibus Rules
authorized representatives; and
Implementing the Labor Code:
(e) Other causes analogous to the foregoing.
Standards of due process: requirements of notice. – In
ART. 298. [283] Closure of Establishment and all cases of termination of employment, the following
Reduction of Personnel. - The employer may also standards of due process shall be substantially
terminate the employment of any employee due to the observed:
installation of labor-saving devices, redundancy,
I. For termination of employment based on just causes
retrenchment to prevent losses or the closing or
as defined in Article 282 of the Code:
cessation of operation of the establishment or
undertaking unless the closing is for the purpose of (a) A written notice served on the employee
circumventing the provisions of this Title, by serving specifying the ground or grounds for termination, and
a written notice on the workers and the Ministry of giving to said employee reasonable opportunity
Labor and Employment at least one (1) month before within which to explain his side;
the intended date thereof.
(b) A hearing or conference during which the
In case of termination due to the installation of labor- employee concerned, with the assistance of counsel if
saving devices or redundancy, the worker affected the employee so desires, is given opportunity to
thereby shall be entitled to a separation pay equivalent respond to the charge, present his evidence or rebut
to at least his one (1) month pay or to at least one (1) the evidence presented against him; and
month pay for every year of service, whichever is
higher. In case of retrenchment to prevent losses and (c) A written notice of termination served on the
in cases of closures or cessation of operations of employee indicating that upon due consideration of
establishment or undertaking not due to serious all the circumstances, grounds have been established
business losses or financial reverses, the separation to justify his termination.
pay shall be equivalent to one (1) month pay or at least
In case of termination, the foregoing notices shall be
one-half (1/2) month pay for every year of service,
served on the employee's last known address.
whichever is higher.

A fraction of at least six (6) months shall be FACTS:


considered one (1) whole year. Sometime in August 1998, ESQ-Nice Transport
Corporation (respondent) hired petitioner as bus
ART. 299. [284] Disease as Ground for dispatcher in its Baguio branch.7 When its Baguio
Termination. - An employer may terminate the branch operations was audited in August 2013,
services of an employee who has been found to be respondent found out that numerous collections were
suffering from any disease and whose continued not deposited in its bank account. 8 Thus, in an
employment is prohibited by law or is prejudicial to August 22, 2013 letter, the respondent gave the
his health as well as to the health of his co-employees: petitioner 72 hours to explain the following
Provided, That he is paid separation pay equivalent to violations:
at least one (1) month salary or to one-half (1/2)
month salary for every year of service, whichever is [a]Unremitted collection of payment of United Van
Assoc. - dated August 3, 2013 - P15,000.00

MNI NOTES |239


[b] Unremitted payment of M. Kaley dated August Petitioner eventually surrendered voluntarily at the
2013 -P.60,000.00 Sta. Lucia Police Station in !locos Sur on February 6,
2014.
[c] Other sales
Petitioner to LA:
Petitioner also sent a similar letter to Alex Garcia
(Garcia), who admitted using the money to pay the filed a complaint for illegal dismissal, underpayment
hospitalization bills of his father. of wages, non-payment of 13th month pay in 2013
and wages for September 15 to 30, 2013 and October
On September 28, 2013, respondent called for a
1 to 9, 2013, service incentive leave pay, overtime
meeting to discuss the matter of undeposited
pay, separation pay in lieu of reinstatement, full
collections. The meeting was attended by the
backwages and attorney's fees.
petitioner and the other concerned employees of the
respondent. Respondent claimed that during the said LA:
meeting, the petitioner admitted appropriating for
*ruled that petitioner had been illegally dismissed
himself numerous proceeds of the company.
given that respondent failed to show any valid cause
Respondent claimed that petitioner and Garcia for his termination.
admitted to having fraudulently taken the undeposited
--> Respondent's claim that petitioner committed
collections in the amount of PS6,710.46 and
qualified theft had not been duly substantiated
P665,090.SS, respectively. As proof, respondent
inasmuch as the prosecutor only found probable cause
submitted a document denominated as Eso-Nice ·
against Garcia. While respondent presented evidence
Transport Corp., Undeposited Collections, January 1
showing that petitioner admitted to taking the amount
to August 31, 2013, which shows the petitioner and
of P56,710.46, the same becomes doubtful in light of
Garcia's handwritten and signed confession dated
his staunch denial of appropriating for himself the
October 3, 2013.
said amount in his August 24, 2013 written
By reason of petitioner's admission, respondent, on explanation.
October 9, 2013, served upon the petitioner a Notice
*the signature of the petitioner in the said document
of Termination dated October 3, 2013.
where he allegedly admitted to pocketing the
On November 29, 2013, respondent filed with the undeposited collections was forged
Baguio City prosecutor's office a complaint against
--> his handwriting was different from his
the petitioner and Garcia for qualified theft.
penmanship in the document where he denied his
Meanwhile, the investigating prosecutor found liability for the missing collections.
probable cause for qualified theft against the
*petitioner was not accorded his right to procedural
petitioner and Garcia.
due process.
RTC:
No hearing was conducted to investigate the alleged
agreed with the finding of probable cause by the complicity of the petitioner for theft.
investigating prosecutor and ordered the issuance of a
NLRC: affirmed the decision of LA in toto.
warrant of arrest against the petitioner and Garcia but
it was returned unserved as the petitioner was *held that other than petitioner's purported admission,
nowhere to be found in Sta. Lucia, !locos Sur. respondent miserably failed to adduce substantial
evidence to justify his termination.

MNI NOTES |240


*The need for a thorough inquiry is brought to greater petitioner's reinstatement to his former position
light by the fact that the petitioner is a mere dispatcher without loss of seniority rights, but without
who is not charged with the custody of daily fare backwages or other monetary benefits.
collections as such task belonged to Garcia being the
company's cashier/teller. Petitioner moved for reconsideration, but the CA
denied it in its February 5, 2016 Resolution.
*also agreed with the Labor Arbiter that a marked
variance exists between petitioner's supposed ISSUE:
signature admitting the theft vis-a-vis his signature
appearing on other documents submitted before it. Whether or not there exists a valid ground for
petitioner's dismissal.
*petitioner's termination cannot be upheld for the
additional ground of want of procedural due process. RULING:
--> The special meeting called by the respondent is For a dismissal to be valid, the rule is that the
not equivalent to the required hearing since no employer must comply with both the substantive and
searching questions were propounded during the the procedural due process requirements. Substantive
meeting to ferret out the truth behind the unremitted due process requires that the dismissal must be
collections. The three days that petitioner was given pursuant to either a just or an authorized cause under
to answer the charge was also not sufficient. Articles 282, 283 or 284 (now Articles 297, 298 and
299, respectively) of the Labor Code. On the other
Respondent moved for reconsideration, but the hand, procedural due process in dismissal cases
NLRC denied it in a Resolution dated November 28, consists of the twin equirements of notice and
2014. hearing.
Respondent elevated the matter before the CA via a Anent the issue of procedural due process, Section 2
Petition for Certiorari, which ruled in its favor in a (I), Rule XXIII, Book V of the Omnibus Rules
Decision. Implementing the Labor Code provides for the
required standard of procedural due process accorded
CA:
to employees who stand to be terminated from work.
*respondent complied with the twin-notice
In asserting the illegality of his dismissal, the
requirement when it gave the petitioner a chance to be
petitioner harped on respondent's non-observance of
heard and subsequently informed him of his dismissal
his right to procedural due process.
from employment for committing qualified theft
against it. *A close scrutiny of the records of this case reveals
that respondent indeed failed to comply with the due
*found the admission of the petitioner that he failed
process requirement.
to deposit the collections in the amount of P56,710.46
coupled with the findings of probable cause for --> In short, the blanket notice, instead of informing
Qualified Theft by both the investigating prosecutor the petitioner of the violation for which his
and the RTC as valid ground for the respondent to explanation is being required, creates confusion on
impose disciplinary action upon the petitioner. the nature of the complaints against him. Aside from
the said deficiency, the notice failed to mention which
*found the penalty of dismissal imposed by the
company rule petitioner violated or the just cause for
respondent upon the petitioner to be not
which his termination is sought.
commensurate to the offense committed --> ordered

MNI NOTES |241


*Instead of five calendar days, he was given only 72 Given that the petitioner was dismissed without just
hours or three short days to explain his side. cause and without due process, he is entitled to
reinstatement, without loss of seniority rights and
However, contrary to the view of the petitioner,
other privileges and to his full backwages, inclusive
violation of the due process requirement does not
of allowances, and to his other benefits or their
automatically result to the illegality of one's dismissal
monetary equivalent computed from the time his
from work.
compensation was withheld from him up to the time
--> If his dismissal was justified, but his right to of his actual reinstatement under Article 294 of the
procedural due process was transgressed, petitioner's Labor Code. However, considering the strained
dismissal will still remain valid, but respondent shall relationship now existing between the parties, the
become liable for damages. grant of separation pay in lieu of reinstatement is
justified.
--> if petitioner's termination is not for a just or
authorized cause, his dismissal shall be invalid and he Ample Opportunity to be Heard
shall be entitled to reinstatement without loss of
seniority rights, and other pertinent benefits and CMP Federal Security Agency v. Reyes, Jr.,
allowances. GR 223082, June 26, 2019
While the respondent claimed that during the DOCTRINE: The test for the fair procedure
September 28, 2013 meeting "petitioner admitted to guaranteed under Article 277(b) cannot be whether
not depositing numerous proceeds of the company there has been a formal pre-termination confrontation
and using the same for his personal gain and to the between the employer and the employee.
damage and prejudice of respondent," the minutes of The 'ample opportunity to be heard' standard is neither
the said meeting which the respondent presented synonymous nor similar to a formal hearing. To
show otherwise. confine the employee's right to be heard to a solitary
--> Nothing in the minutes indicates that it had been form narrows down that right. It deprives him of other
shown during the said meeting or that the petitioner equally effective forms of adducing evidence in his
admitted appropriating for himself any amount that defense. Certainly, such an exclusivist and absolutist
was collected from January 1 to August 31, 2013. interpretation is overly restrictive. The 'very nature of
due process negates any concept of inflexible
It is contrary to human experience that an employee procedures universally applicable to every
would admit to the taking of company funds which imaginable situation.'
was not even established during the investigation
initiated by the company. --> If at all, petitioner's The guiding principles in connection with the hearing
liability would only amount to negligence for not requirement in dismissal cases:
ensuring that funds that came to his possession was (a) 'ample opportunity to be heard' means any
immediately deposited to respondent's bank account meaningful opportunity (verbal or written) given to
or turned over to the personnel in-charge of the employee to answer the charges against him and
collections. submit evidence in support of his defense, whether in
*Negligence is not among the just cause under Article a hearing, conference or some other fair, just and
297 which would validate respondent's act of reasonable way.
terminating the petitioner from employment. (b) a formal hearing or conference becomes
mandatory only when requested by the employee in

MNI NOTES |242


writing or substantial evidentiary disputes exist or a complaints against him contained in the Reply by
company rule or practice requires it, or when similar Indorsement dated July 20, 2013, which states:
circumstances justify it.
You are hereby directed to explain in writing within
(c) the "ample opportunity to be heard" standard in FIVE (5) days upon receipt hereof why you should
the Labor Code prevails over the 'hearing or not be charged [with] the following:
conference' requirement in the implementing rules
and regulations 1. Insubordination: For not: following the
instruction of Mr. Arnel Maningat, Operations
FACTS: Manager, to designate SO Robert Sagun as Shift-in-
CMP Federal is a duly licensed security agency with Charge effective 01 May 2013, and designated him as
petitioner Carolina Mabanta-Piad as its President and ordinary guard instead;
Chief Executive Officer (collectively, petitioners).
2. Negligence (4th Offense): For failure to report to
CMP Federal hired respondent Reyes as Security the Operations Manager the incident pertaining to
Guard and assigned him at the the two (2) security personnel in the persons of SG
Rommy Ramiterre and SG Jesus Sumalbag who were
Mariveles Grain Terminal (MG Terminal) in
confronted by the Personajes Trucking Personnel,
Mariveles, Bataan. He was twice promoted, first as
wherein as Detachment Commander, [you] are
Shift-in-Charge, and then as Detachment
Commander. duty-bound to report to the latter all matters
pertaining to the [operations;
According to Reyes, he was reluctant to accept the
promotion because he was only a high school 3. Violation of Section 1.B.C, Rule X of RA 5487:
graduate with little knowledge about operating For providing confidential information relative to the
computers and petitioners were not in favor of his Cabcaben Vacant Lot takeover, wherein this office
promotion as Detachment Commander because they has received a reports [sic] that you allegedly leak
wanted a certain Robert Sagun (Sagun) for the the information to your subordinates on the drinking
position, but they had to accede to the request of MG session last 02 December 2012 that eventually
Terminal, one of CMP Federal's valued clients. reached the knowledge of the [MG Terminal]
General Manager.
After the promotion, Reyes claimed that CMP Federal
would treat him unaffably and that he would be Failure to comply within the prescribed period shall
rebuked incessantly by his superiors, who told him be construed as [a] waiver of your right to be heard.
that he was not fit for the job and OM Maningat would
For your strict compliance.
relay orders and instructions from the main office to
Sagun, and not to him, for implementation. Reyes timely submitted his explanation but CMP
Federal barred Reyes from reporting to work, and told
He also claimed that he received via e-mail various
him instead to await the decision of the
complaints from Maningat.
management regarding the complaints.
Reyes formally received Offence Notices9 pertaining
to the complaints from CMP Federal and was ordered Reyes still reported for duty until July 2013 but he
immediately suspended until July 20, 2013.10 Upon was verbally informed of his termination and received
the expiry of the suspension period, Reyes reported a Notice of Termination on the said date.
back to work, only to be confronted by additional

MNI NOTES |243


Reyes: a) Pay complainant separation pay in lieu of
reinstatement in [an] amount equivalent to one (1)
lodged a complaint for illegal dismissal, non-payment
month pay for every year of service reckoned from his
of service incentive leave, separation pay,
employment up to finality of this Decision;
reimbursement of expenditures for supplies and cash
bond, with a prayer for payment of moral and b) Pay full backwages to complainant from the time
exemplary damages, as well as attorney's fees. he was illegally dismissed on 20 July 2013 up to
finality of this Decision;
CMP Federal:
c) Pay the amount of Php 5,220.00 to complainant
- denied the complaint and averred that, starting
representing his service incentive pay;
January 2013, Reyes had been remiss in the discharge
of his duties as Detachment Commander at MG d) Pay the amount of Php 8,900.00 to complainant
Terminal. representing reimbursement of expenditures for
supplies;
- Reyes' dismissal was justified because Reyes was
negligent in the performance of his duties as shown e) Pay the amount of Php 3,400.00 to complainant for
by his repeated disregard of company rules. the cash bond; and

- Reyes' position was one of trust and confidence, to f) Pay the amount corresponding to 10% of the
which Reyes proved untrustworthy when he leaked judgment award to complainant as and by way of
confidential information. attorney's fees.

- they observed procedural due process in dismissing --> Reyes committed no serious misconduct that
Reyes from service through the e-mails and Reply by could have warranted his dismissal. Moreover, the
Indorsement, apprised of the specific NLRC held, that in dismissing Reyes, the petitioners
did not comply with the detailed steps of procedural
incidents that led to the charges against him and
due process.
provided ample opportunity to explain himself and
controvert the charges. --> wrongful intent, an indispensable element of
serious misconduct, was not duly established by the
LA: respondent CMP Federal Security Agency, Inc.
petitioners; and that on the contrary, Reyes' Written
is hereby ordered to pay complainant the amount of
Explanation dated July 22, 2013 clearly showed that
₱5,220.00 representing his service incentive leave
there was no deliberate intent on his part to violate
pay.
CMP Federal's rules and regulations.
All other claims are denied.
--> a perusal of the Reply by Indorsement dated July
--> the just cause for Reyes' dismissal was adequately 20, 2013 would show that no hearing or conference
substantiated bythe petitioners who also proved that was scheduled and conducted by petitioners to give
they complied with the due process requirements for Reyes an opportunity to explain and clarify his
termination of employment. defenses from the charges against him, to present
evidence in support of his defenses, and to rebut the
NLRC: reversed the Labor Arbiter's ruling. evidence presented against him.
Respondent CMP Federal Security Agency, Inc. is *Without the benefit of a hearing prior to his
directed to: dismissal and absent just cause for his termination,

MNI NOTES |244


Reyes's dismissal was struck down by the NLRC as the legislative intent to give some degree of flexibility
illegal. or adaptability to meet the peculiarities of a given
situation. To confine it to a single rigid proceeding
Ascribing grave abuse of discretion on the part of the
such as a formal hearing will defeat its spirit.
NLRC for reversing the Labor Arbiter's finding, the
petitioners filed with the CA a Petition for Certiorari Significantly, Section 2(d), Rule I of the
with Prayer for Issuance of Temporary Restraining Implementing Rules of Book VI of the Labor Code
Order. itself provides that the so-called standards of due
process outlined therein shall be observed
CA: Unfortunately for the petitioners, the CA, in its
'substantially,' not strictly. This is a recognition that
assailed Decision, upheld the NLRC's rulings
while a formal hearing or conference is ideal, it is not
--> The CA sustained the NLRC's findings on the an absolute, mandatory or exclusive avenue of due
ground that the standards of due process were not process.
strictly complied with; that, absent proof that an
A hearing means that a party should be given a chance
investigation was conducted by the petitioners or that
to adduce his evidence to support his side of the case
Reyes was given an opportunity to be heard and
and that the evidence should be taken into account in
present his countervailing evidence, it would be
the adjudication of the controversy. 'To be heard' does
unfair for the CA to reverse the NLRC's Decision.
not mean verbal argumentation alone inasmuch as
Petitioners moved for reconsideration, but the CA one may be heard just as effectively through written
explanations, submissions or pleadings. Therefore,
affirmed its August 28, 2015 Decision through its
January 26, 2016 Resolution. while the phrase 'ample opportunity to be heard' may
in fact include an actual hearing, it is not limited to a
ISSUE: formal hearing only. In other words, the existence of
an actual, formal 'trial-type' hearing, although
Whether or not the Honorable Court of Appeals erred preferred, is not absolutely necessary to satisfy the
in affirming the Decision of the NLRC, reversing the employee's right to be heard.
Decision of the Labor Arbiter Fe Cellan in finding
Reyes was afforded ample opportunity to be heard
that the Respondent Reyes was illegally dismissed
(YES) --> Reyes was afforded more than enough chances to
raise intelligent defenses, except that he mostly
RULING: admitted his infractions and apologized for them in
The Petition is impressed with merit. his Written Explanations.
Procedural due process in illegal dismissal cases does --> A formal hearing or conference was not necessary
not require formal hearing or conference since nowhere in any of his Written Explanations did
In determining whether an employee's dismissal has Reyes request for one.
been legal, the inquiry focuses on whether the --> The CA therefore erred in ruling that the NLRC
dismissal violated the employee's right to both did not act with grave abuse of discretion when it
substantial and procedural due process. reversed the Decision of the Labor Arbiter.

Just cause for Reyes's termination


The standard for the hearing requirement, ample Article 297 of the Labor Code enumerates the just
opportunity, is couched in general language revealing causes for the termination of employment.

MNI NOTES |245


--> the Court agrees with the NLRC and the CA that Compounded with the earlier finding that the NLRC
Reyes' infractions did not constitute "serious similarly gravely abused its discretion in finding that
misconduct" as contemplated under the first the procedural due process requirements were not
paragraph of Article 282 of the Labor Code. complied with, the Court is constrained to reverse the
ruling of the CA. The reinstatement of the Labor
Misconduct is defined as an improper or wrong
Arbiter's ruling is therefore in order.
conduct. It is a transgression of some established and
definite rule of action, a forbidden act, a dereliction of
duty, willful in character, and implies wrongful intent
and not mere error in judgment. To constitute a valid Terminating a Probationary Employee
cause for the dismissal within the text and meaning of
Article 282 of the Labor Code, the employee's Concept of Probationary Employment
misconduct must be serious, i.e., of such grave and
aggravated character and not merely trivial or - A situation where the employee upon his
unimportant. engagement is made to undergo a trial period during
which the employer determines his fitness to qualify
--> the explanations proffered by Reyes showed that
for regular employment, based on reasonable
he was not animated by any wrongful intent when he
standards made known to him at the time of
committed the infractions complained of. Moreover,
engagement.
the finding that he was guilty of serious misconduct
was incompatible with the charges for negligence - Employment contract or appointment paper should
which, by definition, requires lack of wrongful intent. expressly and specifically state that the engagement
--> The Court cannot also consider negligence as a of the employee is on probationary basis
valid ground for Reyes' dismissal. To be a valid
- Purpose of probationary employment: To allow the
ground for dismissal, the neglect of duty must be both
employer to test the working habits and other personal
gross and habitual. Gross negligence implies want of
traits of the employee with respect to his fitness for
care in the performance of one's duties. Habitual
regularization in its company.
neglect, on the other hand, implies repeated failure to
perform one's duties for a period of time.
- Prerogative of an employer to place new employees
--> the Court finds that, although Reyes' negligence on probation
was habitual, they could in no way be considered
gross in nature. Prerogative an incident of the employers
inherent right to choose whom to hire and
Nevertheless, the Court rules that there was still just whom to decline.
cause for Reyes' termination - gross inefficiency.
Duration of Probationary Employment of
The CA thus erred in ruling that the NLRC did not act
Ordinary Employees
with grave abuse of discretion in invalidating Reyes'
dismissal for lack of just cause. The NLRC and the
- Generally: Probationary period of employment of
CA should not have fixated itself with the designation
ordinary employees is limited to six (6) months.
of the offense as serious misconduct when it is clear
from the complaints and Reply by Indorsement that - Exceptions:
Reyes was actually being made to answer for his
violation of company policies and standards.

MNI NOTES |246


(a) When the parties to an employment contract or Extension of Probationary Employment
collective bargaining agreement agree on a longer
period; - Can be extended to give the employee a chance to
improve.
(b) When a longer probationary period is established
by company policy; or - Such extension should be done on or before the
expiration of the prescribed period otherwise the
(c) When a longer period is required by the nature of employee will automatically become a regular
work. employee by operation of law

Duration of Probationary Employment of Termination of Probationary Employment


Teachers
- The services of an employee who has been engaged
- The standards set or promulgated jointly by the on a probationary basis may be terminated for:
Department of Education and the Department of
Labor and Employment shall be applied by the (a) Any of the causes enumerated in Articles
Department of Labor and Employment. 282, 283 and 284 of the Labor Code; or

- The probationary period for teaching and academic (b) Failure to qualify as a regular employee in
non-teaching personnel are as follows (DOLE- accordance with reasonable standards made
DECS-CHED-TESDA Order No. 1, 1996): known by the employer at the time of his
engagement.
(a) For elementary and secondary level --
three (3) consecutive school years of - It is not necessary that the entire probationary period
satisfactory service; be exhausted before the employment could be
terminated.
(b) For tertiary and graduate level – six
consecutive semesters of satisfactory service; o Termination may be done even before the
expiration of the probationary period.
(c) For tertiary level on trimester service –
nine (9) consecutive trimesters of satisfactory Limitations on the Right to Terminate a
service. Probationary Employment

- The School, as employer, is the one who is to set the - The power of an employer to terminate a
standards and determine whether or not the services probationary employment is subject to the following
of an employee are satisfactory. limitations:

- It is the right of the employer to shorten the (a) It must be exercised in accordance with the
probationary period if he is not impressed with the specific requirements of the contract.
services of the employee.
(b) The dissatisfaction of the employer must be real
- This prerogative is in accordance with academic and in good faith, not feigned so as to circumvent the
freedom and constitutional autonomy which give contract or the law; and
educational institution the right to choose who should
teach.

MNI NOTES |247


(c) There must be no unlawful discrimination in the available position but later withdrew his application
dismissal. prompting Magtibay to apply for the same.

Daily Inquirer v. Magtibay, 528 SCRA 355 PDI then hired Magtibay on a probationary basis for
a period of 6 months. A week prior to the expiration
DOCTRINE: ART. 294. [279] Security of
of the 6 month probationary period, Magtibay was
Tenure. - In cases of regular employment, the
handed his termination paper for alleged failure to
employer shall not terminate the services of an
meet company standards. For such, he filed a
employee except for a just cause or when authorized
complaint for illegal dismissal and damages before
by this Title. An employee who is unjustly dismissed
the Labor Arbiter.
from work shall be entitled to reinstatement without
loss of seniority rights and other privileges and to his PDIEU later joined the case by filing a supplemental
full backwages, inclusive of allowances, and to his complaint for unfair labor practice.
other benefits or their monetary equivalent computed
from the time his compensation was withheld from LA: ruled in favor of PDI.
him up to the time of his actual reinstatement. --> relied on the abstract language provided for in the
Art. 281. Probationary employment. ̶ Probationary termination paper which stated that: “you did not
employment shall not exceed six (6) months from the meet the standards of the company”, to wit:
date the employee started working, unless it is (1) he repeatedly violated the company rule
covered by an apprenticeship agreement stipulating a prohibiting unauthorized persons from entering the
longer period. The services of an employee who has telephone operators room;
been engaged on a probationary basis may be
terminated for a just cause or when he fails to qualify (2) he intentionally omitted to indicate in his
as a regular employee in accordance with reasonable application form his having a dependent child; and
standards made known by the employer to the (3) he exhibited lack of sense of responsibility by
employee at the time of his engagement. An employee locking the door of the telephone operators room on
who is allowed to work after a probationary period March 10, 1996 without switching the proper lines to
shall be considered a regular employee. the company guards so that incoming calls may be
FACTS: answered by them.

Leonardo Magtibay was hired on a contractual basis NLRC reversed LA's decision - ruled in favor of
by the Philippine Daily Inquirer for a period of 5 Magtibay
months. Before the expiration of the said contract, --> effectively ruling that Magtibay was illegally
they agreed to extend it for another 15 days. After dismissed.
Magtibay’s contractual employment expired, PDI
then announced that they created an available position --> Magtibay’s probationary employment had ripened
for a second telephone operator, who will undergo a into a regular one.
probationary employment. Since it was a practice of
PDI filed a motion for reconsideration - NLRC
PDI to give preference to its regular employees for
denied.
vacancies, Ms. Regina Layague, a regular employee
and a member of the Philippine Daily Inquirer PDI filed a petition for certiorari to CA.
Employees Union (PDIEU), applied for the said
CA: denied the petition.

MNI NOTES |248


--> Petitioner PDI failed to prove that such rules and It is expressly provided in the afore-quoted Article
regulations were included in or form part of the 281 that a probationary employee may be terminated
standards that were supposed to be made known to only on two grounds:
respondent Magtibay at the time of his engagement as
(a) for just cause, or
telephone operator.
(b) when he fails to qualify as a regular employee in
--> nowhere can it be found in the list of Basic
accordance with reasonable standards made known
Responsibility and Specific Duties and
by the employer to the employee at the time of his
Responsibilities (Annex D of the petition) of
engagement.
respondent Magtibay that he has to abide by the
duties, rules and regulations that he has allegedly In claiming that it had adequately apprised Magtibay
violated. The infractions considered by petitioner PDI of the reasonable standards against which his
as grounds for the dismissal of respondent Magtibay performance will be gauged for purposes of
may at most be classified as just causes for the permanent employment, PDI cited the one-on-one
termination of the latter’s employment. seminar between Magtibay and its Personnel
Assistant, Ms. Rachel Isip-Cuzio. PDI also pointed to
ISSUE:
Magtibay’s direct superior, Benita del Rosario, who
1. Whether Magtibay was illegally dismissed. (NO) – diligently briefed him about his responsibilities in
he is still in probationary employment PDI.

Does this make his termination unlawful for being --> assertions were never denied nor controverted by
violative of his right to due process of law? (NO) - It Magtibay.
is undisputed that PDI apprised Magtibay of the
CA erred when it cleared the NLRC of commission of
ground of his termination, i.e., he failed to qualify as
grave abuse of discretion despite the latter’s disregard
a regular employee in accordance with reasonable
of clear and convincing evidence that there were
standards made known to him at the time of
reasonable standards made known by PDI to
engagement, only a week before the expiration of the
Magtibay during his probationary employment.
six-month probationary period.
--> It is on record that Magtibay committed obstinate
RULING:
infractions of company rules and regulations, which
SC GRANT the petition. in turn constitute sufficient manifestations of his
inadequacy to meet reasonable employment norms.
Management and labor, or the employer and the
employee are more often not situated on the same Common industry practice and ordinary human
level playing field, so to speak. Recognizing this experience do not support the CA’s posture.
reality, the State has seen fit to adopt measures
--> All employees, be they regular or probationary,
envisaged to give those who have less in life more in
are expected to comply with company-imposed rules
law. Article 279 of the Labor Code which gives
and regulations, else why establish them in the first
employees the security of tenure is one playing field
place. Probationary employees unwilling to abide by
leveling measure.
such rules have no right to expect, much less demand,
Within the limited legal six-month probationary permanent employment.
period, probationary employees are still entitled to
SC find sufficient factual and legal basis, duly
security of tenure.
established by substantial evidence, for PDI to legally

MNI NOTES |249


terminate Magtibay’s probationary employment Abbott Laboratories v. Alcaraz, GR 192671,
effective upon the end of the 6-month probationary July, 23, 2013
period. FACTS:
It is undisputed that PDI apprised Magtibay of the
• Petitioner Abbott Laboratories, Philippines
ground of his termination, i.e., he failed to qualify as
(Abbott) caused the publication in a major
a regular employee in accordance with reasonable
broadsheet newspaper of its need for a
standards made known to him at the time of
engagement, only a week before the expiration of the Medical and Regulatory Affairs
six-month probationary period. Given this Manager. Alcaraz - who was then a
perspective, does this make his termination unlawful Regulatory Affairs and Information
for being violative of his right to due process of law? Manager at Aventis Pasteur Philippines,
- NO Incorporated (another pharmaceutical
company like Abbott) showed interest and
Due process of law for this second ground consists of submitted her application.
making the reasonable standards expected of the
employee during his probationary period known to • In Abbotts offer sheet, it was stated that
him at the time of his probationary employment. Alcaraz was to be employed on a
--> By the very nature of a probationary employment, probationary basis. Later that day, she
the employee knows from the very start that he will accepted the said offer and received an
be under close observation and his performance of his electronic mail (e-mail) from Abbotts
assigned duties and functions would be under Recruitment Officer, petitioner Teresita C.
continuous scrutiny by his superiors. Bernardo (Bernardo), confirming the same.
Attached to Bernardos e-mail were Abbotts
PDI was only exercising its statutory hiring
organizational chart and a job description
prerogative when it refused to hire Magtibay on a
of Alcarazs work.
permanent basis upon the expiration of the six-month
probationary period. This was established during the
• During Alcarazs pre-employment
proceedings before the labor arbiter and borne out by
orientation, petitioner Allan G. Almazar
the records and the pleadings before the Court.
(Almazar), Hospiras Country Transition
When the NLRC disregarded the substantial evidence Manager, briefed her on her duties and
establishing the legal termination of Magtibay’s responsibilities as Regulatory Affairs
probationary employment and rendered judgment Manager. Petitioner Kelly Walsh (Walsh),
grossly and directly contradicting such clear Manager of the Literature Drug
evidence, the NLRC commits grave abuse of Surveillance Drug Safety of Hospira, will
discretion amounting to lack or excess of jurisdiction. be her immediate supervisor. Petitioner
It was, therefore, reversible error on the part of the Maria Olivia T. Yabut-Misa (Misa),
appellate court not to annul and set aside such void Abbotts Human Resources (HR) Director,
judgment of the NLRC. sent Alcaraz an e-mail which contained an
explanation of the procedure for evaluating
the performance of probationary
employees.

MNI NOTES |250


• During the course of her employment, • NLRC: reversed the findings of the LA
Alcaraz noticed that some of the staff and ruled that there was no evidence
had disciplinary problems. Thus, she showing that Alcaraz had been apprised of
would reprimand them for their her probationary status and the
unprofessional behavior such as non- requirements which she should have
observance of the dress code, complied with in order to be a regular
moonlighting, and disrespect of Abbott employee.
officers. However, Alcarazs method of
management was considered by Walsh to • CA: affirmed the NLRC decision.
be "too strict."
• Hence, this petition.
• Alcaraz was called to a meeting with Walsh
ISSUE:
and Terrible, Abbotts former HR Director,
where she was informed that she failed to Whether Alcaraz was illegally dismissed.
meet the regularization standards for the
position of Regulatory Affairs Manager. HELD:
Walsh, Almazar, and Bernardo personally
handed to Alcaraz a letter stating that her The probationary employee may also be
services had been terminated effective May terminated for failure to qualify as a regular
19, 2005. The letter detailed the reasons for employee in accordance with the reasonable
Alcarazs termination. standards made known by the employer to the
employee at the time of the engagement.
• Alcaraz felt that she was unjustly
terminated from her employment and thus, • A probationary employee, like a regular
filed a complaint for illegal dismissal and employee, enjoys security of tenure.
damages against Abbott and its officers, However, in cases of probationary
namely, Misa, Bernardo, Almazar, Walsh, employment, aside from just or authorized
Terrible, and Feist. She claimed that she causes of termination, an additional
should have already been considered as a ground is provided under Article 295 of
regular and not a probationary employee the Labor Code, i.e., the probationary
given Abbotts failure to inform her of the employee may also be terminated for
reasonable standards for her regularization failure to qualify as a regular employee
upon her engagement as required under in accordance with the reasonable
Article 295 of the Labor Code. standards made known by the employer
to the employee at the time of the
• LA: dismissed Alcarazs complaint for engagement. Thus, the services of an
lack of merit. The LA rejected Alcarazs employee who has been engaged on
argument that she was not informed of the probationary basis may be terminated for
reasonable standards to qualify as a regular any of the following: (a) a just or (b) an
employee. authorized cause; and (c) when he fails to
qualify as a regular employee in accordance

MNI NOTES |251


with reasonable standards prescribed by the she was further notified that Abbott had
employer. only one evaluation system for all of its
employees.
• A punctilious examination of the records
reveals that Abbott had indeed complied • Considering the totality of the above-stated
with the above-stated requirements. This circumstances, it cannot, therefore, be
conclusion is largely impelled by the fact doubted that Alcaraz was well-aware that
that Abbott clearly conveyed to Alcaraz her her regularization would depend on her
duties and responsibilities as Regulatory ability and capacity to fulfill the
Affairs Manager prior to, during the time of requirements of her position as
her engagement, and the incipient stages of Regulatory Affairs Manager and that her
her employment. On this score, the Court failure to perform such would give Abbott
finds it apt to detail not only the incidents a valid cause to terminate her probationary
which point out to the efforts made by employment.
Abbott but also those circumstances which
would show that Alcaraz was well-apprised An employer who terminates an employee for a
of her employers expectations that would, valid cause but does so through invalid
in turn, determine her regularization. procedure is liable to pay the latter nominal
damages.
• Abbott caused the publication in a major
broadsheet newspaper of its need for a • Despite the existence of a sufficient ground
Regulatory Affairs Manager, indicating to terminate Alcarazs employment and
therein the job description for as well as the Abbotts compliance with the Labor Code
duties and responsibilities attendant to the termination procedure, it is readily apparent
aforesaid position. In Abbotts December 7, that Abbott breached its contractual
2004 offer sheet, it was stated that Alcaraz obligation to Alcaraz when it failed to
was to be employed on a probationary abide by its own procedure in evaluating
status. On the day Alcaraz accepted the performance of a probationary
Abbotts employment offer, Bernardo sent employee.
her copies of Abbotts organizational
• Records show that Abbotts PPSE
structure and her job description through e-
procedure mandates, inter alia, that the job
mail. Alcaraz was made to undergo a pre-
performance of a probationary
employment orientation where Almazar
employee should be formally reviewed
informed her that she had to implement
and discussed with the employee at least
Abbotts Code of Conduct and office
twice: first on the third month and second
policies on human resources and finance
on the fifth month from the date of
and that she would be reporting directly to
employment. Abbott is also required to
Walsh. Alcaraz received copies of Abbotts
come up with a Performance
Code of Conduct and Performance
Improvement Plan during the third month
Modules from Misa who explained to her
review to bridge the gap between the
the procedure for evaluating the
employees performance and the standards
performance of probationary employees;

MNI NOTES |252


set, if any. In addition, a signed copy of the
PPSE form should be submitted to
Abbotts HRD as the same would serve as
basis for recommending the confirmation
or termination of the probationary
employment.

• In this case, it is apparent that Abbott


failed to follow the above-stated
procedure in evaluating Alcaraz. For
one, there lies a hiatus of evidence that a
signed copy of Alcarazs PPSE form was
submitted to the HRD. It was not even
shown that a PPSE form was completed
to formally assess her performance.
Neither was the performance evaluation
discussed with her during the third and fifth
months of her employment. Nor did Abbott
come up with the necessary Performance
Improvement Plan to properly gauge
Alcarazs performance with the set
company standards.

• In this light, while there lies due cause to


terminate Alcarazs probationary
employment for her failure to meet the
standards required for her regularization,
and while it must be further pointed out that
Abbott had satisfied its statutory duty to
serve a written notice of termination, the
fact that it violated its own company
procedure renders the termination of
Alcarazs employment procedurally
infirm, warranting the payment of nominal
damages. A further exposition is apropos.

MNI NOTES |253


C. Floating Status; Preventive Suspension over 38 years old and subsequently
advised him to resign which he refused.
Temporary Off-Detail/Floating Status
He reported to the office to collect his 13th
Temporary Off-Detail in security parlance means month pay and was again persuaded to
waiting to be posted. The inactivity should not exceed hand a resignation letter. When he was still
six months, or the security agency is liable for not deployed or reassigned for more than a
constructive dismissal. year, he filed a Complaint for illegal
Fulfillment of Military or Civic Duty dismissal.

The worker’s employment is deemed suspended even • Padilla alleged that he was constructively
if the service rendered to military or civic duties dismissed on the ground that he was placed
exceed six months.
on a floating status. Airborne countered
Requisites: that Padilla was relieved from his post on
account of a client’s request. He was
a.) The employee must signify his desire to resume
work not later than 1 month from his relief from said directed to report to Airborne’s office but
military or civic duty. he failed to comply and went on absence
without leave instead.
The payment of wages and benefits shall be subject to
special laws, decrees and to applicable individual or ISSUE:
collective bargaining agreement and voluntary
employer practice or policy. Is placement in an inordinately long floating status
a ground for constructive dismissal?

Padilla v. Airborne Security Service, Inc., GR HELD:


210080, Nov. 22, 2017
• Yes. Padilla was constructively dismissed
FACTS:
from employment owing to his inordinately
long floating status.
• This is a petition for review on Certiorari
assailing the decision of the CA, which
• According to the case of Reyes v. RP
sustained the decision of the NLRC and
Guardians Security Agency, temporary
LA, dismissing petitioner Macario S.
displacement or temporary off-detail of
Padilla’s (Padilla) complaint for illegal
security guard is, generally, allowed in a
dismissal against respondent Airborne
situation where a security agency’s client
Security Service, Inc. (Airborne).
decided not to renew their service contract
with the agency and no post is available for
• Airborne hired Padilla as a security guard.
the relieved security guard. Such situation
He was relieved from his post at City
does not normally result in a constructive
Advertising Ventures Corporation and was
dismissal. Nonetheless, when the floating
advised to wait for his reassignment order.
status lasts for more than six (6) months,
Pending reassignment, he was told that
the employee may be considered to have
Airborne was having a hard time finding an
been constructively dismissed. In addition,
assignment for him since he was already

MNI NOTES |254


for an employee to be considered to have
abandoned his work, two (2) requisites
must concur, (1) the employee must have
failed to report for work or have been
absent without a valid or justifiable reason
and (2) the employee must have had a
“clear intention to sever the employer-
employee relationship.

• Padilla’s conduct belies any intent to


abandon his work. Right after he received
the first letter, he called Airborne’s head
office, only to be told that he has no
assignment yet. Considering Padilla’s 24
years of uninterrupted service, it is highly
improbable that he would abandon his work
so easily. Padilla was placed on floating
status for more than six (6) months.

• Thus, Padilla was constructively dismissed


when he was placed in inordinately long
floating status.

Philam Homeowners Association, Inc. v. De


Luna, GR 209437, Marc. 17,2021

MNI NOTES |255


D. Reliefs due to Illegal Termination; Strained - Such doctrine should not be applied indiscriminately
Relations since every labor dispute invariably results in strained
relations
Strained Relations
o Mere filing of complaint for ID
Since reinstatement is no longer feasible, such as in
the case of a clearly strained employer-employee  does not by itself justify the
relationship (limited to managerial positions and application of the doctrine of strained
contracts of employment predicated on trust and relations
confidence, such as in this case) or when the work or o Where differences of the ER with EE are
position formerly held by the dismissed employee neither personal nor physical much less
simply no longer exists, separation pay can substitute serious in nature
for reinstatement.
 does not by itself justify the
o Even if dismissal is found to be unjust/illegal,
application of the doctrine of strained
reinstatement should not be ordered anymore if the
relations
relationship between the parties has become so
strained and ruptured as to preclude a harmonious  Otherwise, reinstatement can
working relationship never be possible because some
hostility is engendered between the
o Instead, EE should be afforded separation pay
parties as a result of their
 This way, EE is spared the agony of having disagreement
to work anew with his ER under the
Backwages
atmosphere of antipathy and antagonism
while ER does not have to endure the - It is not the principal cause of action in an illegal
continued services of an EE whom it has lost dismissal case
confidence
- Merely one of the reliefs extended to an EE who is
o Reinstatement is not practical for EE who is no unjustly dismissed
longer welcome and imposing the EE’s position in the
company where he is no longer welcome would only - In illegal dismissal case = Principal cause of action
poison their relations to their mutual prejudice  is the unlawful deprivation of one’s employment by
irritations would only recur if unwanted EE has to be the employer in violation of the right of security of
tolerated by the reluctant ER tenure

 This is not conducive to industrial peace - A form of relief that restores the income that was
lost by reason of unlawful dismissal
When to Invoke Doctrine of Strained
- RATIO: an EE whose dismissal is found to be illegal
Relations is considered as not having left his office so that he is
- Such matter of strained relations should be raised entitled to all the rights and privileges that accrue to
and proved before the LA him by virtue of the office that he held

o UNLESS: strained relations arose after the Distinction between Backwages and Unpaid
filing of the case, as when antagonistic wages
feelings that stemmed from the filing of the
- Backwages = compensation which an EE would
complaint deepened during the 8-year
have earned had he not be unjustly dismissed
pendency of the case

MNI NOTES |256


- Unpaid wages = compensation for services already of dismissal was found to be too harsh, full
rendered by the withheld by the ER Backwages should not be awarded because that
would in effect absolve the EE from his wrongdoing
Amount of Backwages that may be awarded
- Awarded also when there is delay in filing of the
- Art. 279 of the LC provides that an unjustly complaint of ID (e.g. 2 years lapse)
dismissed EE is entitled to full Backwages from time
his compensation was withheld up to the time of his o Period of delay in instituting the action for
actual reinstatement reinstatement may be deducted from liability
for Backwages
o This must not mean that unjustly dismissed
EE is auto-entitled to full Backwages o A ruling that would permit a dismissed
laborer to earn back wages for all time, or for
o LA and NLRC have discretion to determine a very long period of time, is not only unjust
how much Backwages should be awarded to the employer but the same would foster
taking into account the facts and indolence on the part of the laborers.
circumstances of each case
The laborer is not supposed to be relying on
o Note that dismissal could be illegal or a court judgment for his support, but should
unjust because the EE was dismissed: do everything a reasonable man would do; he
 On grounds specifically prohibited should find employment as soon as an
by law (Art. 118, 248(f) and 286 of employment has been lost, especially when
LC) the employment has to depend on a litigation.

 Without any cause whatsoever (EE He should try to minimized the loss that may
not committed an offense) be caused to the employer by looking for
other work in which he can be employed
 Without just cause (EE committed
an offense but penalty of dismissal No Backwages
was not commensurate) - Backwages may not be awarded in any of the
following circumstances:
Full Backwages
o GF on part of ER – as e.g
- Entitles EE who was dismissed on grounds
specifically prohibited by law  ER honestly believed that
o Under Art. Art. 118, 137, 248(f) and 286 of dismissal was the proper penalty for
LC offense committed, reinstatement
without Backwages would be
o Ratio: EE should not have been dismissed appropriate relief
in the first place
 When ER honestly believed that it
- Entitles EE who was dismissed without any cause could dismiss EE based on a closed
shop provision of the CBA
o Ratio: EE does not deserve any penalty
considering he has not committed any offense Cessation of employment brought about
neither by dismissal nor abandonment
Limited Backwages
 Where the EE’s failure to work
- In situation where EE was dismissed without just
was caused neither by his
cause like EE committed an offense and the penalty

MNI NOTES |257


abandonment nor by dismissal, - Cases where reinstatement is no longer possible:
burden of eco. Loss is not rightfully
shifted to ER o Position no longer exist at time of
reinstatement
• Each party must bear his
own loss and hence, ER not  EE should be given substantially
to be liable for bckwages. equivalent position

o Position previously occupied by EE is


already filled up
Reinstatement
 EE should be given substantially
- Relief separate and distinct from Backwages
equivalent position
o Usually is a concomitant of Backwages; but
• To insist on reinstatement
the two are not necessarily complements nor
would merely compound the
award of one is a condition precedent to an
injustice- ER to terminate
award of the other
the services of the new hire
- Simply means, restores the lost position (while who replaced the illegally
Backwages → restores lost income dismissed EE just for latter
to assume former position
- Restoration to state from which one has been
removed/separated - Cases where reinstatement is rendered impossible
and Substantially equivalent position is not available
- Return to position from which he was removed
 REMEDY: separation pay in lieu of reinstatement
- NO reinstatement in cases:
o ER has closed down business
o To a position EE which never occupied
o ER undertook reorganization resulting to
o To a permanent position of an originally
abolition of position previously occupied by
temporary EE
EE
- A reinstated EE may be required to undergo
o ER undertook retrenchment measures or
physical/medical examination in order to determine
drastic reduction of personnel
fitness to work but such should not be a precondition
for reinstatement UNFEASIBLE reinstatement → EE dismissed has
reached retirement age of 60
Meaning of Reinstatement “without loss of
seniority rights” o Relief separation pay is not available

- That upon reinstatement, EE is to be treated in o EE entitled only to Backwages up to time


matters involving rank, position and continuity of when he reached retirement age plus
employment as though he has not been absent from retirement pay
work.
Propriety of Reinstatement
Alternative Relief if reinstatement is no longer - Relief is available only to EEs who is
possible unjustly/illegally dismissed.

MNI NOTES |258


o If not, as when severance of employment o Redundancy
was brought by abandonment/refusal to
work, reinstatement cannot be properly o Installation of labor-saving devices
ordered. o Retrenchment

o Closure of establishment not due to serious


Separation Pay business losses

→ Intended to provide the EE money during period o Disease


in which he will be looking for another employment o Lay-off/suspension of operations for more
→ Distinct from ”Backwages” (which is designed to that 6 mos.
restore income that was lost by reason of unjust - If EE have been unjustly dismissed, is SP proper?
dismissal)
o Exceptional circumstances entitling such
Concept of SP EE to SP:
- An aid given to an EE upon his separation from  If the reinstatement of the EE has
service so that he may have something on which to been rendered impossible by
fall back when he loses his means of livelihood. supervening events (such as:
- Amount designed to provide him with the • closure of establishment,
wherewithal during the period that he is looking for
employment. • sale/transfer of business
ownership,
Purpose of SP
• abolition of position
- a social legislation, to alleviate the difficulties which
confront a dismissed employee thrown into the streets • reduction of personnel
to face the harsh necessities of life.
• physical incapacity of EE)
- It is for this reason that the said statute compels the
 If the reinstatement of the EE is no
employer to dole out money, reasonable under
circumstances, to cushion the adverse effects of longer feasible
sudden separation from employment. • Doctrine of Strained
- This gives the employee a leeway, commensurate to Relations
his years of service, to tide him and his family over in • No substantially equivalent
the meantime that he goes job hunting. position is available
- To one who has been accustomed to a certain type - If EE was dismissed for a just and valid cause, is he
of job in one company, adjustment to other job entitled to SP?
opportunities becomes a problem. Advanced age, too,
may reduce him to a low priority in the labor market o GR: Such worker is not entitled

When SP is Proper o EXCEPTION: SC held that SP may be awarded as


measure of social justice even if the dismissal is found
- Awarded to EE’s who are terminated by reason of: to be valid and justified, but only in those instances

MNI NOTES |259


where EE was validly dismissed for a cause other than a. Installation of labor-saving device
serious misconduct or offenses reflecting on his moral
b. Redundancy
character.
2. ONE-HALF (½) MONTH PAY FOR
- EE resigned from employment, is he entitled to SP? EVERY YEAR OF SERVICE

o GR: NO a. Retrenchment to prevent losses


b. Closure of establishment NOT due
o EXCEPTION: it is stipulated in the employment to serious business losses
contract, CBA or established employer
practice/policy  NO SEPARATION PAY – Closure of
establishment due to losses
- EE retires from employment, entitled to SP?
PHILOSPHY BEHIND THE GRANT OF
o GR: NO SEPARATION PAY
 To enable the employee to have something on
 He is only entitled to retirement pay, which
which to fall back when he loses his job.
is different for SP.
COMPUTATION OF SEPARATION PAY
 Retirement = result of a bilateral act of
 The latest salary shall be used
parties, a voluntary agreement between ER
and EEs whereby latter after reaching a o EXCEPTION – latest salary was reduced
certain age agrees and/or consents to sever by the employer to defeat the intention of the
his employment with the former. Labor Code
o Salary rate before deduction shall be used
 Dismissal = refers to unilateral act of ER
in terminating the services of an EE.  SEASONAL EMPLOYEES – one-half of their
respective average monthly pay during the last season
Entitlement to Separation Pay multiplied by the number of years they actually
General Rule: An employee who voluntarily resigns rendered service
from his employment is not entitled to separation pay. o SEPARATION PAY = ½ of average
Exception: monthly pay last season X number of years
they actually rendered service
- When stipulated in the employment contract
o Worked at least 6 months
- When stipulated in the CBA
E. Retirement
- If sanctioned by established employer
practice or policy. Article 302, LC
AMOUNT OF SEPARATION PAY ART. 302. [287] Retirement.

 At least one (1) month pay or the following amount, Any employee may be retired upon reaching the
whichever is higher retirement age established in the collective bargaining
agreement or other applicable employment contract.
1. ONE (1) MONTH PAY FOR EVERY
YEAR OF SERVICE

MNI NOTES |260


In case of retirement, the employee shall be entitled
to receive such retirement benefits as he may have
earned under existing laws and any collective Laya v. Philippine Veterans Bank, GR
bargaining agreement and other agreements: 205813, Jan. 10, 2018
Provided, however, retirement benefits under any
FACTS:
collective bargaining and other agreements shall not
be less than those provided therein.
• This is a petition for review on Certiorari
In the absence of a retirement plan or agreement filed by petitioner Alfredo Laya, Jr.
providing for retirement benefits of employees in the assailing the decision promulgated by the
establishment, an employee upon reaching the age of CA, which upheld the ruling of the NLRC
sixty (60) years or more, but not beyond sixty-five and LA, finding no illegal dismissal by
(65) years which is hereby declared the compulsory
respondent Philippine Veterans Bank
retirement age, who has served at least five (5) years
in the said establishment, may retire and shall be
(PVB).
entitled to retirement pay equivalent to at least one-
• Petitioner was hired by respondent PVB as
half (1/2) month salary for every year of service, a
fraction of at least six (6) months being considered as its Chief Legal Counsel with a rank of Vice
one whole year. President. Pursuant to a retirement plan
imposed by the company with which the
Unless the parties provide for broader inclusions, the
normal retirement is at age 60, petitioner
term one-half (1/2) month salary shall mean fifteen
was informed thru letter by the private
(15) days plus one-twelfth (1/12) of the 13th month
pay and the cash equivalent of not more than five (5) respondent of his retirement effective on 1
days of service incentive leaves. July 2007.

Retirement is a withdrawal from office, public • On 26 June 2008, private respondent and
station, business, occupation, or public duty upon PVB president Ricardo A. Balbido, Jr.
reaching a certain age or after rendering a certain (Balbido, Jr.) issued a memorandum
number of years of service
directing the petitioner to continue to
Retirement Resignation discharge his official duties and functions
- Bilateral act of the - Unilateral act of en as chief legal counsel pending his request.
employer and employee. However, on 18 July 2007, petitioner was
employee. - Terminates informed by Balbido Jr. that his request for
- Puts an end to the employment relations
employment relations with the employer for an extension of tenure was denied.
upon reaching a certain personal reasons.
age or after rendering a • According to the petitioner, he was made
certain number of years aware of the retirement plan of respondent
of service. PVB only after he had long been employed
and was shown a photocopy of the
Retirement Dismissal Retirement Plan Rules and Regulations and
- Bilateral act of both - Unilateral act of an that Balbido, Jr. had told him upon his
employer and employer in terminating request for extension of his service that the
employee. the services of an same would be denied "to avoid
employee for cause.
precedence.” He sought the reconsideration

MNI NOTES |261


of the denial of the request for the extension appointment letter apparently enumerated
of his retirement, but PVB certified his only the minimum benefits that he would
retirement from the service. enjoy during his employment by PVB, and
contained no indication of PVB having
• The petitioner filed his complaint for illegal given him a copy of the program itself in
dismissal against PVB and Balbido, Jr. in order to fully apprise him of the contents
the NLRC to protest his unexpected and details thereof.
retirement.
• With the petitioner having been thus
ISSUE: dismissed pursuant to the retirement
provision that he had not knowingly and
• Was petitioner validly retired at age 60
voluntarily agreed to, PVB was guilty of
when he did not expressly agree to the
illegal dismissal as to him.
terms of an early retirement plan?

HELD:
United Doctors Medical Center v. Bernadas,
• No, petitioner was not validly retired at age
GR 209468, Dec. 13, 2017
60 when he did not expressly agree to the
terms of an early retirement plan.

• The mere mention of the retirement plan in


the letter of appointment did not Father Saturnino Urios University (FSUU),
sufficiently inform the petitioner of the Inc., v. Curaza, GR 223621, June 10, 2020
contents or details of the retirement
program. To construe from the petitioner's
acceptance of his appointment that he had
acquiesced to be retired earlier than the
compulsory age of 65 years would,
therefore, not be warranted. This is because
retirement should be the result of the
bilateral act of both the employer and the
employee based on their voluntary
agreement that the employee agrees to
sever his employment upon reaching a
certain age.

• Moreover, it was incumbent upon PVB to


prove that the petitioner had been fully
apprised of the terms of the retirement
program at the time of his acceptance of the
offer of employment. PVB did not
discharge its burden, for the petitioner's

MNI NOTES |262

You might also like